You are on page 1of 486

Fundamentals of

Refrigeration
Second Edition
Donald L. Fenton
SI
International System

A Course Book for


Self-Directed or Group Learning

Includes Skill Development Exercises


for PDH, CEU, or LU Credits

SDL_cover_SI.indd 1 10/24/2016 3:24:47 PM


Fundamentals of
Refrigeration

Second Edition

Donald L. Fenton

A Course Book for Self-Directed or Group Learning

Atlanta
Fundamentals of Refrigeration (SI), Second Edition
A Course Book for Self-Directed or Group Learning
ISBN 978-1-939200-14-3 (paperback)
ISBN 978-1-939200-15-0 (PDF)

© 2004, 2016 ASHRAE


All rights reserved.

ASHRAE is a registered trademark in the U.S. Patent and Trademark Office, owned by the American Society of Heat-
ing, Refrigerating and Air-Conditioning Engineers, Inc.

No part of this publication may be reproduced without permission in writing from ASHRAE, except by a reviewer who
may quote brief passages or reproduce illustrations in a review with appropriate credit; nor may any part of this book
be reproduced, stored in a retrieval system, or transmitted in any way or by any means (electronic, photocopying,
recording or other) without permission in writing from ASHRAE. Requests for permission should be submitted at
www.ashrae.org/permissions.

ASHRAE has compiled this publication with care, but ASHRAE has not investigated, and ASHRAE expressly disclaims
any duty to investigate, any product, service, process, procedure, design or the like that may be described herein. The
appearance of any technical data or editorial material in this publication does not constitute endorsement, warranty, or
guaranty by ASHRAE of any product, service, process, procedure, design or the like. ASHRAE does not warrant that the
information in this publication is free of errors. The entire risk of the use of any information in this publication is assumed
by the user.

ASHRAE STAFF
ASHRAE Learning Institute Special Publications Publisher
Karen Murray Mark Owen W. Stephen Comstock
Manager of Professional Editor/Group Manager of
Development Handbook and Special Publications
Martin Kraft Cindy Sheffield Michaels
Managing Editor of Professional Managing Editor
Development Sarah Boyle
Kelly Arnold Assistant Editor
Administrative Assistant Lauren Ramsdell
Assistant Editor
Michshell Phillips
Editorial Coordinator

For course information or to order additional materials, please contact:

ASHRAE Learning Institute Telephone: 404/636-8400


1791 Tullie Circle, NE Fax: 404/321-5478
Atlanta, GA 30329 Web: www.ashrae.org/ali
E-mail: edu@ashrae.org

Errors or omissions in the data should be brought to the attention of Special Publications via SDLcorrections@ashrae.org.

Updates and errata for this publication will be posted on the


ASHRAE website at www.ashrae.org/publicationupdates.
Your Source for HVAC&R Professional Development
1791 Tullie Circle, NE • Atlanta, GA 30329-2305 • Phone: 678.539.1146 • Fax 678.539.2146 • www.ashrae.org

Karen M. Murray kmurray@ashrae.org


Manager of Professional Development

Dear Student,
Welcome to this ASHRAE Learning Institute (ALI) self-directed or group learning course. We look forward
to working with you to help you achieve maximum results from this course.

You may take this course on a self-testing basis (no continuing education credits awarded) or on an ALI-
monitored basis with credits (PDHs, CEUs or LUs) awarded. ALI staff will provide support and you will
have access to technical experts who can answer inquiries about the course material. For questions or tech-
nical assistance, contact us at 404-636-8400 or edu@ashrae.org.

Skill Development Exercises at the end of each chapter will gauge your comprehension of the course mate-
rial. If you take this course for credit, please complete the exercises and either email copies from each
chapter to edu@ashrae.org (preferred method) or fax them to 678-539-2161. Be sure to include your stu-
dent ID number with each set of exercises. (Your student ID can be the last five digits of your Social Secu-
rity number or other unique five-digit number you create.) We will return answer sheets to the Skill
Development Exercises and maintain records of your progress. Please keep copies of your completed exer-
cises for your records.

When you finish all exercises, please submit the course evaluation, which is located at the back of your
course book. Once we receive all chapter exercises and the evaluation, we will send you a Certificate of
Completion indicating 35 PDHs/LUs or 3.5 CEUs of continuing education credit. The ALI does not award
partial credit for self-directed or group learning courses. All exercises must be completed to receive full
continuing education credit. You have two years from the date of purchase to complete each course.

We hope your educational experience is satisfying and successful.

Sincerely,

Karen M. Murray
Manager of Professional Development

ASHRAE
AN INTERNATIONAL ORGANIZATION
Continuing Education Opportunities
from ASHRAE Learning Institute

ASHRAE Learning Institute (ALI) provides professional development through in-depth train-
ing that is timely, practical, and targeted to engineers in consulting practices, facility management,
or supplier support with instruction on applying ASHRAE standards and employing new technol-
ogies essential for advanced building performance.

HVAC Design Essentials and Applications Training—Instructor Led at Approved Locations


Expand your knowledge and understanding of the fundamentals and technical aspects to
design and maintain HVAC systems. Level I covers essentials. Level II instructs on use of
ASHRAE Standards 55, 62.1, 90.1, and 189.1. A companion course explains improving existing
building operations. www.ashrae.org/hvactraining

Online Courses—Instructor Led on the Web


ALI offers high-quality, instructor-led online courses that allow attendees to learn from any-
where with an Internet connection. Course categories include Commissioning, Energy Efficiency,
Environmental Quality, HVAC&R Applications, and Standards and Guidelines. www.ashrae.org/
onlinecourses

ASHRAE Chapter and In-Company Training—Instructor Led at Your Location


ALI offers a wide range of instruction that helps chapters and companies close the gap
between entry-level engineers and seasoned practitioners. ASHRAE’s courses bring your team up
to speed on current standards and explain how to apply new technologies with real-world, bottom-
line emphasis. ASHRAE will arrange for an instructor to visit your location or license use of edu-
cational materials. www.ashrae.org/chaptercourses and www.ashrae.org/companycourses

eLearning—Web-Based Instruction on Demand


ASHRAE eLearning focuses on key skills and practical applications in HVAC&R and related
areas. Because it is web based, students can train from any computer with Internet access. This
makes it ideal for both individual and corporate training. www.ashrae.org/elearning

Self-Directed Learning Texts—Self Study or Texts for Group Instruction


For those seeking traditional book-based instruction, ASHRAE offers Learning Texts for self-
study or group training with instructor materials. Texts cover the basics of what a practicing engi-
neer needs for real-world HVAC&R applications. Skill Development Exercises are included to
evaluate progress. Students receive a course completion certificate designating continuing educa-
tion credits. www.ashrae.org/sdl

ASHRAE Learning Institute · www.ashrae.org/education


Donald L. Fenton was born in Doylestown, PA, and raised in the Tren-
ton, NJ, area. He attended Kansas State University (KSU), and during
his undergraduate summers he worked as an engineer trainee at the
Naval Air Propulsion Test Center near Trenton. In 1969, he obtained a
bachelor's degree in Mechanical Engineering from KSU and went on to
receive master and doctorate degrees in Mechanical Engineering from
the University of Illinois at Urbana-Champaign in 1971 and 1974,
respectively.
Upon completing his graduate studies, Don worked at the Fine Parti-
cles Section of the IIT Research Institute in Chicago, where he con-
ducted contract research in the areas of air-quality assessment and air-
pollution control. In 1977, he relocated to New Mexico State University
in Las Cruces, where he taught classes and researched solar energy and
combustion systems for nine years. In 1986, he moved to KSU, where
he currently teaches a variety of undergraduate and graduate courses,
including Thermal Systems Design, in the thermal science area. He also
conducts research, some ASHRAE sponsored, relating to the energy
field. Don is the department head of the Mechanical and Nuclear Engi-
neering Department at KSU. He is a registered professional engineer
and has consulted on many industrial projects.
Don resides in Manhattan, KS, with his wife, Mary. Their three
sons, Adam, James, and Michael have completed college and are devel-
oping their own careers. A great joy for Mary and Don is to visit their
grandchildren, Charlie, Lydia, Audrey, and George in Newnan, GA.
Table of Contents
Preface . . . . . . . . . . . . . . . . . . . . . . . . . . . . . . . . . . . . . . . . . . . . . . . . . xi
Acknowledgments . . . . . . . . . . . . . . . . . . . . . . . . . . . . . . . . . . . . . . . xiii
Chapter 1: Fundamentals of Refrigeration. . . . . . . . . . . . . . . . . . . . . . 1
Introduction . . . . . . . . . . . . . . . . . . . . . . . . . . . . . . . . . . . . . . . . . . . . . 1
Types of Refrigeration Systems . . . . . . . . . . . . . . . . . . . . . . . . . . . . . . . 2
Major Processes of Vapor-Compression Refrigeration . . . . . . . . . . . . . . 7
Heat-Pump Cycle . . . . . . . . . . . . . . . . . . . . . . . . . . . . . . . . . . . . . . . . . 9
The Field of Refrigeration. . . . . . . . . . . . . . . . . . . . . . . . . . . . . . . . . . . 11
Skill Development Exercises. . . . . . . . . . . . . . . . . . . . . . . . . . . . . . . . . 16
Chapter 2: Fundamentals of Thermodynamics . . . . . . . . . . . . . . . . . 17
Introduction . . . . . . . . . . . . . . . . . . . . . . . . . . . . . . . . . . . . . . . . . . . . 17
Thermodynamic Properties of Fluids . . . . . . . . . . . . . . . . . . . . . . . . . . 20
Thermodynamic Laws and Applications . . . . . . . . . . . . . . . . . . . . . . . . 27
Efficiency of Refrigeration and Heat-Pump Cycles . . . . . . . . . . . . . . . . . 30
Energy Calculations . . . . . . . . . . . . . . . . . . . . . . . . . . . . . . . . . . . . . . . 34
Skill Development Exercises. . . . . . . . . . . . . . . . . . . . . . . . . . . . . . . . . 42
Chapter 3: Multistage and Cascade Refrigeration Cycles . . . . . . . . . 45
Introduction . . . . . . . . . . . . . . . . . . . . . . . . . . . . . . . . . . . . . . . . . . . . 46
Single-Stage Ideal Refrigeration Cycle . . . . . . . . . . . . . . . . . . . . . . . . . . 47
Two-Stage Ideal Refrigeration Cycle . . . . . . . . . . . . . . . . . . . . . . . . . . 50
Cascade Refrigeration Cycle . . . . . . . . . . . . . . . . . . . . . . . . . . . . . . . . 57
Refrigeration-System Performance Parameters. . . . . . . . . . . . . . . . . . . 59
Deviations of Actual Refrigeration Systems from
Ideal Systems. . . . . . . . . . . . . . . . . . . . . . . . . . . . . . . . . . . . . . . . . . . . 62
Refrigeration System Types . . . . . . . . . . . . . . . . . . . . . . . . . . . . . . . . . 63
Skill Development Exercises. . . . . . . . . . . . . . . . . . . . . . . . . . . . . . . . . 79
Chapter 4: Evaporators . . . . . . . . . . . . . . . . . . . . . . . . . . . . . . . . . . . 81
Introduction . . . . . . . . . . . . . . . . . . . . . . . . . . . . . . . . . . . . . . . . . . . . 81
Psychrometrics of Moist-Air Cooling . . . . . . . . . . . . . . . . . . . . . . . . . . 82
Air-Cooling Evaporators . . . . . . . . . . . . . . . . . . . . . . . . . . . . . . . . . . . 91
Liquid-Cooling Evaporators . . . . . . . . . . . . . . . . . . . . . . . . . . . . . . . . 104
Skill Development Exercises. . . . . . . . . . . . . . . . . . . . . . . . . . . . . . . . 110
viii Contents

Chapter 5: Compressors . . . . . . . . . . . . . . . . . . . . . . . . . . . . . . . . . 111


Introduction . . . . . . . . . . . . . . . . . . . . . . . . . . . . . . . . . . . . . . . . . . . 112
Reciprocating Compressors . . . . . . . . . . . . . . . . . . . . . . . . . . . . . . . . 112
Rotary Screw Compressors . . . . . . . . . . . . . . . . . . . . . . . . . . . . . . . . 125
Rotary Vane Compressors. . . . . . . . . . . . . . . . . . . . . . . . . . . . . . . . . 135
Scroll Compressors . . . . . . . . . . . . . . . . . . . . . . . . . . . . . . . . . . . . . . 136
Centrifugal Compressors . . . . . . . . . . . . . . . . . . . . . . . . . . . . . . . . . . 138
Skill Development Exercises. . . . . . . . . . . . . . . . . . . . . . . . . . . . . . . . 148
Chapter 6: Condensers . . . . . . . . . . . . . . . . . . . . . . . . . . . . . . . . . . 151
Introduction . . . . . . . . . . . . . . . . . . . . . . . . . . . . . . . . . . . . . . . . . . . 151
Condensation Processes . . . . . . . . . . . . . . . . . . . . . . . . . . . . . . . . . . 154
Air-Cooled Condensers. . . . . . . . . . . . . . . . . . . . . . . . . . . . . . . . . . . 156
Water-Cooled Condensers . . . . . . . . . . . . . . . . . . . . . . . . . . . . . . . . 160
Evaporative Condensers . . . . . . . . . . . . . . . . . . . . . . . . . . . . . . . . . . 163
Skill Development Exercises. . . . . . . . . . . . . . . . . . . . . . . . . . . . . . . . 171
Chapter 7: Refrigerant Flow in Pipes, Valves, and Pumps . . . . . . . . 173
Introduction . . . . . . . . . . . . . . . . . . . . . . . . . . . . . . . . . . . . . . . . . . . 173
Fluid Flow in Pipes . . . . . . . . . . . . . . . . . . . . . . . . . . . . . . . . . . . . . . . 175
Valve and Pipe-Fitting Equivalent Length . . . . . . . . . . . . . . . . . . . . . . . 184
Pumps. . . . . . . . . . . . . . . . . . . . . . . . . . . . . . . . . . . . . . . . . . . . . . . . 188
Skill Development Exercises. . . . . . . . . . . . . . . . . . . . . . . . . . . . . . . . 196
Chapter 8: Expansion Devices. . . . . . . . . . . . . . . . . . . . . . . . . . . . . 199
Introduction . . . . . . . . . . . . . . . . . . . . . . . . . . . . . . . . . . . . . . . . . . . 199
Capillary Tubes and Short Tube Restrictors . . . . . . . . . . . . . . . . . . . . 200
Pressure Control Valves. . . . . . . . . . . . . . . . . . . . . . . . . . . . . . . . . . . 204
Thermostatic Expansion Devices . . . . . . . . . . . . . . . . . . . . . . . . . . . . 206
Electronic Expansion Valves . . . . . . . . . . . . . . . . . . . . . . . . . . . . . . . . 213
Hand Expansion Valves . . . . . . . . . . . . . . . . . . . . . . . . . . . . . . . . . . . 215
Level Control Valves . . . . . . . . . . . . . . . . . . . . . . . . . . . . . . . . . . . . . 215
Turbo Expanders. . . . . . . . . . . . . . . . . . . . . . . . . . . . . . . . . . . . . . . . 217
Skill Development Exercises. . . . . . . . . . . . . . . . . . . . . . . . . . . . . . . . 225
Chapter 9: Pressure Vessels and Refrigerant Management . . . . . . . 227
Introduction . . . . . . . . . . . . . . . . . . . . . . . . . . . . . . . . . . . . . . . . . . . 227
Suction Line Accumulators . . . . . . . . . . . . . . . . . . . . . . . . . . . . . . . . . 228
Receivers . . . . . . . . . . . . . . . . . . . . . . . . . . . . . . . . . . . . . . . . . . . . . 230
Fundamentals of Refrigeration SI, Second Edition ix

Surge Drums. . . . . . . . . . . . . . . . . . . . . . . . . . . . . . . . . . . . . . . . . . . 247


Flash Tanks and Intercoolers . . . . . . . . . . . . . . . . . . . . . . . . . . . . . . . 247
Refrigerant Inventory . . . . . . . . . . . . . . . . . . . . . . . . . . . . . . . . . . . . . 248
Skill Development Exercises. . . . . . . . . . . . . . . . . . . . . . . . . . . . . . . . 252
Chapter 10: Refrigerant Selection . . . . . . . . . . . . . . . . . . . . . . . . . . 253
Introduction . . . . . . . . . . . . . . . . . . . . . . . . . . . . . . . . . . . . . . . . . . . 254
Types of Refrigerants . . . . . . . . . . . . . . . . . . . . . . . . . . . . . . . . . . . . . 256
Saturation Pressure and Temperature of a Refrigerant . . . . . . . . . . . . 261
Refrigeration Capacity and Efficiency. . . . . . . . . . . . . . . . . . . . . . . . . . 262
Safety of Refrigerants . . . . . . . . . . . . . . . . . . . . . . . . . . . . . . . . . . . . . 265
Environmental Impact of Refrigerants . . . . . . . . . . . . . . . . . . . . . . . . . 269
Codes and Standards. . . . . . . . . . . . . . . . . . . . . . . . . . . . . . . . . . . . . 270
Skill Development Exercises. . . . . . . . . . . . . . . . . . . . . . . . . . . . . . . . 278
Chapter 11: Product Cooling, Freezing Loads, and
Cooling Secondary Fluids . . . . . . . . . . . . . . . . . . . . . . . . . . . . . . 279
Introduction . . . . . . . . . . . . . . . . . . . . . . . . . . . . . . . . . . . . . . . . . . . 280
Cooling Food Products . . . . . . . . . . . . . . . . . . . . . . . . . . . . . . . . . . . 282
Freezing Food Products . . . . . . . . . . . . . . . . . . . . . . . . . . . . . . . . . . . 287
Food Freezing Equipment . . . . . . . . . . . . . . . . . . . . . . . . . . . . . . . . . 293
Cooling Liquid-Food Products . . . . . . . . . . . . . . . . . . . . . . . . . . . . . . 297
Cooling Secondary Fluids. . . . . . . . . . . . . . . . . . . . . . . . . . . . . . . . . . 299
Skill Development Exercises. . . . . . . . . . . . . . . . . . . . . . . . . . . . . . . . 311
Chapter 12: Practical Guide to Refrigeration Systems. . . . . . . . . . . 313
Introduction . . . . . . . . . . . . . . . . . . . . . . . . . . . . . . . . . . . . . . . . . . . 314
Refrigeration Racks and Display Cases . . . . . . . . . . . . . . . . . . . . . . . . 315
Refrigerated Warehouses. . . . . . . . . . . . . . . . . . . . . . . . . . . . . . . . . . 325
System Operation . . . . . . . . . . . . . . . . . . . . . . . . . . . . . . . . . . . . . . . 333
Meat Processing . . . . . . . . . . . . . . . . . . . . . . . . . . . . . . . . . . . . . . . . 334
Pharmaceutical Refrigeration . . . . . . . . . . . . . . . . . . . . . . . . . . . . . . . 341
Refrigeration . . . . . . . . . . . . . . . . . . . . . . . . . . . . . . . . . . . . . . . . . . . 345
Skill Development Exercises. . . . . . . . . . . . . . . . . . . . . . . . . . . . . . . . 347
Appendix A: Sample Refrigerants. . . . . . . . . . . . . . . . . . . . . . . . . . . 349
Appendix B: Pressure-Enthalpy Diagrams . . . . . . . . . . . . . . . . . . . . 359
Appendix C: Supplemental Tables . . . . . . . . . . . . . . . . . . . . . . . . . . 367
Skill Development Exercises
Preface
Refrigeration has been in existence for over 150 years, and over the
years many books have been written explaining its theory and applica-
tion. Despite this, a modern introduction to refrigeration was not pub-
lished until 1999, when ASHRAE recognized this need and
commissioned Donald L. Fenton to author the first edition of Funda-
mentals of Refrigeration. This text is a revised edition of the 1999 text
with the notable addition of a final chapter presenting example applica-
tions of refrigeration.
This second edition continues as an introduction to the field of
refrigeration. The three major areas of refrigeration are covered: com-
fort cooling or air conditioning, commercial refrigeration, and indus-
trial refrigeration. The text is divided into chapters that are sequenced
in a logical progression from the first principles that underpin refrigera-
tion theory, through thermodynamic cycles and practical details, and
finally to four prevalent applications— compressor racks in supermar-
kets, refrigeration warehouses, meat processing, and a pharmaceutical
application. Exercises are available at the end of each chapter for stu-
dents and readers to assess their understanding of the material and to
apply their knowledge to new situations. Current ASHRAE Handbook
volumes are used as references throughout, as these are readily avail-
able and contain the latest results of ASHRAE and other research.
The intended audience for this book is anyone seeking an introduc-
tion to refrigeration based on thermodynamic principles. The text con-
tains the thermodynamic property data and other information necessary
to understand the material and to complete the exercises. Two quite dif-
ferent kinds of users are targeted in this book. One is the recent
mechanical engineering school graduate who didn’t study refrigeration
while in college. The other is the veteran technician who has worked in
the refrigeration field for a number of years. Both of these, and anyone
who falls between, will benefit from this book. The presentations in the
text use familiar lines of reasoning and appropriate explanations suit-
able for all students and readers seeking an introduction to refrigera-
tion. Consequently, this text strives to be both effective and useful as an
introduction to refrigeration.
Acknowledgments
Refrigeration is a major component of ASHRAE’s collective research
effort and knowledge base. Certainly, the ASHRAE Handbook—Refrig-
eration, published by ASHRAE every four years, maintains the best
up-to-date information on refrigeration systems and their application.
Acknowledgment is due to ASHRAE for maintaining these activities
for the past 50 years. Much of the technical data used in this book was
drawn from these Handbooks.
Additionally, ASHRAE is acknowledged for providing support of
the author in developing the text, example illustrations, and homework
exercises.
Several leaders in the refrigeration field have made suggestions that
improved the text, corrected errors, and changed several exercises to
make them more effective. These people include: George Briley, Ron-
ald Cole, Brian Friecke, Daniel Kramer, Douglas Rindl, Donald Siller,
and Ronald Vallort. Additionally, Justin Smith, while a senior mechani-
cal engineering student at Kansas State University and using a draft
copy of the text while studying refrigeration in a special topics course,
corrected numerous errors. Special acknowledgment is made to Profes-
sor Will Stoecker for first introducing me to the industrial refrigeration
field and mentoring me often on the topic.
Very special acknowledgment is reserved for my wife, Mary, who
proofread the entire text and corrected many grammatical and context
errors. Also, sincere thanks to my three sons, Adam, James, and
Michael, for allowing me to develop this work when it meant time not
spent with them.
In addition, the unit conversions from Inch-Pound (I-P) to Systéme
International (SI) completed by Jason Robbins, PE, are gratefully
acknowledged.

Donald L. Fenton
Manhattan, KS

July 2016
Fundamentals of
Refrigeration

Second Edition

Donald L. Fenton
Fundamentals of
Refrigeration

Study Objectives
Chapter 1 introduces the concept of refrigeration and explains how several dif-
ferent refrigeration systems operate. The classification of refrigeration systems
and sample applications of refrigeration are discussed. After studying
Chapter 1, you should be able to

 define refrigeration;
 name and briefly describe at least two methods of refrigeration;
 name and describe the four basic components of a vapor-compression
refrigeration system;
 describe the purpose of a heat-pump system; and
 identify three applications of refrigeration.

Instructions
Read the material in Chapter 1. At the end of the chapter, complete the skill
development exercises without referring to the text. Review those sections of
the chapter as needed to complete the exercises.

Introduction
Refrigeration is the action of removing heat from an enclosed space or material
for the purpose of lowering its temperature (it is important to note that cold is
not added to the material). Consequently, a refrigeration system must provide a
means by which heat can be moved away. Refrigeration systems do this by pro-
viding a cold surface near the material to be cooled. This surface, colder than
the material, causes heat to transfer from that material through the cold surface.
Because heat only flows from a warmer body to a colder body, the temperature
of the cold surface must be less than that of the refrigerated material.
Figure 1-1 shows a diagram of a generic refrigeration system represented
by the rectangle in the center. A refrigeration system is the collection of equip-
ment that generates cold and hot surfaces to perform refrigeration. Observe
that the temperature of the cold surface is less than the temperature of the
2 Chapter 1 Fundamentals of Refrigeration

Figure 1-1 Generic refrigeration system.

refrigerated space or material. Once heat or thermal energy passes through the
cold surface, the objective of the refrigeration system is to transport the energy
to another location—a hot surface whose temperature is higher than the sur-
rounding outside environment. Figure 1-1 shows the movement of heat from
the refrigerated material through the refrigeration system and out to the sur-
rounding atmosphere.
Again, observe that because heat only flows from a warm body to a cold
body, the hot surface temperature must be greater than that of the surroundings.
In summary, a refrigeration system removes heat from one body and transfers it
to another body.

Types of Refrigeration Systems


Several types of refrigeration systems are available, and each accomplishes the
same task of removing heat from a space or material. The material may be a
gas, liquid, or solid. For example, the refrigeration system in a domestic refrig-
erator cools the inside air to a temperature less than the surroundings, thus
maintaining an appropriate temperature for the storage of food. The means by
which the cold surface is generated in the refrigerated space distinguishes one
refrigeration system from another.

Vapor-Compression System
The vapor-compression system is the most common refrigeration system cur-
rently in use. A vapor-compression system passes a fluid called a refrigerant
through four components in sequence: evaporator, compressor, condenser, and
Fundamentals of Refrigeration SI, Second Edition 3

Photograph courtesy of the University of Idaho

Figure 1-2 A small vapor-compression system showing the components comprising the
cycle.

expansion device. Figure 1-2 displays a typical small vapor-compression sys-


tem. Since the fluid circulates through the system and returns to its original
position, the system is also referred to as a cycle. The evaporator is the compo-
nent where the actual cooling occurs. A mixture of liquid and vapor refrigerant
at low pressure and low temperature enters the evaporator. The temperature of
the refrigerant mixture in the evaporator is somewhat less than the temperature
of the refrigerated space or material. Therefore, the evaporator absorbs heat
from the space or material, causing the liquid refrigerant to boil or vaporize.
The heat transported from the refrigerated space or material causes the space to
maintain the needed temperature. The low-pressure, low-temperature refriger-
ant vapor leaves the evaporator and enters the compressor; upon exiting the
compressor, the pressure and temperature are increased to that within the con-
denser. The condenser transfers heat from the high-pressure, high-temperature
refrigerant vapor to the surroundings, causing condensation of the vapor into a
liquid. The liquid refrigerant flows to the expansion device, which decreases
the pressure and causes the formation of a low-temperature mixture of vapor
and liquid that enters the evaporator.
In the vapor-compression system, the refrigerant circulates in sequence
among the four components, which constitutes the vapor-compression cycle.
An important feature of the cycle is the conversion of the refrigerant from liq-
uid to vapor and back again as it moves through the system. In this way, the
4 Chapter 1 Fundamentals of Refrigeration

refrigerant is not consumed during operation of the system. All components of


the system operate continuously, because the refrigerant is flowing steadily
through the system.

Air-Cycle System
The air-cycle system is different from the vapor-compression system in that the
refrigerant (air) does not undergo the conversion from vapor to liquid and back
again. In the air-cycle system, air from the refrigerated space enters a compres-
sor, which increases the air’s pressure and temperature. The air then passes
through a heat exchanger, which results in the high pressure air being cooled to
a temperature near the outside ambient temperature. Next, the air moves
through an expansion device that reduces the air pressure to that of the refriger-
ated space. The temperature of the air is also reduced by the expansion device
to a value somewhat less than that of the refrigerated space. Finally, the chilled
air is re-introduced to the refrigerated space, where it mixes with and lowers
the temperature of the somewhat warmer air. Air continues to be drawn from
the refrigerated space, passing through the refrigeration cycle, until the desired
space temperature is achieved.
A variation of the air-cycle system is used in commercial aircraft to pro-
vide cabin air conditioning. The compressed air is supplied by bleeding a
small airstream from the compressor of the gas turbine engine providing pro-
pulsion. Cooling the high-pressure airstream is accomplished using a heat
exchanger (coil) over which passes high-altitude ambient air. When the cooled
high-pressure air is expanded to cabin pressure, the temperature decreases to
somewhat below the cabin air temperature. The mixing of the expanded cool
air with the cabin air cools the air in the cabin. To keep the cabin pressure
from increasing, an equal amount of warm air is allowed to flow out as the
chilled air is introduced. What differentiates the air-cycle system in aircraft
cabin cooling is that the air does not complete a cycle, but rather begins as air
entering the engine’s compressor and ends by leaking from the aircraft cabin.
This type of cycle is usually referred to as an open cycle because the same air
does not continue to circulate through the cycle.

Absorption System
An absorption system for refrigeration is similar to a vapor-compression sys-
tem in several respects. Figure 1-3 shows a simplified schematic diagram of an
absorption refrigeration system. First, a refrigerant fluid sequentially moves
through the components of the system. Second, the refrigerant moves through a
condenser, expansion device, and evaporator in a manner similar to that of the
vapor-compression cycle. The difference lies in how the low-pressure, low-
temperature refrigerant vapor is changed to a high-pressure, high-temperature
vapor. In the absorption cycle, the vapor leaving the evaporator is absorbed by
a liquid solution in a vessel called an absorber. Heat must be removed from the
Fundamentals of Refrigeration SI, Second Edition 5

Figure 1-3 Simplified schematic diagram of an absorption refrigeration system.

absorber to maintain the affinity that the vapor has for the solution. A pump
increases the pressure of the liquid solution to the level of the condensing pres-
sure; the liquid solution then enters another vessel called a generator. Heat is
added to the generator, driving the refrigerant vapor out of the solution. At this
point, the refrigerant enters the condenser, where the energy absorbed by the
refrigerant is transferred to the atmosphere. The refrigerant then passes through
an expansion device, which lowers its temperature and pressure. Finally, the
low-temperature, low-pressure refrigerant enters the evaporator, completing
the cycle. A significant quantity of heat at a moderate temperature is needed to
operate the absorption system, and a relatively small input of electrical power
is necessary to operate the pump. Thus, for example, when waste heat is avail-
able from a steam boiler, an absorption system may be employed to provide
low-cost refrigeration. Figure 1-4 shows an absorption system used to provide
chilled water for air conditioning a commercial building.
Water and ammonia are commonly paired in absorption refrigeration sys-
tems, with ammonia serving as the refrigerant. Because ammonia is the refrig-
erant, temperatures below the freezing point of water are possible. The success
of the ammonia/water pair is due to ammonia’s large affinity for water.
Another common fluid pair is water and lithium bromide. In this pair, water is
the refrigerant, and the lowest achievable temperature is limited by its freezing
temperature of 0°C. Absorption systems for air conditioning generally use the
6 Chapter 1 Fundamentals of Refrigeration

Photograph courtesy of McQuay.

Figure 1-4 An absorption refrigeration system used to supply chilled water for a commercial
building.

water/lithium bromide pair, because sufficiently low temperatures are pro-


duced.

Thermoelectric System
In the 1960s, semiconductor materials were developed that allowed for com-
mercial production of thermoelectric systems for refrigeration. These systems
depend on the Peltier effect, first observed by Jean Peltier in 1834, which states
that when an electric current passes through junctions of two dissimilar metals,
one junction is cooled and the other is heated. Consequently, a cold surface is
generated, whereupon heat may be absorbed. The hot surface receives the heat
or thermal energy from the cold junction for transfer into the environment.
Many junctions placed in series and attached to a plate forming a cold side and
another plate for the hot side provide one approach to constructing a practical
system. Figure 1-5 shows a thermoelectric cooler in two configurations: with a
flat cold plate and using an air fan.

Evaporative Cooler
In climates where the air contains little moisture (such as the desert Southwest
of the United States), the evaporative cooler is commonly used to cool residen-
tial and commercial buildings. The evaporative cooler typically consists of
Fundamentals of Refrigeration SI, Second Edition 7

Photograph courtesy of Thermoelectric Cooler of America, Inc.

(a) (b)

Figure 1-5 Thermoelectric coolers in two configurations: (a) with a flat cold plate and (b)
using an air fan.

pads over which water is dripped. This promotes contact between the water and
a warm dry airstream entering the evaporative cooler by means of a fan. The air
is cooled by the evaporation of water. Reductions in air temperature may
exceed –6.67°C, depending on how dry the outside air is. For evaporative cool-
ing to be successful, the building must allow some conditioned air to exit,
thereby allowing the cooled air to enter.

Major Processes of
Vapor-Compression Refrigeration
The processes necessary for several different types of refrigeration were
described briefly in the previous section, “Types of Refrigeration Systems.” We
now direct our attention to the widely used vapor-compression refrigeration
system, which is the focus of this course. Figure 1-6 shows the vapor-compres-
sion cycle and the relationship between the four components: evaporator, com-
pressor, condenser, and expansion device. In this system, an expansion device
is selected to carry out the refrigerant’s sudden decrease in pressure. Each of
the components is connected at its inlet and outlet by a pipe transporting the
refrigerant to the other components. As shown in Figure 1-6, the refrigerant in
the evaporator absorbs heat from the cold space, while the condenser rejects
heat to the surroundings.
The expansion valve and compressor divide the system into a low-pressure
side and a high-pressure side. The evaporator is on the low-pressure side and the
8 Chapter 1 Fundamentals of Refrigeration

Figure 1-6 Vapor-compression system.

condenser is on the high-pressure side. In Figure 1-6, the compressor is shown


with a reciprocating piston and two valves, one for the suction line from the
evaporator and one for the discharge line to the condenser. While Figure 1-6
does not accurately show the geometry of the valves in the compressor, note that
when the piston moves away from the compressor’s top (or head), the suction
valve is pulled open by the low pressure in the cylinder, and the discharge valve
is forced closed by the high-pressure refrigerant in the discharge line. When the
piston moves toward the compressor’s head, the suction valve is closed by the
higher pressure inside the cylinder, and the discharge valve opens when the
pressure in the cylinder exceeds the pressure in the discharge line. In this way,
the compressor takes in low-pressure refrigerant vapor and discharges high-
pressure vapor to the condenser. To perform its function, the compressor
requires power, which ultimately goes into the refrigerant and is revealed by an
increase in pressure and temperature.
The refrigerant vapor leaving the compressor is at a temperature exceed-
ing that of the surroundings, thus allowing heat transfer from the refrigerant to
the surroundings. The vapor entering the condenser loses heat to the surround-
ings and begins converting to a liquid (i.e., condenses). It is this condensation
Fundamentals of Refrigeration SI, Second Edition 9

process that rejects most of the heat. The temperature in the condenser, under
ideal conditions, is constant, depending on the pressure of the refrigerant. The
heat transfer rate in the condenser regulates the condensing temperature.
In Figure 1-6, vertical lines are shown on the condenser and evaporator to
represent fins (thin metal plates) augmenting the heat transfer rate. This is one
particular design for these components and the term coil is normally applied to
this configuration. Individual fans may force air through the coils, thereby fur-
ther increasing the heat transfer rate.
The expansion device has two functions in the ideal vapor-compression cycle
shown in Figure 1-6. First, it reduces the pressure of the liquid refrigerant by
passing it through a restriction, thereby forming a mixture of liquid and vapor at
a low temperature for the evaporator. Second, the expansion valve regulates the
flow of refrigerant to the evaporator. In the cycle shown in Figure 1-6, just
enough refrigerant is given to the evaporator so that all the liquid evaporates upon
leaving. Inside the evaporator, the conversion of liquid to vapor occurs at con-
stant temperature and pressure for the ideal cycle. The vaporization or boiling of
the refrigerant inside the evaporator is the process responsible for the absorption
of heat from the cold space. The necessary low temperature is produced by the
low pressure of the refrigerant maintained by the compressor.
Later in this course, this ideal vapor-compression cycle is addressed fur-
ther. Greater detail is provided as to why the system operates as it does, and
demonstrations are given for how to make calculations concerning the heat
transfer rates and other performance characteristics. Variations on this basic
cycle are also presented, including multistage systems for when the needed
pressure increase is too large for a single compressor. Deviations of actual
cycles from the ideal cycle are also discussed.

Heat-Pump Cycle

A refrigeration cycle and a heat-pump cycle differ only in purpose. A refriger-


ation cycle is applied to remove heat from a body. In contrast, a heat-pump
cycle is applied to add heat to a hot body. Heat-pump cycles are used to heat
the interior of a building above the outdoor temperature or to provide a supply
of heat to an industrial process operating at an elevated temperature.
In a vapor-compression system, the same equipment is used to construct
both refrigeration and heat-pump cycles: compressor, condenser, expansion
device, and evaporator. In a vapor-compression heat pump, the working fluid
or refrigerant passes through the components in the above order, but the con-
denser is placed near the need for heat. For example, in an air-source heat-
pump cycle applied to heat a residential building, where both the condenser
and evaporator exchange heat with air, the condenser is located inside and the
evaporator outside. With the use of a reversing valve, the flow direction of the
refrigerant may be reversed and the unit converted to a refrigeration cycle.
Figure 1-7 shows a schematic diagram of the refrigeration and heat-pump
cycles in conjunction with a reversing valve. The reversing of the refrigerant
10 Chapter 1 Fundamentals of Refrigeration

flow direction causes the condenser and evaporator to swap positions. Note
that the figure, while showing how the refrigerant flow direction is reversed,
does not show the use of two separate expansion devices. Two expansion
devices (one used for refrigeration and one used for the heat pump) are needed
to properly control the refrigerant flow rate for each cycle. Generally, check
valves are used to prevent backflow of the refrigerant in actual reversing
cycles.

(a)

(b)

Figure 1-7 Schematic diagrams of (a) a vapor-compression refrigeration system using a


reversing valve and (b) a vapor-compression heat-pump cycle with the refriger-
ant flow reversed.
Fundamentals of Refrigeration SI, Second Edition 11

The Field of Refrigeration

The field of refrigeration can be loosely divided into three categories: domes-
tic, commercial, and industrial refrigeration. Typical cooling capacities for
these categories are as follows:

• Domestic—less than 20 kW
• Commercial—more than 20 kW
• Industrial—small to very large

The refrigeration and air-conditioning fields are intertwined, but each has
its own separate considerations as well. Domestic or residential refrigeration
systems include the household refrigerator, food freezer, and the central air-
conditioning system. Commercial refrigeration includes larger refrigeration
systems than domestic refrigeration requires. Examples of commercial units
include supermarket cold-food display cases as well as restaurant walk-in cool-
ers and freezers. Industrial refrigeration is characterized by the application of
refrigeration systems to achieve specific goals, such as the cooling and freezing
of foods or the removal of heat from material streams in chemical, petroleum,
and petrochemical plants. Within the field of air conditioning, refrigeration
finds its greatest application in the cooling of air for human comfort. In terms
of measures such as units manufactured, total sales volume, and employment
level, comfort air conditioning overshadows both commercial and industrial
refrigeration.
However, human-comfort air conditioning involves considerations beyond
only the cooling of air. The term air conditioning, when applied to human
comfort, refers to those processes of air treatment that control temperature,
humidity, cleanliness, and ventilation to meet the comfort requirements of the
occupants in the space. So, air conditioning involves air-heating processes, air-
quality control, appropriate regulation of air velocity, and thermal radiation in
addition to the refrigeration or cooling processes. Generally, cooling for human
comfort requires that refrigeration equipment operate over a relatively narrow
range of temperatures, approximately 2°C to 13°C. As this temperature range
is small and the market for refrigeration equipment is large, suppliers have
standardized their offerings and packaged them for large-volume production,
thereby achieving lower unit costs.
In contrast to the air-conditioning field, industrial refrigeration involves
custom-designed systems, because the equipment operating temperatures
vary significantly for each system. The temperature range of concern for the
industrial-refrigeration designer extends to approximately –73°C. Tempera-
tures from below –73°C to near absolute zero (–273.15°C) are covered in the
field of cryogenics. These fields are bordered by flexible boundaries, which,
from time to time, refrigeration engineers and technicians successfully cross
to participate in design and development.
12 Chapter 1 Fundamentals of Refrigeration

Refrigeration systems may also be categorized by the type of application


in which they are used. The categories that follow are not absolute. Rather,
they serve to organize the field of refrigeration in several different ways so as
to provide a better understanding. The lines between categories are not abso-
lute, and systems and their applications occasionally cross from one category
to another.

Residential Air Conditioning


Residential air conditioning includes the room air conditioner and unitary or
central system where the condenser and compressor are located outdoors and
the evaporator coil is placed in the air-supply duct near the air-handling unit.
Another system type called a heat pump uses the same basic four components
(see the previous “Vapor-Compression System” section) and provides both
summer cooling and winter heating.

Vehicle Air Conditioning


The most common air-conditioned vehicle is the private automobile, but trac-
tors, buses, trucks, trains, aircraft, and ships are air conditioned as well. This
application is characterized by the refrigerated space receiving a large quantity
of solar radiation. Also, public transportation vehicles present the added chal-
lenge of quickly cooling a large number of people occupying a relatively small
space.

Medium- and Large-Sized-Building Air Conditioning


Commercial buildings may require cooling even during cold outside condi-
tions, depending on the heat loads generated internally by people, lights, and
office equipment. The responsibility of the refrigeration system is to provide
needed cooling so that the people inside the building can function comfortably
and effectively. The refrigeration system may consist of a central water-chilling
plant as part of a mechanical room. The air-conditioned spaces may be served
by one or more air-supply and return-duct systems linked to the central chiller.
Another approach is to pipe the chilled water to heat exchanger coils within the
air-conditioned spaces where a fan forces the conditioned air through the coil.
Single-story building applications sometimes employ rooftop units that supply
conditioned air directly to the occupied space.

Transport Refrigeration
Perishable products transported by trucks, rail cars, and cargo containers use
vapor-compression refrigeration systems to maintain the required temperature,
preventing loss. These transport refrigeration units are similar to those used in
Fundamentals of Refrigeration SI, Second Edition 13

other commercial applications where the compressor is powered by an electric


motor or internal combustion engine (gasoline or diesel) that allows for mobile
operation. Air returning from circulating over the products is cooled by the
refrigeration system and blown again across the product to maintain the desired
interior temperature.

Residential Refrigeration

Vapor-compression is the most-used refrigeration system for domestic full-


sized refrigerators and freezers. The refrigerator’s function is to generate one
or more low temperature spaces suitable for the storage of food. Typically, 0°C
to 4°C is suitable for fresh food storage. Freezer sections of combination
refrigerators-freezers and individual freezers must maintain approximately –
18°C storage temperature. However, smaller units, which may be portable,
may use ammonia-water absorption or thermoelectric refrigeration for applica-
tion in hotel rooms, recreational vehicles, and portable coolers, as their operat-
ing noise is considerably less.

Refrigerated Vending Machines

Refrigerated vending machines use vapor-compression refrigeration systems to


maintain bottle and can beverages as well as other perishable food products
such as ice cream at appropriate temperatures.

Industrial Air Conditioning

This course defines the broad area of industrial air conditioning to include
adjusting hostile environments for worker comfort and generating conditioned
air appropriate for manufacturing processes. The list of activities involving
refrigeration under this category is huge, represented by the following:

• Spot Cooling. Cooled air for worker relief in a hot environment where it is
impractical to cool the entire plant (such as an ore smelting plant).
• Environmental Laboratories. Conditions ranging from the simulation of
hot desert climates to those of severe winters (such as chambers for auto-
mobile tests concerned with hot and cold engine starts).
• Printing. Humidity regulation needed for proper handling of the paper and
drying of ink.
• Textiles. Regulation of humidity so that static electricity is reduced and
fibers have suitable strength and flexibility to travel at high speed through
the machines without failure.
14 Chapter 1 Fundamentals of Refrigeration

• Precision Parts and Cleanrooms. Maintaining the required temperature


and humidity to control dimensional variation for metal parts, prevent rust,
and reduce dust particles with filtration.
• Photographic Products. Proper temperature and humidity to reduce dete-
rioration of photographic raw materials and precisely controlled tempera-
tures for film coating processes.
• Mine Air Conditioning. Air cooling and ventilation for worker safety and
comfort mandated by the mine’s high humidity and high temperature and
the need for adequate oxygen.

Industrial Refrigeration
Industrial Refrigeration is another broad area for the application of refrigera-
tion. A large category within industrial refrigeration is the processing and stor-
age of food, while the pharmaceutical and chemical industries also use
specialized refrigeration systems. A partial list of applications is provided in
the following subsections.

Unfrozen- and Frozen-Food Storage


The time period during which most foods can be safely stored is significantly
increased as temperature is reduced. Foods stored at temperatures slightly
above their freezing temperature include beef, chicken, fish, eggs, bananas,
oranges, apples, tomatoes, lettuce, cabbage, potatoes, onions, and many others.
Foods stored at temperatures below freezing include ice cream, fish, poultry
products, beef, pork, concentrated fruit juice, prepared foods, and candy.

Process Freezing
Process freezing is the rapid freezing of some foods to minimize growth of ice
crystals and extend safe storage time. Common methods of food process freez-
ing include air-blast freezing, where a low-temperature airstream flows past the
product; immersion freezing by a low-temperature brine; and contact freezing,
where the product (such as ice cream cartons) is placed between cold plates.

Food Processing
Refrigeration may be required in food processing where the food’s chemical
nature is altered. Example processes that require refrigeration include the cur-
ing of milk curds for cheese production; beer fermentation, where the removal
of heat is necessary to complete the process; wine storage after fermentation at
below room temperature for a period of six months to two years; and fruit juice
concentrate production, where refrigeration is used to condense water vapor
Fundamentals of Refrigeration SI, Second Edition 15

removed from fruit juice. The freeze-drying process also uses refrigeration to
condense water removed from food products.

Chemical and Process Industries


Numerous processes that require special refrigeration systems include the sep-
aration of gases, the condensation of gases, solidification by freezing one
chemical in a mixture, storage of a liquid at a low temperature avoiding high
pressure, and the removal of heat from a chemical reaction.

Additional Applications
Additional applications include drinking fountains, dehumidifiers, ice makers,
ice-skating rinks, manufacturing plants, environmental test chambers and wind
tunnels, construction (freezing soil for excavation and concrete cooling during
hardening), and water desalination.

Next Step
The information in Chapter 2 provides a review of thermodynamics, which is
important for understanding the processes that comprise the vapor-compression
refrigeration system.

Summary
Chapter 1 introduced the refrigeration system as the means by which heat is
removed from a material or space and transported elsewhere. Refrigeration
system types, including vapor-compression, air cycle, absorption, and thermo-
electric systems, were discussed, and their essential features were provided.
Evaporative cooling was also presented, in which an airstream is cooled by the
evaporation of water. The field of refrigeration was categorized by capacity,
refrigerating temperature, and application. The boundaries of the categories
overlap but, nevertheless, provide structure to the field of refrigeration. You
should now be able to

 define what is meant by the term refrigeration;


 describe the refrigeration systems;
 discuss the similarities and differences between air conditioning and indus-
trial refrigeration;
 describe the basic operation of three refrigeration system types;
 identify and describe the four major components in a vapor-compression
refrigeration system;
 Describe the purpose of a heat-pump system and how it is different from a
refrigeration system; and
 list five major applications of refrigeration and give an example of each.
16 Chapter 1 Fundamentals of Refrigeration

Skill Development Exercises for Chapter 1


Complete these questions by writing your answers on the worksheets at the back of this book.

1-1 After arriving home from a long day at work, you open your refrigerator door
and remove a bottle containing a cold beverage. What refrigeration-cycle pro-
cesses occurred to produce your cold drink?
1-2 Describe in your own words the meaning of the word refrigeration.
1-3 For the vapor-compression refrigeration system, name the four major compo-
nents and their functions.
1-4 Describe in your own words the purpose of the heat pump-system.
1-5 Explain the similarities and differences between the fields of air conditioning,
commercial refrigeration, and industrial refrigeration.
1-6 Name and describe three applications of refrigeration systems in both the air-
conditioning and industrial-refrigeration fields.
Fundamentals of
Thermodynamics

Study Objectives
Chapter 2 presents thermodynamic properties of fluids that are important for
understanding a refrigeration system’s operation. The terms saturated, saturated
mixture, and superheated, as they relate to the refrigerant fluid, are introduced,
along with the idea of the thermodynamic state. The application of the three
laws of thermodynamics to energy calculations for refrigeration processes are
shown. The refrigerant appears in these energy calculations as a liquid, a vapor,
or a mixture of both. After studying Chapter 2, you should be able to

 determine and use thermodynamic fluid properties;


 describe application of the first and second laws of thermodynamics to
refrigeration; and
 use the thermodynamic laws to perform energy calculations on simple
refrigeration processes.

Instructions
Read the material in Chapter 2. Verify the examples presented in the chapter
with your own calculations. At the end of the chapter, complete the skill devel-
opment exercises without referring to the text. Review those sections of the
chapter as needed to complete the exercises.

Introduction

All refrigeration systems involve the movement of heat from a cold region to
a warm region. Thermodynamics describes the manner in which this move-
ment may occur and impacts the design of refrigeration systems. The vapor-
compression system (introduced in Chapter 1) utilizes a refrigerant fluid cir-
culating through the system’s components. As energy moves in and out of the
vapor-compression system, the refrigerant’s condition changes as well. Ther-
modynamics translates these changing refrigerant conditions into quantified
energy exchanges for the components. Consequently, knowledge of thermo-
dynamics is needed to understand the operation of a refrigeration system.
18 Chapter 2 Fundamentals of Thermodynamics

Consider a refrigerant as a fluid that has a fixed chemical structure. Within


a vapor-compression system, the refrigerant must maintain the ability to move
through the system; it must not exist in the solid state. Therefore, the refriger-
ant may be a liquid, a vapor, or a mixture of liquid and vapor. All of these occur
in a refrigeration system. To quantify the energy transfers that occur in the
refrigeration system, the energy content of the refrigerant must be identified.
By following the changes in the energy content of the refrigerant, the energy
that is transferred to and from the system’s components can be determined.
For so-called simple fluids, such as the common refrigerants, thermody-
namics tells us that two properties are needed to specify the state of the fluid.
First, some definitions are needed. A property is any identifiable characteristic
of the fluid. Following are some examples of already familiar properties:

Pressure (p): force acting over a unit area; represents the tendency to burst a
vessel or pipe; measured in kilopascals (kPa).
Temperature (T): represents the average motion of the molecules comprising
the substance; measured in °C (Celsius) or K (Kelvin, absolute scale).
Volume (V): the geometrical size of the space occupied by the refrigerant;
measured in cubic metres (m3).
Mass (m): the quantity of the substance present inside the vessel or any other
volume of concern; measured in kilograms (kg).

The thermodynamic state of a simple fluid is fixed when two properties of


the fluid are specified. This is important because once the thermodynamic
state is identified, all other thermodynamic properties at that state may be
obtained.
Pressure can be measured using a normal pressure gage, typically a Bour-
don tube pressure gage. This gage, often found in a refrigeration plant, con-
sists of a round dial and pointer needle that moves clockwise indicating the
pressure. The Bourdon gage only gives what is called the gage pressure,
which is the pressure greater than that of the surrounding air. Thus,

p abs = p gage + p atm (2-1)

where pgage (kPa [gage]) is the pressure reading from the gage, patm (kPa) is
the atmospheric pressure, and pabs (kPa) is the absolute pressure. The letter
“a” after the psi denotes absolute pressure. It is important to note if the pres-
sure is reported as a gage pressure or absolute pressure, because this influ-
ences the numerical value of the pressure. The atmospheric pressure may be
obtained from a barometer that directly measures the atmospheric pressure.
For example, if the pressure reading from a Bourdon tube pressure gage is
379 kPa (gage), and a barometer gives the atmospheric pressure as
101.325 kPa, then the absolute pressure is obtained by applying Equation 2-1:
379 kPa (gage) + 101.325 kPa = 480.325 kPa.
Fundamentals of Refrigeration SI, Second Edition 19

Thermodynamic properties may also be formed by combining with other


properties, as with the property of density, which is the ratio of mass divided by
volume. For example, if a tank contains a certain liquid volume and it is placed
on a weighing scale, yielding the mass of the liquid, then the density of the liq-
uid is the known mass divided by the known volume.

Example 2-1

Problem A cylindrical tank is partially filled with water, as shown in Figure 2-1. The
diameter of the tank is 0.5 m, and the height of the water inside the tank is
1.2 m. The empty weight of the tank is known to be 5.1 kg, and the combined
weight of the tank and water is 240.7 kg. What is the density of the water in the
tank?

Solution

Volume  V  = --- D 2 h
4

=  ----------  0.5 m  2  1.2 m 


3.14
4
= 0.236 m 3

Weight of water  m  =  Tank + Water weight  –  Tank weight 


m = 240.7 – 5.1
= 235.6 kg

Figure 2-1 Cylindrical tank containing water.


20 Chapter 2 Fundamentals of Thermodynamics

Density    = m
----
V

= 235.6
-------------
0.236
= 1000 kg/m 3

Related to density is specific volume, which is mathematically the recipro-


cal of the density and, therefore, has units of volume/mass. The specific vol-
ume is the amount of volume that a unit mass occupies. Generally, the property
density is used in conjunction with liquids, and specific volume is reserved for
vapors.

Thermodynamic Properties of Fluids

Several thermodynamic properties of fluids are presented in this section that


will be useful for calculations important to refrigeration systems.

Specific Heat

The specific heat is the energy needed to increase the temperature of a unit
mass of a substance by one degree of temperature. In particular, the specific
heat at constant pressure, cp (J/kg·°C), applies to those processes where the
pressure remains relatively constant (such as those involving the flow of a
vapor or liquid). Typical values of cp at a temperature of 0°C are shown in
Table 2-1. The specific heat at constant volume, cv , is associated with the situa-
tion where the volume remains constant but the pressure may vary (such as in a
storage tank with all valves closed). When the change in energy content of a
fluid is needed, where the fluid remains either a vapor or liquid for the process,
the appropriate specific heat is used to make the calculation.

Table 2-1 Constant Pressure and Constant Volume Specific Heats for Several Fluids at 0°C
cp, cv,
J/kg·°C J/kg·°C
Liquid Vapor Liquid Vapor
R-22 1171.47 743.99 904.77 574.85
R-134a 1334.75 883.00 1132.95 749.86
R-717 4615.95 2659.45 3297.52 1899.55
Water 4228.25 1867.73 3179.45 1404.67
Fundamentals of Refrigeration SI, Second Edition 21

Example 2-2

Problem Consider the flow of air through an evaporator coil (shown in Figure 2-2)
where the entering temperature is 4°C and the exit air temperature is –4°C. If the
airflow rate is 0.65 kg/s, what is the rate of energy removed from the airstream?
(Specific heat of air at constant pressure is 0.9877 kJ/kg·°C.)

Solution Use specific heat of air at constant pressure, cp, because the air is passing
through the evaporator without a change of phase and with a negligible pres-
sure drop. Thus,

·
Q = Heat transfer rate
= m· c p  T 2 – T 1 
=  0.65 kg/s   0.9877 kJ/kg·°C   4°C –  – 4°C  
= 5.136 kW

Saturation Condition
A liquid and a vapor in contact with each other, where all the properties remain
constant with time, is said to be in equilibrium. Under this special condition,
the mixture of liquid and vapor is saturated. This situation may be obtained by
filling a vessel with a liquid refrigerant and, with a pipe and valve, draining a
portion of the liquid from the bottom of the vessel. The vapor above the liquid
is then refrigerant vapor.
Figure 2-3a shows the resulting saturated-mixture condition with refrigerant
R-134a after equilibrium is achieved at the indicated room temperature. Here, if
the saturation temperature is 21°C, then the saturation pressure is 590 kPa. If the

Figure 2-2 Evaporator coil in Example 2-2.


22 Chapter 2 Fundamentals of Thermodynamics

vessel is slowly cooled to –26.16°C, then the pressure becomes 101.325 kPa, or
the standard atmospheric pressure, as shown in Figure 2-3b. Were the tempera-
ture and pressure recorded simultaneously as the R-134a is cooled, a unique
relationship between the pressure and temperature would be observed. In other
words, given a saturation temperature, there is only one saturation pressure. The
opposite is also true: for every saturation pressure, there is a unique saturation
temperature.
The tables in Appendix A show the saturation temperatures and pressures
for five common refrigerants. The important feature to note is the one-to-one
relationship between saturation pressure and saturation temperature for all
these refrigerants. Incidentally, the pressures mentioned above are absolute, as
are all the saturation pressures listed in Appendix A.

Superheated Vapor

Consider again the vessel in Figure 2-3, which contains a saturated mixture of
liquid and vapor. Install a pipe at the top of the vessel and allow some of the
saturated vapor to exit. If heat is added to the saturated vapor as it exits, the
warmed vapor is said to be superheated. The temperature elevation above the
saturation temperature inside the tank is referred to as the degree of superheat.
Figure 2-4a shows a saturated vapor leaving the vessel and, with the addition of
heat, changing to a superheated vapor.

Figure 2-3 Saturated liquid/vapor mixture of R-134a after equilibrium is achieved at the indi-
cated room temperature: (a) saturation pressure is 590 kPa at saturation tem-
perature of 21°C and (b) saturation pressure is standard atmospheric pressure
when the vessel is slowly cooled to –26.16°C.
Fundamentals of Refrigeration SI, Second Edition 23

Subcooled Liquid
Consider a vessel with a pipe at the bottom allowing liquid to drain. Inside the
pipe is a saturated liquid at the same pressure and temperature as that of the liq-
uid in the tank. If heat is removed from the liquid in the pipe, then the liquid is
subcooled; for the pressure of the liquid, the temperature is below that of the
corresponding saturation temperature. Figure 2-4b shows the heat removal pro-
cess for producing a subcooled liquid.

Figure 2-4 Superheated vapor and subcooled liquid: (a) saturated vapor leaving the vessel,
combining with heat, and changing to a superheated vapor and (b) the heat-
removal process for producing a subcooled liquid.
24 Chapter 2 Fundamentals of Thermodynamics

Enthalpy
In simple terms, the enthalpy, h (Btu/lb), is a measure of the energy content of a
fluid. The tables in Appendix A list enthalpy values for a saturated liquid, hf,
and a saturated vapor, hg , for five refrigerants. For each of the refrigerants, the
variation of saturated pressure and liquid enthalpy is plotted in that particular
refrigerant’s corresponding figure in Appendix B. Note in the Appendix B fig-
ures that the vertical axis is absolute pressure and the horizontal axis is enthalpy.
The pressure-enthalpy plot for a refrigerant is normally referred to as the
pressure-enthalpy (p-h) diagram. On this plot, the saturation pressure versus
liquid enthalpy is called the saturated-liquid line, and the saturated pressure
versus the vapor enthalpy is the saturated-vapor line, as shown in Figure 2-5a.

(a)

(b)

Figure 2-5 (a) Saturated-liquid and saturated-vapor lines on the p-h diagram and (b) deter-
minations of mixture quality.
Fundamentals of Refrigeration SI, Second Edition 25

Where these two lines meet at the top is called the critical point. The region
under the saturated-liquid and saturated-vapor lines is the mixture region,
where both liquid and vapor coexist. The area to the left of the saturated-liquid
line is the subcooled region and the area to the right of the saturated vapor line
is the superheat region. Observe that in establishing the saturation thermody-
namic state of a vapor or liquid, the two properties needed to define the state
are the saturation condition and either pressure or temperature. The term qual-
ity defines the relative portion of the mixture’s mass that is a saturated vapor.
The mixture quality, x, is given by
Vapor mass -
x = ----------------------------------- (2-2)
Mixture mass
and has a value between zero and one. A quality of zero implies only liquid,
and a quality of one implies only vapor. The relative position of the state point
horizontally from the saturated-liquid and saturated-vapor lines indicates the
relative portion of the mixture that is vapor, or its quality (see Figure 2-5b).
The quality is
h–h length  a  -
x = ---------------f = ----------------------------------- (2-3)
hg – hf length  a + b 
or solving for h gives
h = hf + x  hg – hf 
(2-4)
=  1 – x h f + xh g

Example 2-3

Problem Consider a vessel containing a saturated-mixture refrigerant R-22 at 30°C with


an enthalpy of 361 kJ/kg. Determine first the quality of the mixture and then
the specific volume of the mixture.

Solution The quality is given by


h – hf
x = ---------------
hg – hf

where, from Table A-1, the saturated properties for R-22 are found:
T sat = 30°C
h g = 414.26 kJ/kg
h f = 236.62 kJ/kg
361 – 236.63 - = 0.70
x = --------------------------------------
414.26 – 236.62
26 Chapter 2 Fundamentals of Thermodynamics

To obtain the specific volume of the saturated mixture,

v = vf + x  vg – vf 

where, from Table A-1, we find at Tsat = 30°C,

 f = 1170.7 kg/m 3
1 - = 0.0008542 m 3  kg
v f = ---------------
1170.7
v g = 0.01972 m 3  kg

Finally,

v = 0.0008542 +  0.70   0.01972 – 0.0008542 


= 0.014 m 3  kg

The horizontal difference between the two saturation lines represents the
energy needed to convert from the liquid state to the vapor state. For the p-h
diagram, because h is the energy content of the refrigerant, a horizontal dis-
tance represents a change of energy in the refrigerant. So, the energy needed to
boil or vaporize the saturated liquid to a saturated vapor at the same pressure is
the difference, hg – hf, and the path taken is toward the right. Likewise, moving
from a vapor to a liquid (toward the left) on the horizontal line gives the energy
reduction in the refrigerant for condensation.
A subcooled liquid exists to the left of the saturated liquid line on the p-h dia-
gram. Subcooled implies that the temperature of the liquid is less than the satura-
tion temperature corresponding to the given saturation pressure. Compressed
liquid is another term used to identify the same condition; here, the pressure is
greater than the saturation pressure corresponding to the temperature of the liq-
uid. This condition sometimes occurs in a refrigeration system and requires the
evaluation of thermodynamic properties.
A rule is now stated for the evaluation of thermodynamic properties of a
subcooled liquid: the enthalpy and specific volume of a subcooled liquid may
be obtained using the saturated state at the actual temperature of the liquid.
For example, if subcooled ammonia has a temperature of –10°C and a pres-
sure of 1000 kPa, and treating the ammonia temperature as saturated at the
given temperature, then the enthalpy, h = 154.01 kJ/kg, and the specific vol-
ume, v = 1/651.47 = 0.001535 m3/kg. Note that the ammonia saturation pressure
at –10°C is 291 kPa. This rule comes from the fact that only a small error is made
by assuming that the thermodynamic properties of a subcooled liquid are equal
to that of a saturated liquid at the same temperature. This is often a convenient
practice, especially when subcooled-liquid data are not available.
Fundamentals of Refrigeration SI, Second Edition 27

The remaining region in the p-h diagram is to the right of the saturated vapor
line and is called the superheat region. The energy content of superheated vapor
exceeds that of the saturated vapor at the same pressure, so higher enthalpies are
expected here. In the superheat region, two thermodynamic properties are again
required to determine the state. For our convenience, the properties v, T, and s
(entropy, discussed in the following section) are also plotted in the superheat
region. The consequence of this is that any two properties (p, h, T, v, and s) suf-
fice in establishing the thermodynamic state and, once the state is found, the
remaining properties may be read from the p-h diagram.
For example, we can use R-134a to demonstrate this, where we know the
temperature is 80°C and the pressure is 800 kPa. From Figure B-4, we first fol-
low the pressure line of 800 kPa horizontally toward the right into the superheat
region until crossing the 80°C constant temperature line. This is the state point.
The remaining properties approximately are: h = 465.17 kJ/kg, density () =
30.62 kg/m3, and entropy (s) = 1.8656 kJ/kg·°C.

Entropy
Lines of constant entropy on the p-h diagram (called isentropic lines) are the
upward-sloping lines in the superheat region. We shall confine our concern of
entropy to only that of the superheated region on the p-h diagram. For now, we
shall say that the lines of constant entropy are lines of ideal compression,
observing that the pressure, temperature, and enthalpy all increase when trac-
ing an entropy line toward the right. In a refrigeration system, what we have
traced is the ideal compression process that requires the smallest possible effort
or work to accomplish the compression. This ideal compression has no heat
transfer, and no friction occurs during the process. The determination of the
work needed to accomplish the compression may be obtained directly from the
superheat region of the p-h diagram, as shown in Figure 2-6. The change in
enthalpy that occurs from the beginning to the end of compression is the work
of compression.
In the superheat region of the p-h diagram, lines of constant vapor specific
volume are also shown. Note that these lines are more nearly horizontal, rising
only slightly when moving toward the right.

Thermodynamic Laws and Applications


Thermodynamics is the fundamental science concerned with heat and work. In
its simplest form it is composed of three laws. The first and second laws of
thermodynamics are the most important for making calculations. The third law
of thermodynamics establishes the value of entropy for a substance at absolute
zero—the temperature at which all molecular motion stops. Normally, refriger-
ants are at temperatures much greater than absolute zero, so the value of
entropy at this temperature has no practical value. Furthermore, refrigerant
28 Chapter 2 Fundamentals of Thermodynamics

Figure 2-6 Ideal compression lines and compression work.

property tables identify values of entropy using an arbitrarily assigned refer-


ence value rather than the numerical value provided by the third law.
There is another law of thermodynamics: the zeroeth law, which is con-
cerned with the establishment of equal temperatures between two bodies in
thermal equilibrium with each other. The zeroeth law is necessary for the accu-
rate measurement of temperature; a thermometer in thermal equilibrium with a
body must be at the same temperature as the body.
Numerous variations of the first law have been carefully stated by scientists
and engineers. Rather than develop a theoretical formulation of the first law,
we will develop the idea of an energy balance that is a consequence of the first
law. Consider any component that may exist as part of a refrigeration system.
A number of refrigerant flow streams may enter and leave this component
along with heat transfer and power exchange.
Figure 2-7 shows a diagram of a generic component operating at steady-
state conditions, which implies that all the operating variables for the compo-
nent remain unchanged with time. Three numbered flow streams enter, two
numbered flow streams leave, a heat transfer enters, and power is produced.
The power is achieved in the component by inserting a propeller driven by the
flow attached to a rotating shaft. Remember that power is the time rate at which
work is done.
Along with the energy balance on the component, a mass balance must also
be applied. In the absence of nuclear reactions, mass cannot be created or
destroyed and so remains constant. For the component operating at steady-state
conditions, the total mass flow rate that enters must also leave. So, we have
Fundamentals of Refrigeration SI, Second Edition 29

Figure 2-7 Generic component at steady-state conditions.

 Sum of mass  Sum of mass


0 =  flow rates  –  flow rates 
   
 leaving   entering 

where the leaving mass flow rates are taken as positive and the entering flows
are negative. For the generic component in Figure 2-7, we have

0 =  m· 4 + m· 5  –  m· 1 + m· 2 + m· 3  (2-5)

where m designates mass flow and the subscript identifies the particular
stream. The dots signify a time rate of change, in this case the mass time rate of
change or flow rate. Note that the component is operating at steady state and so
the temperatures, pressures, and mass density inside remain constant with time.
For this reason, the left side of the mass balance equation is zero.
The energy balance can now be stated as follows:

 Sum of all   Sum of all 


 Net heat   Net shaft     
 transfer into –  power out of =  energy flow  –  energy flow 
     streams out of  streams into
 component   component     
 component   component 
30 Chapter 2 Fundamentals of Thermodynamics

And for the generic component in Figure 2-7, we have


· ·
Q in – W out =  m· 4 h 4 + m· 5 h 5  –  m· 1 h 1 + m· 2 h 2 + m· 3 h 3  (2-6)

where
· = time rate of transfer of heat into component, kJ/s or W
Q in
· = time rate of work output from component, kJ/s or W
W out
m· = flow stream mass flow rate, kg/s
h = enthalpy of mass in flow stream, kJ/kg

Observe that the algebraic sign convention is that heat transfer into the
component is positive and that power or work output is positive. In the opposite
direction, heat transfer out and work input are negative. Additionally, if any of
the variables in the energy balance do not exist, then the variable is zero. For
example, if no shaft protrudes from the component producing work, then

·
W out = 0

In summary, we may say that energy coming in must equal the energy
going out.
The second law of thermodynamics has important implications for a refrig-
eration system. From everyday observation, a hot body placed in contact with a
cold body always drops in temperature (for example, a cup of hot chocolate in
a room will cool when in contact with the air in the room). The second law
asserts that heat cannot spontaneously move from a cold body to a hot body.
This squares with our personal experience. However, the second law also states
that with an input of power to a cycle, it is possible to move heat from a cold
body to a warm body, which is precisely what a refrigeration cycle does.
Another important outcome of the second law is the entropy property, which
we have already discussed in this chapter.

Efficiency of Refrigeration and Heat-Pump Cycles

What is the maximum efficiency that a refrigeration cycle can have? Thermo-
dynamics provides the answer to this question.

1. Suppose we have a refrigeration cycle (the actual components that make up


this cycle are unknown) that is in contact with a cold-temperature body and
a hot-temperature body, as shown in Figure 2-8. In Figure 2-8, we see that
· ·
Q C is the rate of heat transfer from the cold body, Q H is the heat transfer
to the hot body, and W· cycle is the power input to this cycle. To quantify the
performance of refrigeration cycles, we use the concept of efficiency,
which, in general terms, is the ratio of the desired effect divided by the
Fundamentals of Refrigeration SI, Second Edition 31

Figure 2-8 Diagram of a refrigeration cycle.

input the effect costs. For a refrigeration cycle, the efficiency is called the
coefficient of performance (COP) and is the heat transfer rate from the cold
body divided by the supplied energy.
Thus,
·
QC
COP = ---------------
· - (2-7)
W Cycle

and applying an energy balance gives

· · ·
W Cycle = Q H – Q C (2-8)

so
·
QC
COP = -------------------- (2-9)
· ·-
QH – QC

When all processes in the cycle are reversible (no friction, no heat trans-
fer across a finite temperature difference, etc.), W· Cycle is a minimum des-
·
ignated as  W Cycle  Rev , and the resulting COP is a maximum.
So,
32 Chapter 2 Fundamentals of Thermodynamics

·
QC
 COP  max = ----------------------------
· (2-10)
 W Cycle  Rev

Also, observe that if the same quantity of heat is transferred from the
cold body to the hot body, W· Cycle is zero and the COP is infinite. This, of
course, is impossible, because the system would violate the second law of
thermodynamics, which states that a refrigeration cycle must have a power
input to move a quantity of heat from a cold temperature body to a hot tem-
perature body.
2. While we have developed a simple expression for the COP of a refrigera-
tion cycle and have means to establish the maximum COP that can be
achieved, the heat transfers used to make the calculation are not easily
obtained. To overcome this difficulty, thermodynamics may be applied to
define a “special” temperature scale that is independent of the properties of
any substance and that connects temperature to the heat transfer rate. The
details are beyond the scope of this discussion, but the results show that the
maximum COP for any refrigeration cycle is given by

TC
 COP  max = ------------------
- (2-11)
TH – TC

where the temperatures are absolute (K). Recall that (COP)max applies to
the special refrigeration cycle in which all processes are thermodynami-
cally reversible (no friction, no heat transfer across a finite temperature dif-
ference, etc.). This reversible cycle is known as the Carnot Cycle. Actual
refrigeration cycles are not composed of reversible processes and, there-
fore, only Equations 2-7 and 2-9 can be used to compute their COP. Equa-
tion 2-11 is important because it establishes the maximum possible COP
for any refrigeration cycle operating between two temperatures.

To summarize, all refrigeration cycles entirely composed of reversible pro-


cesses operating between the same two temperatures have the same maximum
coefficient of performance, (COP)max. All other refrigeration cycles have a
COP less than (COP)max.

Example 2-4

Problem A new company offers a vapor-compression system sized for residential air
conditioning where the required power input is 2.2 kW to remove 10.4 kW of
energy from the building. The evaporator temperature (cold-body tempera-
ture) is 4°C, and the condenser temperature (hot-body temperature) is 43°C.
Fundamentals of Refrigeration SI, Second Edition 33

There are no other heat transfers with the surroundings. Determine whether
this system is feasible.

Solution Determine the COP for this refrigeration cycle as reported.

·
W Cycle =  37,982 kJ/h   1 --------- ---------------- = 2.2 kW
kW 1 h
kJ/s 3600 s
·
QC
· - = 10.4
 COP  Offered = --------------- ---------- = 4.73
W Cycle 2.2

The maximum possible COP is calculated on the basis of the evaporating


and condensing absolute temperatures.

T H = 43°C + 273.15 K = 316.15 K


T C = 4°C + 273.15 K = 277.15 K
TC 277.15
 COP  max = ------------------
- = ------------------------------------------- = 7.1
TH – TC  316.48 – 277.15 

The system offered is reasonable because the COP of the unit offered is
less than that of the maximum possible COP.

Heat-pump cycles also operate between two bodies at different tempera-


tures, as shown in Figure 2-9. The heat-pump cycle COP is different from that
of a refrigeration cycle because its purpose is to transfer heat to a hot body. So,
the COP for a heat-pump cycle is
·
QH
 = ---------------
· - (2-12)
W Cycle

which, after applying an energy balance, gives


·
QH
 = --------------------
- (2-13)
Q· H – Q· C

Applying thermodynamics as before with the refrigeration cycle, again tak-


ing all the heat-pump cycle processes as reversible, we have an expression for
the maximum COP for a heat-pump cycle,
·
QH
   max = ----------------------------
· (2-14)
 W Cycle  Rev
34 Chapter 2 Fundamentals of Thermodynamics

Figure 2-9 Heat-pump cycles also operating between two bodies at different temperatures.

and in terms of absolute temperature,

TH
   max = ------------------
- (2-15)
TH – TC

Energy Calculations
Each of the four components in the basic vapor-compression cycle (evaporator,
compressor, condenser, and expansion device) causes a change in the refriger-
ant as it passes through the cycle. Examples in this section show how calcula-
tions are made concerning the flow of energy in conjunction with these four
components. While these calculations show typical operating conditions and
sizes, they do not represent one particular cycle. Beginning with the compres-
sor, the refrigerant changes as detailed.

Compressor
The refrigerant enters an ideal compressor as a saturated vapor and leaves as a
superheated vapor at an elevated pressure. An input of power is needed to oper-
ate the compressor, and no heat transfer is assumed to occur with the surround-
ing environment.
A mass balance on the compressor in the heat-pump cycle shown in
Figure 2-10 gives

0 = m· 2 – m· 1
(2-16)
m· 1 = m· 2 = m·
Fundamentals of Refrigeration SI, Second Edition 35

Figure 2-10 Ideal compressor.

An energy balance on the compressor is given by


·
– W out = m· 2 h 2 – m· 1 h 1 (2-17)

Substituting m· 1 = m· 2 = m· and multiplying by –1 gives


·
+W out = m· h 1 – m· h 2 = m·  h 1 – h 2  (2-18)
· ·
Realizing that W out = – W in for an ideal compressor, we can write
·
– W in = m·  h 1 – h 2 
· (2-19)
·
W in = m  h 2 – h 1 
·
where W in is the needed power input to the compressor. The enthalpy of the
saturated vapor entering the compressor h 1 is known, so now h 2 must be
obtained. Remembering that the compression is ideal, the compression process
follows lines of constant entropy in the superheat region of the p-h diagram.
Beginning at the suction state point on the saturated-vapor line, one can trace a
line parallel to the constant entropy lines into the superheat region. The path
terminates when reaching the compressor’s discharge pressure. The path traced
is the ideal compression line, while the final point is the state of the discharge
vapor where the enthalpy and temperature may be read from the chart. Note
that the entropy remains constant. For this reason, the ideal compression is
called isentropic compression.

Example 2-5

Problem R-134a enters an ideal compressor as a saturated vapor at 0°C and is dis-
charged at 1000 kPa. If the flow of the refrigerant is 0.9 kg/min, what is the
required power to the compressor?
36 Chapter 2 Fundamentals of Thermodynamics

Solution For an ideal compressor, as shown in Figure 2-10,

· ·
W in = m  h 2 – h 1 

From Table A-4, saturated vapor properties of R-134a at Tsat = 0°C:

h 1 = 398.68 kJ/kg s 1 = s 2 = 1.7274 kJ/kg·K

The enthalpy at state 2 may be determined using two methods:

• Trace the constant entropy line at 1.7274 kJ/kg·K on a p-h diagram (Fig-
ure B-4). When the traced line crosses the p2 = 1000 kPa line, read
h 2  424.2 kJ/kg .
• Use the superheat table in Table A-5 to interpolate between the given values:

p = 1000 kPa
s (kJ/kg·°C) h (kJ/kg)
T = Tsat = 40°C 1.7139 419.99
1.7274 h2
T = 50°C 1.7482 430.91

1.7274 – 1.7139
h 2 = 419.99 +  ---------------------------------------  430.91 – 419.99 
1.7482 – 1.7139
= 424.3 kJ/kg

The compressor power is

·
W in =  0.9 kg/min   1 min/60 s    424.3 – 398.68 kJ/kg 
x = 0.384 kJ/s = 0.384 kW

Condenser

Refrigerant enters a condenser as a superheated vapor and leaves as a saturated


liquid, as shown in Figure 2-11. During the condensing process, heat is
removed from the refrigerant inside the condenser and transferred to the sur-
roundings. An energy balance gives
Fundamentals of Refrigeration SI, Second Edition 37

Figure 2-11 Condenser with entering and exiting flow streams.

·
Q in = m·  h 2 – h 1 
· · (2-20)
Q out = – Q in = m·  h 1 – h 2 

where the mass balance result, m· 1 = m· 2 = m· , is already incorporated.

Example 2-6
Problem Superheated R-22 enters a condenser at 40°C and 1000 kPa and leaves as a sat-
urated liquid. If the refrigerant flow rate is 4 kg/min, what is the heat transfer
rate at the condenser?

Solution An energy balance on the condenser using Figure 2-11 gives


· ·
Q out = – Q in = – m·  h 2 – h 1 
= m·  h 1 – h 2 

From Figure B-1, p-h diagram for R-22: h1  426 kJ/kg (superheat region).
From Table A-1, saturated liquid properties for R-22 at Psat = 1000 kPa:
h2 = 228.3 kJ/kg (interpolated).

·
Q out =  4 kg/min   426 – 228.3 kJ/kg   --------------
1 min
 60 s 
= 13.18 kW

Expansion Device
The expansion device supplies a cold mixture of liquid and vapor to the evap-
orator, as shown in Figure 2-12. The pressure drop across the expansion
device, along with the pressure increase across the compressor, separates the
38 Chapter 2 Fundamentals of Thermodynamics

high-pressure side of the cycle from the low-pressure side. The heat transfer
from an expansion device is negligible due to its very small surface. Conse-
quently, the expansion process occurs with no heat transfer and is, therefore,
said to occur adiabatically.
An energy balance gives

0 = m·  h 1 – h 2 
(2-21)
h1 = h2

Example 2-7

Problem Suppose that saturated R-123 liquid enters an expansion device at 32°C. If the
evaporating temperature is 4°C, what is the low-side pressure and vapor quality
leaving the expansion device?

Solution Using Table A-3 for the saturation properties of R-123, and using Figure 2-12
yields for the low-pressure side,

T 2  sat.  = 4°C p 2  sat.  = 39.1 kPa

when passing through an expansion device.


Thus,

h 1 = 230.33 kJ/kg = h 2

and

h 2 = h f2 + x 2  h g2 – h f2 

Figure 2-12 Expansion device with entering and exiting flow streams.
Fundamentals of Refrigeration SI, Second Edition 39

The R-123 saturation property table gives

h f2 = 203.54 kJ/kg h g2 = 382.16 kJ/kg

so,

h 2 – h f2
x 2 = --------------------
-
h g2 – h f2
230.33 – 203.54
= ---------------------------------------
382.16 – 203.54
= 0.150 or 15%

indicating that 15% of the refrigerant mass is vapor, while 85% is still liquid.

Evaporator

The refrigerant enters an evaporator as a cold saturated mixture of liquid and


vapor, as shown in Figure 2-13. Evaporation occurs inside the evaporator,
absorbing heat from the cold space. An energy balance gives
·
Q in = m·  h 2 – h 1  (2-22)

Example 2-8

Problem The refrigerant leaving the expansion valve in Example 2-7 enters an evapora-
tor operating at 4°C evaporating temperature and exits as a saturated vapor.
What is the rate of heat transfer to the evaporator if the refrigerant flow rate is
2 kg/min?

Figure 2-13 Evaporator with entering and exiting flow streams.


40 Chapter 2 Fundamentals of Thermodynamics

Solution Using Table A-3 (saturation property table for R-123) and Figure 2-13 gives

h 1 = 230.33 kJ/kg h 2 = 382.16 kJ/kg

Thus,

·
Q in = m·  h 2 – h 1 
=  2 kg/min   382.16 kJ/kg – 230.33 kJ/kg 

= 304 kJ/min  -------------


1 min
 60 s 
= 5.1 kW
The heat transfer rate to the evaporator is the refrigerating capacity of the
refrigeration system. Typically, the capacity for a refrigeration system is given
in tons, a term indicating an early tradition in refrigeration. Before mechanical
equipment was applied to refrigeration, ice was harvested from lakes, ponds,
etc., and was stored over the remainder of the year. The ice was periodically
delivered in blocks for use with insulated boxes for the storage of food (hence
the term icebox.) When one pound of ice melts, it absorbs an amount of heat
equal to its latent heat of fusion. At 0°C, the latent heat of fusion of ice is
334.94 kJ/kg. A refrigerating capacity of one ton is the rate of heat absorption
by one ton (907.2 kg) of ice over a 24-hour period. The heat transfer rate is
equal to

· 907.2 kg   334.94 kJ/kg  = 12,660 kJ/h  --------------- 1 h -


Q = -------------------------------------------------------------- = 3.52 kW
24 h  3600 s
(2-23)

The heat that must be added to convert 907.2 kg of ice into 907.2 kg of
water is the same as that which must be removed to convert 907.2 kg of water
into 907.2 kg of ice. In other words, a refrigeration system capable of produc-
ing 907.2 kg of ice per day (water temperature entering and ice temperature
being 0°C) has a refrigerating capacity of one ton.

Next Step
Chapter 3 presents the single- and two-stage vapor-compression refrigeration
cycles and the characteristics for describing their overall performance. Cascade
refrigeration systems are also described.

Summary
Chapter 2 reviewed the thermodynamic properties that identify the state of the
refrigerant and the changes that the refrigerant undergoes. Properties include
Fundamentals of Refrigeration SI, Second Edition 41

pressure, temperature, volume, density, specific heat, enthalpy, and entropy. A


substance is said to be in the saturated state when its liquid and vapor are in
contact with each other and all thermodynamic properties remain constant with
time. Superheated vapor exists when the vapor contains energy in excess of
that required for the saturated state. A subcooled liquid contains less energy
than in its saturated state. Mass and energy balances describe conditions that
must be met by the refrigerant when passing through any component. Mass and
energy balances applied to the ideal vapor-compression cycle reveal how
energy is transported to and from the cycle. The calculation of the coefficient
of performance (COP) for any refrigeration cycle or heat-pump cycle is based
on the quantity of heat transfer rates and the power input. The maximum possi-
ble COP is computed using cold- and hot-body absolute temperatures. You
should now be able to

 define the thermodynamic properties: temperature, pressure, volume, den-


sity, specific volume, enthalpy, and entropy;
 identify and describe the conditions of subcooled liquid, saturated mixture,
and superheated vapor and the method for determining numerical values for
the thermodynamic properties in the refrigerant tables and charts;
 describe the difference between gage pressure and absolute pressure;
 perform calculations using thermodynamic property values concerning the
four components (evaporator, compressor, condenser, and expansion device)
in an ideal vapor-compression refrigeration system; and
 compute the actual COP for a refrigeration cycle or heat-pump cycle and
the maximum possible coefficient using cold- and hot-body absolute tem-
peratures.
42 Chapter 2 Fundamentals of Thermodynamics

Skill Development Exercises for Chapter 2


Complete these questions by writing your answers on the worksheets at the back of this book.

2-1 A water-cooled condenser is used to remove heat from a vapor-compression


refrigeration system. The temperature of the water entering the condenser is
10°C and the exiting temperature is 13.89°C. If the mass flow rate of water
through the condenser is 1.59 kg/s, what is the rate of heat rejected by the con-
denser (kJ/min)?
2-2 A vessel, cylindrical in shape and installed vertically, has a height and diameter
of 5.0 m and 1.5 m, respectively. When partially filled with liquid ammonia, a
pressure gauge indicates 641.9 kPa (gage) for the internal pressure. If the
height of the liquid level is 3.0 m above the vessel’s bottom, what is the quality
of the ammonia in the vessel? The atmospheric pressure is 100 kPa.
2-3 Consider the generic component shown in Figure 2-7 where the following
information is known:

m· 1 = 2.0 kg, m· 2 = 4.0 kg/s, m· 3 = 1.0 kg/s, m· 4 = 2.0 kg/s,


h 1 = 80 kJ/kg, h 2 = 30 kJ/kg,
h 3 = 120 kJ/kg, h 4 = 20 kJ/kg, h 5 = 40 kJ/kg,
·
Q out = 100 kJ

Using mass and energy balances on the component, determine m· 5 (kg/s)


and the power output (kJ/s).
2-4 An industrial refrigeration system is employed to maintain a freezer room at
–23.33°C. For this system, the evaporating temperature is –28.89°C and the
condensing temperature is 35°C. What is the maximum COP for this refriger-
ation system?
2-5 A display freezer cabinet in a supermarket is claimed to operate at a cold tem-
perature of –17.78°C and reject heat to the surrounding air at 43.33°C. While
maintaining these temperatures, the claim further states that 12,660 kJ/h are
withdrawn from the cold space, requiring 0.447 kW power input to the com-
pressor. Evaluate this claim and, using thermodynamics, determine whether it is
true.
2-6 A heat-pump cycle is applied to a residential building whose interior is kept at
20°C when the outdoor temperature is –1.11°C. The rate of heat transfer
through the building’s structure (walls, roof, etc.) is 1530 kJ/h per degree Cel-
sius temperature difference between the indoor and outdoor temperatures. Find
the minimum power needed to drive this heat-pump cycle (kW).
Fundamentals of Refrigeration SI, Second Edition 43

2-7 The suction line to an ideal compressor in a low-temperature application sup-


plies R-23 saturated vapor at 62.22°C. If the refrigerant flow rate is 10 kg/min
and the discharge pressure is 1034.21 kPa, what is the power (kJ/min) input to
the compressor?
2-8 An expansion device is used to control the flow of R-22 to an ideal evaporator.
Saturated liquid refrigerant at 26.67°C with a flow of 20 kg/min enters the
expansion device. The temperature of the evaporator is maintained at –20.55°C,
and the refrigerant leaving the evaporator is saturated vapor. Determine (1) the
pressure in the evaporator (kPa); (2) the quality of refrigerant downstream
from the expansion device; and (3) the rate of heat transfer to the evaporator
(kJ/min).
Multistage and Cascade
Refrigeration Cycles

Study Objectives
Chapter 3 describes the operation of single- and two-stage vapor-compression
refrigeration systems. The pressure-enthalpy (p-h) diagram is presented to help
the reader visualize the processes occurring in the system and to explain the
reasons for using either two compression stages or a cascade cycle when lower
temperatures are needed. Several refrigeration system performance parameters
that are useful for characterizing how well a particular system is operating are
defined. These include coefficient of performance (COP) (discussed previously
in Chapter 2), energy efficiency ratio (EER), kW/ton, and the refrigeration
effect. The deviations that occur with real or actual refrigeration systems from
the ideal system are discussed using a p-h diagram.
The classification of vapor-compression refrigeration systems is presented
using two criteria. The first criterion is based on the refrigerant supply method
for the evaporator—flooded or recirculation direct expansion. Each imposes
requirements on the system’s equipment and configuration. A further consider-
ation of the refrigerant supply method is discussed involving the means by
which the pressure is developed that delivers refrigerant to the evaporator. The
second criterion discussed involves the fabrication of the refrigeration system.
Both unitary and field-erected systems are presented. After studying Chapter 3,
you should be able to

 explain ideal single- and two-stage refrigeration systems;


 show ideal single- and two-stage refrigeration systems on a p-h diagram;
 explain the operation of a cascade refrigeration system;
 determine the thermodynamic state of the refrigerant at the inlet and outlet
of each component in an ideal refrigeration system;
 determine the overall system performance characteristics of a refrigeration
system;
 explain deviations between actual and ideal refrigeration systems using the
p-h diagram;
 identify and explain the two criteria used to classify vapor-compression
refrigeration systems;
 explain two differences between direct expansion and flooded systems that
relate to their operation;
46 Chapter 3 Multistage and Cascade Refrigeration Cycles

 explain why a low-pressure vessel is used in a recirculation system;


 describe, using a diagram, how the mechanical pumping recirculation sys-
tem operates;
 describe, using a diagram, how the vapor pumping recirculation system
operates; and
 explain the difference between unitary and field-erected systems.

Instructions
Read the material in Chapter 3. Verify the examples presented in the chapter
with your own calculations. At the end of the chapter, complete the skill devel-
opment exercises without referring to the text. Review those sections of the
chapter as needed to complete the exercises.

Introduction

The single-stage refrigeration system has one compression step bridging the
evaporating low and high condensing pressures. The expansion device and
compressor separate the low- and high-pressure sides.
The two-stage system is an example of a multiple-pressure system where
two compression steps bridge the evaporating and condensing pressures. Two
stages of compression are needed to achieve a low evaporating pressure or may
be required to serve an intermediate evaporating pressure. In the two-stage sys-
tem, three pressure levels (low, intermediate, and high) exist to accommodate
the two compression steps.
In ideal refrigeration cycles, constant pressure conditions inside the evapo-
rator and condenser cause the temperature of the refrigerant in each of these
components to remain constant. The refrigerant in these two components is a
saturated mixture, so if pressure is uniform, then so must be the temperature.
For this reason, only one pressure and temperature characterize the conditions
inside the ideal evaporator and condenser.
Applications of refrigeration systems for human comfort use single-stage
systems, because evaporating temperatures are above the freezing temperature
of water. Below-freezing temperatures (as mandated in some industrial appli-
cations) require at least two compression stages. Refrigeration used in food
chilling, food freezing, and frozen-food storage, as well as in other industries,
falls under the category of industrial refrigeration.
This chapter also brings together several refrigeration system compo-
nents: evaporators, compressors, condensers, vessels, and piping and flow
metering devices. The objective is to assemble these components into a sys-
tem that accomplishes refrigeration. The knowledge that we have developed
for these components, including both the thermodynamics and the practical
Fundamentals of Refrigeration SI, Second Edition 47

considerations, are applied to three basic system configurations: direct


expansion, flooded, and recirculation.

Single-Stage Ideal Refrigeration Cycle

In the introduction to refrigeration in Chapter 1, Figure 1-2 provided the first


glimpse of the single-stage ideal vapor-compression cycle. Chapter 2 discussed
the processes that the refrigerant undergoes as it travels through the cycle.
Combining the information from these chapters yields a simplified or sche-
matic diagram that may be used to represent the configuration of the single-
stage system, as shown in Figure 3-1. Notice that the thermodynamic state
points of the refrigerant are identified by the numbers 1 through 4.
The numbers of the refrigerant states and the ideal cycle processes are
given below:
States
1: saturated vapor
2: superheated vapor
3: saturated liquid
4: saturated liquid/vapor mixture

Ideal Components and Processes


1–2: compressor, isentropic compression to the condensing pressure
2–3: condenser, constant pressure desuperheating and condensing to satu-
rated liquid

Figure 3-1 Ideal single-stage vapor-compression refrigeration cycle.


48 Chapter 3 Multistage and Cascade Refrigeration Cycles

Figure 3-2 Pressure-enthalpy (p-h) diagram for ideal single-stage vapor-compression cycle.

3–4: expansion valve, constant enthalpy expansion to the low evaporating


pressure
4–1: evaporator, constant pressure evaporating to saturated vapor
Also shown in Figure 3-1 are the energy exchanges that the cycle has with
·
the surroundings. The energy absorbed by the refrigerant in the evaporator Q in
is the refrigerating capacity, or load, of the cycle. The energy delivered to the
·
surroundings by the condenser is Q out , and the power needed by the compres-
·
sor to operate the cycle is W out .
A special diagram is used to display the thermodynamic state points of the
refrigerant. This is referred to as the p-h diagram because the y-axis is pressure
(p) and the x-axis is enthalpy (h). Pressure and enthalpy are chosen because
pressure is readily measured in the cycle, and the enthalpy represents the energy
content of the refrigerant. Both quantities are useful in refrigeration systems.
Figure 3-2 shows a p-h diagram for the ideal single-stage system shown in Fig-
ure 3-1, where the numbers correspond to the refrigerant state points. Note that
the saturated liquid and vapor lines are also shown on the p-h diagram.

Example 3-1

Problem An R-22 ideal single-stage vapor-compression system operates with an evapo-


rating temperature of 2°C and a condensing temperature of 34°C. If the mass
flow rate of the refrigerant is 1.0 kg/min, determine the following: enthalpies
for all state points in the cycle, refrigerating capacity rate, power required to
operate compressor, and heat rejection rate from the condenser.
Fundamentals of Refrigeration SI, Second Edition 49

Solution The enthalpies are obtained from Table A-1 and Figure B-1: h1 = 405.78 kJ/kg;
h2 = 428 kJ/kg; h3 = 241.77 kJ/kg; h4 = 241.77 kJ/kg.

·
Refrigeration rate = refrigeration capacity = Q in
Q· in = m·  h 1 – h 4 
1 min
=  1 kg/min   -------------  405.78 – 241.77 kJ/kg
60 s
= 2.73 kW
· 1 ton
or that Q in =  273 kW   ------------------------ = 0.78 ton
 3.517 kW

·
Compressor power = W c = m·  h 2 – h 1 
· 1 min
W c =  1 kg/min   428 – 405.78 kJ/kg   --------------
60 s
= 0.37 kW

·
Heat rejection rate = Q out
·
Q = m·  h – h 
out 2 3
1 min
=  1 kg/min   428 – 241.77 kJ/kg  --------------
60 s
= 3.1 kW

Extending the analysis of Example 3-1, vary the evaporating temperature


over a range of values, keeping the condensing pressure and the refrigerant
flow rate constant. If the evaporating temperature range is from –28°C to
·
16°C, the variations in three energy flows, Q· in , Q out , and W· c , with evaporat-
ing temperature are plotted in Figure 3-3. Observe that for a constant refriger-
ant flow rate (1 kg/min), increasing evaporating temperature results in
increasing refrigeration capacity and decreasing compressor power and heat
rejection from the condenser. These same trends are also observed with actual
vapor-compression systems. Furthermore, note that an energy balance
applied to the cycle gives

· · ·
Q in – Q out –  – W c  = 0 (3-1)

and at the conditions for Example 3-1, we have

2.73 – 3.1 –  – 0.37  = 0


50 Chapter 3 Multistage and Cascade Refrigeration Cycles

Figure 3-3 Variation of three energy flows for ideal single-stage cycle (R-22, 34°C condensing
temperature, and 1 kg/min refrigerant flow rate).

showing that the energy balance is satisfied. The cycle energy balance holds for
the entire range of evaporating temperatures, as shown in Figure 3-3.

Two-Stage Ideal Refrigeration Cycle


Refrigeration applications in which the evaporating temperature is sufficiently
low may require two stages of compression to reach the condensing tempera-
ture. The difference between the condensing temperature and the evaporating
temperature is called the temperature lift. If the lift is greater than 38°C, then
two compression stages may be required. With two-stage compression, the
refrigerant flows in series through the two compressors. For very large tem-
perature lifts, three or more stages may be needed. The principles that apply to
two-stage systems may be readily extended to three-stage systems. Relative to
single-stage systems operating with the same temperature lift, multistage sys-
tems generally have higher first costs but require less power to operate.
The separation of compression into two stages allows the addition of two
new processes: flash gas removal and intercooling. To accomplish these two
processes, a vessel containing saturated liquid and vapor is added to the cycle
between the two stages of compression. This vessel (called a flash tank) is
shown in Figure 3-4, where the system is restricted to only one evaporating
temperature.
Fundamentals of Refrigeration SI, Second Edition 51

Figure 3-4 Ideal two-stage vapor-compression system.

With the flash tank, the expansion process is interrupted at the intermediate
pressure, identified as state 6, and the resulting liquid and vapor mixture is intro-
duced to the flash tank. The liquid and vapor inside the vessel are saturated. In
Figure 3-4, a float valve is shown maintaining the liquid level of the refrigerant
inside the flash tank. When the liquid level drops below the preset level, the
downward movement of the float opens the valve, thereby passing refrigerant. If
the liquid level rises, the upward movement of the float closes the valve, and
refrigerant flow stops. The improvement is that now liquid refrigerant at a lower
temperature (the intermediate temperature) is available for further expansion to
the evaporating pressure.
This is shown on the p-h diagram in Figure 3-5, where the enthalpy differ-
ence (h1 – h6) is the refrigeration effect without flash gas removal and the
enthalpy difference (h1 – h8) is the larger refrigeration effect associated with
flash gas removal. The influence of flash gas removal is to move state point 6 to
the left on the p-h diagram to state point 7. Continuing the expansion to state
point 8 increases the refrigeration effect or the refrigerating capacity of the cycle.
This becomes more important as the evaporating pressure is lowered due to the
down and left direction of the saturated vapor line on the p-h diagram.
The flash tank also provides cooling of the vapor discharged from the com-
pressor immediately downstream from the evaporator, as shown in Figure 3-4.
The pipe introducing the superheated vapor is submerged, causing the vapor
bubbles to mix and cool to the liquid temperature inside the vessel. A small
52 Chapter 3 Multistage and Cascade Refrigeration Cycles

Figure 3-5 Increased refrigeration effect due to flash gas removal.

amount of liquid refrigerant must evaporate to accomplish the cooling of the


vapor from the booster compressor.
Figure 3-6 shows this intercooling process on the p-h diagram. State 2 rep-
resents the vapor discharge from the booster compressor, and state 2’ represents
the state that the refrigerant would reach if compressed to the condensing pres-
sure in a single stage. Without intercooling, discharge temperatures may reach
high temperatures. This is particularly true with ammonia, where the discharge
temperature may be sufficiently high (greater than about 105°C), causing the
lubricating oil in the compressor to chemically break down and cause failure.
The two-stage system, as shown in Figure 3-4, combines flash gas removal and
intercooling using a flash tank. The p-h diagram for this ideal cycle, Figure 3-7,
reveals the three pressures: high, intermediate, and low. Note that the p-h dia-
gram nicely shows the thermodynamic state points but that the refrigerant flow
rates are not represented. Particularly, refrigerant flows in or out at the flash tank
for states 2, 3, 6, and 7 are all different due to the processes occurring inside.
The compressor power reduction occurring with two-stage systems may
now be explained. The flash gas separated in the flash tank is not expanded any
further and so is only recompressed by the high-stage compressor. In other
words, the booster compressor does not compress the flash gas back to the
intermediate pressure, which represents a savings in power.
Fundamentals of Refrigeration SI, Second Edition 53

Figure 3-6 Intercooling of refrigerant vapor discharge from the booster compressor.

Figure 3-7 Two-stage system with flash gas removal and intercooling using a flash tank.
54 Chapter 3 Multistage and Cascade Refrigeration Cycles

Example 3-2

Problem An R-22 ideal two-stage vapor-compression system operates with a flash


tank in which both flash gas removal and intercooling occur. The evaporating
temperature is –28°C, the condensing temperature is 34°C, and the interme-
diate pressure is selected as 480 kPa. If the refrigerating capacity is 1 ton
(3.516 kW), determine the enthalpies for all refrigerant state points, the refriger-
ant mass flow rates through the booster and high-stage compressors, and the
heat transfer rate rejected by the condenser.

Solution The enthalpies (Btu/lb) are h1 = 393.58, h2 = 418, h3 = 405.05, h4 = 429,


h5 = 241.77, h6 = 241.77, h7 = 200, and h8 = 200.

Refrigerant flow through booster compressor = m· 7 = m· 8 = m· 1 = m· 2


·
· Q in 3.516 kW
m 7 = --------------------- = ------------------------------------------------
- = 0.0182 kg/s
 h1 – h8   393.58 – 200 kJ/kg

Refrigerant flow through a high-stage compressor requires analysis of the


flash tank.

Mass balance: 0 = m· 6 + m· 2 –  m· 3 + m· 7 

Because m· 2 = m· 7 then m· 3 = m· 6

Energy balance: 0 = m· 6 h 6 + m· 2 h 2 –  m· 3 h 3 + m· 7 h 7 

Substituting,

m· 3 h 6 + m· 2 h 2 = m· 3 h 3 + m· 7 h 7
m· 3  241.77  +  0.0182   418  = m· 3  405.05  +  0.0182   200 

m· 3 = 3.9676
---------------- = 0.0243 kg/s
163.28

·
Heat rejection rate = Q out = m· 3  h 4 – h 5 
·
Q out =  0.0243 kg/s   429 – 241.77 kJ/kg = 4.55 kW

The calculations for the example may be extended to include a range of


evaporating temperatures from –40°C to 4°C. Figure 3-8 shows the resulting
Fundamentals of Refrigeration SI, Second Edition 55

Figure 3-8 Variation of energy exchanges and power inputs (R-22, 35°C condensing tempera-
ture, and 211 kJ/min refrigerating capacity).

energy exchanges and the booster and high-stage compressor power inputs for a
3 kW refrigerating capacity. Holding the refrigerating capacity constant results
in decreasing compressor power and a decreasing heat rejection rate from the
condenser with increasing evaporating temperature. In these calculations, the
intermediate pressure was allowed to vary according to

p intermediate = p low p high (3-2)

which is an estimate of the intermediate pressure that gives the minimum com-
pressor power for the cycle. Note that the units for pressure are kPa (absolute).
However, this intermediate pressure is based on the two-stage compression of
air and is somewhat different for a two-stage refrigeration system because the
processes that occur in the flash tank complicate the cycle.
Figure 3-9 shows the variation of compressor power (booster plus high-
stage) versus the temperature with fixed condensing temperature and refrigera-
tion capacity for the same conditions as shown in Example 3-2. Thus the
results shown in Figure 3-9 include the removal of flash gas. The optimum
intermediate temperature is observed to be approximately –5°C, which corre-
sponds to a saturated pressure of about 420 kPa. From Equation 3-2, the opti-
mum pressure is estimated as 480 kPa, where the corresponding saturation
temperature is –1°C. This may first be considered a large difference but, from
Figure 3-9, the difference in the required compressor power is less than 4%.
56 Chapter 3 Multistage and Cascade Refrigeration Cycles

Figure 3-9 Variation of total compressor power for an ideal two-stage system with interme-
diate temperature (R-22, evaporating temperature is –28°C, condensing tem-
perature is 35°C).

Figure 3-10 Liquid subcooler.


Fundamentals of Refrigeration SI, Second Edition 57

An alternative approach to increasing the refrigeration effect involves the


use of a subcooler. Figure 3-10 shows a schematic of a subcooler where high-
pressure liquid from the condenser is passed through a heat exchanger (such
as a coiled tube) submerged in the liquid inside the vessel. The saturated
high-pressure liquid is subcooled to the extent that evaporation of the inter-
mediate liquid occurs inside the vessel.
Figure 3-11 shows the subcooling process on a p-h diagram as well as the
increased refrigeration effect due to the subcooling. The advantage of the sub-
cooler over the flash tank is that the subcooled liquid may be transported a con-
siderable distance in a pipe with the associated pressure drop without flashing.
On the other hand, the high-pressure liquid cannot achieve as low a tempera-
ture as that of the liquid inside the subcooler vessel. Usually, the temperature of
the subcooled liquid is limited to about 5°C above the temperature of the inter-
mediate liquid in the vessel.

Cascade Refrigeration Cycle

The cascade refrigeration cycle consists of a series of independent single-


stage vapor-compression refrigeration cycles, or circuits, connected by heat
exchangers. The refrigerant in each circuit does not mix with the refrigerant
in the other circuits. This allows the use of a different refrigerant in each cir-
cuit. Very low evaporating temperatures are possible with the cascade cycle,
where two or more circuits may be employed. The advantage provided by the
cascade system is that two or more refrigerants may be used that better match
the high and low temperatures and pressures than would any one refrigerant
in a multistage cycle. Thus, high-pressure refrigerants such as carbon diox-
ide, R-23, or R-508 are selected for the low-temperature circuit, while
medium-temperature refrigerants such as ammonia and R-22 are selected for
the high-temperature circuit.
Figure 3-12 is a schematic diagram showing a cascade cycle where ammo-
nia and carbon dioxide are the high- and low-temperature refrigerants, respec-
tively. Observe that the two refrigeration circuits are coupled together by means
of a heat exchanger, or cascade condenser, that serves simultaneously as the
condenser for the low-temperature circuit and as the evaporator for the high-
temperature circuit. With operation of the cascade cycle, the heat absorbed by
the evaporator in addition to the compression heat of the low-temperature circuit
is absorbed by the evaporator of the high-temperature circuit.
A disadvantage of the cascade cycle relative to the multistage cycle is that a
temperature difference must be established in the cascade condenser that allows
heat to pass from the low-temperature circuit to the high-temperature circuit. To
accomplish this, the condensing temperature of the low-temperature circuit
must exceed the evaporating temperature of the high-temperature circuit. A pen-
alty in compression power accompanies this needed temperature difference.
When a cascade system is initiated, the high-temperature compressor is
started first. The low-temperature compressor is started when the evaporator
58 Chapter 3 Multistage and Cascade Refrigeration Cycles

Figure 3-11 Pressure-enthalphy (p-h) diagram for liquid subcooler showing increased refrig-
eration effect.

Figure 3-12 Schematic diagram of a cascade refrigeration cycle utilizing a fade-out vessel.
Fundamentals of Refrigeration SI, Second Edition 59

temperature reaches a sufficiently low temperature. Figure 3-12 shows a spe-


cial vessel called a fade-out vessel, which serves as an expansion tank added to
the low-temperature circuit. This vessel is needed to prevent the pressure in the
low-temperature circuit from becoming dangerously large upon shutdown and
reaching room temperature. The pressures are particularly large when liquid
refrigerant still exists in the system at room temperature. For example, the sat-
uration pressure of R-23 (trifluoromethane) at 24°C is 457 kPa. The volume of
the fade-out vessel is carefully determined so that a reasonable maximum pres-
sure occurs when the liquid has fully evaporated. For greater temperatures, the
pressure only increases in proportion to the absolute temperature.

Refrigeration-System Performance Parameters


High energy consumption by a refrigeration system results in high operating
costs and a larger adverse impact on the environment. Both of these undesir-
able occurrences are due to the increased electrical power required to operate
the compressor. Consequently, in recent years, emphasis has been placed on
reducing the energy consumption, thereby reducing operating costs and the
environmental impact of refrigeration systems.

System Performance
In general, the energy efficiency of a system is a measure of how well the sys-
tem uses the energy supplied (i.e., power) to drive a compressor, which pro-
duces the desired refrigeration effect. The ratio of the desired effect to energy
costs is called the coefficient of performance (COP). This term has special
meaning for a refrigeration system. In equation form, we have

COP = refrigeration capacity-


----------------------------------------------------------- (3-3)
compressor power

where the units for the capacity and power must be the same. With unitary sys-
tems such as home air-conditioning systems, the Seasonal Energy Efficiency
Ratio (SEER) is defined as the ratio of the refrigeration supplied to unit of energy.
The SEER is a modification of the COP, where a seasonal average performance is
determined and where different units (kJ/W) are used. Another cycle performance
indicator is usually referred to as horsepower-per-ton (kW/ton). This ratio, often
used to characterize industrial refrigeration systems, is merely the reciprocal of
the COP and also has mixed units. Written as an equation, the ratio is

total compressor power  kW 


kW  ton = ------------------------------------------------------------------------------- (3-4)
refrigeration capacity  tons 

Figure 3-13 shows the variation of COP and kW/ton for the two-stage system
in Example 3-2, where the refrigerating capacity is 1 ton and the condensing
60 Chapter 3 Multistage and Cascade Refrigeration Cycles

Figure 3-13 Variation of COP and hp/ton for two-stage system with evaporating tempera-
ture (R-22, 35°F condensing temperature, and 3 kW refrigeration capacity).

temperature is fixed at 35°C. Notice that with increasing evaporating tempera-


ture the COP increases and the kW/ton decreases. In these calculations, the
intermediate pressure is allowed to vary according to the estimate given earlier
for the optimum. Also note that the numerical values for the COP and kW/ton
in Figure 3-13 are higher than those that would be measured in an actual sys-
tem, because all the processes are treated as being ideal.
An overall COP may also be developed for the cascade cycle equal to the
refrigerating capacity of the low-temperature circuit divided by the total com-
pressor power for both circuits. Designating the low-temperature circuit COP
as (COP)LT and the high-temperature circuit COP as (COP)HT, the COP for a
two-circuit cascade may be written as
COPLT COP HT
COP Cascade = ---------------------------------------------------- (3-5)
1 + COP LT + COP HT

Compressor Performance
One measure of compressor performance originates with the fact that the ideal
compression work (when the path on the p-h diagram follows the constant
entropy line) is the minimum possible work input to a compressor. For an
actual compressor, the work input is always greater than this ideal or isentropic
work. It is the heat transfer between the walls and refrigerant and the internal
friction that cause the actual compressor work to exceed the isentropic work.
Fundamentals of Refrigeration SI, Second Edition 61

Figure 3-14 Determination of isentropic compressor efficiency.

Figure 3-14 shows two compression processes (one isentropic and the other
actual) starting from the same saturated vapor suction condition on a p-h dia-
gram. Note that the same discharge pressure is reached in both processes to
attain the condensing pressure. Also shown on Figure 3-14 are the enthalpies
associated with the endpoints of the two processes. The isentropic compression
efficiency (also called adiabatic compression efficiency) is given by
h 2s – h 1
 c = ------------------ (3-6)
h2 – h1

where state 1 refers to the suction condition and state 2 refers to the discharge
condition. The word adiabatic refers to the special condition of no heat transfer.
The subscript s indicates the process is isentropic—that the entropy at state 1
equals the entropy at state 2, or s2 = s1.

Example 3-3

Problem A compressor operates in a refrigeration system using ammonia as the refriger-


ant. At the suction inlet, the ammonia enters as a saturated vapor at –6°C. The
discharge conditions are measured as 931 kPa and 94°C. The mass flow rate of
the ammonia through the compressor is known to be 22.5 kg/min. If the atmo-
spheric pressure is 101 kPa, determine the discharge temperature from the
62 Chapter 3 Multistage and Cascade Refrigeration Cycles

compressor if the compression is isentropic, the isentropic efficiency of the


compressor, the isentropic or ideal compressor power input, and the actual
compressor power requirement.

Solution Use Figure 3-14 to show the process on the p-h diagram. T1 = –6°C, saturated
vapor, p2 = 931 kPa + 101 kPa = 1032 kPa. The saturated property table for
ammonia in Appendix A (Table A-7) gives, at the temperature of –6°C, h1 =
1455.51 kJ/kg and s1 = 5.7013 kJ/kg·°C. For isentropic compression, s2s = s1.
Using the ammonia p-h diagram in Appendix B (Figure B-6), trace the constant
entropy line to yield at p2 = 1032 kPa, h2s  1668 kJ/kg, T2s  71°C.
h 2s – h 1
Isentropic compressor efficiency =  c = ------------------
h2 – h1

Use Figure B-6 to locate the actual discharge state point at T2 = 94°C and
p2 = 1032 kPa. Read h2  1610 kJ/kg. Thus,
– 1455.51- = 0.73 or 73%
 c = 1610
------------------------------------
1668 – 1455.51

·
W c isentropic = m·  h 2s – h 1 
=  22.5 kg/min   1610 – 1455.51 kJ/kg = 3476 kJ/min
=  3476 kJ/min   1 kW   kJ/s    1 min  60 s  = 57.9 kW

·
W c actual = m·  h 2 – h 1 
=  22.5 kg/min   1668 – 1455.51 kJ/kg = 4781 kJ/min
=  4781 kJ/min   1 kW   kJ/s    1 min/60 s  = 79.7 kW

From the above compressor example, we see that the isentropic discharge
temperature of 71°C is considerably less than the actual measured temperature
of 94°C. In an actual compressor, the compression process includes both heat
transfer and friction, which elevates the discharge temperature. Also, the com-
pressor isentropic efficiency of about 75% is typical for vapor-compression sys-
tems. It turns out that a typical compressor efficiency falls within the range of
70% to 80% and provides a reasonable estimate of the actual compressor power.

Deviations of Actual Refrigeration Systems from


Ideal Systems
Figure 3-15 is a p-h diagram of an ideal single-stage vapor-compression sys-
tem. Superimposed on the same diagram (using a dashed line) is the diagram of
an actual cycle where the condenser outlet and evaporator inlet pressures are
Fundamentals of Refrigeration SI, Second Edition 63

Figure 3-15 Comparison of ideal single-stage cycle (solid lines) with actual cycle (dashed
lines).

matched. The differences between the ideal and actual cycles are due to pres-
sure drops in the condenser and evaporator, subcooling of the liquid leaving the
condenser, and superheating of the vapor entering the compressor. The pres-
sure drops as the refrigerant flows through the condenser, the evaporator, and
the piping connecting the components together.
The p-h diagram indicates that, because of the pressure drops, heat transfer,
and friction, the actual compressor work must be greater than that of the ideal
cycle. It is important to note that the path of the actual processes cannot be
traced on the p-h diagram and so are shown as dashed lines. In those cycles
using a thermostatic expansion valve to control the refrigerant flow rate on the
basis of superheat, the vapor leaving the evaporator is superheated.

Refrigeration System Types


How the refrigerant is metered to the evaporator defines the type of system.
Direct-expansion (DX) systems use a special device consisting of an orifice
through which the liquid refrigerant from the condenser must pass. The pres-
sure drop causes a fraction of the liquid to flash to a vapor, which reduces the
temperature of the mixture. The mixture enters the evaporator and, as the liquid
evaporates, heat is absorbed from the surrounding cold space or fluid. The
responsibility of the expansion device is to regulate the refrigerant flow so that
all the liquid evaporates and the vapor leaving the evaporator is superheated.
64 Chapter 3 Multistage and Cascade Refrigeration Cycles

The flooded and recirculation systems differ from the DX system in that
only liquid enters the evaporator. In systems using flooded evaporator coils, the
refrigerant is stored in a surge drum and is supplied to the evaporator at a rate
dependent on the rate of evaporation. A mixture of liquid and vapor returns to
the surge drum.
The recirculation system operates similarly to the flooded coil in that only
liquid is supplied to the coil. However, with recirculation, the liquid is forced
into the evaporator with a pump or high-pressure vapor from the system. The
distinguishing feature of the recirculation system is that more liquid is pumped
into the evaporator than will vaporize, which results in a liquid/vapor mixture
at the evaporator outlet. Another vessel (a low-pressure receiver) is needed to
separate the vapor from the liquid before it reaches the suction line.
A unitary system is one that is fabricated, charged, and tested in the manu-
facturing plant where it is built. These units are relatively small in refrigerating
capacity and can thus be produced in large quantities to meet a specific need in
the air-conditioning or refrigeration market. In contrast, field-erected systems
are custom designed and constructed to meet a unique refrigerating load. Typi-
cally, these systems are large in capacity. The components that comprise a
field-erected system are acquired from different manufacturers, and the system
is assembled, piped, charged, and started at the job site.

Direct-Expansion (DX) Systems

Remember that the basic vapor-compression refrigeration cycle consists of


four components: evaporator, compressor, condenser, and expansion device.
Figure 3-16 depicts a basic single-stage vapor-compression system where
both the evaporator and condenser are shown as tube-and-plate fin coils. The
expansion device controls the rate of refrigerant delivery to the evaporator
coil by using an orifice through which the liquid must pass. The pressure
drop across the device, along with the pressure increase across the compres-
sor, separates the high-pressure side from the low-pressure side. Shown in
Figure 3-16 is a thermostatic expansion valve that uses input from the sens-
ing bulb fastened to the outlet tube from the evaporator. The action of the
valve is to regulate the refrigerant flow to maintain a preset quantity of super-
heat at the evaporator outlet. Other expansion devices may also be used.
While automatic expansion valves maintain a constant suction pressure, the
capillary tube (a small-diameter tube providing a restriction to flow) causes a
pressure drop. With any of these expansion devices properly selected, the refrig-
erant vapor leaving the evaporator is superheated by 5°C to 10°C. Superheated
vapor will occupy the last 15% to 20% of the evaporator’s volume, causing
this portion to be relatively poor in heat transfer performance.
The velocity of the refrigerant in the piping connecting the components is
important. The system’s line pressure drops, noise, oil entrainment characteristics,
and operating costs all depend to a varying degree on velocity. With a halocarbon
system, the oil is soluble in the refrigerant and so moves about the system, as does
Fundamentals of Refrigeration SI, Second Edition 65

Figure 3-16 Single-stage vapor-compression system using direct expansion refrigerant feed.

the refrigerant. However, the refrigerant evaporates in the evaporator, thereby


increasing the concentration of the oil in the remaining liquid. Eventually, all the
refrigerant evaporates, leaving the oil behind. To move the oil out of the evapora-
tor, a minimum vapor velocity is needed to entrain the oil into the vapor flow. For
R-22 and R-134a, this minimum velocity is about 4.5 m/s. The upper velocity is
limited by keeping pressure drop and noise to reasonable values. The maximum
velocity for these refrigerants is usually about 20 m/s.
While the thermostatic expansion valve is widely used in residential and
commercial refrigeration equipment, the capillary tube also has applications
in refrigeration. For those systems that are small and where cost is an import-
ant factor, the capillary tube is reliable and low cost. Capillary tubes are sized
to achieve best system performance at rated conditions. However, at high
condensing temperatures, the evaporator is starved for refrigerant and at low
condenser temperatures, the evaporator is overfed. An example where capil-
lary tubes are used is in room air conditioners with capacities ranging from
1.75 to 3.5 kW.
Compared to the other refrigerant feed methods, the expansion device is the
least expensive. These are typically applied to halocarbon refrigerants at mod-
erate temperatures. As the evaporating temperature is lowered, the superheated
refrigerant in the latter portion of the evaporator imposes a progressively more
severe penalty in heat transfer performance and in overall operating efficiency
of the system. Consequently, expansion devices are not commonly used in low-
temperature applications. Furthermore, when ammonia is used, the small
refrigerant flow rates needed are difficult to achieve with reliability.
66 Chapter 3 Multistage and Cascade Refrigeration Cycles

Figure 3-17 Single-stage vapor-compression system using a flooded evaporator.

Flooded Systems
A single-stage flooded vapor-compression system is depicted in Figure 3-17.
The evaporator and condenser are shown as tube-and-plate fin coils. The refrig-
erant supply rate is governed by gravity. As the evaporator surface absorbs heat
from the cold space, vapor bubbles form in the coil. The bubbles reduce the
average density of the fluid from its value when it is entirely liquid. The supply
leg is filled with liquid, and the resulting force imbalance causes the fluid to
move upward through the coil. Thus, the movement of the refrigerant in the
coil is generated by gravity.
The surge drum provides two functions for the flooded evaporator. The first
is to separate the vapor from the liquid as it enters the drum. Only vapor then
enters the suction line leading to the compressor. The second function is to pro-
vide a reservoir of liquid to adequately supply the evaporator. The surge drum is
classified as a pressure vessel and so falls under the jurisdiction of ASHRAE
Standard 15-2007 (ASHRAE 2007) and, in particular, the ASME Boiler and
Pressure Vessel Code (ASME 2007). Sudden changes in the refrigerating load
cause the liquid level in the surge drum to change quickly, potentially resulting
in liquid entering the suction line. The thermal performance of the flooded evap-
orator is excellent, because all tubes remain wet with refrigerant and thereby
maintain a high heat transfer coefficient. The level of refrigerant in the surge
drum is usually controlled by a low- or high-side float-control valve.
When a halocarbon refrigerant is used, the suction line leaving at the top
of the surge drum (see Figure 3-17) cannot transport oil. Oil-return lines are
Fundamentals of Refrigeration SI, Second Edition 67

added to facilitate the return of oil. These lines are connected to the surge
vessel just below the liquid level and to the bottom of the float valve cham-
ber. They then connect to a lower point on the suction line, thereby using
gravity to move the oil.
With ammonia systems, the oil is typically not miscible with ammonia, so
it does not go into the solution. However, a small quantity of oil always leaves
the compressor oil separator and eventually finds its way to the evaporator.
Here the oil is cold and so does not readily flow. As a result, it cannot return
with the ammonia vapor entering the suction line. The oil then accumulates in
the evaporator coil and surge drum, which must be drained to maintain good
evaporator performance.
The flooded coil is applied to low-temperature systems to avoid the disad-
vantages of direct expansion; namely, reduced coil performance due to super-
heat. However, the flooded coil is a complex device that requires individual
controls and a surge drum. Generally, if more than four coils are needed in a
system, then an alternative coil configuration, such as recirculation, is used for
economic reasons.

Recirculation Systems

The third method of refrigerant supply is the liquid recirculation system. This
system is also referred to as overfeed, pumped-liquid recirculation, or simply
recirculation. Figure 3-18 depicts a simple single-stage vapor-compression
system using recirculation to supply the refrigerant to the evaporator. The key
feature of this system is that liquid is supplied to the evaporator at a rate that
exceeds the vaporization rate in the evaporator. Consequently, liquid refriger-
ant leaves the evaporator and returns to the low-pressure receiver along with
the vapor. Separation of the vapor from the liquid takes place inside the low-
pressure receiver, and the vapor enters the suction line on its way to the com-
pressor. The returned liquid refrigerant and the liquid supplied by the level con-
trol valve fall to the bottom of the vessel for pumping back to the evaporator.
As with the other refrigerant-supply methods, the expansion that occurs across
the valve achieves the desired liquid temperature with the production of some
flash vapor. However, in contrast to the DX system, the flash vapor does not
enter the evaporator coil.
The recirculation system shown in Figure 3-18 utilizes a mechanical pump to
force the liquid to the evaporator. The pressure increase may also be generated by
vapor pressure from the system’s high side. In both situations, the pressure is suf-
ficient to force the liquid through the pipe and attain an adequate pressure drop
across the flow balancing valve to provide adjustment of the liquid flow rate. In
practice, a hand expansion valve is installed for adjusting the refrigerant flow rate
to the evaporator, while a solenoid shutoff valve in series provides ON and OFF
control. Similarly, a low-pressure receiver and liquid pump (or high-side pres-
sure) may be used to deliver liquid refrigerant to a flooded coil.
68 Chapter 3 Multistage and Cascade Refrigeration Cycles

Figure 3-18 Single-stage vapor-compression system using recirculation refrigerant feed.

Liquid recirculation systems are usually applied to industrial refrigeration


systems. The size of the system provides an economy of scale, an advantage
for recirculation systems. In the residential and commercial refrigeration fields
where systems are small and evaporating temperatures are moderate, the rela-
tively complicated recirculation system does not provide a significant advan-
tage and so would be too expensive.
The liquid-recirculated coil provides a slight improvement due to the
higher velocity that the refrigerant has when passing through the coil. The
refrigerant-side heat transfer coefficient increases with increasing velocity. A
parameter, defined as n, quantifies the amount of the overfeed as

refrigerant mass flow rate supplied to evaporator


n = ------------------------------------------------------------------------------------------------------------------------------------------- (3-7)
refrigerant mass flow rate vaporized in evaporator

where n is the circulation ratio.


When the circulation ratio is greater than 1.0, then liquid is returned to
the low-pressure receiver. The selection of a value for n depends on several
factors. From only the heat-transfer viewpoint, the higher the value of n, the
higher the value of the heat transfer coefficient. Opposing this trend is the
associated increase in pressure drop in both the evaporator and piping. Addi-
tionally, as the pressure drop increases across the coil, the evaporating tem-
perature rises somewhat.
The consequence of the conflicting trends associated with the circulation
ratio is that an optimum value has been determined for a given refrigeration
system. Experimental data obtained by Wile (1962) and reported by Stoecker
(1988) are given in Figure 3-19. The evaporator’s actual capacity is divided by
Fundamentals of Refrigeration SI, Second Edition 69

©Business News Publishing Co. Reprinted with permission.

Figure 3-19 Influence of circulation ratio on heat transfer coefficient.

its capacity if operated with an expansion valve, yielding slightly superheated


vapor at the outlet; the result is plotted against the circulation ratio. The evapo-
rator tested was a steel tube-and-plate fin using ammonia and cooling air. The
outside tube diameter was 1.5875 cm.
From Figure 3-19, it can be seen that a circulation ratio of 3 improves the
coil’s performance by about 25%. Further observation indicates that an opti-
mum value of n lies somewhere between 3 and 5. Circulation ratios higher
than 5 reduce performance of the coil due to the higher evaporating tempera-
ture resulting from the increased pressure drop. The recirculation ratio nor-
mally recommended for R-22 is 3.
The recirculation system offers several advantages over other refrigerant
feed methods:

• The entire evaporator heat-transfer surface is wet, resulting in high values


of the heat transfer coefficient.
• The refrigerant leaving the evaporator is a liquid/vapor mixture that pro-
ceeds back to the low-pressure receiver. The vapor entering the suction line
is first nearly saturated and second travels a relatively short distance to the
compressor. With the other two refrigerant feed methods, the vapor travels
a potentially large distance from the evaporator to the compressor.
70 Chapter 3 Multistage and Cascade Refrigeration Cycles

• Regardless of the condensing pressure, liquid refrigerant is delivered to the


evaporators at a constant pressure.
• The flash gas is generated in the low-pressure receiver, which is located in
the mechanical room along with the compressor. Therefore, large transport
distances for the flash gas are avoided.
• The removal of oil from the system’s low side may be accomplished at the
low-pressure receiver. The liquid flow of refrigerant through the evaporator
continuously flushes oil from the internal surfaces.
• The central location of the low-pressure receiver and its control simplifies
the system from individual controls at each evaporator.

Disadvantages of the liquid recirculation system include the following:

• The low-pressure receiver, the larger line sizes, and the pumping equipment
increase the initial system cost, particularly for small industrial systems.
• The liquid lines supplying the evaporators are cold and so must be insulated.
• The refrigerant inventory is larger than that of the DX and flooded systems.
• The equipment providing the pressure forcing the liquid to the evaporators
has a significant operating expense.

For a large industrial refrigeration system, the advantages of the liquid


recirculation system generally outweigh the disadvantages.
Both top-feed and bottom-feed evaporators are available to the refrigeration-
system designer. The evaporator coil, shown in Figure 3-18, is bottom fed. The
advantages of the bottom-fed coil are better distribution of the refrigerant
through the coil’s circuits and increased refrigerant-side heat transfer coeffi-
cient. The top-fed coil offers these advantages: better transport of oil out of the
coil, natural draining of the coil during the defrost sequence, and a reduced
refrigerant charge. When top-fed coils are selected, the recommended circula-
tion ratio increases to 6 or 7.

Gas-Pumped Systems
Vapor from the high-pressure side of the system may be used to force the liquid
through the piping to the evaporators. These gas-pumped systems (typically
applied to industrial systems) began appearing in the late 1940s. The mechani-
cal pumps (both positive displacement and centrifugal) available at that time
were not sufficiently reliable. A gas-pumped system, through the use of vessels
and control valves, provides the pressure needed to force the liquid to the evap-
orators. The gas-pumped system has undergone development and refinement
over the years and is now a safe and reliable method to pump liquid.
Mention should also be made of the word gas used in conjunction with
these systems. This is how the refrigeration industry refers to the system;
Fundamentals of Refrigeration SI, Second Edition 71

however, the appropriate word is vapor, because the high-pressure refrigerant


vapor is at a condition below its critical point. Gas usually refers to conditions
that are above the critical point.
Figure 3-20 depicts one approach to pump liquid using gas pressure in a
two-stage system. The operating features of the system in Figure 3-20 include
the following:

• The low-pressure receiver operates normally.


• The pumping vessel obtains liquid from the low-pressure receiver as it
drains through the check valve. The vent line is opened to the low-pressure
receiver by the three-way valve.
• When the liquid level in the pumping vessel rises to a certain height, the
three-way valve applies high-pressure vapor to the pumping vessel where
the check valves allow the passage of liquid to the controlled-pressure
receiver (CPR) but prevent liquid flow to the low-pressure receiver.
• Pressure in the CPR is controlled by venting vapor to the low-pressure
receiver through pressure-regulating valve A (PRV-A).
• The ON-OFF valve supplies liquid to the CPR when liquid is not available in
the low-pressure receiver. This maintains an appropriate liquid level in the

©Business News Publishing Co. Reprinted with permission.

Figure 3-20 Typical gas-pumping system.


72 Chapter 3 Multistage and Cascade Refrigeration Cycles

low-pressure receiver. The leg labeled “common” for the three-way valve is
the line that is always open.
• Valve PRV-B provides vapor to the CPR when sufficient flash gas from the
high-pressure liquid is not adequate. The pressure setpoint for the PRV-B
valve is somewhat less than the pressure setpoint for valve PRV-A. Appli-
cation of gas pumping occurs in both single- and two-stage systems. The
controlled-pressure receiver supplies a constant-pressure liquid to the evap-
orators that is not influenced by the condensing pressure. Gas pumping
configurations that differ from the one shown in Figure 3-20 are also used
in the HVAC&R industry.

The power needed to operate a gas-pumped system is somewhat higher


than that of a mechanically pumped system. One source of energy loss in gas
pumping is the expansion of the high-pressure vapor to a lower pressure in the
pumping vessel when switching from supplying the CPR to draining the low-
pressure receiver. Another energy loss results from the heat transfer that occurs
between the warm vapor and the cold liquid in the pumping vessel. Estimates
suggest that energy consumption of gas pumping is greater than that of
mechanical pumping by 50% to 100%.

Mechanically Pumped Systems

The availability of reliable liquid pumps has led to the widespread application
of mechanically pumped liquid-recirculation systems. Two mechanically
pumped systems are depicted in Figures 3-21a and 3-21b. Two liquid pumps
allow the system to continue operating while one pump undergoes maintenance.
Oil drains are indicated in the figures. These are important for systems using
ammonia and are always located where the refrigerant velocity is low. Angle
valves reduce the pressure drop that occurs in the line. This is especially import-
ant for the inlet to the pumps, as shown in Figure 3-21a. Shutoff, check, and
relief valves are also shown, which complete the recirculation pumping subas-
sembly. The liquid pump is the key component, and its selection depends on the
pressure, or head, to supply the needed liquid flow rate. Both centrifugal and
positive displacement pumps may be used, but current practice favors the cen-
trifugal pump. The arrangements shown in Figure 3-21a and 3-21b are suitable
for centrifugal pumps. The selection of the centrifugal pump is based on the
needed flow and pressure and also on the net positive suction head. A centrifu-
gal pump requires a minimum liquid depth above the pump’s inlet to avoid cav-
itation. The pressure increase across the pump must overcome all the pressure
drops that occur in the liquid recirculation circuit. These pressure drops include
pipe friction in the supply line, valves in the liquid supply line, the balancing
valve, the evaporator coil, pipe friction in the liquid/vapor return line, and the
pressure resulting from the elevation difference between the low-pressure
Fundamentals of Refrigeration SI, Second Edition 73

(a)

(b)

©Business News Publishing Co. Reprinted with permission.

Figure 3-21 Mechanically pumped liquid supply system (a) using a backup pump and (b)
using a backup pump and a common drop.
74 Chapter 3 Multistage and Cascade Refrigeration Cycles

receiver and the evaporators. The determination of pressure drops and their cal-
culation are addressed in Chapter 7.
Figure 3-22 depicts a simplified two-stage pumped recirculation system
where two low-temperature evaporators are served. Balancing valves are
located at the inlets of the evaporators to establish the required refrigerant flow
that meets the refrigeration load.

Unitary Systems
A unitary system is one in which the evaporator, compressor, and condenser are
incorporated into a single unit. These units are built, charged, and tested at the

Figure 3-22 Two-stage vapor-compression system using recirculation refrigerant feed to two
evaporators.
Fundamentals of Refrigeration SI, Second Edition 75

manufacturing plant. The components are matched for a particular unitary


product, which then comprises an entry into the product line. Unitary systems
are referred to as split systems when the components are separated. One exam-
ple is the domestic home air conditioner, where the evaporator coil and expan-
sion valve are located inside at the air handler, while the compressor and
condenser are located outside. Figures 3-23a and 3-23b show examples of uni-
tary systems.
Unitary systems are classified into three groups: residential, light commer-
cial, and commercial. The residential group consists of units generally smaller
than a capacity of 6 tons and are designed specifically for the domestic market.
The light commercial group includes systems up to a capacity of 12 tons,
which are used in small businesses and commercial properties. The commer-
cial class includes systems greater than 12 tons in capacity, which are designed
for use in large commercial buildings.
Unitary systems are also available for low-temperature systems in capaci-
ties of less than approximately 20 tons. For example, in a refrigerated chamber,
the compressor and condenser combination is mounted outdoors (maybe on the
roof), and penetration through the building’s structure connects the system to
the evaporator coil.
All unitary systems are electrically wired at the plant to minimize job site
wiring during installation. Residential systems are typically powered with
single-phase 115 VAC electrical power. Light commercial and commercial
unitary systems are usually powered by three-phase electrical power.

Field-Erected Systems

Field-erected systems (sometimes called central plants) include commercial


and industrial refrigeration systems. Large commercial building water chillers
are fabricated at the factory and transported to the project site. The piping, con-
denser, cooling tower, controls, and ancillary equipment are all installed at the
site. Similarly, for large industrial systems, the components selected by the
designer (including condenser, compressors, vessels, piping, controls, etc.) are
transported to the site. The assembly, including the fabrication of the piping
system, is performed at the job site. Transport of the entire central system is not
possible due to its physical size and weight
Quality control at the site is important. The system’s components must sus-
tain no damage and must remain internally clean during construction. All
installation practices (as well as the design and building of the foundations on
which the equipment rests) must conform to all applicable codes. The electrical
power wiring and the control wiring are also done at the job site. The charging
of the system with refrigerant, startup of the system, and any final refinements
to the system are performed at the job site as well.
76 Chapter 3 Multistage and Cascade Refrigeration Cycles

(a)

(b)

Figure 3-23 (a) Indoor/outdoor split system and upflow residential furnace—actual view
(left) and cross-section view (right)—and (b) typical room air conditioner.
Fundamentals of Refrigeration SI, Second Edition 77

Next Step
Chapter 4 discusses the configuration and operation of the evaporator compo-
nent used in refrigeration systems.

Summary
This chapter reviewed single-stage and two-stage vapor-compression refrigera-
tion systems and cascade refrigeration systems, with particular emphasis on the
thermodynamics of the processes and the cycles. The thermodynamic state of
the refrigerant at points within the cycle in addition to the application of mass
and energy balances permits calculations describing the operation and the per-
formance of these systems. Deviations of actual refrigeration cycles from ideal
cycles occur through pressure drops, heat transfer in piping and in the com-
pressor, and by friction.
This chapter discussed refrigeration system types based on two criteria.
The first criterion is the method of refrigerant feed. Direct expansion is widely
used in smaller systems or in larger systems where moderate temperatures are
to be maintained. Both flooded and recirculation systems maintain a com-
pletely wet heat-transfer surface in the evaporator, which improves the capacity
of the coil by as much as 20%.
Recirculation systems further improve the evaporator’s performance by
supplying a refrigerant flow that exceeds the vaporization rate. The circulation
ratio is the actual refrigerant mass flow rate divided by the vaporization rate in
the evaporator. The optimum circulation ratio for R-22 and ammonia recircula-
tion systems is approximately 3.
The second criterion is based on how the system is constructed. Small sys-
tems mass produced in a factory are called unitary systems; their components
are placed together, usually inside an enclosure. Field-erected systems gener-
ally use components so large that the system cannot be shipped. Consequently,
the individual components are shipped to the job site and assembled there. You
should now be able to

 determine the thermodynamic state points of the refrigerant in the cycle for
ideal single-stage and two-stage vapor-compression refrigeration systems;
 describe the thermodynamic processes that occur in a single-stage refrigera-
tion system;
 describe the thermodynamic processes that occur in a two-stage system
using a flash tank;
 perform calculations for ideal single- and two-stage systems, yielding tem-
perature, pressure, refrigerant flow rate, compressor input power, refrigerat-
ing rate and the heat rejection rate;
78 Chapter 3 Multistage and Cascade Refrigeration Cycles

 define the terms COP, kW/ton and compressor isentropic efficiency and per-
form the calculations for ideal cycles and for cycles with compressors with
an efficiency of less than one;
 discuss the differences between ideal and actual single-stage vapor-com-
pression refrigeration cycles;
 identify and describe the three refrigerant feeding methods;
 give examples of three devices used to carry out direct expansion of the
refrigerant and the systems that may use these devices;
 describe the operation of the gas-pumped liquid recirculation system;
 describe the operation of the mechanically pumped liquid recirculation
system; and
 explain the differences and similarities of the unitary and field-erected
refrigeration systems.

References and Bibliography


ASHRAE. 2007. ANSI/ASHRAE Standard 15-2007, Safety Standard for
Refrigeration Systems. Atlanta: ASHRAE.
ASHRAE. 2008. ASHRAE Handbook—HVAC Systems and Equipment.
Atlanta: ASHRAE.
ASME. 2007. Boiler and Pressure Vessel Code. New York. American Society
of Mechanical Engineers.
Stoecker, W. 1988. Industrial Refrigeration. Troy, MI: Business News Publish-
ing Co.
Wile, D. 1962. Evaporator performance with liquid refrigerant recirculation.
Proceedings of Annex 1962-1, Washington, DC.
Fundamentals of Refrigeration SI, Second Edition 79

Skill Development Exercises for Chapter 3


Complete these questions by writing your answers on the worksheets at the back of this book.

3-1 Consider an ideal single-stage system, as shown in Figure 3-1, using R-22 as
the refrigerant. If the evaporating temperature is 7.22°C, the condensing tem-
perature is 35°C, and the refrigerant mass flow rate is 0.453 kg/min, calculate
(1) enthalpies for all state points in the cycle, (2) refrigerating capacity rate, (3)
power needed to operate the compressor, (4) the heat rejection rate from the
condenser, and (5) the COP. Additionally, describe the effect of the evaporating
temperature increase on Q· in , Q· out , and W· c .
3-2 Reconsider Example 3-2, where an ideal two-stage refrigeration system oper-
ates with R-22 as the refrigerant and incorporates a flash tank. The evaporat-
ing temperature drops to –40°C. Remaining the same are the condensing
temperature at 35°C and the intermediate pressure at 482.63 kPa. On the basis
of 3.52 kW of refrigerating capacity, determine (1) the enthalpies for all
refrigerant state points in the cycle, (2) refrigerant mass flow rates through the
booster and high-stage compressors, (3) the heat transfer rate rejected by the
condenser, (4) the ideal compressor power input (isentropic efficiency is 1) for
both the booster and high-stage compressors, and (5) the COP and kW/ton.
3-3 A subcooler is used in a refrigeration system where ammonia serves as the
refrigerant. The subcooler is configured as shown in Figure 3-10. The entering
liquid mass flow is 45.36 kg/min. The liquid leaving the condenser is saturated
at 26.67°C. The liquid leaving the subcooler is 15.55°C and is at the condens-
ing pressure. The liquid and vapor inside the vessel are saturated at –1.11°C.
Calculate the mass flow rates of the subcooled liquid and the mass flow rate of
the vapor to the compressor.
3-4 A compressor operates in a system using R-134a as the refrigerant where the
mass flow rate is 3.63 kg/min. The conditions at the suction port of the com-
pressor are 172.37 kPa and –6.67°C. The pressure at the discharge port is
1206.6 kPa. This compressor is known to have an isentropic efficiency of 80%.
Calculate (1) actual refrigerant temperature at the discharge port of the com-
pressor and (2) actual power input requirement to the compressor.
3-5 Rework Exercise 3-2 where all the conditions remain the same but both the
booster and high-stage compressors have an isentropic efficiency of 75%. R-22
is the refrigerant. Also, determine the booster and high-stage compressor dis-
charge temperatures.
3-6 Sketch and label a diagram showing the major components and interconnecting
piping for a two-stage mechanically pumped liquid recirculation system. The
refrigerating load is divided between a low-temperature and an intermediate-
temperature load.
80 Chapter 3 Multistage and Cascade Refrigeration Cycles

3-7 Explain why the thermal performance of the direct-expansion coil is less than
that of an equally sized flooded or liquid-recirculated coil operating under the
same conditions. Why is this important for coils used in low-temperature appli-
cations?
3-8 Why does a pressure drop in the refrigerant flowing through an evaporator coil
increase the evaporating temperature of the coil?
3-9 List the similarities and differences between unitary systems and field-erected
refrigeration systems.
Evaporators

Study Objectives
Chapter 4 presents the configuration and operation of the evaporator. Heat
transfer in the evaporator occurs by convection and conduction. In particular,
convection involves the evaporation of a liquid as well as the cooling of an air-
stream. Conduction is concerned with the transfer of heat through a material.
Psychrometrics of moist air describes the cooling and dehumidification of an
airstream flowing through an evaporator.
The psychrometric chart shows these processes and the associated changes
in the thermodynamic properties of the moist air. The configuration and use of
air-cooling evaporators and the methods of refrigerant feed are discussed.
Additionally, the configuration and use of liquid-cooling units are discussed.
After studying Chapter 4, you should be able to

 explain conduction and convection heat transfer and use the equations to
estimate a heat transfer rate;
 given the air inlet conditions of an air-cooling unit, determine the air exit
conditions and the rate of heat exchange;
 describe the three methods of refrigerant feed to an evaporator;
 describe an air-cooling evaporator and give example applications; and
 describe a liquid-cooling evaporator and give example applications.

Instructions
Read the material in Chapter 4. Verify the examples presented in the chapter
with your own calculations. At the end of the chapter, complete the skill devel-
opment exercises without referring to the text. Review those sections of the
chapter as needed to complete the exercises.

Introduction
The evaporator in a refrigeration system absorbs heat from the space or fluid
stream that is to be chilled. In the process of absorbing the heat or thermal
energy, two methods of heat transfer occur: convection and conduction. Radia-
tion is a third method of heat transfer but is not significant for evaporators.
For heat transfer to exist, a temperature difference must be present between
two points. Convection heat transfer results from the temperature difference
82 Chapter 4 Evaporators

between a fluid and a surface. Furthermore, convection occurs when the fluid is
moving adjacent to the surface. The rate of convection heat transfer is given by
Newton’s Law of Cooling:

·
Q = hc A  Tw – Tf  (4-1)
·
where Q is the heat transfer rate (kW/h), A is the area (m2) available for heat
transfer, T w is the surface or wall temperature (°C), T f is the fluid tempera-
ture (°C) away from the wall, and hc is the convection heat transfer coefficient
(kJ/s·m2·°C). Two items need to be mentioned here. First, the fluid tempera-
ture, T f , is that of the bulk fluid, which is not influenced by the presence of the
wall or the heat transfer. Second, h c is a parameter that depends on a number
of characteristics including fluid velocity and fluid properties.
Conduction heat transfer results from a temperature difference inside a
solid material or completely stagnant fluid. Fourier’s Law gives the heat con-
duction rate as

· kA
Q = ------  T 1 – T 2  (4-2)
x
·
where Q is the heat transfer rate (kW), k is the thermal conductivity (kJ/s·m·°C), A
is the area available for heat transfer (m2), x is the thickness of the material
(m), and  T 1 – T 2  is the temperature difference across the thickness of the
material. The Fourier heat conduction equation given here is only valid for a
plane (a flat surface such as a wall or door). Curved surfaces (such as pipes)
need more complicated equations for heat conduction.
The evaporator typically employs metal to separate the refrigerant from the
fluid being cooled. Thus, convection heat transfer occurs next to the metal on
both sides, involving the refrigerant on one side and the fluid to be cooled on
the other side. Through the metal, only heat conduction occurs. Also note that
the heat transfer rate that enters the evaporator from the outside (cooled-fluid
side) must also pass through the metal and to the refrigerant on the other side at
the same rate for steady-state operation. If this were not the situation, then the
evaporator would either cool or warm with time, which it does not do.

Psychrometrics of Moist-Air Cooling

Predominantly, air is a mixture of two gases: nitrogen and oxygen. While many
other gases exist in the atmosphere in trace quantities, one trace gas is particu-
larly important with refrigeration equipment. This gas is water vapor. It is
referred to as a vapor because it exists in the atmosphere at a temperature below
374°C, which is the critical temperature of water. For our consideration of refrig-
eration systems, atmospheric air consists of two parts: dry air, which represents
the mixture of all the gases, and water vapor, which is the moisture in the air.
Psychrometrics is the study of moist air and its thermodynamic properties. Here
Fundamentals of Refrigeration SI, Second Edition 83

we present several highlights of psychrometrics that are important to the under-


standing of evaporators that cool an airstream.
Moist air is composed of two parts that do not interact with each other.
Therefore, each of these parts behaves as an ideal gas:

dry air = p a V = n a RT
(4-3)
water vapor = p w V = n w RT

where
pa = partial pressure of dry air, kPa
pw = partial pressure of water vapor, kPa
V = total mixture volume, m3
na = number of moles of dry air
nw = number of moles of water vapor
R = universal gas constant, 8.314 kJ/kmol·K
T = absolute temperature, K

The pressure that is measured by a barometer is the sum of the dry air and
water vapor pressures,

p = pa + pw (4-4)

where p (kPa) is the total pressure of the air. We now define several quantities
that are important in the study of psychrometrics:

saturated air: water vapor in the moist air at its saturated pressure and tem-
perature. Table 4-1 shows the thermodynamic properties of saturated liquid and
saturated vapor for water where for each saturated temperature there is a
unique saturated pressure.
dry-bulb temperature: air temperature, T, as measured by an ordinary ther-
mometer.
wet-bulb temperature: temperature measured by a thermometer moving rap-
idly in moist air that has its sensing element covered with a wetted cotton wick.
A psychrometer (shown in Figure 4-1) measures simultaneously the dry-bulb
and wet-bulb temperatures. To obtain an accurate indication of the wet-bulb
temperature, the psychrometer must be rotated rapidly in the moist air. Evapo-
ration from the wick cools the sensing bulb.
thermodynamic wet-bulb temperature: temperature, T*, to which water (or
ice), when evaporating into moist air at a specific dry-bulb and humidity ratio,
W, can bring the air to saturation adiabatically while the pressure remains con-
stant. Measuring the wet-bulb temperature is a different process than that for
thermodynamic wet-bulb temperature, but the numerical differences between
the wet-bulb and thermodynamic wet-bulb temperature are very small and so
may be neglected.
84 Chapter 4 Evaporators

Table 4-1 Refrigerant 718 (Water/Steam) Properties of Saturated Liquid and Saturated Vapor
Source: 2009 ASHRAE Handbook—Fundamentals, Chapter 30
Enthalpy, Entropy, Specific Heat cp , Velocity of Viscosity, Thermal Cond.,
Density, Volume, Surface
Temp.,* Pres- kg/m3 m3/kg
kJ/kg kJ/(kg·K) kJ/(kg·K) cp /cv Sound, m/s Pa·s mW/(m·K)
Tension, Temp.,*
°C sure, MPa Liquid Vapor Liquid Vapor Liquid Vapor Liquid Vapor Vapor Liquid Vapor Liquid Vapor Liquid Vapor mN/m °C
0.01a 0.00061 999.8 205.990 0.00 2500.92 0.0000 9.1555 4.220 1.884 1.329 1402 409.0 1791.2 9.22 561.0 17.07 75.65 0.01
5 0.00087 999.9 147.010 21.02 2510.06 0.0763 9.0248 4.205 1.889 1.328 1426 412.6 1518.3 9.34 570.5 17.34 74.94 5
10 0.00123 999.7 106.300 42.02 2519.21 0.1511 8.8998 4.196 1.895 1.328 1447 416.2 1306.0 9.46 580.0 17.62 74.22 10
15 0.00171 999.1 77.8750 62.98 2528.33 0.2245 8.7803 4.189 1.900 1.328 1466 419.7 1137.6 9.59 589.3 17.92 73.49 15
20 0.00234 998.2 57.7570 83.91 2537.43 0.2965 8.6660 4.184 1.906 1.327 1482 423.2 1001.6 9.73 598.4 18.23 72.74 20
25 0.00317 997.0 43.3370 104.83 2546.51 0.3672 8.5566 4.182 1.912 1.327 1497 426.6 890.1 9.87 607.2 18.55 71.97 25
30 0.00425 995.6 32.8780 125.73 2555.55 0.4368 8.4520 4.180 1.918 1.327 1509 430.0 797.4 10.01 615.5 18.89 71.19 30
35 0.00563 994.0 25.2050 146.63 2564.55 0.5051 8.3517 4.180 1.925 1.327 1520 433.4 719.3 10.16 623.3 19.24 70.40 35
40 0.00738 992.2 19.5150 167.53 2573.51 0.5724 8.2555 4.180 1.931 1.327 1529 436.7 653.0 10.31 630.6 19.60 69.60 40
45 0.00959 990.2 15.2520 188.43 2582.43 0.6386 8.1633 4.180 1.939 1.327 1536 440.0 596.1 10.46 637.3 19.97 68.78 45
50 0.01235 988.0 12.0270 209.34 2591.29 0.7038 8.0748 4.182 1.947 1.328 1542 443.2 546.8 10.62 643.6 20.36 67.94 50
55 0.01576 985.7 9.5643 230.26 2600.09 0.7680 7.9898 4.183 1.955 1.328 1547 446.4 504.0 10.77 649.2 20.77 67.10 55
60 0.01995 983.2 7.6672 251.18 2608.83 0.8313 7.9081 4.185 1.965 1.328 1551 449.5 466.4 10.93 654.3 21.19 66.24 60
65 0.02504 980.5 6.1935 272.12 2617.50 0.8937 7.8296 4.187 1.975 1.329 1553 452.6 433.2 11.10 659.0 21.62 65.37 65
70 0.03120 977.7 5.0395 293.07 2626.10 0.9551 7.7540 4.190 1.986 1.330 1555 455.6 403.9 11.26 663.1 22.07 64.48 70
75 0.03860 974.8 4.1289 314.03 2634.60 1.0158 7.6812 4.193 1.999 1.331 1555 458.5 377.7 11.43 666.8 22.53 63.58 75
80 0.04741 971.8 3.4052 335.01 2643.02 1.0756 7.6111 4.197 2.012 1.332 1554 461.4 354.3 11.59 670.0 23.01 62.67 80
85 0.05787 968.6 2.8258 356.01 2651.33 1.1346 7.5434 4.201 2.027 1.333 1553 464.2 333.3 11.76 672.8 23.51 61.75 85
90 0.07018 965.3 2.3591 377.04 2659.53 1.1929 7.4781 4.205 2.043 1.334 1550 466.9 314.4 11.93 675.3 24.02 60.82 90
95 0.08461 961.9 1.9806 398.09 2667.61 1.2504 7.4151 4.210 2.061 1.335 1547 469.6 297.3 12.10 677.3 24.55 59.87 95
99.97b 0.10133 958.4 1.6732 419.06 2675.53 1.3069 7.3544 4.216 2.080 1.337 1543 472.2 281.8 12.27 679.1 25.09 58.92 99.97
100 0.10142 958.3 1.6718 419.17 2675.57 1.3072 7.3541 4.216 2.080 1.337 1543 472.2 281.7 12.27 679.1 25.10 58.91 100
105 0.12090 954.7 1.4184 440.27 2683.39 1.3633 7.2952 4.222 2.101 1.339 1538 474.7 267.6 12.44 680.5 25.66 57.94 105
110 0.14338 950.9 1.2093 461.42 2691.06 1.4188 7.2381 4.228 2.124 1.341 1533 477.1 254.7 12.61 681.7 26.24 56.96 110
115 0.16918 947.1 1.0358 482.59 2698.58 1.4737 7.1828 4.236 2.150 1.343 1527 479.5 242.9 12.78 682.6 26.85 55.97 115
120 0.19867 943.1 0.89121 503.81 2705.93 1.5279 7.1291 4.244 2.177 1.346 1520 481.7 232.1 12.96 683.2 27.47 54.97 120
125 0.23224 939.0 0.77003 525.07 2713.10 1.5816 7.0770 4.252 2.207 1.349 1512 483.9 222.1 13.13 683.6 28.11 53.96 125
130 0.27028 934.8 0.66800 546.38 2720.08 1.6346 7.0264 4.261 2.239 1.352 1504 486.0 212.9 13.30 683.7 28.76 52.93 130
135 0.31323 930.5 0.58173 567.74 2726.87 1.6872 6.9772 4.272 2.274 1.355 1496 487.9 204.4 13.47 683.6 29.44 51.90 135
140 0.36154 926.1 0.50845 589.16 2733.44 1.7392 6.9293 4.283 2.311 1.359 1486 489.8 196.5 13.65 683.3 30.14 50.86 140
145 0.41568 921.6 0.44596 610.64 2739.80 1.7907 6.8826 4.294 2.351 1.363 1476 491.6 189.2 13.82 682.8 30.86 49.80 145
150 0.47616 917.0 0.39245 632.18 2745.93 1.8418 6.8371 4.307 2.394 1.368 1466 493.3 182.5 13.99 682.0 31.60 48.74 150
155 0.54350 912.3 0.34646 653.79 2751.81 1.8924 6.7926 4.321 2.440 1.373 1455 494.8 176.1 14.16 681.1 32.35 47.67 155
160 0.61823 907.4 0.30678 675.47 2757.44 1.9426 6.7491 4.335 2.488 1.379 1443 496.3 170.2 14.34 680.0 33.13 46.59 160
165 0.70093 902.5 0.27243 697.24 2762.81 1.9923 6.7066 4.351 2.540 1.385 1431 497.6 164.7 14.51 678.6 33.93 45.50 165
170 0.79219 897.5 0.24259 719.08 2767.90 2.0417 6.6650 4.368 2.594 1.392 1419 498.9 159.6 14.68 677.0 34.75 44.41 170
175 0.89260 892.3 0.21658 741.02 2772.71 2.0906 6.6241 4.386 2.652 1.399 1405 500.0 154.7 14.85 675.3 35.59 43.30 175
180 1.00280 887.0 0.19384 763.05 2777.21 2.1392 6.5840 4.405 2.713 1.407 1392 501.0 150.1 15.03 673.3 36.45 42.19 180
185 1.12350 881.6 0.17390 785.19 2781.41 2.1875 6.5447 4.425 2.777 1.416 1378 501.9 145.8 15.20 671.1 37.33 41.07 185
190 1.25520 876.1 0.15636 807.43 2785.28 2.2355 6.5059 4.447 2.844 1.425 1363 502.7 141.8 15.37 668.8 38.24 39.95 190
195 1.39880 870.4 0.14089 829.79 2788.82 2.2832 6.4678 4.471 2.915 1.436 1348 503.4 137.9 15.54 666.1 39.16 38.81 195
200 1.55490 864.7 0.12721 852.27 2792.01 2.3305 6.4302 4.496 2.990 1.447 1332 503.9 134.3 15.71 663.3 40.11 37.67 200
205 1.72430 858.8 0.11508 874.88 2794.83 2.3777 6.3930 4.523 3.068 1.459 1316 504.3 130.9 15.89 660.3 41.09 36.53 205
210 1.90770 852.7 0.10429 897.63 2797.27 2.4245 6.3563 4.551 3.150 1.472 1299 504.6 127.6 16.06 657.0 42.09 35.38 210
215 2.10580 846.5 0.09468 920.53 2799.32 2.4712 6.3200 4.582 3.237 1.486 1282 504.8 124.5 16.24 653.4 43.11 34.23 215
220 2.31960 840.2 0.08609 943.58 2800.95 2.5177 6.2840 4.615 3.329 1.501 1264 504.8 121.5 16.41 649.7 44.17 33.07 220
225 2.54970 833.7 0.07840 966.80 2802.15 2.5640 6.2483 4.650 3.426 1.518 1246 504.6 118.7 16.59 645.6 45.26 31.90 225
230 2.79710 827.1 0.07150 990.19 2802.90 2.6101 6.2128 4.688 3.528 1.536 1228 504.4 116.0 16.76 641.3 46.38 30.74 230
235 3.06250 820.3 0.06530 1013.77 2803.17 2.6561 6.1775 4.728 3.638 1.556 1209 503.9 113.4 16.94 636.7 47.53 29.57 235
240 3.34690 813.4 0.05970 1037.55 2802.96 2.7020 6.1423 4.772 3.754 1.578 1189 503.3 110.9 17.12 631.8 48.73 28.39 240
245 3.65120 806.2 0.05465 1061.55 2802.22 2.7478 6.1072 4.819 3.878 1.601 1169 502.6 108.4 17.31 626.7 49.97 27.22 245
250 3.97620 798.9 0.05008 1085.77 2800.93 2.7935 6.0721 4.870 4.011 1.627 1148 501.6 106.1 17.49 621.2 51.26 26.04 250
255 4.32290 791.4 0.04594 1110.23 2799.07 2.8392 6.0369 4.925 4.153 1.655 1127 500.5 103.9 17.68 615.4 52.61 24.87 255
260 4.69230 783.6 0.04217 1134.96 2796.60 2.8849 6.0016 4.986 4.308 1.686 1105 499.2 101.7 17.88 609.2 54.03 23.69 260
265 5.08530 775.7 0.03875 1159.96 2793.49 2.9307 5.9661 5.051 4.475 1.720 1083 497.7 99.6 18.07 602.8 55.53 22.51 265
270 5.50300 767.5 0.03562 1185.27 2789.69 2.9765 5.9304 5.123 4.656 1.757 1060 496.0 97.5 18.28 595.9 57.11 21.34 270
275 5.94640 759.0 0.03277 1210.90 2785.17 3.0224 5.8944 5.202 4.855 1.798 1037 494.1 95.5 18.48 588.7 58.80 20.16 275
280 6.41660 750.3 0.03015 1236.88 2779.87 3.0685 5.8579 5.289 5.073 1.845 1013 491.9 93.5 18.70 581.1 60.61 18.99 280
285 6.91470 741.3 0.02776 1263.25 2773.73 3.1147 5.8209 5.385 5.314 1.896 988 489.5 91.6 18.92 573.2 62.57 17.83 285
290 7.44180 731.9 0.02555 1290.03 2766.70 3.1612 5.7834 5.493 5.582 1.954 962 486.9 89.7 19.15 565.0 64.71 16.66 290
295 7.99910 722.2 0.02353 1317.27 2758.70 3.2080 5.7451 5.614 5.882 2.019 936 483.9 87.8 19.40 556.3 67.05 15.51 295
300 8.58790 712.1 0.02166 1345.01 2749.64 3.2552 5.7059 5.750 6.220 2.094 909 480.7 85.9 19.65 547.4 69.65 14.36 300
310 9.86510 690.7 0.01833 1402.22 2727.95 3.3510 5.6244 6.085 7.045 2.277 853 473.3 82.2 20.21 528.7 75.84 12.09 310
320 11.28430 667.1 0.01547 1462.22 2700.59 3.4494 5.5372 6.537 8.159 2.528 793 464.4 78.4 20.85 509.2 83.91 9.86 320
330 12.85810 640.8 0.01298 1525.87 2666.03 3.5518 5.4422 7.186 9.753 2.889 729 453.7 74.5 21.61 489.1 94.94 7.70 330
340 14.60070 610.7 0.01078 1594.53 2621.85 3.6601 5.3356 8.210 12.24 3.45 658 440.7 70.4 22.55 468.5 110.91 5.63 340
350 16.52940 574.7 0.00880 1670.89 2563.64 3.7784 5.2110 10.120 16.69 4.46 578 424.4 65.9 23.82 447.4 135.95 3.67 350
360 18.66600 527.6 0.00695 1761.66 2481.49 3.9167 5.0536 15.000 27.36 6.83 480 402.4 60.3 25.72 425.7 181.51 1.88 360
370 21.04360 451.4 0.00495 1890.69 2334.52 4.1112 4.8012 45.160 96.60 21.15 360 362.8 52.1 29.68 425.0 323.84 0.39 370
373.95c 22.06400 322.0 0.00311 2084.26 2084.26 4.4070 4.4070    0 0.0 — —   0.00 373.95
*Temperatures on ITS-90 scale a Triple point b Normal boiling point c Critical point
Fundamentals of Refrigeration SI, Second Edition 85

Figure 4-1 Sling psychrometer.

humidity ratio: defined as W, the ratio of water vapor mass to the dry air mass
in a given quantity of moist air. Thus, W = humidity ratio = mw/ma, where mw is
the water-vapor mass and ma is the dry-air mass.
relative humidity: defined as , the ratio of the actual water-vapor pressure,
pw , in the moist air to the saturated water-vapor pressure, pws, for the same tem-
perature. Thus, relative humidity =  = pw / pws.
dew-point temperature: temperature, Td, of moist air that is saturated at the
same pressure and humidity ratio as that for the given sample of moist air.
specific volume of moist air: defined as the moist-air-mixture volume per unit
mass of dry air. The ideal gas equation of state gives
86 Chapter 4 Evaporators

Ra T
v = -------------------
- (4-5)
 p – pw 

where v is the specific volume (m3/kg dry air), Ra = R/Ma is the gas constant
for air, and Ma is the molecular weight of air (28.97 kg/kmol).
enthalpy: sum of the enthalpies for dry air and water vapor that comprise the
moist air. Thus, h = ha + Whg, where hg is the enthalpy for the saturated water
vapor at the temperature of moist air.
The thermodynamic properties in Table 4-1 can be combined into a single
plot called the psychrometric chart. Two examples of such a chart are given in
Figures 4-2 and 4-3 for above-freezing and below-freezing water temperatures,
respectively.
The dry-bulb temperature is on the x-axis, and the humidity ratio, W, is on
the y-axis located on the right side of the chart. Inside these two boundaries are
several families of lines. Constant relative-humidity lines curve up toward the
right and are labeled as percentages. Lines of constant wet-bulb temperature
are straight lines moving up toward the left.
The enthalpy scale is identified above the chart, and straight lines of con-
stant enthalpy penetrate the chart and down toward the right. Notice that the
slopes of the wet-bulb lines on the low-temperature chart shift at 0°C due to
freezing of the water. Finally, lines of constant specific volume slope sharply
up to the left.
The two psychrometric charts given here are valid only for sea level (baro-
metric pressure = 76 cm mercury). For other atmospheric pressures corre-
sponding to higher altitudes, other charts, or the defining equations for moist-
air thermodynamic properties, must be used.

Example 4-1

Problem Moist air at sea level has a dry-bulb temperature of 29°C and a wet-bulb
temperature of 14°C as measured by a sling psychrometer. Using the psy-
chrometric chart, determine the following thermodynamic properties:
humidity ratio, enthalpy, specific volume, and the dew-point temperature for
the moist air.

Solution Use the psychrometric chart in Figure 4-2. Locate 29°C dry-bulb temperature
on the horizontal axis. Move vertically up to cross the 14°C wet-bulb tempera-
ture line (slopes downward toward the right). The intersection determines the
state of the moist air. The following properties are read from the psychrometric
chart:
humidity ratio: W  0.0044 kg moisture/kg dry air
enthalpy: h  40 kJ/kg
Fundamentals of Refrigeration SI, Second Edition

Figure 4-2 Psychrometric chart for above-freezing temperatures.


87
88
Chapter 4 Evaporators

Figure 4-3 Psychrometric chart for below-freezing temperatures.


Fundamentals of Refrigeration SI, Second Edition 89

specific volume: v  0.86 m3/kg dry air


dew-point temperature: Td  1.5°C

Note that Td  Twet-bulb .

Example 4-2

Problem An aluminum can containing a soft drink at a temperature of 4°C is taken from
a refrigerator to a location outdoors, where the dry-bulb temperature is 24°C
and the relative humidity is 80%. Using the psychrometric chart, explain why
droplets of moisture form on the surface of the can.

Solution The moist air next to the soft drink can is cooled at a constant pressure. The par-
tial pressure of the water vapor thus remains constant as long as condensation
does not occur. When the dew-point temperature is achieved, additional cooling
results in condensation. Using the psychrometric chart for above-freezing tem-
peratures gives Td  20.3°C. Because the can’s surface temperature is less than
the surrounding air’s dew-point temperature, moisture must condense on the can
surface.

When air passes through an evaporator, heat is transferred from the air to the
refrigerant. An example of this cooling process for the air is shown schemati-
cally in Figure 4-4, where state 1 identifies the air inlet condition and state 2
identifies the air outlet condition. The solid line represents the process that the
air undergoes while the heat transfer rate is given by

·
Q in air = m· h 2 – m· h 1 (4-6)

or, multiplying by –1,

· ·
– Q in air = Q out air = m·  h 1 – h 2  (4-7)

·
where m· is the mass flow rate of air (kg/s) and Q out air is the heat transfer rate
from the air to the refrigerant.
Figure 4-4 also shows that the air-cooling process may be broken into two
parts: a sensible and a latent portion. The sensible part of the heat transfer
involves a change in the dry-bulb temperature but no change in the moisture
content of the air. In contrast, the latent part involves a change in the moisture
content as seen by the change in the humidity ratio, but the dry-bulb tempera-
ture remains constant.
90 Chapter 4 Evaporators

Figure 4-4 Cooling and dehumidifying of moist air.

In summary,

sensible heat transfer = m·  h j – h 2  = m· c p  T 1 – T 2 


latent heat transfer rate = m·  h 1 – h j  = m·  W 1 – W 2   2257 kJ/kg 

where 2257 kJ/kg is an approximate value for the latent heat of vaporization (the
energy needed to evaporate one pound of saturated liquid water to a saturated
vapor). The process shown in Figure 4-4 is referred to as cooling and dehumidi-
fying, where we see that the temperature, enthalpy, and humidity ratio all
decrease. Another process that occurs with airstreams passing through evapora-
tors is sensible cooling only, where no change in moisture content occurs, and
the process appears on the psychrometric chart as a horizontal line to the left.
Often, the internal surfaces on the air side of an evaporator or coil are wet.
Additionally, we will later learn that the evaporative condenser employs a wet-
ted surface to improve heat transfer. For both of these situations, the straight-
line law provides an important insight: When air is in contact with a wetted or
iced surface, its condition on the psychrometric chart drives toward the satura-
tion line at the temperature of the surface.
Going back to Figure 4-4 and applying the straight-line law where Tw is the
temperature of the wetted surface, we see that this is the direction that the air
cooling and dehumidifying process takes on the psychrometric chart.
Fundamentals of Refrigeration SI, Second Edition 91

Example 4-3

Problem Consider a coil with an inlet face area of 6 m2 that is operating with the follow-
ing air inlet conditions: average face velocity = 2.5 m/s, 1 = 80%, and dry-
bulb temperature = –6°C. The outlet dry-bulb temperature is –12°C and the
temperature of the iced surface in the coil = –15°C. Estimate the relative
humidity of the outlet air and the refrigerating rate for this coil.

Solution Using the psychrometric chart:


State 1: T1 = –6°C, 1 = 80%
Wetted (or iced) surface temperature = Tw = –15°C
Apply the straight-line law connecting state 1 to Tw on the saturated line.
Read at T2 = –12°C, 2  95%
The refrigerating rate is

· A1 V1
Q = m·  h 1 – h 2  = ------------  h 1 – h 2 
v1

From the psychrometric chart: h1  –1.59 kJ/kg, h2  –8.98 kJ/kg, v1 


0.747 m3/kg

·  6 m 2   2.5 m/s -
Q = ---------------------------------------  – 1.59 –  – 8.98    kJ/kg 
 0.747 m 3 /kg 
= 148.4 kJ/s = 148.4 kW

Air-Cooling Evaporators
Applications that involve air cooling typically use evaporators that contain
refrigerant inside the tube and a fan to force or draw the air through the evapora-
tor. As considerable tube length is required to achieve the needed air-cooling
load, the tubes may traverse the evaporator several times. For this reason, air-
cooling evaporators are usually called coils. Some coils may use a secondary
fluid inside the tubes. Our consideration of air coils is limited to those that use a
refrigerant. Evaporators that chill a secondary fluid are discussed in the next
section.
Figure 4-5 is a simple diagram of an air coil, and Figure 4-6 shows the
tubes and fins of an air coil. Several terms are defined below that describe the
important features of a coil:
face area: cross-sectional area at the entrance of the coil for the entering air-
stream.
92 Chapter 4 Evaporators

Figure 4-5 Air coil with finned tubes.

Photograph used with permission of Aerofreeze Systems.

Figure 4-6 The tubes and fins of an air coil.


Fundamentals of Refrigeration SI, Second Edition 93

face velocity: velocity of the air entering the coil, calculated by dividing the
volume flow rate of the air by the face area.
surface area: coil surface area in contact with the air passing through the coil.
number of rows of tubes: the number of tubes in the direction of airflow
through the coil, sometimes referred to as the depth.
The flow of the refrigerant inside the tubes of an air coil is complex. For
situations where the refrigerant enters the coil as a mixture of liquid and vapor
(such as that provided by direct expansion), typical flow patterns, such as that
shown in Figure 4-7a, are likely to exist in a horizontal tube. Liquid first occu-
pies the lower portion of the tube’s interior. The vapor velocity above the liquid
is faster, causing the development of waves on the liquid surface. As evapora-
tion of liquid occurs along the tube, the vapor velocity increases until the liquid
climbs the walls and wets the entire tube surface. This is annular flow.
If the entering refrigerant is all liquid, then the mass flow is greater and the
flow patterns that occur are shown in Figure 4-7b. Liquid slugs may form
where the liquid alternately fills the tube cross section separated by vapor
pockets. It is this general sloshing about and wetting of the tube interior surface
that generates the high convection heat transfer coefficient on the tube side.

Heat Transfer and Overall Heat Transfer Coefficient


Heat transfer from the airstream to the refrigerant inside the tubes begins on the
air side of the coil, continues through the tube wall material, and finally enters

Figure 4-7 Refrigerant flow patterns in horizontal tube evaporator: (a) entering liquid-vapor
refrigerant and (b) entering subcooled liquid refrigerant.
94 Chapter 4 Evaporators

Table 4-2 Analogy between Electricity and Heat Transfer


Quantity Electricity Heat
Flow rate I (ampere) Q (kJ/s or kW)
Potential V (volt) xT(C) 1-  ---------
C-
---------------------- or -----
Resistance R (ohm) kA kA kJ/h

the refrigerant inside the tubes. The air side presents the largest barrier to heat
transfer due to the low convection heat transfer coefficient. To assist our con-
sideration of heat transfer, we introduce an analogy between electricity and
heat transfer, which is summarized in Table 4-2.
In Table 4-2, the potentials V and T are the voltage and temperature dif-
ferences, respectively. Ohm’s Law is given by

I = V
------- (4-8)
R
which gives the relationship between flow rate, potential, and resistance. For
the air coil, we have the following from Figure 4-8:

Air-to-Tube
· T o – T m o
Q = ----------------------
- (4-9)
1 
 -----------
 h o A o

where Ao is the outside tube surface area.

Through Tube Material


· T m o – T m i
Q = --------------------------
- (4-10)
x
 -- A 
 k mean
Ai + Ao
where Amean = ----------------- , which is the average cylindrical area of the tube.
2
Tube Surface-to-Refrigerant
· T m i – T i
Q = --------------------
- (4-11)
1 
 ---------
 h i A i

where Ai is the inside tube surface area.


Because the heat transfer is the same through either side of the tube and the
·
tube itself, all the Q s above must be equal. Combining Equations 4-9, 4-10,
and 4-11 and eliminating the temperatures Tm,o and Tm,i gives
Fundamentals of Refrigeration SI, Second Edition 95

Figure 4-8 Heat transfer through a tube in an air coil.

 
·  1 
Q =  ----------------------------------------------------  T o – T i  (4-12)
1 + ----------------
 ----------- x - + --------- 1 
 h o A o kA mean h i A i

where we note the convenience of eliminating the tube surface temperatures, as


they are difficult to measure.
We now define the overall heat transfer coefficient, U, by the relationship

·
Q = UA  T o – T i  (4-13)

Equation 4-12 for the heat transfer through a coil involves two different
areas, presenting a difficulty. However, realizing that the heat transfer rates
through Ao and Ai are the same, we have,

·
Q = Uo Ao  To – Ti  = Ui Ai  To – Ti  (4-14)
96 Chapter 4 Evaporators

Considering Equations 4-12, 4-13, and 4-14 together gives

1 - = ----------
1 - = -----------
1 + ----------------
x - + ---------
1
------------ (4-15)
Uo Ao Ui Ai h o A o kA mean h i A i

where, again, the analogy to electricity is observed. The total resistance is the
sum of the individual resistances in the heat transfer path,

R total = R air + R tube + R refrigerant (4-16)

Example 4-4

Problem Consider a coil fabricated with several rows of unfinned or bare steel pipe.
The air-side convection heat transfer coefficient is 40.3 W/m2·°C. The refrig-
erant-side convection heat transfer coefficient is 1360 W/m2·°C. The nominal
pipe diameter is 2 cm, Schedule 40; so the tube outside diameter is 2.67 cm
and the inside diameter is 2.1 cm. The thermal conductivity of the steel is 45
W/m·°C. Calculate the inside surface overall coefficient for heat transfer, or U-
value, for this coil.

Solution We know that

1 - = -----------
1 + ----------------
x - + ---------
1
----------
Ui Ai h o A o kA mean h i A i

Multiplying both sides by Ai gives

Ai
1 = ----------- xA i 1-
----- + ----------------
- + ---
Ui h o A o kA mean h i

where the cylindrical area ratios and tube thicknesses are

A D 2.1- = 0.787
------i = ------i = ---------
Ao Do 2.67

Ai Ai Di 2.1
- = ----------------------
------------- - = ------------------------ = -----------------------------
- = 0.88
A mean o + A i D o + D i
A -----------------  -------------------  2.67 + 2.1
------------------------
 2   2   2 

– 2.1- = 0.285 cm
x = 2.67
-----------------------
2
Fundamentals of Refrigeration SI, Second Edition 97

So,

 0.285
-------------  0.88 
1 = 0.787  100  1 - = 0.0203
----- ------------- + ---------------------------------- + -----------
Ui 40.3 45 1360
= 0.0195 + 0.000056 + 0.00074 = 0.02029
U i = 49.3 W/m 2 ·°C

Additionally, the U-value based on the tube outside area is

Ai
U o = U i  ------ =  49.3   0.787  = 38.8 W/m 2 ·°C
 A0 

From this example, we see that the air-side thermal resistance is by far the
largest. The situation remains unchanged, even if the air-side heat transfer were
to double. Certainly, the performance of an air coil depends on the air-side
resistance, and that is where additional attention should be directed.
The answer to reducing the air-side thermal resistance to heat transfer is
revealed by Equation 4-13. Any change that increases the UA product improves
the heat transfer, given a potential temperature difference. Equation 4-15 sug-
gests that UA is increased when either ho or Ao is increased. Focusing first on
improving ho, higher face velocities are needed. Doubling the face velocity
alone increases ho by a factor of about 1.5 but also increases the fan power for
the coil by a factor of eight. As a result, the surface area available for heat
transfer is increased to improve coil performance. Equation 4-15 can be
arranged to give

Ai
1 = ----------- xA i 1-
----- + ----------------
- + --- (4-17)
Ui h o A o kA mean h i

clearly showing that to increase Ui (or decrease 1/Ui), the ratio Ai /Ao must
decrease. Therefore, we wish to increase Ao.
A common approach taken by manufacturers to increase area Ao is to add
extended surfaces, or fins, to the tube. An example of circular fins is shown in
Figure 4-9, where in this situation the fins are stamped from flat sheet metal
and punched for placement over the tube.
Good thermal contact between the fin and the tube is achieved by mechani-
cally or hydraulically expanding the tube against the fin. If the fin was at the same
temperature as the tube outside surface, then the air-side resistance is Ai/hoAo.
Under these conditions, the fin effectiveness is 100%. However, Figure 4-9 sug-
gests that the fin temperature increases at progressive distances away from the
tube. Thus, the fin surface is less than 100% effective.
As a consequence, we identify the fin’s effectiveness or efficiency as f and
note that its value must fall in the range from zero to one. The air-side resistance
98 Chapter 4 Evaporators

Figure 4-9 Circular fins attached to refrigerant tube.

term for the overall heat transfer coefficient is modified with f as shown in the
expression for Ui:

Ai
1 = ---------------- xA i 1-
----- - + ----------------
- + --- (4-18)
Ui h o A o  f kA mean h i

Values for f typically vary from 0.3 to about 0.9. The variation of fin effi-
ciency for circular fins on a tube as a function of fin thickness, fin diameter, fin
material thermal conductivity, and air-side convection heat transfer coefficient
is shown in Figure 4-10. Figure 4-11 suggests an approximation for equivalent
rectangular fins, and Figure 4-12 shows several common flat fin arrangements.
Manufacturers of coils generally have responsibility for the dimensions of the
coil and the fins to optimize overall performance. The refrigeration-system
designer selects suitable coils from the manufacturer’s catalog rather than
designing coils for each application.

Example 4-5

Problem Reconsider Example 4-4, where now circular steel fins are added. Fin spacing
is 0.635 cm, fin thickness is 0.159 cm, and the outer fin diameter is 9.8 cm. If
all other conditions remain the same, what is the overall heat transfer coeffi-
cient based on the inside tube diameter?
Fundamentals of Refrigeration SI, Second Edition 99

Figure 4-10 Efficiency of circular fins.

©Prentice-Hall, Inc. Reprinted with permission.

Figure 4-11 Approximation for equivalent rectangular fins.


100 Chapter 4 Evaporators

Figure 4-12 Fin and tube arrangements.

Solution Using Figure 4-10


xi = inside tube radius = 2.1/2 = 1.05 cm
xb = outside tube radius = 2.67/2 = 1.34 cm
xe = fin radius = 9.8/2 = 4.9 cm
W = fin height from tube outside radius = xe – xb = 4.9 –1.34
= 3.56 cm or 0.0356 m

ho 40.3
- =  0.0356  -------------------------------------
W ------- - = 1.19  1.20
ky b  45   0.000795 

x 4.9
----e- = ---------- = 3.7
xb 1.34

Read from Figure 4-10,

fin efficiency =  f  0.54

The area Ai is given by

A i = 2x i l

where xi is the inside surface tube radius and l is the length of the tube. Defin-
ing n as the number of fins per inch, we can determine the area Ao as
Fundamentals of Refrigeration SI, Second Edition 101

 tube outside   tube outside   fin 


A o =  surface area –  surface area  +  surface
     
 without fins   covered by fins  area 

A o = 2x b l – 4x b y b nl + n 2  x e2 – x b2  + 4x e y b l

If l is taken to be 2.5 cm,

2.1
A i =  2      -------  2.5  = 16.5 cm 2
2

A o =  2   1.34   2.5  –  4   1.34   0.0795   1.575   2.5 


+  1.575   2       4.9  2 –     1.34  2  +  1.575      4.9   0.0795   2.5 
= 240.4 cm 2

We now have

1 = ---------------- xA i
A - + ---------------- 1-
----- - + ---
Ui h o A o  f kA mean h i

where the numerical values for the second and third terms are the same as were
determined in Example 3-4. So,

1 = -------------------------------------------------
 16.5 
----- + 0.000056061 + 0.00073427
Ui  40.3   240.4   0.54 
U i = 253.5 W/m 2 ·C

Additionally, we may summarize that an area ratio

A o original = 2x b l =  2   1.34 cm   2.5 cm  = 21 cm 2

A o new
- = 253.5
----------------------- ------------- = 12.1  12
A o original 21

yields an improvement in the U-value by the ratio

U i new
- = 253.5
----------------------- ------------- = 5.1  5
U i original 49.3

indicating that the fin area is not as effective transferring heat as is the tube
surface.
102 Chapter 4 Evaporators

Configuration of Air Coils

Air coils are manufactured in three basic configurations, depending on the


method of refrigerant feed. Figure 4-13 shows schematic diagrams for the three
basic types of refrigerant metering: direct expansion, flooded, and recirculation.
The direct-expansion (DX) coil shown in Figure 4-13a is sometimes referred
to as a dry coil because it has less liquid refrigerant in the tubes than do the
flooded or recirculation coils. Typically, liquid refrigerant fills about one-fourth
of the DX coil when operating. The expansion valve is controlled by superheat
sensed at the outlet of the coil. As a consequence, a small portion of the inside
surface of the tube is dry, resulting in very little refrigeration. This dry-out region
may occupy 10% to 20% of the DX coil’s internal surface area, ensuring that liq-
uid will not enter the compressor. The DX coil is routinely applied to situations
where a temperature difference of 8°C or 11°C is selected between the air and
refrigerant. Applications include commercial and residential air-conditioning
systems, domestic refrigerators, and automotive air conditioners.
The flooded-coil diagram in Figure 4-13b incorporates a vessel called a
surge drum that supplies the coil with liquid refrigerant. By natural convection,
the refrigerant moves through the coil and, as it does, vapor bubbles form and
grow in size. The existence of vapor in the coil reduces the fluid density, which
causes the liquid in the liquid leg to flow naturally into the coil. Both liquid and
vapor enter the surge drum, whereupon they separate with the vapor entering
the suction line. The liquid (float) control valve maintains a constant liquid
level in the surge drum by admitting liquid equivalent to that vaporized in the
coil. An operating characteristic of the flooded coil is that variations in the
refrigerating load may cause surging—undesirable changes in the liquid level
inside the surge drum.
In contrast to the DX coil, the flooded coil has a completely wetted internal
tube surface. Therefore, if a DX coil and a flooded coil have the same physical
characteristics (namely, heat transfer surface area) and operate at the same con-
ditions, the flooded coil will provide more refrigerating capacity than the DX
coil. This is because a portion of the DX coil is not wetted with refrigerant and
so is not performing refrigeration. If the DX coil in Figure 4-13b had about
15% of its surface dry, then the capacity of the DX coil is approximately 15%
less than the flooded coil. An advantage of the flooded coil is that saturated
vapor (rather than superheated vapor) enters the suction line to the compressor.
This reduces compressor power and discharge temperature while at the same
time extending compressor life. The initial cost of the flooded evaporator, rela-
tive to the DX coil, is higher due to the need for a surge vessel and the associ-
ated piping. The flooded coil requires a larger quantity or inventory of
refrigerant than does the DX coil. Additionally, lubrication oil may accumulate
in the surge drum and low-elevation portions of the coil.
The remaining coil, shown in Figure 4-13, is the recirculation or overfeed
air coil. Here, liquid refrigerant is supplied at a rate greater than that needed to
replace the vaporized refrigerant. A vessel, not shown in Figure 4-13, receives
Fundamentals of Refrigeration SI, Second Edition 103

Figure 4-13 Basic configurations of air coils: (a) direct expansion air coil, (b) flooded air coil
fins, and (c) recirculation air coil with bottom feed of refrigerant.
104 Chapter 4 Evaporators

the mixture of liquid and vapor from the recirculation air coil. Performance is
similar to that of the flooded coil because all the internal tube heat transfer sur-
face is wetted. Additionally, saturated vapor enters the suction line of the
flooded coil. Application of recirculation air coils generally involves multiple
air coils supplied with refrigerant by a vessel and pump located some distance
from the coils. Multiple recirculation coils are needed before costs are compa-
rable to those of flooded coils.
The higher recirculation rate reduces the surging noted with flooded coils
associated with changing loads. Oil accumulation only occurs in the larger sep-
arating vessel, because this is the only location where the refrigerant velocities
are low.

Liquid-Cooling Evaporators

A liquid cooler is an evaporator whose function is to cool a fluid, usually water


or a brine. Examples of brines include mixtures of glycol and water and of cal-
cium chloride and water. Several configurations of liquid coolers are used.
In a DX cooler, the refrigerant evaporates inside the tubes, cooling the
water or brine as shown in Figure 4-14. Uniform refrigerant distribution among
all the tubes is necessary to achieve the needed refrigerating capacity and to
eliminate liquid entering the suction line. One approach achieving good distri-
bution utilizes a distributor consisting of an array of orifices (each connected to
a small tube) that uniformly transports the refrigerant to all the tubes. Baffles
inside the shell direct the cooled fluid in a serpentine path, causing good mix-
ing and a higher velocity perpendicular to the tubes. The shell-and-tube liquid
cooler, due to its design and construction, cannot easily be disassembled for
cleaning.
A variation on the configuration of the DX cooler is the plate heat
exchanger, composed of panels stacked next to each other that resemble a loaf
of sliced bread. Figure 4-15 shows an exploded view of a plate heat exchanger
where the panels have deformations that generate many small channels for the
liquid and refrigerant to pass through. This configuration provides a large heat-
transfer surface area in a relatively small volume. Relatively easy disassembly
of the plate heat exchanger permits cleaning of the plates.
A flooded cooler, shown in Figure 4-16, has the refrigerant on the outside
of the tubes so vaporization occurs on the shell side. Refrigerant enters the bot-
tom of the shell and is distributed over the bottom, achieving uniform flow
upward across the tubes. Boiling occurs on the tube surfaces, cooling the fluid,
and vapor rises to the upper portion of the shell. Notice that the tubes do not
occupy this upper volume. The liquid refrigerant is fed to the cooler so that the
top row of tubes is just covered by liquid. The vapor space above affords sepa-
ration of the refrigerant droplets and vapor by the action of gravity.
The quantity of refrigerant contained within the flooded cooler is consider-
able. A modification to the flooded cooler that reduces refrigerant charge is the
Fundamentals of Refrigeration SI, Second Edition 105

Figure 4-14 Direct expansion shell-and-tube liquid cooler.

©APV Canada Inc. Reprinted with permission.

Figure 4-15 Plate heat exchanger.

spray-type shell-and-tube cooler. A pump supplies the liquid refrigerant from


the shell’s bottom to an array of spray nozzles keeping all the tube surfaces wet.
Figure 4-17 shows a simple shell-and-coil cooler where the refrigerant liquid
enters the coiled tube and exits as a vapor. The cooled fluid may be stored for a
given length of time and, therefore, kept available to meet large intermittent
loads. Typical applications, such as the cooling of drinking water, are character-
ized by low storage capacities. A simplified analysis of a liquid cooler is possible
by examining the refrigerant: it enters as a saturated liquid, passes through the
106 Chapter 4 Evaporators

Figure 4-16 Flooded shell-and-tube liquid cooler.

Figure 4-17 Shell-and-coil liquid cooler.


Fundamentals of Refrigeration SI, Second Edition 107

cooler with no pressure drop, and leaves as a saturated vapor at the same tem-
perature. A further restriction is that the fluid must not undergo any phase change
(no partial freezing). For this situation, the heat transfer rate is given by

·
Q = UAT m (4-19)

where Tm is the log-mean temperature difference between the refrigerant and
fluid. For the simplified liquid cooler, we have

 Te – T1  –  Te – T2 
T m = -------------------------------------------------
- (4-20)
T – T
ln  -----------------
e 1
 T e – T 2

where Te is the evaporating temperature, T1 is the entering fluid temperature,


and T2 is the fluid exiting temperature.

Example 4-6

Problem A flooded cooler is used to chill 22 000 kg/h of water entering at 12°C. The
water exits at 6°C. The temperature of the refrigerant inside the cooler is uni-
form and equal to 2°C. The specific heat of water is 4.2 kJ/kg·°C. The heat
transfer area on the outside of the tubes is 46 m2. Using the simplified analysis,
estimate the value of the overall heat transfer coefficient based on the outside
surface area.

Solution The heat transfer is given by

·
Q = U o A o T m

where the log-mean temperature difference between the refrigerant and fluid is

 Te – T1  –  Te – T2 
T m = -------------------------------------------------
-
 T e – T 1
ln -----------------
 T e – T 2
2 – 12  –  2 – 6 - = – 26.9C
= -----------------------------------------
2 – 12
ln  ---------------
 2–6
108 Chapter 4 Evaporators

Also, for the water stream,


·
Q = m· c pw  T 2 – T 1 
=  22 000 kg/h   4.2 kJ/kg·°C   6 – 12 °C
=  – 554 400 kJ/h   1 h/3600 s 
= –154 kW

So,
·
Q
U o = -----------------
A o T m
 –154 kW  -
= -------------------------------------
 46 m 2   – 6C 
2
= 0.56 kW/m ·°C

The Next Step


Chapter 5 introduces the compressor, the component to which the refrigerant
travels after leaving the evaporator. The compressor increases the pressure of
the refrigerant vapor so that it will condense.

Summary
This chapter discussed the properties of moist air and applied them to psychro-
metric processes, paying close attention to the flow of air through an air coil.
Three configurations of common air coils (direct expansion, flooded, and recir-
culation) were presented, showing the need for fins to achieve good heat trans-
fer performance in all three. Also presented were liquid coolers that have the
objective of chilling liquids and where the shell-and-tube configuration is prev-
alent. In addition, the plate heat exchanger was discussed as an alternative
physical configuration for the liquid cooler. You should now be able to

 define the following quantities associated with psychrometrics: saturated


air, dry-bulb temperature, wet-bulb temperature, humidity ratio, relative
humidity, and dew-point temperature;
 use a psychrometric chart to analyze the process that moist air undergoes
when passing through an air coil;
 describe the three basic configurations of an air coil and the reason for fins
on the air side;
 given values of the convection heat transfer coefficient for the air and refrig-
erant sides of an air coil and the physical dimensions of the coil’s compo-
nents, estimate the overall heat transfer coefficient for the coil; and
 describe three configurations of liquid coolers.
Fundamentals of Refrigeration SI, Second Edition 109

References and Bibliography


ASHRAE. 2009. ASHRAE Handbook—Fundamentals. Atlanta: ASHRAE.
ASHRAE. 1992. Psychrometric Chart 2, Low Temperature—Sea Level.
Atlanta: ASHRAE.
ASHRAE. 2008. ASHRAE Handbook—HVAC Systems and Equipment.
Atlanta: ASHRAE.
Cogan, P. 1993. Plate heat exchangers in refrigeration. Proceedings of the IIAR
15th Annual Meeting, Vancouver, British Columbia.
Howell, R., H. Sauer, and W.J. Coad. 2010. Principles of Heating, Ventilating
and Air Conditioning, 6th ed. Atlanta: ASHRAE.
Kuehn, T., J.W. Ramsey, and J.L. Threlkeld. 1998. Thermal Environmental
Engineering. Upper Saddle River, NJ: Prentice-Hall, Inc.
110 Chapter 4 Evaporators

Skill Development Exercises for Chapter 4


Complete these questions by writing your answers on the worksheets at the back of this book.

4-1 A water pipe whose surface temperature is 7.22°C passes through a basement
where the air temperature is 20°C. What is the maximum relative humidity that
the air may have before condensation occurs on the surface of the pipe?
4-2 An air coil has an inlet face area of 2.6 m2 operating at the following air inlet
conditions: average air velocity = 2.03 m/s; relative humidity = 70%; dry-bulb
temperature = 32.22°C. The outlet dry-bulb temperature is 23.89°C, and the
temperature of the wetted surface inside the coil is 18.33°C. Using the psy-
chrometric chart, find the relative humidity of the air leaving the coil and the
refrigerating rate for this coil at these conditions.
4-3 The fins on an air coil are fabricated from copper sheets 0.07874 cm thick. The
coil’s tubing is also copper with a nominal outside diameter equal to 1.27 cm
(actual outside diameter is 1.5875 cm). The spacing between tubes follows the
pattern shown in Figure 4-11, where the horizontal distance between tube cen-
terlines is 3.81 cm and the vertical distance between centers is 4.7 cm. The air-
side convection heat transfer coefficient is 45.43 W/m2·°C and the thermal con-
ductivity of copper is 1271.93 W/m2·°C. Use Figures 4-10 and 4-11 to estimate
the fin efficiency.
4-4 Explain the physical events that occur with a refrigerant in a flooded air coil
with (1) a sudden decrease in refrigeration load and (2) a sudden increase in
load.
4-5 A drinking-water cooler is designed to supply 6.667°C water at a continuous
flow rate of 3.674 kg/h. A shell-and-coil cooler is used to chill the water. If the
temperature of the supply water is 26.67°C, estimate the product of UA for the
coil. Assume that the refrigerant enters the coil as a saturated liquid and leaves
as a saturated vapor at 1.67°C with no pressure drop.
Compressors
Study Objectives
Chapter 5 begins by recalling the thermodynamic process of gas compression
and the work input required. The concepts of positive displacement and aero-
dynamic gas compression are identified in conjunction with the respective
compressors that utilize these concepts. For the reciprocating compressor
(which is a positive displacement machine), performance characteristics indi-
cating how well it is operating are presented. These include displacement,
compression ratio, volumetric efficiency, and adiabatic efficiency.
The other configurations of positive displacement compressors (rotary
screw, rotary vane, and scroll) are also described. The centrifugal compressor’s
operating principle is shown to be different than that of the positive displace-
ment compressor in that the inertia of the refrigerant itself accomplishes the
compression. After studying Chapter 5, you should be able to

 describe the basic gas compression process, show the process on a p-h dia-
gram, and determine the work of compression;
 describe the gas compression process, displacement, and the compression
ratio for the reciprocating compressor, screw compressor, centrifugal com-
pressor, vane compressor, and scroll compressor;
 define two reciprocating compressor efficiencies and calculate values using
actual performance;
 describe overexpansion and underexpansion in a screw compressor and
why variable volume ratio is used;
 explain the performance characteristics and operating efficiency of the cen-
trifugal compressor; and
 explain the performance characteristics and operating efficiencies of the
rotary vane and scroll compressors.

Instructions
Read the material in Chapter 5. Verify the examples presented in the chapter
with your own calculations. At the end of the chapter, complete the skill devel-
opment exercises without referring to the text. Review those sections of the
chapter as needed to complete the exercises.
112 Chapter 5 Compressors

Introduction
Recall that the compressor is one of the four basic components of the single-stage
vapor-compression refrigeration system. Its function in the vapor-compression
refrigeration cycle is to increase the pressure of the refrigerant vapor from the
evaporator so that it will condense. From the saturation tables (see Appendix A) of
the refrigerant used, we see that the discharge pressure from the compressor must
be sufficiently high such that the corresponding saturation temperature is greater
than that of the environment. Only in this way can the condenser reject heat. Fur-
thermore, the compressor maintains the low-side pressure so that the evaporator
operates as intended.
On the basis of operating principle, compressors fall into two common
types. One type is the positive-displacement compressor, where the refrigerant
vapor enters on the suction side and is geometrically trapped. The reduction in
volume of the trapped refrigerant by virtue of the work applied to the compres-
sor’s internal mechanism accomplishes the compression process. The other
compressor type is the aerodynamic, or more simply, the dynamic compressor.
In these compressors, the high rotating velocity of the vapor produces a centrif-
ugal force that increases the vapor pressure of the refrigerant.
Chapter 2 presented the ideal compressor in which a saturated mixture
enters and a superheated vapor is discharged. With ideal compression, remem-
ber that the path followed is one of constant entropy (as shown in Figure 2-5).
Furthermore, the corresponding change in enthalpy, proceeding from the initial
saturated vapor state to the final superheated state of the discharge vapor, is the
work input needed for one pound of refrigerant. For this special situation
involving the ideal compressor (no heat transfer and no internal friction), the
enthalpy change is the least possible or minimum value for a given change in
refrigerant vapor pressure. Because the entropy is constant in an ideal compres-
sion, the process is called isentropic.

Reciprocating Compressors
The reciprocating compressor, a positive displacement type, is prevalent in
today’s refrigeration industry. Inside the compressor, a piston moves back and
forth in conjunction with the alternate opening and closing of valves and raises
the pressure of the refrigerant vapor. The refrigerant enters at the suction pres-
sure and leaves at the discharge pressure. Reciprocating compressors are man-
ufactured over a wide range of sizes: from 0.75 kW to over several hundred
kilowatts. Compressors may be configured as a single cylinder or as a multicyl-
inder machine where the possible arrangements of the cylinders are in-line,
“V,” “W,” or radial.
Several arrangements of the motor and compressor are available. One
arrangement, the open-type compressor, has a crankshaft penetrating the com-
pressor’s housing, externally coupled to the motor output shaft. A seal is needed
between the crankshaft and housing to stop leakage. Otherwise, refrigerant will
Fundamentals of Refrigeration SI, Second Edition 113

leak out when the pressure inside the compressor is greater than the surrounding
atmospheric pressure. In the situation where the refrigerant pressure is less than
the atmospheric pressure, the seal prevents air from leaking into the compressor.
Another common arrangement encloses the motor and compressor inside a
single container. Here, the cold vapor from the suction side passes over the
motor, keeping it cool. This arrangement, the hermetically sealed compressor,
is prevalent in the small-compressor motor units applied to residential air con-
ditioners, refrigerators, and freezers. The hermetic compressor has only the
suction line, the discharge line, and the electrical power cable passing through
the container or “can.” Consequently, the absence of moving parts generates a
long-term reliable seal. An example of a small hermetically sealed compressor
is shown in Figure 5-1. A modification to the above two arrangements is the
semihermetic compressor (sometimes called the serviceable hermetic), in
which the cylinder heads are accessible for maintenance.

Fundamental Characteristics

The operation of the reciprocating compressor is best shown in Figure 5-2, in


which the cylinder is oriented horizontally so that the position of the piston
directly corresponds to the volume to the left of the piston. The pressure-volume

Figure 5-1 Hermetically sealed reciprocating compressor.


114 Chapter 5 Compressors

©McGraw-Hill. Reprinted with permission.

Figure 5-2 Ideal reciprocating compressor and pressure-volume plot.

diagram above the horizontal cylinder shows the variation of pressure with vol-
ume as the piston moves back and forth inside the cylinder.
We will now examine several characteristics of reciprocating compressors:
pressure ratio, displacement rate, clearance volumetric efficiency, actual volu-
metric efficiency, isentropic efficiency, and coefficient of performance.

Pressure Ratio
Pressure ratio is defined as the discharge or exit pressure divided by the suc-
tion or inlet pressure, where both pressures must be in absolute units (kPa):
Fundamentals of Refrigeration SI, Second Edition 115

p dis
pressure ratio = --------
- (5-1)
p suc

where pdis is the discharge pressure (kPa) and psuc is the suction pressure (psia).
Figure 5-2 shows several possible suction pressures (p1, p2, and p3) a recipro-
cating compressor operating with one discharge pressure.

Displacement Rate
When the piston is moving to the right in Figure 5-2, the suction valve is open and
refrigerant vapor is entering the cylinder. Under ideal conditions, the vapor that
enters is at the same thermodynamic state as it was in the suction line; the specific
volumes are the same. The volume of refrigerant vapor drawn into the cylinder
during one suction stroke (piston moving to the right) is defined as Vd and is the
volume swept by the piston (m3) as it moves to the right. From Figure 5-2, we see
that during the suction stroke, the suction valve is pulled open because the pres-
sure inside the cylinder is less than the pressure in the suction line. We also see,
by using the volumes labeled in Figure 5-2, that

Vd = V3 – Vc (5-2)

where Vd is the compressor’s displacement volume, V3 is the volume above the


piston at bottom-center, and Vc is the clearance volume or the volume above
the piston at top-center. As the compressor operates, the piston reciprocates
inside the cylinder, sweeping out the displacement volume for each revolution
of the crankshaft. The cylinder bore (diameter), B, and piston stroke length, L,
determine the displacement volume for a cylinder. We have


V d = --- B 2 L (5-3)
4
for a single-cylinder compressor. For a multiple-cylinder compressor, we mul-
tiply by the number of cylinders to obtain the compressor’s displacement vol-
ume. The compressor shown in Figure 5-2 is a single-acting compressor in
that one suction stroke occurs for every revolution of the crankshaft. Double-
acting compressors where suction occurs during both piston strokes are not
common in refrigeration today. The displacement rate for a multiple-cylinder
single-acting compressor is given by

· (5-4)
V d = n cyl V d N

where V· d is the displacement rate or the volume swept by the pistons per unit
time (m3/s), ncyl is the number of cylinders, and N is the rotational speed of the
crankshaft (revolutions/min or rpm). For a double-acting compressor, multiply
the right side of Equation 5-4 by 2. Importantly, V· d is the ideal and maximum
volumetric flow rate that could enter the compressor.
116 Chapter 5 Compressors

Clearance Volumetric Efficiency


In Figure 5-2, a plot of the pressure-volume relationship is shown above the hor-
izontal cylinder where the volume above the piston and piston position are noted
to coincide. The left-most position that the piston attains is referred to as top-
center and the right-most position is bottom-center. Consequently, the piston
reciprocates between these two extreme positions and is dependent on the con-
necting rod length and on the throw of the crankshaft. Because the top-center
position of the piston must always offer space for the suction and discharge
valves, there is a small volume above the piston called the clearance volume.
Further considering Figure 5-2 leads to the development of the compres-
sor’s clearance volumetric efficiency. This efficiency, resulting from the exis-
tence of the clearance volume, is defined as the ratio of the compressor’s
volumetric vapor flow rate at suction to the displacement rate.
Looking at the pressure-volume diagram in Figure 5-2, let’s follow the
events through one revolution of the crankshaft. Beginning at bottom-center
when the pressure is p1 and the volume above the piston is V3, the piston starts
moving to the left. As the compressor is ideal, the suction and discharge valves
open or close the instant the pressure in either the suction line or the discharge
line is different from that in the cylinder. Then, when the piston begins moving
to the left, the suction valve closes immediately. The discharge valve remains
closed because the pressure in the discharge line is greater than the pressure in
the cylinder.
The pressure increases as the piston moves left, until the pressure in the
cylinder equals the discharge pressure. At this moment, the discharge valve
opens and the remaining leftward piston movement pushes refrigerant vapor
into the discharge line, and the vapor pressure inside the cylinder is constant at
pressure pdis. The delivery of refrigerant vapor to discharge is complete when
the piston reaches top-center.
Observe that the refrigerant vapor occupying the clearance volume remains
in the cylinder. When the piston begins moving to the right, clearance volume
pressure inside the cylinder is less than that in the discharge line, and the dis-
charge valve immediately closes, preventing any leakage of vapor back into the
cylinder. The pressure decreases further in the cylinder as the piston continues
toward bottom-center.
When the cylinder pressure drops to the suction line pressure, the suction
valve immediately opens (for example at p1). The clearance vapor has now
expanded to volume V1. As the piston completes the rightward movement,
vapor from the suction line enters the cylinder above the piston. When bottom-
center is reached, one revolution of the crankshaft is complete, and the process
repeats itself again.
Keeping the discharge pressure fixed at pdis, the pressure-volume diagram
shows the influence of suction pressure on the operation of the compressor. At a
suction pressure equal to p1, the refrigerant vapor volume that enters the cylin-
der is V3 – V1. Observe that this volume is less than Vd. If the suction pressure is
Fundamentals of Refrigeration SI, Second Edition 117

reduced to p2, the entering vapor volume per crankshaft revolution decreases
and is equal to V3 – V2.
Furthermore, we see that the suction pressure may be sufficiently low so
that during the suction stroke the cylinder pressure never falls below the suc-
tion line pressure and the suction valve does not open. This is shown in Figure
5-2 with the pressure p3. At suction pressure p3, the compressor cannot deliver
refrigerant vapor to the discharge line, and there is a limiting discharge-to-suc-
tion pressure ratio below which the reciprocating compressor cannot pump
refrigerant.
To develop an expression for the clearance volumetric efficiency, we only
need to analyze one revolution of the crankshaft of the reciprocating compres-
sor. The volume of vapor that enters the compressor involves the piston move-
ment toward bottom-center after the suction valve opens. If we select suction
pressure p1, we have for the clearance volumetric efficiency

V3 – V1
 vc = ------------------  100%  (5-5)
V3 – Vc

given as a percentage. Note that the clearance volumetric efficiency varies from
0% to 100%, where 100% corresponds to the condition where the suction and
discharge pressures are equal.
Unfortunately, Equation 5-5 is awkward to apply because V1 is not gener-
ally known. An approximate expression for vc may be developed assuming
the following conditions:

• Refrigerant does not leak past closed discharge and suction valves.
• Refrigerant does not leak past the piston sealing ring.
• No heat transfer occurs from refrigerant to compressor internal metal sur-
faces.
• No pressure drop occurs to refrigerant vapor passing through discharge and
suction valves.

Now, referring to the curve in Figure 5-2 for suction pressure p1, we realize
that the mass of the refrigerant in the cylinder volume above the piston when at
top-center is

V V
m c = -----c = -------c- (5-6)
vc v dis

where Vc is the clearance volume (m3) and vc is the specific volume (m3/kg) of
the refrigerant inside Vc. The above conditions indicate that the specific vol-
ume in the clearance volume is the same as the specific volume of the refriger-
ant in the discharge line, vdis. As the refrigerant vapor expands upon piston
movement toward bottom-center, the vapor mass remains the same inside the
118 Chapter 5 Compressors

cylinder until the suction valve opens. If the point where the suction valve
opens is identified as state 1 in Figure 5-2, the vapor mass is given by

V V1
m 1 = m c = -----1- = --------
- (5-7)
v1 v suc

At state 1, the refrigerant is at the suction condition. Equating the two


expressions for mass gives

V V1
-------c- = --------
- (5-8)
v dis v suc

or

v suc
V 1 = --------- V c (5-9)
v dis

Substitution into Equation 5-5 for vc yields

v suc v suc V c  Vc 
V 3 – --------- V c 1 –  ---------  ------  ----- - 
V3 – V1 v dis v dis V 3 1 v V
= ------------------ = ---------------------------- = ------------------------------------ = --------------- –  ---------  ---------------
suc 3
 vc
V3 – Vc V3 – Vc V V  v dis   V
1 – -----c- 1 – -----c-  1 – -----c-
V3 V3  V 3

where, with more manipulation, we can write

Vc v suc
 vc = 1 +  ------------------  1 – --------
-
 V 3 – V c  v dis 

Defining the reciprocating compressor’s volume ratio as

Vc V
- = -----c-
r vc = ----------------- (5-10)
V3 – Vc Vd

vc becomes

v suc 
 vc = 1 – r vc  --------
- – 1 (5-11)
v dis

The clearance volumetric efficiency vc is useful in explaining the part


of the compressor’s losses that are due to the clearance volume. Moreover,
vc decreases as the pressure ratio increases, much as in an actual compres-
sor. Depending on the compressor's geometry and the suction and discharge
pressures, clearance volumetric efficiency accounts for about a third of all
Fundamentals of Refrigeration SI, Second Edition 119

losses. Heat transfer and friction (difficult to quantify) make up the other
losses.

Example 5-1

Problem A reciprocating compressor (Figure 5-3) composed of two cylinders, horizon-


tally opposed, compresses R-22 entering as a saturated vapor at 4.44°C. At dis-
charge, the refrigerant leaves as a superheated vapor at 37.78°C. The bore is
7.62 cm and the stroke is 6.35 cm for each cylinder. When the piston is at top-
center, the linear dimension above the piston is 0.508 cm The top of the piston
and cylinder are flat. If the compressor is considered ideal and operating at a
crankshaft rotating speed of 1725 rpm, calculate the following quantities: dis-
placement for one cylinder (m3), compressor displacement rate (m3/min), oper-
ating pressure ratio for the compressor, and clearance volumetric efficiency for
the compressor (%).

Solution The displacement for a single cylinder of the compressor is computed using the
geometrical characteristics: B = 7.62 cm; L = 6.35 cm.

Vd   7.62 2 6.35
- = --- B 2 L =  ---  ----------  ---------- = 0.00028958 m 3
--------
cyl. 4  4  100   100 

The displacement rate is given by

·   7.62 2 6.35
V d =  no. cylinders  --- B 2 LN =  2   ---  ----------  ----------  1725  = 1.0 m 3 /min
4  4  100   100 

Figure 5-3 Schematic diagram of the compressor operating conditions in Example 5-1.
120 Chapter 5 Compressors

The pressure ratio is computed by pdis/psuc. The suction pressure obtained


from the R-22 saturation property table is 574 kPa. To determine the discharge
pressure, the ideal compression is isentropic. Because s2 = s1 = 0.922352 kJ/kg·K,
we have a second property at state 2, permitting the calculation of p2. Using the p-
h chart for R-22 and tracing the constant entropy process until T2 = 37.78°C is
reached, we obtain that p2  1069 kPa.
The clearance volumetric efficiency is given by

v suc 
 vc = 1 – r vc  --------
-–1
v 
dis

On a per-cylinder basis, we compute

L c = linear dimension above piston when at top-center

  7.62 2 0.508
V c = --- B 2 L c =  ---  ----------  ------------- = 0.000023167m 3
4 4 100 100

V
r vc = -----c- = 0.000023167
------------------------------- = 0.0800
Vd 0.000028958

v suc = 0.041 m 3 /kg  saturation tables 


v dis  0.0228 m 3 /kg  p-h chart 

and

0.041- – 1 = 0.934
 vc = 1 –  0.0800   ---------------
0.0228 

or vc = 93.4%.

Actual Volumetric Efficiency


The actual volumetric efficiency is defined as the actual refrigerant vapor vol-
ume flow rate at the suction port divided by the compressor’s displacement
rate, Vd, shown as

·
V actual
 va = ---------------
· - (5-12)
Vd
Fundamentals of Refrigeration SI, Second Edition 121

Example 5-2

Problem If the compressor in Example 5-1 has a measured refrigerant flow rate at the
suction port equal to 0.89 m3/min, calculate the actual volumetric efficiency.
·
V actual
Solution  va = ---------------
· -
Vd

m 3 /min = 0.890 = 89%


 va = 0.89
------------------------------
1.0 m 3 /min

Isentropic Efficiency
The isentropic efficiency for a compressor is given by the ratio of the ideal
(isentropic) compression work to the actual compression work, as we have
already studied in Chapter 3.

h 2s – h 1
 c = ------------------ (5-13)
h2 – h1

This efficiency includes the effects of pressure drop across the suction and
discharge valves and the heat transfer to the refrigerant vapor from the com-
pressor’s internal surfaces. Additionally, friction resulting from the rubbing of
mechanical parts (such as piston rings against the cylinder wall) is included in
the isentropic efficiency. However, leakage remains unaccounted for in this
efficiency.

Example 5-3

Problem Consider again the compressor in Examples 5-1 and 5-2. If the compressor
requires 6.71 kW to operate at the given conditions, estimate the isentropic
efficiency of the compressor.

Solution The isentropic efficiency is defined as

h isentropic
 s = --------------------------
-
h actual
122 Chapter 5 Compressors

Intermediate calculations are needed to determine the values of enthalpy:


·
V actual = 0.89 m 3 /min
1
m· actual =  0.89 m 3 /min   -----------------------------
- = 21.7 kg/min
 0.041 m 3 /kg
kW   60 - = -------------------------
6.71   60 - = 18.55 kJ/kg
h actual = -----------------------
·
m actual 21.7
h isentropic  267.5 – 251.94 = 15.56 kJ/kg
 using R-22 saturation tables and p-h chart 
Thus, s = 15.56/18.55 = 0.84 or s = 84%.

Coefficient of Performance
The coefficient of performance (COP) is a measure of how well the refrigera-
tion system uses its input power to produce refrigeration. The COP was first
introduced in Chapter 3 with our study of the refrigeration cycle. It is given by

COP = refrigerating capacity-


----------------------------------------------------------- (5-14)
compressor power
and is dimensionless because the units for capacity and power are the same. We
now refine the definition of COP to accommodate the hermetic compressor by
including the electric motor efficiency:

refrigerating capacity
COP  hermetic  = ------------------------------------------------------------------------------------
- (5-15)
electrical power input to motor
On the other hand, if the compressor is open-drive as with industrial refrig-
eration systems, the COP is

refrigerating capacity
COP  open  = -------------------------------------------------------------------------------------------------------
- (5-16)
power input to compressor drive shaft
where the motor efficiency is excluded.

Configuration Types
Single-acting reciprocating compressors (in which the piston is driven by a
connecting rod and crank assembly) characterize almost all reciprocating com-
pressors in use today. The configuration of a reciprocating compressor is iden-
tified by two features: application of the refrigeration system and the drive
method of the compressor.
Medium-temperature refrigeration systems utilize single-stage compressors
where evaporating temperatures vary from about –17.78°C to 4.44°C. This tem-
Fundamentals of Refrigeration SI, Second Edition 123

perature range includes air-conditioning applications. However, lower evapora-


tor temperatures are achievable with single-stage compressors when
appropriately designed. For low-temperature applications (to –50°C), booster
compressors are used, especially with industrial systems using ammonia or R-
22 as refrigerants. The booster compressor, receiving refrigerant vapor from the
evaporator, partially raises the pressure. A high-stage compressor completes the
compression, achieving the condensing pressure at its discharge.
The separation of compression into two (or more) stages is particularly
important for reciprocating compressors due to their maximum pressure ratio
of 8 or 9. Additionally, integral two-stage compressors are capable of obtaining
the same low evaporating temperatures. Within these compressors, the cylin-
ders are divided into booster and high-stage sections, taking into account the
needed volumetric flows and pressure ratios. Connections between the booster
and high-stage sections permit interstage vapor cooling.
The drive method also characterizes the type of reciprocating compressor.
Open-drive and hermetic configurations (already discussed) represent the two
extremes available. The semihermetic compressor falls between open-drive
and hermetic configurations and offers the possibility of field service in that
the seal is maintained by bolts or other accessible methods. Typical refrigerat-
ing capacities for hermetic and semihermetic reciprocating compressors cover
the range from a small fraction of a kilowatt to approximately 100 kW. These
compressors are common in domestic and commercial refrigeration applica-
tions. Multicylinder open-drive compressors are available in capacities from
about 100 to 875 kW and are used in the industrial refrigeration field.

Accessories and Subcomponents

Several subcomponents are needed for reliable operation of the reciprocating


compressor and the refrigeration system where it is used. Following are some of
the tasks that must be accomplished by these accessories and subcomponents.

Head Cooling
Actual compressor temperatures are considerably higher than the predicted
temperatures based on isentropic compression. This is especially true for
ammonia, where, at low evaporating temperatures, actual discharge tempera-
tures may exceed 163°C. A typical discharge temperature limit on industrial
reciprocating compressors is in the vicinity of 150°C to 163°C, which is based
on the material properties of the relatively thin plates inside the compressor;
beyond a threshold temperature, the plates lose their temper. As a result,
ammonia compressors are provided with water-cooled heads that reduce the
refrigerant discharge temperature. An added benefit of head cooling is the pre-
vention of carbonization (breakdown) of the lubricating oil. Water-cooled
heads may also be used to advantage on compressors using other refrigerants.
124 Chapter 5 Compressors

Capacity Control
A refrigeration system must usually meet a varying refrigerating capacity. For
example, air-conditioning loads are high during hot summer days but are much
lower when outdoor temperatures are moderate. If the refrigeration system
operated constantly at the design capacity, the evaporator temperature would
decline until the system’s capacity matched the evaporator load. For this rea-
son, capacity control is needed.
Several methods are available for capacity control:

• Cycle the compressor ON and OFF to meet the needed capacity. This is usu-
ally applied to small systems.
• Throttle the suction line to regulate the pressure of the vapor between the
evaporator and compressor. Good capacity control is achieved, but this
method is not efficient.
• Return a portion of the discharge vapor to the suction line. This provides
good capacity control but is inefficient and sometimes causes the compres-
sor to operate hot.
• With multicylinder compressors, individual cylinders may be de-activated
by holding open the suction valve while keeping the discharge valve
closed. This is called cylinder unloading.
• Change the compressor operating speed.

Capacity control is also possible by modifying the geometrical dimensions


of the compressor (such as the clearance volume and stroke length), but this is
impractical. Multicylinder compressors (using open suction valves to unload
the corresponding cylinders) allow the vapor to pass back into the suction
header piping during the compression stroke. While variable-speed drive
equipment costs are decreasing (enhancing this approach to capacity control),
cylinder unloading for multicylinder reciprocating compressors is yet the
method of choice. The suction valve is opened by valve-lifting pins actuated by
high-pressure oil or compressor discharge vapor controlled by a solenoid
valve. The vapor moving in and out of the unloaded cylinders results in some
loss of compressor efficiency. However, when the compressor is partially
unloaded, the condensing temperature declines and the evaporator temperature
climbs due to the lower heat transfer rates. This adjustment somewhat compen-
sates for the inefficiencies caused by the unloaded cylinders. Typically, if a
compressor operates at 50% full capacity, then the power input to the compres-
sor is approximately 55% full power.

Lubrication and Oil Cooling


Small compressors obtain good lubrication and cooling of their internal compo-
nents by using a simple splash system. Larger compressors, particularly those used
in industrial refrigeration systems, use forced lubrication. A positive-displacement
pump accepts oil from the crankcase sump and distributes it to bearings, shaft
Fundamentals of Refrigeration SI, Second Edition 125

seals, and cylinder walls. The forced lubrication system needs several supporting
items: oil cooler, crankcase heater, filters, vents, equalizers, and safety cutouts.
Crankcase heaters are used during compressor shutdown to prevent refrigerants
from condensing in the crankcase or dissolving in the oil. Oil lubrication safety
cutouts shut off the compressor due to low oil pressure or high oil temperature.
The oil cooler may be a water-cooled heat exchanger with a recommended water
flow rate from the compressor manufacturer.

Lubrication Oil Separation


Oil separators are used when oil in the refrigeration system deteriorates refrig-
erating capacity. The separator, if needed, is located at the discharge port and
captures oil for return to the crankcase. Oil separation is required for lubricants
that are immiscible in the refrigerant (such as mineral oil in ammonia). In con-
trast, oils miscible in the refrigerant do not require oil separation; rather, these
systems must be designed so that the oil returns with the refrigerant to the com-
pressor. However, even with those refrigerants, oil separation may improve oil
management for a refrigeration system.

Rotary Screw Compressors


The rotary screw compressor is a positive-displacement compressor. As with
the reciprocating compressor, the refrigerant vapor enters the suction port,
becomes trapped inside an internal chamber, is compressed by the shrinking
volume chamber and, when compression is complete, leaves through the dis-
charge port. However, a precise geometrical description of the compression
process in a rotary screw compressor is much more difficult because the pro-
cess is now three-dimensional. In contrast, the compression process in a recip-
rocating machine is simple: the circular cross-section piston moves back and
forth in a straight line inside a circular cross-section cylinder.
Screw compressors are manufactured in two configurations: single screw
and twin screw. Figure 5-4 shows a cut-away view of a single-screw compres-
sor typically applied to refrigeration. A single cylindrical main rotor sur-
rounded by a case is flanked on opposite sides by two gaterotors. The main
rotor is carefully machined to have a number of helical grooves of special
shape or profile. The two gaterotors are also machined to have teeth that fit
inside the rotor’s grooves. A slot in the compressor’s casing permits the gatero-
tors’ teeth to rotate as the main rotor turns. The compressor is driven by apply-
ing power to the main rotor shaft. Figure 5-5 shows and describes the
compression process as it occurs inside a single-screw compressor.
The second configuration is the twin-screw rotary compressor in which two
helical-grooved rotary screws are engaged with each other. Figure 5-6 shows the
cross sections of the twin helical screws, where the male rotor (right) has lobes,
and the female rotor (left) has flutes or gullies. The male rotor, by means of a
shaft, drives the female rotor inside a stationary case. The suction port is located
at the top for the twin-screw compressor in Figure 5-6, and the discharge goes
126 Chapter 5 Compressors

Figure 5-4 Single-screw rotary compressor.

out the bottom. Figure 5-7 shows the compression process in a twin-screw com-
pressor.

Fundamental Characteristics

The screw compressor (single or twin) is a fixed-volume-ratio compressor. We


will now use the term screw compressor to include both the twin- and single-
rotor configurations. What this means is that the geometrical volumes at the
beginning and ending of compression are constant and are not influenced by the
properties of the refrigerant vapor. In other words, it is the shape of the slots
between the helical teeth on the rotor(s) and the shape of the suction and dis-
charge ports that determine the volume ratio to which the refrigerant is com-
pressed. In contrast to the screw compressor, the reciprocating compressor is a
variable-volume-ratio compressor. Recall from Figure 5-2 that, as the evaporat-
ing pressure drops when holding the condensing pressure constant, the location
along the stroke where the suction valve opens varies due to the expansion of
the trapped vapor in the clearance volume. Furthermore, during the compression
stroke, the location where the discharge valve opens depends on the discharge
pressure. As a consequence, the volume that undergoes compression and the
Fundamentals of Refrigeration SI, Second Edition 127

Figure 5-5 Compression process in a single-screw rotary compressor.

volume compression ratio in the reciprocating compressor depend on the suc-


tion-to-discharge pressure ratio as well as its geometrical characteristics.
When operating, the screw compressor delivers refrigerant vapor at a pres-
sure that is dependent only on the suction vapor conditions and the compres-
sion volume ratio, Vi, which is defined as

internal volume when suction port covered


V i = ---------------------------------------------------------------------------------------------------------------------------------- (5-17)
internal volume when discharge port uncovered
128 Chapter 5 Compressors

Figure 5-6 Twin-screw rotary compressor.

Figure 5-7 Compression process in a twin-screw rotary compressor.

The discharge-to-suction pressure ratio is given by

p dis
- = V ik
-------- (5-18)
p suc

where k is the ratio of constant pressure to constant volume specific heat, cp /cv .
Equation 5-18 is only valid when the compression process is isentropic—i.e., no
heat transfer and no friction. When the compression process is not isentropic,
the exponent k is replaced with the exponent n and is not equal to k. The value
for n is 1 for a compression process, where the temperature of the vapor is con-
stant. For actual compressors, the value of n falls between k and 1.
Fundamentals of Refrigeration SI, Second Edition 129

Figure 5-8 Efficiency comparison for a single-screw compressor.

The discharge pressure at the exit of the screw compressor may or may not
match the pressure in the discharge piping of the refrigeration system. If the
two pressures match, then the compressor operates at highest efficiency. If the
pressures do not match, then we have either undercompression or overcom-
pression. Both situations involve an abrupt change to the vapor in achieving the
conditions in the discharge piping. Undercompression (where the discharge
pressure from the compressor is less than that in the piping) results in the vapor
already in the discharge piping suddenly expanding into the compressor’s dis-
charge port. Overcompression occurs when the vapor is compressed to a pres-
sure higher than that in the discharge piping. Again, the vapor suddenly
expands, but now from the discharge port into the piping.
Figures 5-9 and 5-8 show the variation of isentropic efficiency with pres-
sure ratio for several Vi ratios for the single- and twin-screw compressors,
respectively. These plots are restricted to ammonia as the refrigerant, but simi-
lar trends occur with other refrigerants such as R-22. For a specific Vi , the isen-
tropic efficiency displays a maximum value when the discharge port and piping
pressures match. Undercompression, occurring to the right of the maximum,
shows gradually increasing losses as the extent of the pressure difference
increases. To the left of the maximum, observe the result of overcompression: a
130 Chapter 5 Compressors

Figure 5-9 Efficiency comparison for a twin-screw compressor.

severe drop in isentropic efficiency with a growing mismatch in pressure.


Clearly, overcompression generates the higher efficiency loss.

Example 5-4

Problem A rotary twin-screw compressor has a fixed Vi equal to 3.5. The refrigerant is
ammonia and enters the suction port at –6.67°C and 145 kPa. Assuming that
the compression process inside the compressor is isentropic, estimate the dis-
charge pressure (psia) and the discharge-to-suction pressure ratio.

Solution V i = 3.5 ; T suc = – 6.67 C; p suc = 145 kPa

Note in Appendix B, Figure B-6, that suction vapor is slightly superheated.


Corresponding suction properties are

 suc  1.073 kg/m 3 s suc  5.9 kJ/kg·C


1 - = ------------
1 - = 0.932 m 3 /kg
v suc = ---------
 suc 1.073
Fundamentals of Refrigeration SI, Second Edition 131

The volume ratio is

V suc mv suc v suc


V i = ---------
- = -------------
- = --------
-
V dis mv dis v dis

where m is the refrigerant mass being compressed that factors out of the frac-
tion because it is the same at suction and discharge. So

v suc
- = 0.932
v dis = -------- ------------- = 0.266 m 3 /kg
Vi 3.5
 dis = 3.8 kg/m 3

Because sdis = ssuc  5.9 kJ/kg·°C and dis  3.8 kg/m3, we have two proper-
ties that locate the discharge state point. The p-h chart gives pdis  655 kPa.
Compressor pressure ratio (isentropic compression) is

p dis 655
--------
-  --------- = 4.5
p suc 145

and we note that the pressure ratio is not equal to the Vi .

Also plotted in Figures 5-8 and 5-9 is the actual volumetric efficiency
showing its relationship to the Vi and the pressure ratio. The clearance volume
(an important characteristic of the reciprocating compressor) cannot be identi-
fied readily for the screw compressor. Hence, the screw compressor partially
avoids the limitation associated with clearance volume vapor expansion, and so
the volumetric flow rate at suction is less sensitive to the pressure ratio when
compared to the reciprocating compressor.
The isentropic efficiency for the screw compressor is the same as that for
the reciprocating compressor, as given by Equation 5-13. The COP is com-
puted in the same manner as that for the reciprocating compressor. Screw com-
pressors equal in capacity to reciprocating compressors are smaller in size due
to their higher operating speeds. Refrigerating capacities range from 70 to over
3500 kW and are popular in industrial refrigeration.

Accessories and Subcomponents


Several features enhance the operation of the basic screw compressor as
described in the previous section. However, one feature—oil injection—is
essential for good performance of the screw compressor. Other features only
add to the ease with which the screw compressor may be applied to refrigera-
tion systems. Lubricating oil not only lubricates the internal parts of the com-
pressor but also serves in cooling the refrigerant vapor during compression,
sealing the pathways by which leakage would occur, and actuating the capacity
132 Chapter 5 Compressors

control and the variable Vi features. More detailed discussions of these items
follow.

Oil Injection
With the twin-screw compressor, oil is injected from locations very near but
outside of the rotors so that lobes and gullies are lubricated; this is essential
because the male rotor must touch the female rotor to drive it. Most importantly,
the oil also seals the space between the two rotors as well as the space between
each rotor and the case. This sealing is mandatory to attain the high pressure
ratios typical in refrigeration applications. Also, the oil, being in direct contact
with the refrigerant vapor at a lower temperature, cools the refrigerant vapor.
The refrigerant discharge temperature from screw compressors with oil injec-
tion is much lower than that from a comparable reciprocating compressor. How-
ever, the oil must be cooled to maintain its lubrication properties and to prevent
breakdown of the oil. The single-screw compressor relies on lubricating oil in
the same way that the twin-screw compressor does. Additionally, the oil pro-
vides lubrication to the moving parts inside the compressor, serves as the
hydraulic fluid positioning the valves providing capacity control and variable Vi,
and cools the refrigerant vapor during the compression process.

Capacity Control
Control of the screw compressor’s capacity is needed to meet varying refriger-
ation loads. Regulation of capacity is possible by the modulation of compres-
sor drive speed, the throttling of suction, and the adjustment of compressor
displacement. The preferred approach to achieving capacity control with screw
compressors is the use of a slide valve. Figure 5-10 shows a cut-away view of a
twin-screw compressor and a common slide-valve arrangement. At full load,
the slide-valve block is at the extreme left position, thereby preventing any
bypass flow of the vapor back to suction. At part load, the slide moves to the
right, opening the bypass and concurrently reducing the vapor flow capacity.
While other approaches (such as lift valves) may afford capacity control, the
slide valves remain the most efficient because they partially correct the volume
ratio along with the capacity. The part-load efficiency of a screw compressor is,
in general, somewhat less than that of a reciprocating compressor operating at
the same conditions.

Variable Vi
We have seen that the screw compressor with a fixed Vi will deliver only one dis-
charge pressure given a suction pressure. Typical operation of a refrigeration sys-
tem involves the variation of condensing temperature and the related condensing
pressure. For example, during relatively cool days, the condensing pressure may
be allowed to drop, resulting in improved system operating efficiency. As a result
of this operating strategy, the discharge pressure from the screw compressor will
not match the condensing pressure. Variable Vi permits the screw compressor to
Fundamentals of Refrigeration SI, Second Edition 133

Figure 5-10 Twin-screw compressor and slide valve.

change the extent of the volume compression that occurs to match the compres-
sor’s discharge pressure to that in the discharge piping. The variable Vi in a screw
compressor is achieved by adding a second slide to the original slide, thereby
accomplishing capacity control. Figure 5-11 shows a schematic view of the slide
valves used to achieve fixed low Vi, fixed high Vi, and the second slide valve pro-
viding variable Vi. The two slides permit, by their relative positions, the bypass of
vapor back to suction for capacity control and the adjustment of the discharge
port’s geometrical size for variable volume ratio.

Oil Separator
Lubricating oil injected into the compressor contacts the refrigerant vapor. For
applications that cannot tolerate oil entering the remainder of the system, an oil
separator captures the oil and circulates it back after cooling to the compressor.
When the presence of oil on internal surfaces adversely impacts the heat transfer
rates at the evaporator, separation equipment capable of limiting the oil concen-
tration to 5 ppm of refrigerant is normally employed. In low-temperature appli-
cations, two-stage separators with a coalescing-type second stage may be used
where ammonia is the refrigerant or where oil could be trapped in flooded evap-
orators. These same high-performance oil separators may also be applied to
reciprocating compressors.
134 Chapter 5 Compressors

Figure 5-11 Slide valves.

Economizer Port
Both single- and twin-rotor screw compressors may have a secondary suction
port positioned between the main suction and discharge ports. In this manner,
the compressor may operate as a two-stage compressor. Improvements in
both capacity and efficiency are possible with the economizer port due to the
lower vapor quality entering the evaporator. The compressor accepts two
refrigerant vapor streams: the main suction line and another from the flash-
gas removal vessel.
Fundamentals of Refrigeration SI, Second Edition 135

The location of the secondary or side port is determined by the compres-


sor’s manufacturer to meet the required intermediate pressure. However, the
side port is at a fixed location, so the compressor does not have the ability to
maintain a constant intermediate pressure subject to changing suction and dis-
charge pressures. It is important to recall from Chapter 3 that with the use of
separate booster and high-stage compressors, the intermediate pressure can be
maintained when the system is facing variable suction and discharge pressures.
An additional consequence of the economizer port is that the slide valve pro-
ducing the variable capacity feature also influences the intermediate pressure
when moving to control capacity. Typically, at capacities less than about 75%,
the pressure at the side port is about the same as that of the main suction port,
eliminating any improvement from the economizer.

Rotary Vane Compressors

Two configurations encompass the rotary vane compressor: the single-vane, or


rolling-piston, type and the multiple-vane type. Both configurations fall under
the category of positive-displacement compressors. The dominant application
of vane compressors is in domestic refrigerators, freezers, and air-conditioners,
where capacities are limited to about 2 kW. Applications of multiple-vane
compressors occur in the transport field and occasionally in industrial refriger-
ation as booster compressors.
An example of the rolling-piston compressor is shown in cross section in
Figure 5-12, where the vane, located at the top, is spring loaded to maintain
contact with the rolling-piston surface. The vane separates the suction side
from the discharge side of the compressor. Only a discharge valve is needed,
usually a reed type. Note that the rolling piston is mounted eccentrically on the
driving shaft. Thus, the rolling piston, as it rotates inside the case called the
cylinder block, has a point on its circumference that stays nearly in contact
with the internal surface. Furthermore, the contact point sweeps once around
the cylinder block for every revolution of the rolling piston. The displacement
of this compressor is determined from the rolling piston and cylinder block
diameters, DA and DB, respectively. Thus,


V d = --- H  D A2 – D B2  (5-19)
4
where H is the height of the cylinder block. In use, the suction gas enters the
suction port at the compressor and discharges into the compressor housing
shell. As a consequence, heat transfer to the suction vapor is reduced, thereby
improving efficiency and simplifying the lubrication system. The clearance
between the rolling piston and cylinder block is sufficiently small, allowing the
formation of a hydrodynamic oil seal controlling the rate of leakage. These
compressors have good volumetric efficiency due to a very small clearance
volume and the associated small losses from re-expansion.
136 Chapter 5 Compressors

Figure 5-12 Single-vane or rolling piston compressor.

The multiple-vane compressor differs from the rolling-piston type in that


the rotor rotates about its centerline, but the centerlines of the rotor and cylin-
der block are not coincident. Figure 5-13 shows one configuration using eight
vanes where the contact point is fixed at the top of the cylinder block. The
vanes generate eight internal volumes, or cells, that shrink in volume upon
rotation to accomplish the compression. While suction and discharge valves
are not required, check valves are usually employed to prevent oil clogging
during shutdown and reverse rotation. Lubrication of the vanes is needed and,
depending on the application and refrigerant used, oil separation may also be
needed.
Both types of rotary vane compressors exhibit a constant-volume ratio and
so are characterized as fixed-volume-ratio compressors. The calculation meth-
ods applied to the rotary screw compressor also apply to the rotary vane com-
pressor. These compressors are suited to handling large volumetric flow rates
of low-temperature vapor. Refrigerating capacities range from 100 to 700 kW.

Scroll Compressors
The scroll compressor, the newest positive-displacement compressor, has wide
application in the residential, commercial, and automotive fields. The power
input range for scrolls is from about 0.75 to 15 kW. Figures 5-14 and 5-15 are
schematic views of the scroll compressor’s internal parts. To begin, two identi-
cally shaped spiral plates (or scrolls), one rotated 180° from the other, are posi-
Fundamentals of Refrigeration SI, Second Edition 137

Figure 5-13 Multiple-vane rotary compressor.

tioned to fit together as shown in Figure 5-15. The scrolls are open on one side
and fastened to a base plate on the other side. One scroll is stationary while the
other rotates in an orbital fashion. It is this motion that first forms the chambers
at the outer diameters, generating vapor suction between points of contact
between the scrolls; next traps the vapor inside the just-formed chamber; then
shrinks the volume of the chamber, producing compression; and finally
releases the compressed vapor to the discharge port.
Observe in Figure 5-15 the steps in the compression process for a scroll
compressor. Note that the compression begins outside and progresses toward
the center until the discharge port is reached. Figure 5-15 also reveals that two
pockets undergo compression simultaneously and symmetrically around the
centerline of the scrolls. Because the scrolls are inflexible, sophisticated manu-
facturing procedures are used to fabricate them to the needed tolerances to pre-
vent leakage past the contact points. The sealing of the scrolls at the contact
points is essential for good performance of the compressor. Both compliant and
noncompliant designs are available and provide good seals.
Scroll compressors are fixed-volume-ratio machines. The geometry of the
scrolls and the discharge port determine the actual volume compression that
occurs. Consequently, the equations developed for the rotary screw compressor
apply to scroll compressors as well. Capacity control for scroll compressors is
available using variable-speed drives and porting holes in the fixed-scroll mem-
ber to vary the displacement of the compressor. A control device that sequen-
tially opens the ports venting refrigerant vapor to the suction port progressively
138 Chapter 5 Compressors

Figure 5-14 Side view of two scrolls in a scroll compressor.

reduces capacity. Other means of capacity control already mentioned, such as


suction throttling, are suitable.
The performance of a scroll compressor using R-22 as the refrigerant is
shown in Figure 5-16, where the volumetric and isentropic efficiency are given
for a 48.89°C condensing temperature. The scroll compressor has neither
clearance volumes nor suction and discharge valves, which results in good vol-
umetric efficiencies. In air-conditioning applications, discharge-to-suction
pressure ratios in the vicinity of 2.5 to 3.5 provide acceptable losses associated
with undercompression and overcompression. Figure 5-16 suggests that at
these pressure ratios the isentropic compressor efficiency is about 70%, which
is comparable to efficiencies for other compressors. Refrigerating capacities of
the scroll compressor are limited to about 140 kW due to the large physical size
of the scroll.

Centrifugal Compressors
The centrifugal compressor develops the action of compression on the basis of
a principle that involves imparting a high velocity to the refrigerant vapor and
converting this velocity (which is actually kinetic energy) to a pressure. The
compressor types previously discussed trap the vapor in a chamber and geo-
metrically reduce the volume. However, in the centrifugal compressor, the
vapor is never trapped during the compression process.
A common application of the centrifugal compressor is water chilling for a
central air-conditioning system. Centrifugal compressors are used in refrigera-
tion systems that range in capacity from roughly 175 to 8800 kW. Despite the
air-conditioning application where the evaporating temperature is typically
Fundamentals of Refrigeration SI, Second Edition 139

Figure 5-15 Orientation of scrolls and the compression process.

from 4.5°C to 8°C, multistage centrifugal compressors can support evaporating


temperatures down to –100°C. Furthermore, the multistage centrifugal com-
pressor permits the removal of flash vapor between the stages, which in turn
improves the refrigeration cycle efficiency. Now only a portion of the flash
vapor is compressed from the evaporating pressure.
140 Chapter 5 Compressors

Figure 5-16 Performance of scroll compressor using R-22.

The centrifugal compressor consists of an impeller driven to a high rotating


speed. The refrigerant vapor enters the compressor through a set of inlet guide
vanes that direct the flow to the center or eye of the impeller. The impeller has
blades shaped in a manner that channel the refrigerant vapor to flow outward,
by centrifugal force, at ever-increasing speeds. The action of the impeller on
the vapor increases the pressure and the kinetic energy (higher velocity). After
leaving the impeller, the vapor enters the diffuser, where some of the kinetic
energy is converted to pressure. Figure 5-17 shows a cross-section view of the
internal flow passages of a two-stage centrifugal compressor where two impel-
lers accomplish the compression. The suction vapor enters from the left and
passes through the inlet guide vanes before reaching the first impeller. A typi-
cal impeller is shown in Figure 5-18, where the blades are curved backward.
The field of turbomachinery embraces centrifugal pumps and fans of vari-
ous configurations. The impeller in a centrifugal compressor is another exam-
ple of a turbomachinery unit. The performance characteristics of a centrifugal
compressor are tied closely to the operation of the impeller. Figure 5-19 is a
plot (usually called a map in the field of turbomachinery) where the coordi-
nates are carefully selected to conveniently show the important features. Con-
siderable information is shown on this one map and reveals how impeller
Fundamentals of Refrigeration SI, Second Edition 141

Figure 5-17 Two-stage centrifugal compressor with refrigerant vapor entering from the left.

Figure 5-18 Front view of impeller for centrifugal compressor.


142 Chapter 5 Compressors

Figure 5-19 Performance characteristics of a centrifugal compressor.

speed, efficiency, and the surge line are influenced by the discharge-to-suction
pressure ratio and flow rate through the compressor. Note that the lines drawn
on the map represent constant values of impeller rotating speed and efficiency.
Also note that no lines are drawn to the left of the surge line. Isentropic effi-
ciencies for centrifugal compressors typically fall in the vicinity of 70% to
80%.
The simplest expression for the performance of a centrifugal compressor
results from the approximation that the tip speed of the impeller is equal to the
tangential velocity of the refrigerant vapor. This is the condition of very low
refrigerant flow rate. Consequently,

V t = RN
----------- (5-20)
30
Fundamentals of Refrigeration SI, Second Edition 143

where Vt is the tangential velocity (m/s), R is the radius (m) of the impeller at
the vapor exit point, and N is the rotating speed of the impeller (rpm). The
power applied to the impeller is given by

m· V t2
P = ----------- (5-21)
gc J

where P is the power (kJ/s or kW), m· is the refrigerant mass flow rate (kg/s),
gc is a constant equal to 1 kg·m/N·s2, and J is a constant equal to 1 N·m/J, the
conversion between mechanical and thermal energy. Another expression for
the power input to the impeller may be obtained from the isentropic work of
compression

P = m  h s  (5-22)

where hs is the enthalpy change for the isentropic work. The two expressions
for the power must be equal, so

m· V t2
----------- = m·  h s 
gc J

Solving for Vt yields

Vt = g c J  h s  (5-23)

This expression for the impeller tip speed, although inaccurate, does give
adequate estimates for the purpose of comparison.

Example 5-5

Problem Estimate the tip speed of the impeller of a centrifugal compressor needed to
compress three different refrigerants (R-123, R-134a, and R-717) from a satu-
rated vapor at 15.55°C to a condensing pressure corresponding to 37.78°C.

Solution The isentropic compression of R-123 from a saturated vapor at 15.55°C to a


saturated condensing temperature of 37.78°C yields an enthalpy change.
Using the p-h chart in Appendix B for R-123 (Figure B-3) gives

h s  242 kJ/kg – 221 kJ/kg = 21 kJ/kg


144 Chapter 5 Compressors

The estimated impeller tip speed is

kg·m  N·m
 1000 ------------ kJ
Vt = 1 ----------  21 ------ = 145 m/s
 N ·s 2  kJ   kg

The compression of R-134a under the same conditions and using the infor-
mation in Appendix B, Figure B-4, gives

h s  272 – 235 = 37 kJ/kg


Vt =  1000   1   37  = 192 m/s

For compression of R-717 under the same conditions and using Appendix B,
Figure B-6

h s  605 – 512 = 93 kJ/kg


Vt =  1000   1   93  = 305 m/s

Example 5-5 demonstrates that the refrigerant selected for use in a centrifu-
gal compressor influences the diameter of the impeller. The impeller’s tip
speed is limited to approximately 305 m/s; the actual value depends on the
material used for the impeller. Additionally, refrigerants with properties similar
to those of R-123 are preferred because a smaller required tip speed results in a
smaller impeller diameter.
The performance of a centrifugal compressor depends on the refrigerant and
on two dimensions: the diameter at the exit and the width of the flow passages
of the impeller. With the refrigerant selected, the discharge pressure determines
the impeller’s exit diameter because, for a fixed rotating speed provided by the
motor, the impeller’s diameter is adjusted to deliver the needed tip speed. In
other words, higher pressure ratios require higher tip speeds and, thus, larger
impeller diameters. Returning to Example 5-5, we observe the influence of the
selected refrigerant on the impeller tip speed. To illustrate the refrigerant’s
impact, let’s begin with a typical fixed impeller rotating speed of 28,800 rpm
(speed range for centrifugal compressors is from 1800 to 90,000 rpm). Calcula-
tion of the impeller’s diameter is based on the dynamics equation relating rotat-
ing speed and radius to tangential velocity given by
V tang = r impellor 

and

V tang
r impellor = -----------
-

where Vtang is the tangential velocity (m/s) and  is the rotating speed (radians/s).
Fundamentals of Refrigeration SI, Second Edition 145

Example 5-6

Problem Use the results from Example 5-5 to predict the diameters of the impellers at
their exit for the three refrigerants R-123, R-134a, and R-717. The impeller
rotating speed is fixed at 3600 rpm.

Solution Convert the rotating speed to units radians/second (rad/s):

3600 rev 1min 2


 =  ---------------------  ------------  ------------ = 377 rad/s
 min   60 s   1 rev

Compute the impeller exit radius for each refrigerant:

V tang 118 m/s- = 0.31 m


R-123: r impellor = -----------
- = ---------------------
 377 rad/s
193 m/s- = 0.51 m
R-134a: r impellor = ---------------------
377 rad/s
305 m/s- = 0.81 m
R-717: r impellor = ---------------------
377 rad/s

In Table 5-1, we see that the impeller diameter for ammonia (R-717) is
impractical due to its size and that the tip speed is at the mechanical limitation
of 305 m/s. Consequently, refrigerants with thermodynamic properties similar
to R-123 are best suited for centrifugal compressors. However, centrifugal
compressors may be applied to ammonia with the modification of two (or
more) compression stages, therefore decreasing the required tip speeds to
acceptable levels.
The flow passage width of the impeller is the space between the faces chan-
neling the vapor flow through the impeller. This width determines the compres-
sor’s capacity. Increases in capacity result from increases in the passage width.
At small capacities, the low refrigerant flow requires that the flow passages be
very small (to achieve the required tip speed), which in turn generates signifi-

Table 5-1 Impeller Tip Speeds and Diameters for Refrigerants R-123, R-134a, and R-717
Refrigerant Impeller Tip Speed, m/s Impeller Diameter, m
R-123 118 0.64
R-134a 193 1.036
R-717 305 1.615
146 Chapter 5 Compressors

cant frictional losses. As a result, small centrifugal compressors have difficulty


maintaining the operating efficiencies that large machines have. Again, the
need for a refrigerant with a low vapor density (such as R-123) is emphasized;
the impeller width should be as large as possible.
The surge line (first mentioned in conjunction with Figure 5-19) is the
boundary to the left of which no performance data is shown. For a specified
impeller tip speed, beginning at a point near the highest efficiency (point A), a
reduction in refrigeration capacity or load shifts the compressor’s operating
point to the left on the same curve to eventually reach the surge line, point B. A
further reduction in refrigeration capacity propels the operating point to C.
Here, the compressor is operating at a lower pressure ratio due to the impeller’s
backward-curved blade characteristics and the change in flow pattern inside
the passages. The flow pattern changes because the vapor flow separates from
the inside surface, much like that of an airplane wing that is too steep, which
results in a stall.
Going back to point C, the evaporator’s load does not change and continues
to produce refrigerant vapor. The vapor flow rate is sufficient to increase the
evaporating pressure, thereby decreasing the pressure ratio. The compressor
eventually shifts operation back to point A, and the situation repeats again.
This is called surging and may repeat itself every 2 to 5 s. The system experi-
ences a large variation in load at the compressor and motor, accompanied by an
annoying noise. Because steady operating conditions do not exist to the left of
the surge line, no performance curves are provided.
Capacity control for the centrifugal compressor is usually accomplished by
partially closing the inlet guide vanes upstream from the first impeller or vary-
ing the impeller’s rotating speed. While all the methods mentioned before may
be employed to regulate capacity, adjusting condenser pressure and bypassing
the discharge gas are not used because they are inefficient. The inlet guide
vanes provide efficient adjustment of capacity when near the full-open posi-
tion. However, when the guide vanes are almost closed, they behave more like
a throttling valve.

The Next Step


Chapter 6 introduces the condenser, the component in the refrigeration system
that delivers thermal energy to the outdoor environment. The condenser
accepts the superheated refrigerant vapor from the compressor and condenses
the vapor to a liquid. This liquid is then fed to the evaporator, thus completing
the refrigeration cycle.

Summary
This chapter presented the prevalent types of compressors applied to refrigera-
tion systems. The positive-displacement compressors include the reciprocating,
rotary screw, rotary vane, and scroll types. The remaining compressor is the cen-
Fundamentals of Refrigeration SI, Second Edition 147

trifugal compressor, a dynamic compressor, in which the pressure gain is mani-


fested by the conversion of centrifugal force into pressure. The process for
accomplishing compression was identified and, where possible, appropriate rela-
tionships for the estimation of performance characteristics were provided.
Auxiliary equipment was discussed as significant to the operation of the
compressor and the refrigeration system. These included oil lubrication, oil
separation, and capacity control. While the reciprocating compressor was
noted as being a variable-volume-ratio compressor, the rotary screw compres-
sor requires a slide valve to achieve variable-volume-ratio operation. You
should now be able to

 list four positive-displacement compressors and describe the compression


process that occurs in each;
 name one compressor type that does not fall under the positive-displacement
category and describe the manner in which the compression is accom-
plished;
 define the clearance volumetric and isentropic efficiencies for a reciprocat-
ing compressor and, if given the suction and discharge conditions for the
refrigerant vapor, be able to estimate values for these two efficiencies;
 explain why the basic configurations of the single- and twin-screw rotary
compressor are fixed-volume-ratio machines and how the addition of the
slide valve allows both capacity control and the adjustment of the volume
ratio;
 list the available approaches for capacity control and indicate an appropri-
ate method for the rotary screw, rotary vane, scroll, and centrifugal com-
pressor types; and
 explain the important characteristic of a refrigerant that makes it suitable
for use in a centrifugal compressor.

Bibliography
ASHRAE. 2008. ASHRAE Handbook—HVAC Systems and Equipment.
Atlanta: ASHRAE.
Howell, R., H. Sauer, and W.J. Coad. 2010. Principles of Heating, Ventilating
and Air Conditioning, 6th ed. Atlanta: ASHRAE.
Stoecker, W., and J. Jones. 1982. Refrigeration and Air-Conditioning. New
York: McGraw-Hill Publishing Co.
148 Chapter 5 Compressors

Skill Development Exercises for Chapter 5


Complete these questions by writing your answers on the worksheets at the back of this book.

5-1 Describe the suction, compression, and discharge processes for a reciprocating
compressor in conjunction with the pressure-volume diagram. Also, discuss
the two limiting cases where the suction and discharge pressures are equal and
the suction pressure is sufficiently low that the compressor no longer pumps
refrigerant.

5-2 A reciprocating compressor has the following specifications: number of cylin-


ders = 6, bore = 11.43 cm, stroke = 11.43 cm, volume ratio rvc = 0.04, and oper-
ating speed = 1200 rpm. This compressor is planned for use in an ammonia
refrigeration system. When the evaporating saturation temperature is –12.22°C
and the condensing temperature is 35°C, the power required by the compressor
is –73.7 kW, which produces 257.4 kW of refrigeration. For this compressor
operating at these conditions, estimate the following quantities: pressure ratio
across the compressor, clearance volumetric efficiency, actual volumetric effi-
ciency, and isentropic compressor efficiency.

5-3 Consider a reciprocating compressor that consists of eight cylinders with cylin-
der dimensions of 10.16 × 12.065 cm for the bore and stroke, respectively. The
refrigerant is CO2 (R-744), which enters the suction port as a saturated vapor at
–40°C and is discharged at a pressure corresponding to a –17.78°C condensing
pressure. If the operating speed of the compressor is 1500 rpm, determine (1)
compressor displacement rate (cm3/s), (2) refrigerant mass flow rate (kg/min),
(3) the pressure ratio across the compressor, and (4) the ideal (minimum)
power (kW) to drive the compressor.

5-4 Describe the features that result in the reciprocating compressor operating as a
variable-volume-ratio compressor. Compare the reciprocating compressor to
the fixed-volume-ratio rotary twin-screw compressor.

5-5 A rotary twin-screw compressor is used in a single-stage industrial refrigera-


tion system with R-22 as the refrigerant. The evaporating temperature is –15°C
and the condensing temperature is 40.56°C. The exponent in Equation 5-18 is
adjusted to the value of 1.2 to account for heat transfer from the refrigerant
vapor to the lubricating oil. Under the condition of negligible pressure drop in
the suction and discharge piping, estimate the volume ratio, Vi, that is required
for the screw compressor to achieve a matching discharge pressure.

5-6 For a centrifugal compressor, estimate the speed of the impeller tip that is
required to compress saturated R-123 vapor at 4.44°C to a pressure corre-
sponding to a condensing temperature of 37.78°C.
Fundamentals of Refrigeration SI, Second Edition 149

5-7 An R-134a refrigeration system utilizes a compressor where the refrigerating


capacity and power requirement specified by the manufacturer are based on
saturated liquid leaving the condenser and saturated vapor entering the com-
pressor. At conditions of –6.67°C evaporating temperature and 37.78°C con-
densing temperature, predict the percentage of change in refrigerating capacity
and power required if the vapor leaving the evaporator and entering the com-
pressor has –6.67°C of superheat.
Condensers

Study Objectives
Chapter 6 presents the basic convection heat transfer processes concerning the
condensation of the refrigerant vapor discharged from the compressor. Three
configurations of the condenser—air cooled, water cooled, and evaporative
cooled—are discussed in conjunction with the condenser’s operation. The psy-
chrometric chart demonstrates changes undergone by air passing through air-
cooled and evaporative condensers but is not relevant to water-cooled condens-
ers. Noncondensable gases are shown to enter the refrigeration system and
degrade the performance of the condenser. These noncondensable gases must
be purged from the system using special equipment. After studying Chapter 6,
you should be able to

 describe the condensation process that occurs on the refrigerant side of a


condenser;
 describe the operation of the air-cooled condenser and show the air-side pro-
cess on the psychrometric chart;
 describe the operation of the water-cooled condenser;
 describe the operation of the evaporative condenser and show the air-side
processes on the psychrometric chart; and
 explain why purging the noncondensable gases from the refrigeration sys-
tem may be necessary and how to determine if purging is needed.

Instructions
Read the material in Chapter 6. Verify the examples presented in the chapter
with your own calculations. At the end of the chapter, complete the skill devel-
opment exercises without referring to the text. Review those sections of the
chapter as needed to complete the exercises.

Introduction
The condenser takes the superheated vapor from the compressor and first cools
it to the condensing temperature and then condenses it. Only liquid refrigerant
exits from the condenser. The condenser must remove from the refrigeration
system all the heat absorbed by the evaporators and the compression heat
152 Chapter 6 Condensers

added by the compressors. To accomplish this task, the condenser must have
sufficient surface area for the heat transfer to occur. Both the evaporator and
condenser are heat exchangers. In the condenser, the entering refrigerant vapor
is converted to a liquid, thus removing heat (thermal energy) from the system.
The evaporator boils the entering liquid refrigerant, thereby removing heat
(thermal energy) from the cold space. The direction of heat transfer in the con-
denser is opposite to that in the evaporator. We will consider in greater detail
the heat transfer associated with condensation when examining the configura-
tions of condensers.
Along with enough surface area to transfer the heat, the condenser must
have enough internal volume for the superheated refrigerant vapor from the
compressor to undergo desuperheating. Furthermore, the condenser’s volume
must be sufficient to allow for the separation of liquid and vapor. If the volume
for separation is too small, the condenser fills with liquid refrigerant. With this
condition, the volume available for desuperheating the vapor is reduced and the
vapor flow rate to the condenser drops.
The condenser types employed in the refrigeration field are air cooled,
water cooled, and evaporative. The smaller refrigeration systems typical of the
domestic and commercial fields commonly employ air-cooled condensers. For
larger HVAC applications in which a central water-chilling plant is used, the
water-cooled condenser finds application due to its small size; it fits into the
limited space.
However, special consideration must be made in securing the supply of
water to the condenser. For example, a cooling tower that cools water by spray-
ing warm water from the condenser in ambient air is a common choice. In a
large commercial building, the cooling tower is generally located some dis-
tance from the mechanical room to have access to outdoor air. Consequently,
water is pumped between the cooling tower and the water-cooled condenser.
This eliminates long refrigerant piping runs in the building and also minimizes
the system’s refrigerant charge.
Industrial refrigeration utilizes all three condenser types, but the evapora-
tive condenser is preferred in larger refrigeration plants. However, evaporative
condensers are not used with refrigerants suitable for centrifugal compressors
(e.g., R-11, R-123) because of the refrigerants’ large specific volume. The tube
diameters become so large as to be impractical.
As we have seen, the condenser must reject the heat absorbed by the evapo-
rator and the compression heat added by the compressors. A special term, heat
rejection ratio, has been devised to relate the condenser and the evaporator heat
flows:

condenser heat rejection rate, kW


heat rejection ratio = --------------------------------------------------------------------------------------------------- (6-1)
evaporator heat absorption rate, kW

The condenser’s heat transfer rate is the sum of the refrigerating capacity
and the compressor power. With refrigeration systems using open-drive com-
pressors, only the shaft power developed by the motor goes into the refrigerant.
Fundamentals of Refrigeration SI, Second Edition 153

Condensers in refrigeration systems using hermetic compressors have higher


heat rejection ratios compared to those with open compressors. This is because
the heat from the electric motor (caused by electrical resistance heating of the
coil windings) in a hermetic compressor is absorbed by the refrigerant vapor
passing through it.
An additional point to mention is that, for situations in which a reciprocating
compressor has separate head cooling or where a screw compressor uses sepa-
rate oil cooling, the cooling rate supplied by the separate system must be sub-
tracted from the heat the condenser must reject. Figure 6-1 shows typical values
for the heat rejection ratio using R-22 for both hermetic and open compressors.

Adapted from a figure ©McGraw-Hill.

Figure 6-1 Typical values for heat rejection ratio.


154 Chapter 6 Condensers

Condensation Processes

There are two types of condensation, dropwise and film. Dropwise condensation
occurs when liquid collects on the condensing surface in lumps or drops, leaving
a portion of the surface in contact with the vapor. In contrast, film condensation
is characterized by the condensed liquid covering all of the condensing surface.
Dropwise condensation requires a clean surface onto which the droplets may
attach. Compared to film condensation, dropwise condensation results in higher
heat transfer coefficients. Unfortunately, clean surfaces cannot be guaranteed, so
heat transfer calculations regarding condensers are approached conservatively by
using the film condensation heat transfer coefficient.
Consider the situation in which refrigerant condenses on the outside of a
tube. Cool water passing through the inside of the tube maintains the outer tube
surface below the condensing temperature. Thus, refrigerant vapor coming in
contact with the surface condenses onto the surface. After enough liquid has
condensed on the surface, the liquid drains off the tube, eventually falling to
the bottom of the shell. Figure 6-2 presents a cross-section view of a water-
cooled condenser where condensation occurs on the outside of the tube bundle.

©Business News Publishing Co. Reprinted with permission.

Figure 6-2 Water-cooled condenser showing the condensation process.


Fundamentals of Refrigeration SI, Second Edition 155

In 1916, Professor Willhelm Nusselt, using only the basic principles of


thermodynamics and fluid mechanics, developed a relationship for the heat
transfer coefficient for condensation on a cold vertical plate. Later, experiments
performed by other researchers modified Nusselt’s equation to accommodate
condensation on the outside of horizontal tubes arranged in a bundle. The con-
densation heat transfer coefficient for this situation is

g 2 h fg k 3 1  4
h c = 0.64  ---------------------- (6-2)
 tND 
where
hc = condensing heat transfer coefficient, W/m2·°C
g = acceleration due to gravity = 9.81 m/s2
 = condensate liquid density, kg/m3
hfg = latent heat of vaporization, kJ/kg
µ = condensate viscosity, kg/m·h
t = temperature difference between vapor and the tube surface, °C
N = number of tubes in a vertical row
D = tube outside diameter, m

Considering the complexity of the condensation process on the outside sur-


faces of a tube bundle, Equation 6-2 is a remarkably simple result. The influ-
ence of the refrigerant’s properties, as well as the geometry of the tube bundle,
is accounted for in this equation. Typical values (29.44°C condensing tempera-
ture and six tubes of 2.54 cm outside diameter in a vertical row) for the con-
densing heat transfer coefficients for the chlorofluorocarbon refrigerants range
from about 850 to 1135 W/m2·°C; the value for ammonia is 5110 W/m2·°C
because of its high latent heat.
Air-cooled and evaporative condensers carry out condensation on the
inside of the tubes. Condensation inside tubes is even more complex than con-
densation on the outside of tubes. The complexity stems from the different
flow regimes that the refrigerant vapor and liquid can take when flowing
through the tube. At the entrance of the tube, the refrigerant is superheated and
the heat transfer coefficient is relatively small. Despite the superheated condi-
tion of the refrigerant vapor at the entrance, condensation occurs straight away,
resulting in the formation of liquid drops. Farther along the tube, surface con-
densation occurs where the coefficient increases to a peak value and then
decreases due to lower vapor velocities and smaller surface areas available for
condensation.
When the tube is filled with liquid, the coefficient takes on a value corre-
sponding to the liquid flowing in a tube. As condensation occurs, the flow
regimes (geometry of the vapor and liquid streams inside the tube) change,
which in turn affects the heat transfer coefficient. As a consequence, a simple
expression, similar to Equation 6-2 developed by Nusselt, is not possible.
156 Chapter 6 Condensers

However, Figure 6-3 shows the relative variation of the inside tube condensa-
tion heat transfer coefficient.

Air-Cooled Condensers
An air-cooled condenser consists of a tube coil on the outside of which are fas-
tened fins to aid in the transfer of heat. Figure 6-4 depicts a single-row, single-
pass air-cooled condenser and the condensation process inside the tube as it
might occur. A propeller fan may be used to force air through the condenser to
enhance the rate of heat transfer. At the entrance, the entering superheated
vapor undergoes cooling until the saturated condition is reached; this corre-
sponds to the pressure inside the condenser. Further extraction of heat produces
condensation of the refrigerant, forming drops that grow and slide down the
surface to the bottom of the tube. Near the exit, the tube is completely filled
with liquid. Any removal of heat now subcools the liquid.
As the air passes through the condenser, it becomes warmer. Figure 6-5
shows the changes occurring in the air that passes through the condenser on a
representative psychrometric chart. Figure 4-2 in Chapter 4 is a psychrometric
chart for sea-level conditions. Observe that it is the dry-bulb temperature that

©Business News Publishing Co. Reprinted with permission.

Figure 6-3 Relative variation of condensing heat transfer coefficient along the length of the
tube.
Fundamentals of Refrigeration SI, Second Edition 157

©Business News Publishing Co. Reprinted with permission.

Figure 6-4 Single-row, single-pass air-cooled condenser.

Figure 6-5 Air heating process for an air-cooled condenser.


158 Chapter 6 Condensers

drives the heat transfer in the air-cooled condenser. The temperature Ts is the
surface temperature of the tube, while the temperature Ta is the entering air
dry-bulb temperature. The air warms as it moves through the condenser,
thereby removing heat from the refrigerant inside the coil. Because moisture
content of the air remains constant, the air-heating process line is horizontal on
the psychrometric chart. If Ta is greater than Ts, then Ts increases until it is
greater than Ta. Consequently, the condensing temperature (and corresponding
pressure) increases.
With smaller units, air movement is caused by the natural tendency of
warm air to rise, which then draws new, cooler air into and through the con-
denser. If a fan is used in the same application to force the air through the con-
denser, the condenser will be smaller. For example, natural airflow is used with
the condensers for domestic refrigerators. Integral self-contained refrigeration
units using hermetically sealed compressors and a condenser fan are limited to
about 7.5 kW. Beyond this size, it is difficult to locate the entire condensing
surface in front of the fan. However, condensing capacities of 350 kW or more
are achieved with roof-mounted units. These larger condensers are designed
with appropriate heat transfer area and airflow to yield the desired perfor-
mance.
Small air-cooled condensers are manufactured in a number of configura-
tions, several of which are shown in Figure 6-6. The single-row, single-pass
condenser has the least capacity and is composed of one tube bent to produce a
serpentine flow path for the refrigerant. This condenser or coil is applied to
fractional horsepower units, as found in domestic applications. For a given
frontal area, the four-row, four-pass unit has the highest capacity of those
shown in Figure 6-6b as a result of its greater surface available for heat trans-
fer. Multiple-row condensers divide the refrigerant flow among the tubes that
comprise the condenser by using headers, as suggested in Figure 6-6b. This
also enhances the ability of the condenser to handle larger quantities of liquid
in the bottom. The three- and four-row condensers are used in larger refrigera-
tion systems.
Extended surfaces are used in air-cooled condensers for the same reason
they are used in air-cooling evaporators. The air-side resistance severely limits
the heat transfer rate. A larger surface area, resulting from the presence of fins,
improves the condenser’s performance, even though the fin surface tempera-
ture is the same as the outside tube temperature only, where the fin contacts the
tube surface. The fin temperature decreases and approaches the air temperature
at points on the fin more distant from the tube surface.
Let’s now examine more closely the operation of the air-cooled condenser.
As already mentioned, the entrance portion of the tube desuperheats the refrig-
erant vapor. In so doing, the temperature of the refrigerant drops—with or
without a pressure drop. Likewise, subcooling may exist in the liquid leaving
the condenser, which also involves a drop in temperature.
Figure 6-7a shows a plot of the temperature changes that occur in the
refrigerant and the airstream as they pass through the condenser. As shown, no
Fundamentals of Refrigeration SI, Second Edition 159

©Business News Publishing Co. Reprinted with permission.

Figure 6-6 Typical configurations for air-cooled condensers: (a) single-row single-pass and
double-row single-pass air-cooled condensers and (b) double-row double-pass
three-row three-pass, and four-row four-pass air-cooled condensers.

pressure drop occurs in that portion where condensation exists, so the tempera-
ture of the refrigerant is constant. Notice that the air and refrigerant flow
streams move in opposite directions or in counterflow. However, with actual
condensers, the flow paths are not exactly counterflow, as we can see by look-
ing at Figure 6-6. In practice, the temperature of the refrigerant is taken as con-
stant and the changes that occur during desuperheating and during subcooling
are usually ignored, as shown in Figure 6-7b.
With this adjustment, the heat transfer rate is

·
Q = UAT m (6-3)

where
·
Q = heat transfer rate, kJ/h
UA = product of overall heat transfer coefficient and heat transfer area,
kJ/h·°C
To – Ti
T m = ----------------------------------------------------
- = log-mean temperature difference, °C
ln   T c – T i  T c – T o  
Tc = refrigerant condensing temperature, °C
160 Chapter 6 Condensers

Ti = entering air temperature, °C


To = leaving air temperature, °C

This use of the log mean temperature difference is reasonable because the
lower heat transfer coefficient in the desuperheating section is somewhat offset
by the higher temperature difference that exists there.

Water-Cooled Condensers
When introducing condensers, the condensation process inside the water-
cooled condenser, as shown in Figure 6-2, was discussed. Remember that the

©Business News Publishing Co. Reprinted with permission.

Figure 6-7 Actual and idealized temperature distributions in an air-cooled condenser.


Fundamentals of Refrigeration SI, Second Edition 161

condenser consists of a bundle of tubes arranged to fit within a shell. Figure 6-8
shows a side view of the condenser depicting the tubes within which the cool-
ing water flows, with the refrigerant vapor entering the top and the refrigerant
liquid leaving the bottom.
Two approaches are typically used when describing water-cooled or air-
cooled condensers for use in a refrigeration system. The heat rejection ratio is
one method. The heat rejection ratio is the sum of the refrigerating capacity and
the compressor power divided by the refrigerating capacity. As we have already
seen, the heat rejection ratio depends on both the evaporating and condensing
temperatures (Figure 6-1). This definition applies to small reciprocating com-
pressors where head cooling is not needed and where heat transfer to the sur-
roundings is negligible. The heat rejection-ratio method is valid for both water-
cooled and air-cooled condensers. Manufacturers supply numerical values for
the heat rejection ratio along with specific correction factors for evaporating and
condensing temperatures different than those for which the unit is rated. It
should be noted that, when screw compressors are used, the manner in which
the lubricating oil is cooled may influence the heat rejection ratio. When the oil
is cooled by an independent water-cooling circuit, the cooling provided is sub-
tracted from the condenser’s heat transfer rate. Additionally, when a reciprocat-
ing compressor’s heads are cooled by water, the heat removed must be deducted
from the condenser’s load. On the other hand, for screw compressors in which

©Business News Publishing Co. Reprinted with permission.

Figure 6-8 Internal flow arrangement in a water-cooled condenser.


162 Chapter 6 Condensers

the oil is cooled by heat exchange with the refrigerant or by direct injection of
liquid refrigerant, the condenser’s load remains the same when computing the
heat rejection ratio.
The second approach to describing air- and water-cooled condensers uses
Equation 6-3, which is given again below

· To – Ti
Q = UA ----------------------------
Tc – Ti
ln  -----------------
 T c – T o

where the log mean temperature difference is inserted into the equation. The
above expression is approximated by

·
Q =  constant   T o – T i  (6-4)

where the constant results from the performance of the condenser at the evapo-
rating and condensing temperatures.
Recall Figure 6-7b and the discussion of air-cooled condensers where the
temperature of the entering refrigerant vapor and the exiting liquid are treated
as equal. The same approximation may be made with water-cooled condensers.
Consequently, applying the second approach (Equation 6-4) determines the
appropriate value of the constant from the manufacturer’s ratings and makes
adjustments that account for the actual evaporating and condensing tempera-
tures. Furthermore, condenser performance is reduced if the heat transfer sur-
face becomes scaled or fouled.
The water-cooled condenser may be supplied using a pump with water
from a cooling tower’s sump. In a cooling tower, the water spray and the asso-
ciated evaporation removes heat from the water left behind. The cooled water
is then pumped back to the water-cooled condenser. This arrangement is preva-
lent with water chillers used to air condition commercial buildings.

Example 6-1

Problem A manufacturer’s catalog shows the performance of a 0.3048 × 2.74 m long


water-cooled condenser where the water flow rate is 696.5 L/min and the con-
stant is 159,700 kJ/h·°C. The condenser is rated at the following conditions:
29.44°C inlet water temperature and 35°C refrigerant condensing temperature.
Determine the heat rejection rate for this condenser.

Solution We know that

·
Q =  constant   T c – T i 
Fundamentals of Refrigeration SI, Second Edition 163

and that

constant = 159,700 kJ/h·°C; Tc = 35°C; Ti = 29.44°C

so

· kJ
Q =  159,700 -----  C  35 – 29.44 C = 888,000 kJ/h
h

which is the heat rejection rate at the condenser.

Evaporative Condensers
The evaporative condenser may be considered a merger between the cooling
tower and the air-cooled condenser. The refrigerant vapor condenses inside the
tubes, while a water spray driven by an airflow passes over the outside of the
tubes. As with the cooling tower, the evaporation of water into the air transfers
most of the heat from the refrigerant. The ambient wet-bulb temperature gov-
erns the performance of the evaporative condenser.
The evaporative condenser is complex, and its performance cannot be
described, as for air-cooled and water-cooled condensers, by a simple tempera-
ture difference relationship (Equations 6-3 or 6-4). Figure 6-9 is a schematic
diagram of an evaporative condenser. As with all evaporative condensers, the
tubes (or condenser coils) that contain the refrigerant are within the enclosure
and in contact with the airflow and water spray. The airflow through the con-
denser is upward, while the spray is downward, causing a counterflow arrange-
ment. Variables that influence the performance of an evaporative condenser
include ambient wet-bulb temperature, airflow velocity, water-spray flow rate,
refrigerant flow rate, and the geometry of the coil. It should be noted that if the
water is chemically treated in an appropriate manner, the outside tube surfaces
do not develop a scale deposit that fouls the surface.

Air-Side Heat and Mass Transfer


The water and air passing through the evaporative condenser involve the trans-
fer of heat and the evaporation of water. These are psychrometric processes,
and the straight-line law, previously studied in Chapter 4, applies. Remember
that the straight-line law dictates that when air is in contact with water, the path
followed by the air on a psychrometric chart is a straight line directed toward
the saturation line at the temperature of the water. This law explains what hap-
pens to the air when it passes over the coil in an evaporative condenser.
The psychrometric chart in Figure 6-10 shows the normal operating situa-
tion with an evaporative condenser. The temperature of the water in the sump is
164 Chapter 6 Condensers

Figure 6-9 Blow-through evaporative condenser.

identified as Tw. The airflow entering the condenser has dry-bulb and wet-bulb
temperatures that correspond to the ambient conditions. Lines of constant
enthalpy and of constant wet-bulb temperature are, for practical purposes, par-
allel to each other on the psychrometric chart. Point a on the chart shows the
state of the air entering the condenser where the water temperature is greater
than the entering air’s wet-bulb temperature.
According to the straight-line law, the process path on the psychrometric
chart drives toward the water temperature, Tw, as shown. The path implies that
the enthalpy of the air increases. Therefore, the water’s enthalpy must decrease
Fundamentals of Refrigeration SI, Second Edition 165

Figure 6-10 Air-heating process path for an evaporative condenser.

to satisfy the energy balance for the air and water. As a result, the temperature
of the spray water decreases.
As the evaporative condenser operates, the refrigerant condensing tempera-
ture is dependent on the heat transfer rate from the coil. This rate depends on
the water temperature that is in contact with the coil’s surface and its total sur-
face area. Therefore, depending on the wet-bulb temperature, the refrigerant
condensing temperature adjusts up or down with changes in the wet-bulb tem-
perature. If the wet-bulb temperature rises, then the refrigerant’s condensing
temperature also rises, while the unit maintains the same capacity. In contrast,
if the wet-bulb temperature falls, then the condensing temperature falls for the
same capacity. Manufacturers of evaporative condensers supply information
that allows designers and users to adjust the nominal performance ratings
based on different wet-bulb and condensing temperatures.

Configurations and Applications


The evaporative condenser diagram shown in Figure 6-9 is referred to as the
blow-through type. The axial flow fan draws in ambient air and pushes it up
past the condenser coil. A centrifugal fan may also be used if low noise
requirements must be met. An alternative configuration is the draw-through
166 Chapter 6 Condensers

type shown in Figure 6-11. Here, the fan is located at the top of the condenser’s
enclosure and pulls the air up past the coil. Both configurations use a down-
ward water spray to keep the coil outer surface wet with water. Both the blow-
through and the draw-through configurations have float valves placed in the
water sump at the bottom of the units. The float valve provides for the addition
of makeup water needed to replenish the water that evaporates into the airflow
past the coil. Not shown in either figure is a blowdown line and valve directed
down from the water sump. Blowdown—a deliberate, steady removal of a por-
tion of the water from the sump—is necessary to prevent the dissolved mineral
solids collecting in the sump from becoming unacceptably high.
The objective of the condenser’s drain piping is to allow for the movement
of liquid refrigerant from the condenser. In industrial refrigeration systems,

Figure 6-11 Draw-through evaporative condenser.


Fundamentals of Refrigeration SI, Second Edition 167

this is accomplished by appropriate piping from the condenser drain to the


receiver. Here, the piping must be sized so that a small pressure drop occurs
when the condenser is operating and so that an equalizer line vents the vapor
from the receiver. Industrial refrigeration systems generally utilize multiple
evaporative condensers, further complicating the piping requirements for good
draining. With multiple evaporative condensers, refrigerant pressure drops
must be accommodated and are critical to achieve good draining.
An exception to the good condenser draining objective applies to commer-
cial refrigeration systems. For systems that operate all year, the condenser
pressure must be maintained at a sufficiently high pressure so that the thermal
expansion valves pass an adequate flow rate of refrigerant. Supermarkets and
other commercial systems may allow the condenser pressure to float rather
than flood the condenser. However, one method to ensure that the condenser
pressure remains high is to cause liquid to back up into the condenser coil, thus
reducing the condenser’s capacity. This keeps the condenser pressure high.
This chapter presents three types of condensers: the air-cooled condenser,
the water-cooled condenser, and the evaporative condenser. When comparing
these condenser types, several generalizations may be stated as discussed
below.

Air-Cooled Condenser
The air-cooled condenser for a given capacity is likely to have the lowest first
cost and smallest operating and maintenance costs due to the absence of the
circulating water. However, it is the dry-bulb temperature that governs the per-
formance of the air-cooled condenser, so the refrigerant’s condensing tempera-
ture is higher.

Water-Cooled Condenser
The shell-and-tube configuration with the cooling water passing through the
tube side is the most common water-cooled condenser. Typically, the water is
pumped from a cooling tower. The wet-bulb temperature (never higher than
the dry-bulb temperature) drives the heat transfer process, causing the con-
densing temperature to be lower than that provided by the air-cooled con-
denser. Consequently, the water-cooled condenser and the cooling tower
comprise a two-component subsystem operating at a lower effectiveness than
is possible with the evaporative condenser. Thus, the condensing temperature
is somewhat higher than that of the evaporative condenser.

Evaporative Condenser
Because the water is in contact with the condensing coil, the condensing tem-
perature is the lowest of the three temperatures. The condenser is relatively
small due to its configuration. The evaporative condenser is applied primarily
in the industrial refrigeration field, as it provides low condensing temperatures.
168 Chapter 6 Condensers

Low condensing temperature reduces compressor power consumption, which


is particularly important for large systems.

Purging Noncondensable Gases


Air and other noncondensable gases enter the refrigeration system when the
low-side pressure is below atmospheric pressure. This situation exists with suf-
ficiently low evaporating temperatures. For example, the refrigeration system
operates in vacuum pressures for evaporating temperatures below –34°C with
ammonia and –40°C with R-22. Leakage of air may occur through seals, gas-
kets, and uncapped valves. Air also enters the refrigeration system when the
evaporator, piping, compressor, or other components are opened for servicing.
The presence of noncondensable gases inside the refrigeration system eventu-
ally degrades the thermal performance of the condenser. The gases entering
from the low-pressure side migrate and collect in the condenser and in the pip-
ing and vessels immediately downstream from the condenser. The refrigerant
vapor enters the condenser coil from the compressor discharge line at a rela-
tively high velocity. At this location, the vapor and noncondensable gases are
well mixed.
As condensation occurs inside the tube, the noncondensable gases are
pushed toward the tube’s wall, which inhibits the continuation of additional
condensation of the refrigerant. In this way, the noncondensable gases block
the refrigerant vapor from contacting the tube surface and condensing. If con-
siderable noncondensable gas is present, then the performance of the con-
denser is adversely affected.
As condensation proceeds in the condenser coil, the velocity of the vapor
decreases and the concentration of noncondensable gases in the refrigerant
vapor increases. In the vicinity of the condenser coil exit and downstream in
the piping and receiver vessel, the vapor and gas velocity is relatively low. To
return condenser performance to normal levels, the noncondensable gases must
be removed from the refrigeration system. This is known as purging.
A simple check is available to determine if purging is needed. At a location
where the vapor and liquid refrigerant are in equilibrium (such as in a receiver
vessel downstream from the condenser), measure the pressure and temperature.
If the measured pressure is significantly higher than the saturation pressure for
the measured temperature, then purging should be performed. Small systems
may need intermittent purging, but larger systems usually employ purging sys-
tems that operate continuously.

Example 6-2

Problem An ammonia system is suspected of containing noncondensable gases. The fol-


lowing measurements are made at the receiver: 32.22°C and 1200 kPa. The
atmospheric pressure is 100 kPa. Determine if purging should be conducted.
Fundamentals of Refrigeration SI, Second Edition 169

Solution We first determine the absolute pressure of the refrigerant in the receiver:

p  abs  = 1200 kPa (gage) + 100 kPa = 1300 kPa

From the saturation table for ammonia (Appendix A, Table A-7), the satu-
ration pressure corresponding to 32.22°C is 1246 kPa. Because the saturation
pressure is less than the measured pressure at the receiver (1300 kPa), purging
is needed.

Recommended locations for purging include those places on the high-


pressure side where the velocity of the vapor is low or near zero and where
only vapor exists. Examples of purging locations are at the top of the drain
line from the condenser and at the top of the receiver vessel.
Techniques employed for purging involve the transport of the vapor and
noncondensable mixture through piping into a special station where the refrig-
erant may be recovered and the remaining noncondensable gases released. One
method, common with centrifugal-compressor water chillers, first compresses
the purged mixture, then condenses the refrigerant, and finally vents the
enhanced air concentration mixture to the atmosphere. This method is not used
with industrial refrigeration systems.
Another method, prevalent in industrial refrigeration systems, condenses
the refrigerant using a small evaporator and then vents the remaining mixture
to the atmosphere. The advantage of the second method of refrigerated purger
is that a separate compressor is eliminated. Purging of noncondensable gases
from refrigeration systems must comply with the United States Environmental
Protection Agency (EPA) or other government regulations.

The Next Step


Chapter 7 presents important characteristics of refrigerant flow in piping that
connects the components in a refrigeration system. Valves regulating flow, and
pumps developing flow, will also be studied.

Summary
The condenser serves two functions in a refrigeration system: transfer of heat
from the refrigerant to the surroundings and the conversion of the refrigerant
vapor discharged from the compressor to a liquid. For the heat to transfer to the
surroundings, the discharge pressure from the compressor must be sufficiently
high so that the condensing temperature is greater than the temperature of the
surroundings. The heat rejected by the condenser is the energy absorbed by the
evaporator and the energy added by the compressor.
Three types of condensers were considered: air-cooled, water-cooled, and
evaporative condensers. Each plays an important role in refrigeration systems.
The air-cooled condenser is used in relatively small unitary systems where first
170 Chapter 6 Condensers

costs are significant. The dry-bulb temperature governs the performance of the
air-cooled condenser. The water-cooled condenser (typically a shell-and-tube
configuration) is used with central-plant water chillers where a cooling tower
supplies the water. Performance of the evaporative condenser depends on the
wet-bulb temperature, which is always less than the dry-bulb temperature.
Industrial systems use the evaporative condenser so that the condensing tem-
perature is low. After studying Chapter 6, you should be able to

 explain how the condensation process occurs inside the condenser;


 show, using a psychrometric chart, how an air-cooled condenser operates;
 define the heat rejection ratio and explain the importance of the evaporating
and condensing temperatures;
 estimate the product UA for an air-cooled or water-cooled condenser based
on performance conditions of a particular condenser;
 explain the operation of the evaporative condenser and show the air-side
processes on the psychrometric chart;
 explain why the application of the evaporative condenser provides the low-
est condensing temperatures; and
 explain why purging noncondensable gases is required and the procedure
for determining whether purging is needed.

Bibliography
ASHRAE. 2008. ASHRAE Handbook—HVAC Systems and Equipment.
Atlanta: ASHRAE.
King, G. 1986. Basic Refrigeration: Principles, Practice, Operation. Troy, MI:
Business News Publishing Co.
Stoecker, W. 1988. Industrial Refrigeration. Troy, MI: Business News Publish-
ing Co.
Stoecker, W., and J. Jones. 1982. Refrigeration and Air-Conditioning. New
York: McGraw-Hill Publishing Co.
Fundamentals of Refrigeration SI, Second Edition 171

Skill Development Exercises for Chapter 6


Complete these questions by writing your answers on the worksheets at the back of this book.

6-1 Using equations, explain why extended surfaces, or fins, are used with air-
cooled condensers.
6-2 A vapor-compression refrigeration system using R-134a provides a refrigerat-
ing capacity of 10.55 kW. The conditions at the exit of the evaporator are
7.22°C and 241 kPa. The condenser operates at 60°C and 862 kPa. Estimate
the heat rejection ratio for this condenser.
6-3 In a manufacturer’s catalog, a particular air-cooled condenser is noted to have a
refrigerating capacity of 42.2 kW (at the evaporator) when the condensing tem-
perature is 48.89°C and the evaporating temperature is 4.44°C. The air enters
the condenser at 32.22°C and exits at a temperature of 43.33°C. The refrigerant
is R-22. Determine the product UA for this condenser at the given conditions,
and estimate the condensing temperature if the refrigeration load drops to half
its original value, or 21.1 kW. (Hint: Begin with Figure 6-1 and approximate
the heat rejection ratio.)
6-4 Consider again the air-cooled condenser in problem 6-3 above operating at the
same conditions and capacity of 42.2 kW. However, it is now known that the
relative humidity of the air entering the condenser is 60%. Find (1) the change
in enthalpy of the air as it passes through the condenser, (2) the mass flow rate
(kg/min) of the flowing air, and (3) the volumetric flow rate (m3/kg) at the inlet
to the condenser.
6-5 Consider an evaporative condenser starting operation with an ambient wet-bulb
temperature greater than the temperature of the water in the sump. Explain
what happens to the air and water using a psychrometric chart and the operat-
ing temperature of the condenser.
6-6 A water chiller using R-123 is checked to determine whether purging of the
noncondensable gases is needed. Pressure and temperature measurements are
carefully made at a location in the system where the liquid and vapor are in
equilibrium downstream from the condenser. The results are 71.7 kPa and
43.33°C. The barometric pressure is measured using a barometer that indicates
754.9 mm. Should purging be recommended in this situation?
Refrigerant Flow in
Pipes, Valves, and Pumps

Study Objectives
Chapter 7 presents the two modes of flow that a fluid in a pipe may assume:
laminar and turbulent. The use of the Darcy-Weisbach equation and the Moody
chart that yield the friction factor are explained in conjunction with the calcula-
tion of pressure over a length of pipe. The concept of equivalent pipe length is
applied to pipe fittings and valves, which enables the estimation of pressure
drop in these devices. Centrifugal and positive-displacement liquid pump con-
figurations are given along with a discussion of their performance characteris-
tics. The special term net positive suction head (NPSH) is also presented. After
studying Chapter 7, you should be able to

 explain laminar and turbulent flow and give an example of each;


 describe the flow of a fluid in a pipe and, using the Darcy-Weisbach equa-
tion and the Moody chart, calculate the pressure drop over a length of pipe
given the pipe diameter, pipe length, and fluid flow rate;
 estimate the pressure drop between two locations in a piping system that
consists of vertical and horizontal pipe lengths, pipe fittings, and valves;
 describe the operation of the positive-displacement and centrifugal pumps
and applications where each would be suitable; and
 explain NPSH and its importance to centrifugal pump operation.

Instructions
Read the material in Chapter 7. Verify the examples presented in the chapter
with your own calculations. At the end of the chapter, complete the skill devel-
opment exercises without referring to the text. Review again those sections of
the chapter as needed to complete the exercises.

Introduction
Piping in a refrigeration system transports the refrigerant between the compo-
nents. Depending on its location in the system, the refrigerant may be a liquid,
a vapor, or a mixture of liquid and vapor. The amount of pressure drop over a
length of pipe or tube depends on the phase of the refrigerant that passes
174 Chapter 7 Refrigerant Flow in Pipes, Valves, and Pumps

through it. While liquid and vapor flows may be estimated using simple rela-
tionships, the prediction of pressure drop associated with the flow of mixtures
is very complex and is a topic of current research in the refrigeration field. Two
types of fluid flow exist in a pipe: laminar and turbulent. The pressure drop
that occurs over a length of pipe is influenced by the type of flow that the fluid
assumes inside the pipe. Laminar flow is characterized by a smoothness in the
flow. If we consider the fluid as made up of liquid particles, then the flow may
be visualized as the orderly movement of fluid particles in a single direction. In
contrast, turbulent flow exhibits an irregular pattern in the movement of the
fluid particles. While the particles move in bulk in a certain direction, the indi-
vidual particles move about in a random fashion in all directions.

An example that shows both laminar and turbulent flow is the smoke from
a cigarette placed on an ashtray in a room containing still air. Near the ciga-
rette, the smoke moves upward in a smooth column. That portion of the smoke
exhibits laminar flow. However, further up along the smoke column, sudden
turns begin to occur that tend to break up the column. This is turbulent flow.
A special dimensionless parameter, the Reynolds number (Re), is
employed to differentiate laminar and turbulent flows of liquids and vapors. In
pipe flow,

VD
Re = ------------ (7-1)

where
V = superficial velocity of the fluid, m/s
D = inside pipe diameter, m
 = fluid density, kg/m3
µ = fluid viscosity, kg/m·s

The fluid’s density and viscosity are properties of the fluid. The refrigerant
saturated property tables in Appendix A, first introduced in Chapter 2, provide
density and viscosity data over a range of conditions. The superficial fluid
velocity is determined by dividing the volumetric flow rate by the inside cross-
sectional area of the pipe:

Q
V = ----------------- (7-2)
D 2  4

where Q is the volume flow rate of fluid (m3/s) and D is the inside diameter of
the pipe (m). This velocity is called the superficial velocity because it is the aver-
age velocity in the pipe that exists at a cross section of the pipe at a particular
length. For flow inside pipes, if Re is less than 2100, then the flow is laminar. If
Re is greater than 3000, then the flow is turbulent. If Re falls between 2100 and
3000, the flow is referred to as being in the transition regime. In refrigeration sys-
Fundamentals of Refrigeration SI, Second Edition 175

tems, laminar flow seldom occurs. However, in the flow of lubricating oil, which
has a substantially higher viscosity, the flow is likely to be laminar.

Fluid Flow in Pipes


Fluid motion in a pipe is caused by the application of pressure. However, as the
fluid moves, friction against the pipe’s internal surface retards the motion,
which results in a pressure drop along the length of the pipe. In the design of a
refrigeration system, the pipe diameter is selected to meet a specific pressure
drop requirement.
Because refrigeration system power consumption is influenced by the com-
pressor suction and discharge temperatures, the saturation temperature is often
used to indicate the pressure drop. Over a length of pipe transporting a vapor,
the pressure drop that occurs may be converted to a drop in saturation tempera-
ture by noting the corresponding values in the saturation tables. Additionally,
the refrigerant chosen affects the pressure and temperature drops.

Darcy-Weisbach Equation
When the fluid consists of a single phase (either liquid or vapor), the Darcy-
Weisbach equation gives the pressure drop in a circular pipe as a function of
several variables. We have

L V2
p = f  ----  ------  (7-3)
 D  2g

where
p = pressure drop, N/m2
f = friction factor, dimensionless
L = pipe length, m
D = pipe inside diameter, m
 = fluid density, kg/m3
g = constant, 9.81 m/s2

The superficial velocity, V, is the same as that used to evaluate Re. The fric-
tion factor f is a function of two variables, Re and the relative roughness, /D.
Both of these variables are dimensionless. Values for the roughness, , for several
pipe materials are given in Table 7-1.
Numerical values for friction factor are obtained using the chart given in
Figure 7-1. This chart was first published in 1944 by L.F. Moody and is based
on carefully performed experiments. It plots the friction factor against Re along
the x-axis using a family of curves representing variations in the relative rough-
ness, /D. However, the laminar flow region is identified by a single curve, so
176 Chapter 7 Refrigerant Flow in Pipes, Valves, and Pumps

Table 7-1 Effective Roughness of Pipe Internal Surfaces

Pipe Material ε, mm

Smooth brass, copper, plastic pipe 0.00152

Steel 0.046

Galvanized steel 0.15

Cast iron 0.26

Figure 7-1 Variation of the friction factor with the Reynolds number and relative roughness.
Fundamentals of Refrigeration SI, Second Edition 177

relative roughness does not influence the friction factor. Consequently, for lami-
nar flow, a simple equation may be used to calculate the friction factor:
64-
f = ----- (7-4)
Re
However, in refrigeration systems, laminar flow almost never occurs due to
the unacceptably low fluid velocities. A transition region separates laminar and
turbulent flow and is best avoided in practice because pressure drops are high
and predictions are uncertain. Turbulent flow friction factors may be determined
from Figure 7-1 for a range of Reynolds numbers and relative roughness values.

Example 7-1

Problem Calculate the pressure drop for a 30.5 m horizontal length, 100 mm, steel
Schedule 40, ammonia suction line. The mass flow rate is 18.14 kg/min and the
temperature of the saturated vapor is –30°C.

Solution 100 mm line


Schedule 40
Dinside = 10.24 cm = 0.1024 m
Specific volume = v = 0.916 m3/kg
Viscosity = µ = 0.0296 kg/m·h (1 h/3600 s) = 8.22 × 10–6 kg/m·s
Q m· v- = ------------------------------------
18.14   0.916 - = 2018 m/min  1-------------
min
- = 33.6 m/s
V = ---------- = ---------
 2  2    60 s
--- D --- D ---  0.1024  2
4 4  4

Re = VD 33.6   0.1024   1.0917  = 456,952  4.6  10 5


------------ = ----------------------------------------------------------
  8.22  10 – 6 
- = 0.046
--- ------------- = 0.00045
D 102.4

From the Moody chart (Figure 7.1), f  0.018

L V2 30.5  33.6  2
p = f  ----  ------  =  0.018   ---------------- -----------------------  1.0917 
 D  2g  0.1024  2   9.81 
kg f
= 337 --------
m2
337
=  ------------------- = 3.3 kPa
101.325

T = 0.61C (by interpolation using ammonia saturated property table)


178 Chapter 7 Refrigerant Flow in Pipes, Valves, and Pumps

The Darcy-Weisbach equation (Equation 7-3) shows how the refrigerant’s


properties and the dimensions of the pipe affect pressure drop. As expected,
pressure drop is proportional to pipe length: if the pipe length is doubled, then
the pressure drop is doubled. The pressure drop is also observed to be propor-
tional to fluid density. So, for a given velocity and pipe size, low-pressure suc-
tion vapor has a lower pressure drop than high-pressure discharge vapor.
However, the velocity of the flow is also important. For a given rate of vapor
mass flow, discharge lines are smaller than suction lines because the velocity is
much less.
Another important trend is that the pressure drop changes inversely with
the pipe diameter raised to the fifth power. In other words, if the pipe diameter
is doubled, then the pressure drop is reduced by a factor equal to 25, or 32.
Stated another way, if the pipe is reduced to one-half its original diameter, then
the pressure drop increases by a factor of 32. Accordingly, the selection of pipe
sizes is important.
In industrial refrigeration systems, liquid refrigerant may occasionally flow
up in a vertically oriented pipe or riser. Here, the fluid’s static head also plays a
role in the pressure drop. The static head originates from the weight of the fluid
column in the riser and decreases to zero as the top is reached. The pressure
drop due to static head is given by

p static head = h (7-5)

where h is the height of the riser (m) and  is the fluid density (kg/m3). At the
same time, the pressure drop due to friction still exists. Consequently, the sum
of the two pressure drops yields the total pressure drop in a riser. We have

p total = p friction + p static head


L V2 (7-6)
= f  ----  ------  + h
 D  2g

which gives the total pressure drop in the pipe. The static head that results in a
vertical riser carrying a vapor is usually neglected due to the vapor’s relatively
low density. Vapor in a liquid line feeding an expansion valve or float control
valve causes the valve to pass an inadequate quantity of refrigerant flow.

Example 7-2

Problem A liquid line supplying R-22 has a pressure and temperature equal to 724 kPa
(gage) and 15.56°C, respectively, upstream from a vertical riser. The riser
height is 25 ft as shown in Figure 7-2. Does the R-22 liquid flash (partially
vaporize) in the riser before reaching the higher horizontal pipe?

Solution p 1 = 724 kPa (gage) + 101.325 kPa = 825.3 kPa


Fundamentals of Refrigeration SI, Second Edition 179

Figure 7-2 Riser in Example 7-2.

where the atmospheric pressure is assumed to be 101.325 kPa. Neglecting the


additional pressure drop due to friction in the vertical length of pipe, we have

p 2 = p 1 – h

 1227 kg -------f  7.62 m 


 m 3
= 825.3 kPa – -----------------------------------------------
 101.325 kg f /m 2
---------------------------------------
 1 kPa 
= 825.3 kPa – 92.3 kPa
= 733 kPa

The saturation pressure for R-22 at 15.56°C is 802.5 kPa and is greater than
p2. Yes, the R-22 liquid will flash, and it will be a vapor-liquid mixture at state
2. The temperature will drop to 14.1°C, which is the saturation temperature
that corresponds to the pressure of 733 kPa. Consequently, if this line supplies
an expansion valve, the vapor “binds” the valve, thereby reducing the rate of
liquid flow.
Ammonia is less prone to flashing in liquid risers because its liquid density
is less by about a factor of two than that of the halocarbon refrigerants.
A liquid line may also be directed vertically down, whereupon the static
head provides a pressure recovery. Thus, vertically downward flow of liquid
generally provides a pressure increase in the pipe. A special flow condition
180 Chapter 7 Refrigerant Flow in Pipes, Valves, and Pumps

exists when the pipe is sloped downward at an angle such that the pressure
drop due to friction balances the pressure gain from static head recovery. Pipe
slope angles greater than this balance condition cause the liquid to separate
from the top of the pipe. This is sometimes called gravity flow. An example of
a sloped pipe producing gravity flow is the drain line from a single evaporative
condenser unit, where the liquid flows down and the space above allows vapor
to flow up counterflow to the liquid.
A special situation exists in the return line from a recirculated evaporator,
where the flow of a vapor and liquid mixture occurs in a pipe. Remember that,
in a recirculation system (usually applied to large industrial refrigeration sys-
tems), the evaporator is supplied with more liquid refrigerant than will vapor-
ize. Consequently, both liquid and vapor return to the low-pressure receiver.
This flow situation is an example of two-phase flow. These return lines are typ-
ically sized by first selecting a size to accommodate the vapor flow rate alone
and then increasing the selection by one pipe size to handle the flowing liquid.
It is important to recognize that two-phase flow in pipes may substantially
increase the pressure drop over that of the vapor flow alone.

Pressure-Drop Estimation

Charts have been developed to simplify the determination of pressure drop in


lines. Examples are shown in Tables 7-2 and 7-3 for ammonia suction and dis-
charge vapor lines, respectively. Similar charts are available for liquid lines as
well as for other refrigerants.

Example 7-3

Problem Re-examine Example 7-1 and use Table 7-3 to estimate the pressure drop and
drop in suction pressure for a 30.5 m horizontal length of 100 mm Schedule 40
pipe that conveys 18.14 kg/min of ammonia vapor at –30°C.

Solution 1. Table 7-3 gives pressure drops for ammonia suction lines.
2. Enter table on left-hand side and find 100 mm nominal size.
3. Follow 100 mm line over until reaching –30°C saturated suction.
4. Read pressure drop as approximately 116.3 Pa/m and saturation tempera-
ture drop as approximately 0.02 K/m.
5. Multiply P and T by total length of 30.5 m to get P of 3.55 kPa and T
of 0.61 K.
6. Comparing the results to those obtained in Example 7-1 using the Darcy-
Weisbach equation suggests good agreement.
Fundamentals of Refrigeration SI, Second Edition 181

Table 7-2 Suction Line Capacities in Kilowatts for Ammonia


with Pressure Drops of 0.005 and 0.01 K/m Equivalent
Saturated Suction Temperature, °C
Steel –50 –40 –30
Nominal Line
Size, mm t = 0.005 K/m t = 0.01 K/m t = 0.005 K/m t = 0.01 K/m t = 0.005 K/m t = 0.01 K/m
p = 12.1 Pa/m p = 24.2 Pa/m p = 19.2 Pa/m p = 38.4 Pa/m p = 29.1 Pa/m p = 58.2 Pa/m
10 0.19 0.29 0.35 0.51 0.58 0.85
15 0.37 0.55 0.65 0.97 1.09 1.60
20 0.80 1.18 1.41 2.08 2.34 3.41
25 1.55 2.28 2.72 3.97 4.48 6.51
32 2.39 3.51 4.43 6.47 7.66 11.14
40 3.68 5.41 6.85 9.94 11.77 17.08
50 9.74 14.22 16.89 24.50 27.57 39.82
65 15.67 22.83 27.13 39.27 44.17 63.77
80 28.08 40.81 48.36 69.99 78.68 113.30
100 57.95 84.10 99.50 143.84 161.77 232.26
125 105.71 153.05 181.16 261.22 293.12 420.83
150 172.28 248.91 294.74 424.51 476.47 683.18
200 356.67 514.55 609.20 874.62 981.85 1402.03
250 649.99 937.58 1107.64 1589.51 1782.31 2545.46
300 1045.27 1504.96 1777.96 2550.49 2859.98 4081.54
Saturated Suction Temperature, °C
Steel 20 5 +5
Nominal Line
Size, mm t = 0.005 K/m t = 0.01 K/m t = 0.005 K/m t = 0.01 K/m t = 0.005 K/m t = 0.01 K/m
p = 42.2 Pa/m p = 84.4 Pa/m p = 69.2 Pa/m p = 138.3 Pa/m p = 92.6 Pa/m p = 185.3 Pa/m
10 0.91 1.33 1.66 2.41 2.37 3.42
15 1.72 2.50 3.11 4.50 4.42 6.37
20 3.66 5.31 6.61 9.53 9.38 13.46
25 6.98 10.10 12.58 18.09 17.79 25.48
32 12.47 18.03 19.22 28.67 28.32 36.02
40 19.08 27.48 29.45 42.27 43.22 54.88
50 42.72 61.51 76.29 109.28 107.61 153.66
65 68.42 98.23 122.06 174.30 171.62 245.00
80 121.52 174.28 216.15 308.91 304.12 433.79
100 249.45 356.87 442.76 631.24 621.94 885.81
125 452.08 646.25 800.19 1139.74 1124.47 1598.31
150 733.59 1046.77 1296.07 1846.63 1819.59 2590.21
200 1506.11 2149.60 2662.02 3784.58 3735.65 5303.12
250 2731.90 3895.57 4818.22 6851.91 6759.98 9589.56
300 4378.87 6237.23 7714.93 10 973.55 10 810.65 15 360.20
Note: Capacities are in kilowatts of refrigeration resulting in a line friction loss per unit equivalent pipe length (p in Pa/m), with corresponding change in saturation tem-
perature per unit length (t in K/m).
182 Chapter 7 Refrigerant Flow in Pipes, Valves, and Pumps

Table 7-3 Suction, Discharge Line, and Liquid Capacities in Kilowatts for Ammonia
(Single- or High-Stage Applications)
Discharge Lines
Suction Lines (t = 0.02 K/m) t = 0.02 K/m, p = 684.0 Liquid Lines
Steel Pa/m Steel
Nominal Nominal
Saturated Suction Temp.,
Line Saturated Suction Temperature, °C Line
°C
Size, Size, Velocity = p =
mm –30 –20 –5 +5 mm 0.5 m/s 450.0
–40
p = p = p = p = –40 –20 +5
p = 76.9
116.3 168.8 276.6 370.5
10 0.8 1.2 1.9 3.5 4.9 8.0 8.3 8.5 10 3.9 63.8
15 1.4 2.3 3.6 6.5 9.1 14.9 15.3 15.7 15 63.2 118.4
20 3.0 4.9 7.7 13.7 19.3 31.4 32.3 33.2 20 110.9 250.2
25 5.8 9.4 14.6 25.9 36.4 59.4 61.0 62.6 25 179.4 473.4
32 9.5 16.16 25.7 46.4 57.6 96.2 107.0 98.9 32 311.0 978.0
40 14.4 24.60 39.4 60.4 88.2 146.0 163.8 151.4 40 423.4 1469.4
50 35.4 57.2 88.1 155.7 218.6 355.2 364.9 374.7 50 697.8 2840.5
65 56.7 91.6 140.6 248.6 348.9 565.9 581.4 597.0 65 994.8 4524.8
80 101.0 162.4 249.0 439.8 616.9 1001.9 1029.3 1056.9 80 1536.3 8008.8
100 206.9 332.6 509.2 897.8 1258.6 2042.2 2098.2 2154.3 — — —
125 375.2 601.8 902.6 1622.0 2271.4 3682.1 3783.0 3884.2 — — —
150 608.7 975.6 1491.4 2625.4 3672.5 5954.2 6117.4 6281.0 — — —
200 1252.3 2003.3 3056.0 5382.5 7530.4 12 195.3 12 529.7 12 864.8 — — —
250 2271.0 3625.9 5539.9 9733.7 13619.6 22 028.2 22 632.2 23 237.5 — — —
300 3640.5 5813.5 8873.4 15568.9 21787.1 35 239.7 36 206.0 37 174.3 — — —
Notes:
1.Table capacities are in kilowatts of refrigeration. 4. Values are based on 30°C condensing temperature. Multiply table capacities by
the following factors for other condensing temperatures:
p = pressure drop due to line friction, Pa/m
t = corresponding change in saturation temperature, K/m Condensing Suction Discharge
2.Line capacity for other saturation temperatures t and equivalent lengths Le Temperature, °C Lines Lines

t 0.55
20 1.04 0.86
Line capacity = Table capacity  Actual
-----------------------
 Table t  30 1.00 1.00
3.Saturation temperature t for other capacities and equivalent lengths Le 40 0.96 1.24
50 0.91 1.43

 Actual capacity- 1.8


------------------------------------
t = Table t 5. Liquid line capacities based on 5°C suction.
 Table capacity 

Charts and tables have been developed by the industry to further expedite
the tedious task of computing the pressure drop in refrigeration system piping.
Examples of two charts that provide estimates for the required refrigerant mass
flow rates per kW of refrigerating capacity are shown in Figures 7-3 and 7-4
for R-22 and ammonia, respectively. These two plots show that the refrigerant
flow rate depends on the temperature of the liquid supplied to the evaporator’s
feed valve and the evaporator temperature. Furthermore, these figures assume
that the refrigerant leaves the evaporator as a saturated vapor.
The refrigeration industry has also developed tables to assist in the selec-
tion of pipe sizes. Examples are given in Tables 7-2 and 7-3 for ammonia suc-
Fundamentals of Refrigeration SI, Second Edition 183

Figure 7-3 Required R-22 mass flow rate/ton of capacity.

tion and discharge lines. Similar tables are available for other refrigerants. In
these tables, the pipe sizes are listed along the left side, and the different satu-
rated suction temperatures are listed along the top toward the right. Note that
for Table 7-2 under each saturated suction temperature, two levels of pressure
drop are identified, 0.005 K/m and 0.01 K/m. For Table 7-3, line carrying
capacities are listed only for a saturated temperature drop of 0.02 K/m. The
footnotes included with Table 7-3 indicate how pressure drops may be obtained
for conditions other than those on which the table is based.

Example 7-4

Problem Estimate the capacity that the ammonia suction line in Example 7-1 may
accommodate. The evaporating temperature is –30°C, the mass flow rate is
18.14 kg/min, and the Schedule 40 pipe has a nominal diameter of 100 mm.

Solution Use Table 7-3 and note that when saturation temperature is –30°C, the line
capacity is 332.6 kW. From Example 7-1, T = 0.61°C. Apply footnote 2 from
Table 7-3. This gives
184 Chapter 7 Refrigerant Flow in Pipes, Valves, and Pumps

Figure 7-4 Required ammonia mass flow rate/kW of capacity.

 corrected   0.61°C-
0.55
  =  332.6 kW   ----------------
0.55°C
= 352 kW
 line capacity 

Practical use of tables like Tables 7-2 and 7-3 involves identifying the type
of line and the required carrying capacity of the line. With the desired saturated
temperature drop per metre length, enter the table and find the saturated tem-
perature column that corresponds to the application. Move down the column
until the capacity listed exceeds the capacity required. The associated pipe size
indicates the selection. Interpolation between table values for suction or dis-
charge temperatures and saturation temperature drops is acceptable for applica-
tions that consist of a single pipe size.

Valve and Pipe-Fitting Equivalent Length


The piping used in a refrigeration system incorporates fittings and valves that
also generate a pressure drop. A convenient way to characterize the pressure
Fundamentals of Refrigeration SI, Second Edition 185

drop is to identify a length of the same pipe that has a pressure drop equal to
that in the valve or fitting. This is the equivalent pipe length Le. As a conse-
quence, the complete piping arrangement, which consists of straight pipe, fit-
tings, and valves, is reduced to a single equivalent length of straight pipe. Line-
sizing tables (such as Tables 7-2 and 7-3 for ammonia) may be used in con-
junction with appropriate corrections.
Tables 7-4 and 7-5 give values for equivalent pipe lengths for valves and
common fittings, respectively. For example, a standard 90° elbow in a 8 cm
line has an equivalent length of 2.3 m; the pressure drop for this elbow is the
same as for a 2.3 m length of pipe.
Insight on valve selection may also be gained by attentive observation of
Table 7-4 and the equivalent lengths of globe and angle valves, both manual
shutoff valves. The equivalent lengths of angle valves for any size are at least
one-half the equivalent lengths of globe valves. Over the economic life of a
system, the pumping energy costs may be substantially reduced by selecting
angle valves over globe valves. Angle valves have smaller pressure drops,
because the flow path is somewhat larger and the flow does not make a sharp
turn when passing through the valve.

Example 7-5

Problem A 8.0 cm, Schedule 40, steel pipe discharge line carries ammonia vapor from a
compressor. The discharge pressure and temperature are 1055 kPa and
48.89°C, respectively. The ammonia condensing temperature is 30°C, and the
flow rate is 31.75 kg/min. The piping arrangement, as shown in Figure 7-5,
contains a globe valve, an 24.4 m horizontal pipe run, and a 9.1 m riser. Deter-
mine the pressure drop between points 1 and 2.

Solution Begin by assuming that no flashing occurs in the pipe. The pressure drop is
given by

p total = p globe + p horiz. + pstd. + p static head + p vertical


valve 24.4 m elbow 9.1 m 9.1 m

p
=  -------- L e + h
(m)

and

L e = 26 + 24.4 + 2.3 + 9.1 m = 61.8 m

where the globe valve is taken as fully open, as given in Table 7-4.
186 Chapter 7 Refrigerant Flow in Pipes, Valves, and Pumps

Table 7-4 Valve Losses in Equivalent Metres of Pipe


Nominal Pipe Swing
60° 45° Lift
or Tube Size, Globea Anglea Gateb
mm
Wye Wye Checkc Check

10 5.2 2.4 1.8 1.8 0.2 1.5


15 5.5 2.7 2.1 2.1 0.2 1.8 Globe
and
20 6.7 3.4 2.1 2.1 0.3 2.2
vertical
25 8.8 4.6 3.7 3.7 0.3 3.0 lift
32 12 6.1 4.6 4.6 0.5 4.3 same as globe
40 13 7.3 5.5 5.5 0.5 4.9 valved
50 17 9.1 7.3 7.3 0.73 6.1
65 21 11 8.8 8.8 0.9 7.6
80 26 13 11 11 1.0 9.1
90 30 15 13 13 1.2 10
100 37 18 14 14 1.4 12
125 43 22 18 18 1.8 15
150 52 27 21 21 2.1 18
200 62 35 26 26 2.7 24
250 85 44 32 32 3.7 30 Angle
300 98 50 40 40 4.0 37 lift
350 110 56 47 47 4.6 41 same as angle
400 125 64 55 55 5.2 46 valve
450 140 73 61 61 5.8 50
500 160 84 72 72 6.7 61
600 186 98 81 81 7.6 73
Note: Losses are for valves in fully open position and with screwed, welded, flanged, or flared connections.
a
These losses do not apply to valves with needlepoint seats.
b
Regular and short pattern plug cock valves, when fully open, have same loss as gate valve. For valve losses of short pattern plug cocks above 150 mm, check with man-
ufacturer.
c
Losses also apply to the in-line, ball-type check valve.
d
For Y pattern globe lift check valve with seat approximately equal to the nominal pipe diameter, use values of 60° wye valve for loss.

Using Table 7-3 gives

p-
--------  0.684 kPa
m

Next, obtain the density of the ammonia vapor using the pressure-enthalpy
diagram given in Figure B-6 or calculate the density by using the ideal gas
equation of state. Let’s use the ideal gas equation of state where we must first
determine the gas constant for ammonia.

kJ
8.31 -------------------
k mol  K kJ
R NH3 - = 0.488 -----------
= -----------------------------
kg kg·K
17.03 ----------
k mol

Now, applying the ideal gas equation to obtain the density, we have
Fundamentals of Refrigeration SI, Second Edition 187

Table 7-5 Fitting Losses in Equivalent Metres of Pipe


(Screwed, Welded, Flanged, Flared, and Brazed Connections)
Smooth Bend Elbows Smooth Bend Tees
90° 90° Long- 90° 45° 45° 180° Flow Straight-Through Flow
Stda Radiusb Streeta Stda Streeta Stda Through
Nominal
Branch No Reduced Reduced
Pipe or
Tube Reduction 1/4 1/2
Size,
mm
10 0.4 0.3 0.7 0.2 0.3 0.7 0.8 0.3 0.4 0.4
15 0.5 0.3 0.8 0.2 0.4 0.8 0.9 0.3 0.4 0.5
20 0.6 0.4 1.0 0.3 0.5 1.0 1.2 0.4 0.6 0.6
25 0.8 0.5 1.2 0.4 0.6 1.2 1.5 0.5 0.7 0.8
32 1.0 0.7 1.7 0.5 0.9 1.7 2.1 0.7 0.9 1.0
40 1.2 0.8 1.9 0.6 1.0 1.9 2.4 0.8 1.1 1.2
50 1.5 1.0 2.5 0.8 1.4 2.5 3.0 1.0 1.4 1.5
65 1.8 1.2 3.0 1.0 1.6 3.0 3.7 1.2 1.7 1.8
80 2.3 1.5 3.7 1.2 2.0 3.7 4.6 1.5 2.1 2.3
90 2.7 1.8 4.6 1.4 2.2 4.6 5.5 1.8 2.4 2.7
100 3.0 2.0 5.2 1.6 2.6 5.2 6.4 2.0 2.7 3.0
125 4.0 2.5 6.4 2.0 3.4 6.4 7.6 2.5 3.7 4.0
150 4.9 3.0 7.6 2.4 4.0 7.6 9 3.0 4.3 4.9
200 6.1 4.0 — 3.0 — 10 12 4.0 5.5 6.1
250 7.6 4.9 — 4.0 — 13 15 4.9 7.0 7.6
300 9.1 5.8 — 4.9 — 15 18 5.8 7.9 9.1
350 10 7.0 — 5.5 — 17 21 7.0 9.1 10
400 12 7.9 — 6.1 — 19 24 7.9 11 12
450 13 8.8 — 7.0 — 21 26 8.8 12 13
500 15 10 — 7.9 — 25 30 10 13 15
600 18 12 — 9.1 — 29 35 12 15 18
a R/Dapproximately equal to 1.
b
R/D approximately equal to 1.5.

pv = RT
1
p  --- = RT
 
p
 = -------
RT
Inserting numerical values gives
1 kPa
 1055 kPa   -------------2 
 N·m 
 = -----------------------------------------------------------------------------
 0.488 ----------- kJ 
 48.89 + 273.15 K
 kg·K
kg
 = 6.71 ------3-
m
188 Chapter 7 Refrigerant Flow in Pipes, Valves, and Pumps

Figure 7-5 Piping arrangement for Example 7-5.

Thus,

 6.71 ------ kg-


 9.1 m 
0.684 kPa  m 3
 
p = ------------------------  61.8 m  + ------------------------------------------- = 42.3 + 0.6
 m  101.325
p = 42.9 kPa

Pumps
The piping in a refrigeration system transports discharge vapor from a com-
pressor, suction vapor to the compressor, and liquid to the evaporators. The
prime mover for the vapor flow is the compressor. If the refrigeration system is
large (such as a two-stage industrial system), then liquid may be pushed to the
evaporators by a pump. When secondary fluids are used to transfer heat from a
Fundamentals of Refrigeration SI, Second Edition 189

cold space, instead of directly to the refrigerant, pumps are used to circulate the
fluid between the cold space and the refrigeration system. Therefore, applica-
tions of pumps in refrigeration systems include liquid refrigerants and second-
ary heat-transfer fluids.
As discussed, the piping is characterized as having a pressure drop for a
specific flow rate. Pump performance is plotted as a curve showing the rela-
tionship between pressure difference and flow rate. Superimposed on the same
plot is the variation of power input to the pump with flow rate. Different pump
types yield different shapes for the pressure-difference-versus-flow-rate perfor-
mance curves.

Centrifugal Pumps
In centrifugal pumps, the liquid is thrown out to the pump’s outer case by means
of a rotating impeller, thereby increasing its pressure. The pump casing directs
the liquid from the impeller to the point of discharge. Technically, the energy
transfer to the liquid results from a change in angular momentum of the liquid.
Figure 7-6 shows a cross section of a centrifugal pump where the liquid enters
from the right and exits at the top on the right. The mechanical components to
the left of the impeller and casing are needed to support and drive the impeller.
A typical centrifugal pump performance curve is shown in Figure 7-7. Four
curves, beginning at the left and designated by varying impeller diameters,
show how this pump’s generated pressure difference changes with volumetric
flow rate. Note that the pressure difference is given in terms of total head in
feet. This is the pressure difference that results from a column of water equal in
height to the amount of head. For example, 14.6 m of water head may be con-
verted to kPa as follows:

kg f  1 kPa
pressure  kPa = h =  1000 --------  14.6 m   -------------------------------------2- = 143 kPa
 m 3   101.97 kg f / m 

where 21.1°C is assumed as the water temperature for the evaluation of its den-
sity. For an impeller diameter equal to 20.95 cm, point A in Figure 7-7 indi-
cates the operating condition that corresponds to maximum efficiency, about
82%. Lines of equal efficiency are drawn and appear as U-shaped curves
around the maximum efficiency point. Points B and C indicate off-design
points for the 20.95 cm impeller at about 76% efficiency, which suggests a
compromise between pressure difference and flow rate.
The performance of a centrifugal pump also includes the variation of pump
power as a function of flow capacity and pressure difference. Interpolation may
be used to estimate the power at a specific operating condition. At point B, the
power is approximately 5.2 kW.
An important feature of the centrifugal pump is shown by the shape of the
pressure-difference-versus-flow-capacity curves. Notice that the pump may
190 Chapter 7 Refrigerant Flow in Pipes, Valves, and Pumps

Cross section of a centrifugal pump.


Figure 7-6
Fundamentals of Refrigeration SI, Second Edition 191

Figure 7-7 Performance curves for a centrifugal pump.

operate with no flow, as suggested by the performance curves. In practice, this


condition is avoided because the liquid may partially vaporize and cause a con-
dition known as cavitation. The curves indicate reduced pressure difference
with increasing flow capacity that becomes severe at high flows.
The remaining item plotted on Figure 7-7 is the net positive suction head
(NPSH), required for the pump. NSPH is defined as the difference between
the static pressure at the pump suction (inlet) and the vapor pressure of the
fluid. The NPSH identifies the condition at which a pump will experience
cavitation: vapor bubbles form in the liquid when the liquid pressure is less
than the saturated vapor pressure of the liquid. The bubbles disrupt the
smooth flow through the pump, causing mechanical agitation and accelerated
wear of the pump’s components as well as significant reductions in the oper-
ating efficiency. Cavitation is avoided when the pump is located where the
192 Chapter 7 Refrigerant Flow in Pipes, Valves, and Pumps

©McGraw-Hill. Reprinted with permission.

Figure 7-8 Positive-displacement gear pump.

actual liquid height above the inlet exceeds the NPSH required by the pump.
Careful consideration of the pump’s NPSH requirements as indicated by the
performance plots eliminates pump cavitation. Centrifugal pumps used to
supply liquid refrigerant typically have a lower NPSH than those shown in
Figure 7-7.

Positive-Displacement Pumps
Many configurations of positive-displacement pumps are manufactured, but all
operate on the principle of trapping a certain volume of liquid and squeezing it
around a closed case. Figure 7-8 shows a schematic drawing of a two-shaft
pump where the spur gears operate as impellers. The large number of teeth in
contact with the pump’s case reduces leakage. Observe that, for this gear
Fundamentals of Refrigeration SI, Second Edition 193

Figure 7-9 Performance characteristics for a gear pump.

pump, the liquid is trapped at the point of intermesh, which results in noisy
operation and relatively low efficiency. Sophisticated arrangements of the
gears and higher operating speeds improve operation.
Figure 7-9 is a plot of performance characteristics for a gear pump. The
pressure-difference-versus-flow-capacity curves are very steep when compared
to those of the centrifugal pump. Consequently, the gear pump can deliver a
volume rate of flow over a wide range in pressure. These capacity curves are
vertical if the pump has no leakage. Also plotted in Figure 7-9 are lines of con-
stant power showing that for a given speed, as capacity falls, so does the pump
194 Chapter 7 Refrigerant Flow in Pipes, Valves, and Pumps

power. An application well suited to the characteristics of the positive-dis-


placement pump is the transfer of liquid from a location of low pressure to
another location at high pressure; this is particularly important in larger indus-
trial refrigeration systems. An important caution must be mentioned concern-
ing the use of positive-displacement pumps: they require a pressure relief valve
under the condition of no flow. If the flow were suddenly stopped, continuing
to operate the pump would generate dangerously high pressures. The use of a
pressure relief valve near the pump discharge eliminates this problem. When
abnormally high pressures occur, the relief valve opens and bypasses liquid to
a location of low pressure, such as a vessel serving the inlet side of the pump.

The Next Step


Chapter 8 presents a series of expansion devices whose function in a refrigera-
tion system is to regulate the flow of refrigerant in the system. Expansion
devices also separate the high and low sides of the system.

Summary
Pipes convey refrigerant between components in the system, and accompanied
with this flow is a pressure drop. In a refrigeration system, pressure drops rep-
resent a loss that adversely affects operating efficiency. The Darcy-Weisbach
equation gives the pressure drop as a function of several variables, including
flow velocity, pipe length, pipe inside diameter, fluid density, and friction fac-
tor. The Moody chart (Figure 7-1) provides values of the friction factor that
depend on the surface roughness of the pipe and Reynolds number (Re). Re is a
dimensionless number equal to VD/µ and is greater than 2300 for turbulent
flow in a pipe.
Charts and figures based on calculations using the Darcy-Weisbach equa-
tion may be used to predict pressure drop in pipes. The pressure drop associ-
ated with fittings and valves is given as an equivalent length of pipe. Liquids
are driven through pipes by pumps. The centrifugal pump delivers relatively
large flows at moderate pressure, while the positive-displacement pump readily
provides high discharge pressures. The differences in the performance curves
for these two types of pumps result from the method used to generate the dis-
charge pressure. You should now be able to

 describe the differences between laminar and turbulent flow in a pipe and
identify the distinguishing value of the Re;
 calculate the pressure drop in a length of pipe using the Darcy-Weisbach
equation in conjunction with the Moody chart (Figure 7-1);
 use charts and figures similar to the examples shown in this chapter to predict
pressure drop and the drop in saturation temperature over a length of pipe;
 determine the equivalent length of a piping arrangement consisting of fit-
tings, valves, and straight lengths of pipe;
Fundamentals of Refrigeration SI, Second Edition 195

 estimate the pressure drop for a change in elevation associated with a


riser; and
 describe the operation of the centrifugal and gear pumps and explain the
shape of the pressure-difference-versus-flow-capacity performance curve
for each.

Bibliography
ASHRAE. 2006. ASHRAE Handbook—Refrigeration. Atlanta: ASHRAE.
ASHRAE. 2008. ASHRAE Handbook—HVAC Systems and Equipment.
Atlanta: ASHRAE.
ASHRAE. 2009. ASHRAE Handbook—Fundamentals. Atlanta: ASHRAE.
Baumeister, T. 1978. Marks’ Standard Handbook for Mechanical Engineers.
New York: McGraw-Hill Book Co.
Howell, R., H. Sauer, and W.J. Coad. 2010. Principles of Heating, Ventilating
and Air Conditioning, 6th ed. Atlanta: ASHRAE.
Wile, D. 1977. Refrigerant Line Sizing. Atlanta: ASHRAE.
196 Chapter 7 Refrigerant Flow in Pipes, Valves, and Pumps

Skill Development Exercises for Chapter 7


Complete these questions by writing your answers on the worksheets at the back of this book.

7-1 Calculate the pressure drop for a 30.5 m length of a 15.2 cm steel Schedule 40
ammonia suction line. The mass rate flow is 45.4 kg/min from a –28.9°C evap-
orating temperature.
7-2 Using the equations presented in the section “Fluid Flow in Pipes,” show that
the pressure drop over a length of pipe is inversely proportional to D raised to
the fifth power

1
 p  ------
 D 5

7-3 A steel piping layout for an ammonia liquid line is shown the figure below,
where all the pipe runs are horizontal. The ammonia liquid enters at a pressure of
448 kPa (gage) and 4.44°C and has a mass flow rate of 136 kg/min. Estimate the
pressure drop for this piping layout and determine if the ammonia flashes before
it reaches the exit.

7-4 Calculate the influence that a pressure drop in a suction line has on the com-
pressor power to operate a refrigeration system using R-134a that consists of
four components: compressor, condenser, expansion valve, and evaporator. The
refrigerating capacity is 14 kW. The condensing pressure is 1207 kPa, and the
refrigerant exits the condenser as a saturated liquid. The evaporating pressure
Fundamentals of Refrigeration SI, Second Edition 197

is 345 kPa, and superheated vapor leaves the evaporator at 15.55°C. The suc-
tion line between the evaporator and compressor is 4 m long, 1.27 cm nominal
diameter type-K copper tube (actual inside diameter is 1.34 cm). The compres-
sor’s isentropic efficiency is 70%. The pressure drop in the other lines that con-
vey refrigerant, in the condenser, and in the evaporator may be neglected. It is
suggested to proceed through the following intermediate steps:

1. Calculate the pressure drop in the suction line using R-134a thermody-
namic data. (Hint: Viscosity of the superheated vapor may be approximated
by the saturated vapor viscosity at the same temperature.)
2. Estimate the actual inlet conditions to the compressor. (Hint: Enthalpy dif-
ference from the inlet to exit of the suction line is approximately p   .)
3. Calculate the actual thermodynamic conditions at the discharge of the com-
pressor.
4. Calculate the actual compressor power input.
5. Recalculate the compressor power input if the suction line has no pressure
drop.
6. Compare the two estimated compressor powers by calculating a percentage
change relative to the no-pressure-drop ideal situation.

7-5 The figure below shows a schematic of an evaporative condenser. The centrifu-
gal pump planned for use has the performance characteristics shown in
Figure 7-7. The required flow is 1.06 m3/min with pressure difference of
110 kPa. Determine the following: impeller diameter, power needed to drive the
pump, the pump’s efficiency at this condition, the pump’s required NPSH, and
whether the height provided by the sump and inlet pipe is sufficient for this
application.
198 Chapter 7 Refrigerant Flow in Pipes, Valves, and Pumps

7-6 A recirculation system that uses ammonia as the refrigerant has an insulated
steel line delivering liquid to a cluster of evaporators. The refrigerating load
at the evaporators is 700 kW, and the evaporating temperature is –40°C. The
circulation ratio is 4. The liquid line is a 50 mm Schedule 40 (internal diame-
ter of 5.25 cm) with roughness equal to 0.000381 cm. The equivalent length
of the pipe is 305 m with an elevation from the supply to the evaporator of
9.1 m, and the pressure drop across the balancing valve at the evaporator is
69 kPa. Determine the pressure that must be provided by the pumping system
to deliver the required ammonia to this evaporator cluster, and estimate the
temperature of the ammonia in the low-pressure receiver so that the liquid
entering the evaporator is –40°C.
Expansion Devices

Study Objectives
Chapter 8 presents information concerning expansion valves, one of the four
basic components in a vapor-compression refrigeration system. Several expan-
sion devices are discussed in this chapter, all of which regulate the flow of
refrigerant to the evaporator and separate the high- and low-pressure sides. The
thermal expansion valve is explained in conjunction with the operation of the
evaporator. Other devices discussed that involve the expansion and control of
refrigerant flow include: level control valves, pressure regulating valves, and
hand expansion valves. Additionally, we consider the special application of
turbo expanders in two different refrigeration cycles, one of which uses air as
the refrigerant. After studying Chapter 8, you should be able to

 describe the configuration and operation of a thermostatic expansion valve;


 list the similarities and differences between the pressure control and thermo-
static expansion valves;
 explain the application of level control valves in a refrigeration system;
 explain why hand expansion valves may be used in a refrigeration
system; and
 describe the thermodynamic advantage given by the turbo expander when it
is applied to the refrigeration cycle and give one example of an application.

Instructions
Read the material in Chapter 8. Verify the examples presented in the chapter
with your own calculations. At the end of the chapter, complete the skill devel-
opment exercises without referring to the text. Review those sections of the
chapter as needed to complete the exercises.

Introduction
Without an expansion device, the refrigeration system will not operate properly.
Expansion devices have two important functions in a vapor-compression sys-
tem: (1) regulate the refrigerant flow to the evaporator for its proper operation
and (2) provide a pressure drop where the refrigerant temperature is lowered so
that heat moves from the cold space into the evaporator. The expansion device
also separates the high pressure in the condenser from the lower pressure in the
200 Chapter 8 Expansion Devices

evaporator. In larger systems, the expansion device controls the liquid level in a
vessel operating at a lower pressure.
An expansion device obstructs the flow of high-pressure liquid by provid-
ing a relatively small area or orifice through which the flow must pass. The
simplest devices are the capillary tube and short tube restrictor, both of which
consist of a very small flow passage diameter. The pressure drop from the inlet
to the exit of the flow passage is considerable and results from fluid friction
and flashing vapor restricting or binding the flow of refrigerant. The combina-
tion of length and diameter determines the pressure drop. Consequently, the
capillary tube and short tube restrictor are only applied to small refrigeration
systems where the refrigeration load is relatively constant. Generally, the capil-
lary tube or flow restrictor is located directly upstream from the evaporator.
When an expansion device that produces a liquid and vapor mixture is located
there, the system is described as employing direct expansion. Applications
include room air conditioners, residential refrigerators, and home freezers.
Several expansion devices are described in this chapter. The pressure con-
trol valve and the thermostatic expansion valve are used in direct-expansion
systems where the valve is located just upstream from the evaporator. Hand
expansion valves and level control valves are applied to more complex systems
that involve refrigerant storage vessels and multiple evaporators. The turbo
expander is a special expansion device that allows the extraction of work from
the expanding refrigerant, but it is limited to certain applications.

Capillary Tubes and Short Tube Restrictors


For small refrigeration systems where the refrigerating capacity is roughly less
than 10 kW, the capillary tube or short tube restrictor are nearly always
applied. Examples of these refrigeration systems include domestic refrigera-
tors, freezers, dehumidifiers, and room air conditioners.
A capillary tube is simply a long and small-diameter tube. Typical lengths
vary from about 1 to 6 m, and internal diameters (bore) vary from about 0.5 to
2 mm. The capillary tube’s function in the refrigeration system is to provide a
pressure drop sufficient to achieve the desired evaporating temperature. The
processes that occur in the capillary tube pressure drop are complex. One pro-
cess is fluid friction against the capillary tube’s inside surface. The other pro-
cess occurs after the pressure has dropped below the saturation pressure and
the refrigerant begins to flash into a vapor that binds or throttles the flow. At
this point, the refrigerant travels at a greater velocity, which generates a greater
pressure drop. While the refrigeration system capillary tube is very narrow,
capillary flow does not occur in the tube, because its diameter is much too
large for fluid surface tension forces to be important.
The capillary tube serves as the liquid line connecting the condenser to the
evaporator, while, at the same time, it regulates the flow of refrigerant to the
evaporator. The regulation of refrigerant flow must be such that the evaporator
operates at the desired pressure. Additionally, the capillary tube and compressor
Fundamentals of Refrigeration SI, Second Edition 201

operate in series, which means that the refrigerant flow rate through the capillary
tube must equal the refrigerant vapor flow rate through the compressor. In order
for the refrigeration system to operate at the design condensing and evaporating
pressures, the flow capacity of the capillary tube for these pressures must be
equal precisely to the pumping capacity of the compressor. The pressure drop
across the capillary tube is determined by its length and bore. Because these
dimensions are fixed for a given capillary tube, the refrigeration system is not
free to adjust to changes in load, suction pressure, or condensing pressure.
If the flow capacity of the capillary tube is greater or less than the pumping
capacity of the compressor at design conditions, then the operating balance
point of the system shifts to a different condition from that of design. To illus-
trate, if the tube has inadequate flow resistance, implying that the tube’s bore is
too large or too short, then the tube’s flow capacity is greater than the pumping
capacity of the compressor at the design conditions. This situation results in
overfeeding of the evaporator where slugging the compressor with liquid is
possible. Also, the entrance to the capillary tube is now a mixture of liquid and
vapor and not liquid-only as it should be. The increased vapor flow (and conse-
quent decreased liquid flow) to the evaporator reduces the system’s refrigerat-
ing capacity as a result of the additional latent heat. Moreover, with the
increased refrigerant flow through the capillary tube, the evaporator pressure
rises because the pumping capacity of the compressor is insufficient. On the
other hand, if the capillary tube’s flow resistance is too large, the evaporator is
starved of refrigerant.
A refrigeration system using a capillary tube operates at maximum effi-
ciency only at the design condition. At all other operating conditions, the
refrigeration system’s efficiency is less. The capillary tube, however, does self-
compensate to some extent at off-design conditions. As the load on the refrig-
eration system increases or decreases from the design condition, the flow
capacity of the capillary tube changes and a greater or lesser amount of refrig-
erant is passed. This is due to changes in condenser pressure and the degree to
which subcooling exists in the condenser. For example, as system load
decreases, condenser pressure and the degree of subcooling decreases, which
in turn reduces the flow capacity of the capillary tube. Nevertheless, the com-
pensation possible by the capillary tube is not perfect.
Short tube restrictors that consist of a relatively short tube length, com-
pared to capillary tubes, are also widely used with small refrigeration systems.
Typical dimensions of short tube restrictors are lengths from 5 to 30 mm and
bores ranging from 0.7 to 2 mm. For these devices, the bore, more than the
length, controls the pressure drop. As with the capillary tube, the pressure dif-
ference across the restrictor and the degree of subcooling determines its refrig-
erant flow capacity. Short tube restrictors are manufactured in two basic
configurations: stationary and movable. Figure 8-1 shows a cut away view of a
short tube restrictor with the special feature of a moving piston. Upon reversal
of the refrigerant flow, the piston moves, offering very little pressure drop. This
202 Chapter 8 Expansion Devices

Figure 8-1 Cross section of a typical short-tube restrictor with a movable piston.

feature is particularly important for heat-pump cycles that require the reversal
of the refrigerant flow direction.
The operation of capillary tubes and short tube restrictors in a refrigeration
system is the same. Both devices provide the needed pressure drop between the
high- and low-pressure sides of the system while, at the same time, establishing
the desired flow capacity. When the system is not operating, called the OFF-
cycle, the pressure in the system equalizes between the high- and low-pressure
sides. Consequently, the compressor starts in an unloaded condition, which
permits the use of a low-starting torque motor. One advantage of the capillary
tube is that, due to its length, it may be put into contact with the suction line to
provide for heat transfer, which results in some subcooling of the liquid. The
subcooling delays the onset of flashing in the capillary tube, which allows
more liquid to enter the evaporator thereby improving the thermal efficiency of
the refrigeration cycle.
Charts are available that give the performance rating of capillary tubes and
short tube restrictors. Figures 8-2 and 8-3 are charts appropriate for capillary
tubes without heat transfer to the suction line used with refrigerant R-134a
(ASHRAE 2006). Figure 8-2 shows the R-134a flow rate through a reference
3300 mm length and 0.86 mm bore capillary tube as a function of inlet pressure
and degree of subcooling. Also observe that Figure 8-2 shows the influence of
refrigerant quality at the inlet when its value is not one. Figure 8-3 yields the
correction flow factor,   to multiply the flow rate from Figure 8-2 when the
geometry of the capillary tube is different than that of the reference. Thus, if
the mass flow rate from Figure 8-2 is m· uncorrected , we may obtain the cor-
rected mass flow rate through the capillary tube using
Fundamentals of Refrigeration SI, Second Edition 203

Figure 8-2 Mass flow rate of R-134a through a capillary tube with 0.086 cm inside diameter
and 330 cm length.

Figure 8-3 Mass flow rate correction factor φ for R-134a as a function of capillary tube
length, inside tube diameter, and refrigerant inlet condition.
204 Chapter 8 Expansion Devices

m· corrected = m· uncorrected (8-1)

Charts for capillary tubes using R-134a, R-22, and R-410A with and with-
out heat transfer with the suction line are available (ASHRAE 2006). Charts
for short tube restrictors for R-22 are also available (ASHRAE 2006).

Example 8-1

Problem Refrigerant R-134a flows without heat exchange through a capillary tube that
has a bore of 1 mm and a length of 2540 mm. The operating characteristics of
the system are 1380 kPa condensing pressure (tube inlet pressure) and 2.78°C
subcooling. What is the mass flow rate of R-134a through this capillary tube at
these conditions?

Solution From Figure 8-2, at conditions 1380 kPa and 2.78°C subcooling, read the mass
flow rate as approximately 5.53 kg/h.
Obtain flow factor correction from Figure 8-3 as 1.75 for a capillary tube of
1 mm bore and 2540 mm length.
So,
m· corrected = m· uncorrected
=  1.75   5.53 
= 9.7 kg/h

The refrigerant charge in systems using either device is very important. The
charge should be just enough to satisfy the operating requirements of the evap-
orator and provide a liquid seal between the condenser and the capillary tube or
short tube restrictor. When the system cycles off, any liquid in the condenser
moves to the low-pressure region or, specifically, to the evaporator. Liquid
refrigerant stays in the evaporator until the compressor starts again. Conse-
quently, the refrigerant charge quantity in these systems is very important. The
importance of charge is further emphasized by the fact that these small systems
do not use a receiver vessel. For example, if the charge is too great, the excess
refrigerant backs up in the condenser and causes higher condenser pressure,
which results in a new operating balance point for the system.

Pressure Control Valves


The pressure control valve (sometimes called an automatic expansion valve)
has been used in refrigeration systems for a long time but is rarely used now.
However, the word automatic may be applied to a number of expansion devices
and is not unique to this valve. A more accurate name for this valve would be
Fundamentals of Refrigeration SI, Second Edition 205

the constant evaporator pressure expansion valve, because it keeps the evapo-
rator outlet pressure at a constant value with changing refrigerating load and
liquid inlet pressure.
Figure 8-4 is a cross section of a typical pressure control valve. The liquid
refrigerant enters the valve by passing through the inlet. The restriction in the
valve that meters the refrigerant flow is located between the valve seat and the
valve pin. From the diagram, observe that the equalizer passage transmits the
evaporator pressure to the underside of the diaphragm. In Figure 8-4, either the
external or the internal equalizer may be used, but not both. The evaporator
pressure multiplied by the exposed area of the diaphragm is the force exerted
on the underside. On top of the diaphragm, a spring fixed at one end exerts a
balancing force. The valve pin is positioned where the diaphragm moves to
balance these two forces. An adjusting screw at the top of the spring assembly
permits changes to the spring force, which in turn changes the position of the
valve pin and, ultimately, the valve’s pressure setting. Note that increasing
evaporator pressure closes the valve and that decreasing evaporator pressure
opens the valve.

Figure 8-4 Constant-pressure valve.


206 Chapter 8 Expansion Devices

The pressure control valve responds to changes in the refrigeration load by


changing the refrigerant flow rate to maintain constant pressure in the evapora-
tor. With a decrease in load, the evaporator pressure drops below the valve’s
pressure setting, which allows the valve pin to move downward. This increases
the flow of refrigerant to the evaporator. At first, we see this as opposite the
action that we wish: the refrigerant flow should decrease with a reduction in
load. However, the evaporator pressure does increase until the pressure setting
is reached again. If the refrigeration load increases, the evaporator pressure
increases, which causes the diaphragm to move upward. This in turn decreases
the refrigerant flow and brings the evaporator pressure back to the pressure set-
ting—again, the action opposite that we would expect. However, with this con-
trol action, we see that the constant expansion pressure valve maintains
evaporation of the refrigerant inside the evaporator at a set pressure.
Further investigation into the operation of the constant pressure valve is
needed to understand the control action. When a refrigeration system operates
for a sufficient length of time, the flow of liquid refrigerant metered by the con-
stant pressure valve balances the vapor flow drawn by the compressor. The
actual pressure in the evaporator is adjusted to the desired value by changing the
compression force on the opening spring by turning the screw. The necessary
adjustment falls between preventing liquid from reaching the compressor during
low load conditions and preventing the pressure from increasing and overload-
ing the compressor during high loads. In other words, this valve overfeeds the
evaporator at low loads and starves it at high loads. Another characteristic of the
valve is that, as the evaporator pressure is controlled, so is the evaporator tem-
perature. Moreover, the constant pressure valve has a tendency to starve the
evaporator for refrigerant at high altitudes.
The operational characteristics of the constant pressure valve restrict its
application to constant refrigeration loads. Typical applications of these valves
are domestic refrigerators, unitary room air conditioners, water coolers, and
drink dispensers. Another use of the constant pressure valve is to return a por-
tion of the compressor discharge vapor to reduce compressor capacity. This
superheated discharge vapor may be introduced into the suction line or to the
inlet of the evaporator.

Thermostatic Expansion Devices


The thermostatic expansion valve is the most common refrigerant flow meter-
ing device applied to larger refrigeration systems. This valve regulates the
flow based on the amount of vapor superheat at the evaporator exit. When all
the liquid refrigerant in the evaporator boils, additional heat absorbed by the
evaporator superheats the vapor, while the pressure remains essentially con-
stant. If the temperature of the superheated vapor is 11°C above the evaporat-
ing temperature (saturation temperature), then the vapor is said to have 11°C
of superheat. The term thermostatic expansion valve does not correctly
describe the valve. The valve controls the refrigerant flow so that a level of
Fundamentals of Refrigeration SI, Second Edition 207

superheat in the refrigerant vapor leaving the evaporator is maintained. In


doing this, the valve ensures that the metered refrigerant into the evaporator
equals the vaporization rate of refrigerant inside the evaporator. Because
superheat exists at the evaporator’s outlet, a portion of the evaporator’s heat
transfer surface is dry and superheats the vapor. As mentioned previously, the
dry portion of the evaporator transfers only a small amount of heat to the
refrigerant. Incidentally, the heat that does go into superheating the refrigerant
vapor is occasionally called useful superheat.
Figure 8-5 shows a cross section of a thermostatic valve in which three
important pressures are identified in the drawing to the right of the valve.
These pressures are labeled as follows:
p1 = pressure on top of the diaphragm, originating from the thermostatic ele-
ment, that tends to open the valve
p2 = transmitted pressure of the evaporator, directed to the bottom of the dia-
phragm, that tends to close the valve

Figure 8-5 Thermal expansion valve.


208 Chapter 8 Expansion Devices

p3 = equivalent pressure, exerted by the spring located below the diaphragm,


that tends to close the valve

Remembering that pressure is force multiplied by an area, the equivalent


pressure is that resulting from the spring force acting on the diaphragm’s face.
At a steady-state operating condition, the forces resulting from the action of
these pressures are balanced. The valve pin is positioned according to the bal-
anced position of the diaphragm inside the valve.
Figure 8-6 depicts a thermostatic valve that supplies an evaporator—a
direct-expansion (DX) configuration. A liquid/vapor mixture of R-134a enters
the evaporator, and superheated vapor leaves. A bulb is secured to the tubing
near the exit so that good thermal contact is made with the vapor inside the
tube. In the basic valve, the fluid in the sensing bulb is the same as the refriger-
ant used and must contain a small amount of liquid. Therefore, the fluid in the
bulb is always at saturation. The capillary tube transmits the pressure in the
bulb back to the valve. Sometimes the fluid in the sensing bulb is referred to as
the power fluid. As with the pressure control valve, the thermostatic expansion
valve may use either internal or external pressure equalization.

Figure 8-6 Thermostatic valve in DX configuration using R-134a as refrigerant and power
fluid.
Fundamentals of Refrigeration SI, Second Edition 209

Example 8-2

Problem The evaporator shown in Figure 8-6 uses R-134a and is controlled to a satura-
tion temperature of 4.44°C by a thermostatic expansion valve. Using the ther-
modynamic property tables in Appendix A, estimate the pressures at locations
A, B, and C in Figure 8-6 and the superheat at point C, the exit of the evapora-
tor.

Solution The evaporating pressure (saturated) is 241.3 kPa (gage), assuming that atmo-
spheric pressure is 101.325 kPa. The spring is adjusted so that the equivalent
exerted pressure on the diaphragm is 71.7 kPa (gage). The liquid passing
through the valve expands to a mixture of liquid and vapor refrigerant at a
lower pressure and temperature shown as point A. Heat transfer to the evapo-
rator boils the liquid until no more liquid remains. Under the condition of no
pressure drop, the portion of the evaporator from points A to B maintains con-
stant temperature and pressure, 4.44°C and 241.3 kPa (gage), respectively.
Downstream from point B, continuing heat transfer superheats the vapor until
the outlet is reached. At point C, the temperature is 10°C, the pressure is
241.3 kPa (gage), and the superheat is 5.56°C (10°C – 4.44°C = 5.56°C).

The operation of the thermostatic expansion valve may be explained by


referring to the three pressures acting within (the bulb pressure on top of the
valve, the evaporator pressure, and the equivalent spring pressure below the
diaphragm). For an increase of refrigeration load, the bulb pressure is greater
than the spring and evaporator pressures added together, and the valve’s pin
moves downward, causing the valve to open further and pass more refrigerant
to the evaporator. The amount of superheat is, consequently, reduced at the
evaporator outlet. When the load is decreased, the bulb pressure drops, and the
valve pin moves upward, somewhat closing the orifice. The refrigerant flow is
reduced, and the degree of superheat increases to achieve its set value.
Notice in Example 8-2 that it is the spring adjustment that sets the degree
of superheat for the valve. The actual situation is slightly more complicated.
Figure 8-7 is a representative performance curve for a thermostatic expansion
valve—a plot of refrigerating capacity resulting from flow through the valve
versus the degree of superheat. Point A identifies the superheat on the curve
that corresponds to the condition of no load. This is called the static superheat
and is set by adjusting the superheat spring force. This static superheat spring
force must be sufficient to maintain the valve in the closed position when the
refrigeration system is not operating.
Figure 8-7 also indicates that when the valve is operating, the value of
superheat controlled by the valve is greater than the static value. The degrees of
superheat between points A and B, where B is the superheat at rated capacity
for the valve, is called the opening superheat. The valve’s performance curve is
210 Chapter 8 Expansion Devices

Figure 8-7 Performance curve for a thermostatic expansion valve.

nearly linear over the operating range, so the valve’s capacity rating is roughly
proportional to the superheat degrees. Another term, operating superheat,
refers to the sum of the static and opening superheat. The highest capacity at
which the valve can operate when fully open usually exceeds the rated capac-
ity; the difference between the two capacities is known as reserve capacity. The
degree of superheat associated with the reserve capacity is shown by the length
BC in Figure 8-7. The flow rate through the valve increases with increasing
pressure drop across the valve. When the valve is fully open, the velocity of the
refrigerant through the valve is given by

V = C 2  p  (8-2)
Fundamentals of Refrigeration SI, Second Edition 211

where V is the velocity through the valve (m/s), p is the pressure difference
across the valve (kPa), and C is a constant experimentally measured for a spe-
cific valve. Manufacturers of expansion valves generally specify a refrigerating
capacity for the mass flow rate for which the valve is capable. This mass flow
rate may be approximated by the following:

m· = AV = AC 2  p  (8-3)

where  is the density of the liquid (kg/m3) and A is the cross-sectional area
through which the flow passes (m2).
An intriguing thermodynamic feature is associated with the expansion pro-
cess. Flashing of liquid to vapor occurs downstream of the valve’s orifice, so
only liquid exists inside the valve’s body. Thus Equation 8-2, which only
applies to liquids, is correct for expansion of liquid refrigerants flowing
through an orifice.
Actual evaporators have a drop in refrigerant pressure (about 13 to 40 kPa)
that occurs from the inlet to the outlet. If the valve includes a refrigerant dis-
tributor, as shown in Figure 8-8, the pressure drop is on the order of 170 to
240 kPa. A significant pressure drop across the coil and or the distributor
causes the thermostatic expansion valve to control to an unacceptably high
degree of superheat. This occurs because the pressure at the outlet of the valve
is greater than the pressure at the outlet of the evaporator, and, as a result, the
evaporator is starved for refrigerant. The pressure drop across the coil changes
with the refrigeration load, which prevents simple correction by adjusting the
superheat spring force. Compensation for this pressure drop is accomplished
by adding an external equalizing port to the lower side of the diaphragm inside
the thermostatic expansion valve. Unlike with the pressure control valve, ther-
mostatic expansion valve operation is not influenced by altitude. An external
equalizing line is shown in Figure 8-8.
We know the thermostatic expansion valve controls the degree of superheat
at the evaporator outlet. However, this valve does not control the evaporating
pressure or temperature. Refrigeration systems using compressors operating at
a constant displacement rate (typical of relatively small refrigeration systems)
alter their evaporating temperature and pressure with load. Figure 8-9 shows a
typical situation at high and low refrigeration loads. Where the compressor
performance curve crosses the thermostatic expansion valve’s operating curve,
the system’s balance, or specific operating condition, is determined. Two valve
performance curves are shown: one at 50% rated capacity to meet the low load
and the other at 100% rated capacity to meet the high load. At the high-load
condition, the evaporating pressure is relatively high, while at a low load, the
evaporating temperature is reduced along with the refrigeration flow rate to the
evaporator.
The thermostatic expansion valve is by far the most widely used metering
device in the residential and commercial refrigeration fields. Application
does occur in industrial refrigeration systems but is not as widespread. When
the required evaporating temperature falls, the superheat in the evaporator
212 Chapter 8 Expansion Devices

Figure 8-8 External equalizer line applied to thermostatic valve.

©Business News Publishing Co. Reprinted with permission.

Figure 8-9 Evaporator temperature versus refrigeration load.


Fundamentals of Refrigeration SI, Second Edition 213

progressively degrades the evaporator performance. The power required to


drive the compressor and the refrigerating capacity drop significantly with
each degree of temperature reduction in the evaporator. For this reason, the
degree of superheat to which the thermostatic expansion valve is asked to
control is decreased, which in practice may be difficult to achieve. Further-
more, when ammonia is the refrigerant, the mass flow rate needed for the
same refrigeration rate is less than it would be for a halocarbon refrigerant
such as R-22. Because small flow rates are considerably more difficult to
control accurately with thermostatic expansion valves, other methods are
employed.

Electronic Expansion Valves

Over the past 15 years, electronic expansion valves have become popular in the
commercial refrigeration field. For example, separate electronic valves and
their associated sensors are used in supermarket display cases containing
unfrozen food products such as vegetables. This application requires a narrow
control range of interior case temperature to prevent freezing. With electronic
expansion valves, the degree of superheat is tightly controlled, as are the inte-
rior temperature, defrost, drain, and anti-sweat heaters. Small digital comput-
ers (such as personal computers) serve as the controllers, which can sense
numerous inputs, perform complex calculations, and send electronic signals to
the actuators.
Two basic approaches are taken with electronic expansion valves. The first
is to use a screw-type metering valve actuated by a stepping motor. The regula-
tion of liquid refrigerant flow is accomplished in the same manner as with the
expansion valves previously considered—by changing the orifice area through
which the liquid must pass. The second approach uses a special electrical sig-
nal called pulse-width modulation to open and close the valve periodically.
Thus, the fraction of time that the valve is fully open determines the feed rate
of refrigerant.
A stepping (or stepper) motor is a multipole motor that rotates a certain
fraction of a revolution when a certain set of pulses is applied. These motors
are well suited for positioning expansion valves because the rotational motion
of the motor’s rotor is accurately controlled by the pulses delivered by the con-
troller. These motors may be controlled by a computer using digital electronics
where only two bits, or registers, are needed. For example, a stepping motor
may be controlled in the following manner. Each bit may be either high or low.
If one bit is high, the motor rotates one step to open the valve, and if the other
bit is high, the motor rotates one step in the opposite direction to close the
valve. When both bits are low, the motor does not rotate. When actuating an
expansion valve, the stepping motor sequentially receives a stream of high and
low signals, causing movement of the valve to reach the desired position. An
example of a stepping motor actuating signal is given in Figure 8-10a. Stepping
214 Chapter 8 Expansion Devices

motors are classified by the construction of the rotor and may be either the
variable reluctance type or the permanent magnet type.
With pulse width modulation, the electronic expansion valve is either open
or closed. The actuator only opens when receiving a signal and remains open for
the period of time that the signal occurs. For the rest of the time, the valve is
closed. Consequently, the width (actually the time interval) of the pulse controls
the open time of the expansion valve. Therefore, the fraction of time that the
valve is open regulates the refrigerant flow. Figure 8-10b shows a pulse-width
modulated actuator signal where pulses of varying widths are shown. One appli-
cation of electronic expansion valves in conjunction with pulse-width modula-
tion involves the simultaneous control of superheat (4.4°C–5.5°C) and the
refrigerating capacity.

Figure 8-10 Actuator signals for electronic expansion valves—(a) stepping motor and (b)
pulse-width modulation.
Fundamentals of Refrigeration SI, Second Edition 215

Hand Expansion Valves


The hand expansion valve is essentially a needle valve used to make fine
adjustments in the refrigerant flow to an evaporator. During the early days of
refrigeration, only hand expansion valves were available. An obvious disad-
vantage of these valves is the personal attention required to regulate the refrig-
erant flow. Virtually all domestic and commercial refrigeration systems in
operation today use an automatic valve to meter the refrigerant.
However, two applications of hand expansion valves remain important
today. One is the control of refrigerant flow in a bypass line around an auto-
matic valve. In conjunction with hand shutoff valves, the repairs to or replace-
ment of the automatic valve may be accomplished without shutting down the
refrigeration system.
The other application is in the field of industrial refrigeration. Typically,
recirculation systems use a number of evaporators arranged in parallel where
liquid is supplied intermittently by a temperature-controlled ON/OFF valve.
When the thermostat calls for cooling, the ON/OFF valve opens to the fully open
position. A hand expansion valve or balancing valve is used to reduce the
refrigerant flow to an acceptable level and to evenly distribute the refrigerant to
all the evaporators on the circuit. Hand expansion valves are also added down-
stream from ON/OFF valves metering the liquid refrigerant into a storage vessel.
To guard against huge pressure fluctuations inside the vessel resulting from the
ON/OFF action of the valve, the hand expansion valve is adjusted to lessen the
flow and, therefore, manage the pressure fluctuations. In this application, real-
ize that the hand expansion valve does not operate as a true expansion valve.

Level Control Valves


The level control valve, or float valve, is a type of expansion valve that controls
the liquid level in an evaporator or vessel. When the liquid level is constant in
an evaporator’s surge drum, the float valve maintains a balanced condition
between the metered refrigerant and the flow of vapor to the compressor. For
example, if the refrigerating load decreases, the evaporating temperature and
pressure decrease, which reduces the vapor flow to the compressor. Conse-
quently, the liquid level rises in the evaporator and causes the float valve to
close somewhat, which reduces the flow of liquid to the evaporator. In this way,
the level is controlled. Similar events occur when the refrigeration load
increases.
A typical high-side float valve is shown in Figure 8-11, where the inlet is
located on the left side of the body. The inlet accepts refrigerant liquid from the
condenser, which in turn raises the float. The float, connected to the float arm,
positions the valve pin in the port, which adjusts the extent of the opening.
After the liquid passes through the port, it expands and enters the evaporator.
Because the float valve necessarily controls a liquid level, the evaporator must
be the flooded type, and a significant portion of the refrigerant charge is in the
216 Chapter 8 Expansion Devices

Figure 8-11 Typical high-side float valve.

evaporator. At steady-state operation, the mass flow rate of refrigerant admitted


by the high-side float valve is equal to the flow rate of vapor to the compressor,
as it is for all expansion devices.
A high-side float valve is normally used in conjunction with one evapora-
tor, condenser, and compressor. A disadvantage of the high-side float valve is
that the system’s operation is very sensitive to refrigerant charge. When the
system has a greater-than-optimum charge, floodback of liquid to the compres-
sor occurs, and if the charge is less, then the system operates at reduced refrig-
erating capacity.
A low-side float valve also controls the level of liquid, but the float is
located in the low side of the refrigeration system. Figure 8-12 is a cross
section of a low-side float valve that shows the valve port serving as the ori-
fice regulating the liquid flow. Operation is similar to the high-side float
valve: if the liquid level falls, the float drops and the valve port opens, which
allows more refrigerant to pass.
In a low-side float valve, the float ball may be positioned inside the evapo-
rator or placed inside a small vessel or surge drum located above the evapora-
tor. Sometimes the float ball is located in a small chamber adjacent to the
evaporator or surge drum so the ball may be isolated from the boiling action of
the refrigerant liquid. Isolating the ball is important in those applications where
Fundamentals of Refrigeration SI, Second Edition 217

Figure 8-12 Typical low-side float valve.

the boiling action is strong, as the bubbles present in the evaporator or surge
drum may cause the ball to rise, generating an erroneously high level and caus-
ing an incorrect liquid feed to the evaporator. When the compressor stops, the
boiling ceases and the float ball drops, passing more refrigerant into the evapo-
rator to restore the liquid level. After a period of time, the compressor starts,
and if the evaporator is overfilled, flood-back may occur.
Furthermore, if the low-side float valve is in a separate chamber external to
the evaporator or surge drum, equalizing lines are required to connect the
chamber to the evaporator or surge drum. Equalizing lines are two generously
sized lines: a vapor line to the top and a liquid line to the bottom of the evapo-
rator or surge drum. The sizes of the equalizing lines must be sufficient to
counteract a potentially reversing effect of the liquid level near the float valve.

Turbo Expanders
In the vapor-compression cycle, all the expansion devices we have considered
so far have flashed the liquid refrigerant from the compressor to a low-pressure
and low-temperature vapor and liquid mixture. It may have occurred to you
218 Chapter 8 Expansion Devices

that the expanding refrigerant is capable of producing useful work that could
be returned to the system and decrease compressor power input. An expansion
device that does this is called a turbo expander. In this section, the application
of a turbo expander to a different refrigeration cycle is described. Next, a rela-
tively recent development where a turbo expander is used in a vapor-compres-
sion cycle is examined.

Gas-Cycle Refrigeration

A practical refrigeration cycle may be built using a gas (such as air) as the
refrigerant. In contrast to a vapor-compression cycle, the gas would only warm
and cool in heat exchangers. The basic gas refrigeration cycle is the Brayton
refrigeration cycle, shown in Figure 8-13. The component labeled “turbine” is
the expansion device, or turbo expander. As the gas flows through the turbo
expander, work is extracted, resulting in a drop in gas temperature and pressure.
The output shaft from the turbine is coupled to the input shaft of the compressor,
which reduces the external power (from a motor) that would otherwise be
needed to drive the system. A line for compressed makeup air is needed to over-
come leaks.
·
The refrigeration effect, labeled Q in , occurs from state 4 to state 1. Heat
·
rejection Q out occurs from states 2 to 3. The processes that the gas undergoes

Figure 8-13 Ideal Brayton refrigeration cycle.


Fundamentals of Refrigeration SI, Second Edition 219

for an ideal Brayton refrigeration cycle, where there are no losses in the piping
connecting the components, are given below:

1-2: isentropic compression

2-3: constant-pressure heat rejection

3-4: isentropic expansion

4-1: constant-pressure heat addition

An isentropic process (either compression or expansion) involves a simul-


taneous change in pressure and temperature from the initial state and depends
on the ratio of specific heats for the gas. This relationship is

k–1
T2 p 2 -----------
----- =  ----- k (8-4)
T1 p1

where k is the ratio of specific heats, cp/cv , T is the temperature (K), p is the
pressure (kPa), and subscripts 1 and 2 refer to the initial and final states, respec-
tively. Figure 8-14 shows the Brayton refrigeration cycle on a p-h diagram.
The Brayton refrigeration cycle is analyzed using the following equations:

·
1-2: W compr = m·  h 2 – h 1  = m· c p  T 2 – T 1 
·
2-3: Q out = m·  h 2 – h 3  = m· c p  T 2 – T 3 
· (8-5)
3-4: W turb = m·  h 3 – h 4  = m· c p  T 3 – T 4 
·
4-1: Q in = m·  h 1 – h 4  = m· c p  T 1 – T 4 

where m· is the mass flow rate of the gas. An ideal Brayton refrigeration cycle
has the conditions p1 = p4 and p2 = p3. The coefficient of performance (COP) is
given by

COP = Refrigeration effect-


----------------------------------------------------
Net power input
m· c p  T 1 – T 4 
= ----------------------------------------------------------------------
- (8-6)
m· c p  T 2 – T 1  – m· c p  T 3 – T 4 
 T1 – T4 
= --------------------------------------------------
 T2 – T1  –  T3 – T4 
220 Chapter 8 Expansion Devices

Figure 8-14 Pressure-enthalpy diagram for ideal Brayton refrigeration cycle

Using these expressions, it can be shown that

1
COP = ----------------------------
- (8-7)
– 1-
k----------
p
 ----2- k – 1
 p 1

When air is used as the refrigerant, typical low and high pressures are 690 kPa
(gage) and 2000 kPa (gage), respectively.

Example 8-3

Problem Determine the work extracted from the turbo expander and the COP of an ideal
Brayton refrigeration cycle using air as the refrigerant. The refrigerating capac-
ity is 88 kW. The low pressure is 690 kPa (gage) and the high pressure is
2070 kPa (gage). The temperature entering the compressor is –6.67°C, and the
temperature entering the turbo expander is 26.67°C. For air, the value of cp is
0.24 Btu/lb·°R and k is 1.4. The atmospheric pressure is 101.325 kPa.

Solution Using the diagram and labeled states shown in Figure 8-14, we have the fol-
lowing: p1 = p4 = 790.1 kPa (gage); p2 = p3 = 2170 kPa (gage); T1 = 266.67 K;
T3 = 300 K.
Fundamentals of Refrigeration SI, Second Edition 221

Applying an energy balance to the low-pressure evaporator or using


Equation 8-3 gives

·
Q in = m· c p  T 1 – T 4 

Need T4: Expansion across turbine is isentropic.

k – 1- 1.4 – 1-
p4 ----------
790.1 ---------------
T 4 = T 3  ----- k
=  300   ------------- 1.4
= 225 K
 p 3  2170 

So,

· Q· in  88 kW 
m = ----------------------------- = ---------------------------------------------------------------------
- = 2.1 kg/s or 125.6 kg/min
Cp  T1 – T4  kJ 
 1 -----------  266.67 K – 225 K 
 kg·K

· · ·
W net = W compr – W turb
= m· c p  T 2 – T 1  – m· c p  T 3 – T 4 

Need T2: Compression process is isentropic.

– 1-
k---------- 1.4 – 1-
p2 2170 ---------------
T 2 = T 1  ----- k
=  266.7   ------------- 1.4
= 356K
p1 790.1

So,

·
W net =  2.1   1   356 – 266.7  –  2.1   1   300 – 225 
= 30 kW

·
Q in  88 
· - = ----------
COP = ----------
 30 
 3.0
W net

The COP may also be computed using Equation 8-6:

1 1
COP = ----------------------------
- = ----------------------------------------
- = 3.0
k–1 1.4 – 1
p -----------
 ----2- k – 1 2170  1.4
 ------------
----------------
- –1
 p 1  790.1
222 Chapter 8 Expansion Devices

A prevalent application of gas-cycle refrigeration is commercial aircraft


cabin cooling. Figure 8-15 shows a simplified diagram of a representative sys-
tem where the cycle is designated as an open cycle; the refrigerant is released
to the environment and not recycled.

Vapor-Compression Refrigeration Cycle


Over the last several years, the turbo expander has also been applied to the
vapor-compression cycle. The major technical difficulty that has prevented the
use of turbo expanders is the damage resulting from the liquid impacting and
eroding the internal surfaces. Described here is one approach applied to cen-
trifugal compressors used in commercial water chillers.

Figure 8-15 Gas refrigeration cooling of an aircraft cabin


Fundamentals of Refrigeration SI, Second Edition 223

The approach taken was to carefully design an impulse-type turbine. This


turbine is composed of a fixed ring of nozzles and a rotor to which blades are
attached. The blades move past the nozzle exits where the energy in the refrig-
erant flow is first converted to kinetic energy and then imparted to the blades,
causing the rotor to rotate. Inside the nozzle, the liquid is flashing, and due to
the curvature of the nozzle’s internal surface, the liquid droplets break up into
smaller droplets. These small liquid droplets closely follow the flow direction
of the vapor, avoiding corrosion of the nozzle and blade surfaces. The rotor’s
blades were also specially designed to accommodate the presence of liquid,
where the blade’s exit was made particularly sharp to reduce friction. The tur-
bine efficiency achieved in this approach was greater than 40%.
The motivation behind the development and application of the wet turbo
expander was that a commercial water chiller using a centrifugal compressor
and R-134a as the refrigerant operates at about a 5% lower COP than one
using R-123. The application of the turbo expander offsets the penalty of
using R-134a.

The Next Step


Chapter 9 presents the types of vessels used in refrigeration systems. Also, the
application of vessels in regard to refrigerant storage and liquid-vapor separa-
tion is discussed.

Summary
The expansion device in a refrigeration system has two functions: reducing
the refrigerant temperature so that refrigeration occurs and separating the
high-pressure side from the low-pressure side. The flow of a liquid refriger-
ant may be regulated by a capillary tube, short tube restrictor, or a valve that
opens and restricts the cross-sectional area of its port. Capillary tubes and
short tube restrictors have fixed geometrical dimensions and so are designed
to meet the system’s design conditions. Expansion valves, on the other hand,
adjust to changing conditions of the refrigeration system maintaining design
conditions in the evaporator. Four valve types, based on their control action,
are applied to the vapor-compression refrigeration system. These are the
pressure control valve (evaporator pressure), the thermostatic expansion
valve (evaporator superheat at exit), the level control valve (liquid level), and
the hand expansion valve (manual control).
Another expansion device, the turbo expander, differs from the valves in
that it extracts work from the expanding refrigerant. The application of the
turbo expander includes the ideal Brayton refrigeration cycle using air as the
refrigerant. A new application of a turbo expander capable of accepting liquid
refrigerant has been used in conjunction with a centrifugal compressor. After
studying Chapter 8, you should be able to
224 Chapter 8 Expansion Devices

 explain the operation of the capillary tube and short tube restrictor when
feeding an evaporator;
 explain the operation of the pressure control valve when feeding an
evaporator;
 draw a diagram of a thermostatic expansion valve feeding an evaporator and
explain its operation;
 explain why hand expansion valves are used in refrigeration;
 describe the control action of either a low-side or high-side level control
valve;
 identify the advantage to a refrigeration system that a turbo expander pro-
vides; and
 describe the Brayton refrigeration cycle and identify its four basic com-
ponents.

References and Bibliography


ASHRAE. 2006. ASHRAE Handbook—Refrigeration. Atlanta: ASHRAE.
Brasz, J. 1995. Improving the refrigeration cycle with turbo-expanders. Pro-
ceedings of the IIR Conference, The Hague, The Netherlands.
Stoecker, W. 1988. Industrial Refrigeration. Troy, MI: Business News Publish-
ing Co.
Fundamentals of Refrigeration SI, Second Edition 225

Skill Development Exercises for Chapter 8


Complete these questions by writing your answers on the worksheets at the back of this book.

8-1 Refrigerant R-134a is used in conjunction with a capillary tube that has a bore
of 0.762 mm and length of 4060 mm. The operating conditions are 1448 kPa
condensing pressure and 0°C subcooling. Assume that heat transfer to the capil-
lary tube is negligible. Determine the mass flow of R-134a through the capillary
tube (kg/h). Also, if the same capillary tube operating at the same condensing
pressure had a quality of 20% at the inlet to the tube, what would be the new R-
134a mass flow rate?

8-2 Explain why the thermostatic expansion valve should be called a superheat-
controlled valve.

8-3 R-22 passes through a thermostatic expansion valve without external equalizer
lines. As supplied by the manufacturer, this valve is set to open at an evaporator
temperature of 1.67°C when the superheat is 5.56°C. What is the pressure dif-
ference between the top and bottom sides of the diaphragm needed to open the
valve? If the entering pressure to the evaporator is 690 kPa and the pressure
drop from the entrance to the exit of the evaporator is 13.8 kPa, what is the
degree of superheat needed to open the valve?

8-4 An expansion valve has a refrigerating capacity equal to 42.2 kW when the
pressure difference across the valve is 483 kPa. The conditions at which this
valve is rated include only R-22 liquid (no vapor bubbles) at 37.8°C entering
and an evaporating temperature of 4.44°C. Estimate the new rating of this
valve if the pressure difference increases to 1172 kPa.

8-5 Explain the control action of a low-side float valve when the refrigeration load
increases. Use a plot of the refrigerant mass flow rate versus the suction pres-
sure to explain your answer.

8-6 Reconsider the ideal Brayton refrigeration cycle from Example 8-3 where the
high-side pressure is now decreased to 1379 kPa (gage). All other values for the
cycle remain the same, including the air mass flow rate, which is 125.6 kg/min
or 2.1 kg/s. Determine the following: the net work rate required by the com-
pressor, the refrigeration rate, and the COP.

8-7 Reconsider Example 8-3 again, where all the cycle information is unchanged
with the exception that the isentropic efficiencies of the compressor and tur-
bine (turbo expander) are now 80%. Calculate the net work rate required by the
compressor, the refrigeration rate, and the COP.
Pressure Vessels and
Refrigerant Management

Study Objectives
Chapter 9 discusses the application of vessels that may be included in a refrig-
eration system. Vessels are used to store liquid refrigerant and to separate
vapor from liquid. The vessel types used are classified according to their loca-
tion and function in the system. Discussed in this chapter are low-pressure
receivers, high-pressure receivers, suction line accumulators, flash tanks, inter-
coolers, and surge drums.
Mathematical relationships yielding the liquid refrigerant volume associ-
ated with a specific liquid-level height are given to determine the refrigerant
mass in a system. The separation of vapor from liquid relies on the dimensions
of the vapor space above the liquid surface inside the vessel and on the velocity
of the vapor inside the vessel. After studying this chapter, you should be able to

 describe four classifications of vessels in a refrigeration system and the


function each performs;
 calculate the mass of refrigerant in a vessel from its internal dimensions and
the liquid depth;
 describe how a vessel accomplishes the separation of vapor from liquid; and
 explain how to determine the refrigerant charge in a system.

Instructions
Read the material in Chapter 9. Verify the examples presented in the chapter
with your own calculations. At the end of the chapter, complete the skill devel-
opment exercises without referring to the text. Review those sections of the
chapter as needed to complete the exercises.

Introduction
A vessel in a refrigeration system is a volume strategically located to store liq-
uid refrigerant, to separate vapor from liquid, or both. It is these functions that
clearly identify the vessels in a refrigeration system. Obviously, the piping
transporting refrigerant between components are not classified as vessels.
228 Chapter 9 Pressure Vessels and Refrigerant Management

Additionally, evaporator coils, condenser coils, pipe headers, pumps, and com-
pressors are not classified as vessels.
Design guidelines and codes provide specific definitions for what consti-
tutes a vessel. Generally, vessels in a refrigeration system serve as receptacles
for refrigerant, and because they must contain the refrigerant at a pressure
greater than atmospheric pressure, they are also pressure vessels. Under this
classification, important design codes must be adhered to in the manufacture,
certification, and installation of vessels. The governing authority for pressure
vessels is the Boiler and Pressure Vessel Code (ASME 2007). Two other codes,
ANSI/ASHRAE Standard 15-2007, Safety Standard for Refrigeration Systems
(ASHRAE 2007) and ANSI/IIAR Standard 2-1999, Equipment, Design and
Installation of Ammonia Mechanical Refrigeration Systems (IIAR 1999), spec-
ify important considerations for refrigeration systems.
Furthermore, all pressure vessels must be provided a means to relieve
excessive pressure that would otherwise rupture the vessel. Another code,
American Petroleum Institute’s Recommended Practice 520, Sizing, Selection
and Installation of Pressure-Relieving Devices in Refineries (API 2000),
describes acceptable methods for pressure relief. While the mechanical design
of pressure vessels is certainly important, this chapter focuses on the function
of the vessel.
Small refrigeration systems may not incorporate a vessel. In domestic sys-
tems where the load is relatively constant (such as with a chest freezer), the
control action of the refrigerant metering device is fast enough to prevent liquid
from entering the suction line and damaging the compressor. However, capil-
lary tubes are not effective at small refrigeration loads and may permit liquid to
enter the suction line. When the metering device cannot keep pace with the
load, a vessel in the suction line (or suction line accumulator) may be used to
trap the liquid, thus protecting the compressor. Other components sometimes
classified as vessels are hermetic compressor housings and refrigerant driers.
Larger refrigeration systems (especially industrial systems) utilize pressure
vessels. High-pressure receivers, flash tanks, and low-pressure receivers are
used in two-stage systems that employ liquid recirculation to the evaporators.
Receivers contain liquid, so they hold most of the system’s refrigerant charge.
Calculating the refrigerant inventory for a system involves estimating the liquid
volume in all vessels as well as in the piping, condenser, and evaporators. The
shape of the pressure vessel and the height of the liquid inside must be known
to determine the mass of refrigerant in a system. Surge drums are used in con-
junction with flooded evaporators to separate vapor from liquid and to handle
the occasional liquid volume entering during a defrost or sudden refrigerating
load increase.

Suction Line Accumulators


A suction line accumulator is a vessel inserted into the suction line to trap liquid
refrigerant and protect the compressor (see Figure 9-1). When a vapor/liquid
Fundamentals of Refrigeration SI, Second Edition 229

Figure 9-1 Suction line accumulator.

mixture enters the accumulator, the liquid falls to the bottom and the vapor
leaves from the top. Suction accumulators are needed when the metering device
on an evaporator is not able to follow the changes in refrigeration load and when
capillary tubes cannot completely shut off under very small loads. In these situ-
ations, liquid refrigerant may enter the suction line. The accumulator prevents
the liquid from entering the compressor and causing damage.
The separation of the liquid from the vapor is a complex process. It involves
the motion of liquid droplets in the vapor stream. Inside the suction accumula-
tor, the vapor velocity is reduced significantly, which, due to the action of grav-
ity, causes the liquid droplets to fall out of the vapor stream. The vapor
eventually makes its way to the top, and the droplets collect on the bottom of the
accumulator. The collected liquid vaporizes in the accumulator due to heat
transfer from the outside or by heat directly supplied to the accumulator.
Provision must also be made for the return of lubricating oil to the system.
Moreover, the suction accumulator must be large enough so that it does not
overfill with liquid.
In industrial refrigeration systems, a suction line accumulator may also be
installed for compressor protection. Multiple flooded evaporators and their
associated surge drums may occasionally spill liquid refrigerant into the suc-
tion line. A coil submerged in the trapped liquid may be used to increase the
vaporization rate by passing a warm refrigerant liquid through it. Another
method to remove the liquid is to pump it to the high-pressure receiver using an
appropriate positive-displacement pump and suitable piping.
230 Chapter 9 Pressure Vessels and Refrigerant Management

Receivers
Receivers are vessels in a refrigeration system used for storing refrigerant. If
the vessel is located immediately downstream of the condenser, it is called a
high-pressure receiver. The liquid level in a high-pressure receiver changes
depending on the operating condition of the system. In a two-stage system, a
vessel may be located in the low-pressure side and serve both the storage func-
tion as well as the separation of liquid and vapor function. This vessel is called
a low-pressure receiver and is sometimes used in conjunction with a liquid
recirculation system. Here, liquid is pumped from the low-pressure receiver to
the evaporators, and a saturated liquid/vapor mixture returns to the same ves-
sel. The vapor volume in the low-pressure receiver allows separation of the
vapor from the liquid so that only vapor enters the suction line. In this course,
we shall assume all vessels are cylindrical in shape unless information is pro-
vided to indicate otherwise.
Receiver vessels are most common in industrial and commercial refrigera-
tion systems. Here the storage function provides a volume of liquid refrigerant
to ensure liquid flow to the system as the refrigerating load changes from high
to low and vice versa. When the plant is at full load, less refrigerant is in the
evaporators, which causes the liquid level in the receiver to rise. Commercial
systems may or may not use receiver vessels, depending on the advantage that
a stored refrigerant liquid may provide in the operating flexibility of the sys-
tem. Refrigeration systems used in domestic applications, due to their simplic-
ity and small refrigerant charge, generally do not employ receivers.

High-Pressure Receivers
High-pressure receivers are generally cylindrical in shape and are usually ori-
ented horizontally. The liquid level adjusts up and down inside the vessel to
accommodate the operating conditions of the system. The low liquid level limit
is determined by the requirement that only liquid enter the liquid outlet line, as
indicated by a sight glass placed in the outlet liquid line showing bubbles. The
high liquid level limit is set so that a vapor volume will always exist at the top
of the vessel.
The estimation of the liquid volume in a receiver is important for determin-
ing the refrigerant mass or inventory in the system. For a cylindrical vessel ori-
ented upright, the volume of the liquid is given by


V f = --- D 2 H f (9-1)
4

where D is the internal diameter (m) of the vessel, Hf is the height of the liquid
(m), and Vf is the liquid volume in the vessel (m3). Typically, the mass of refrig-
erant in the vapor volume is negligible relative to that in the liquid and is,
therefore, not included in the calculation for refrigerant inventory.
Fundamentals of Refrigeration SI, Second Edition 231

The liquid volume in a horizontally oriented vessel is more difficult to cal-


culate. Figure 9-2 depicts a high-pressure receiver vessel where the liquid level
is shown as height Hf, measured from the bottom. With the aid of Figure 9-3,
the volume of the liquid is given by

R–H
cos  = ---------------f
R

Figure 9-2 Horizontally oriented cylindrical high-pressure receiver.


232 Chapter 9 Pressure Vessels and Refrigerant Management

Figure 9-3 Geometry associated with a horizontal cylindrical vessel.

Liquid Cross-Sectional Area


A f = R 2  ----------- – R  R – H f  sin 
 180

Liquid Volume


V f = A f L R 2  ----------- – R  R – H f  sin  (9-2)
 180

where
 = angle between the vessel’s vertical centerline and the radius drawn
toward the edge of the liquid surface, degrees
R = vessel inside radius, m
L = axial length of vessel, m
Hf = liquid height measured along vertical centerline from the bottom of
the vessel, m
Equation 9-2 applies when the vessel is less than half full. When the vessel
is more than half full, the liquid volume is computed using the following steps:

1. Compute the full internal volume of the cylindrical vessel.


2. Compute the vapor space volume above the liquid surface using Equation 9-2.
3. Subtract the vapor volume from the full volume to obtain the liquid volume.
Fundamentals of Refrigeration SI, Second Edition 233

Table 9-1 Variation of Horizontal Cylindrical Vessel Liquid Capacity with Depth
% Depth % Vessel % Depth % Vessel % Depth % Vessel % Depth % Vessel
Filled Capacity Filled Capacity Filled Capacity Filled Capacity
1 0.20 26 20.73 51 51.27 76 81.50
2 0.50 27 21.87 52 52.55 77 82.60
3 0.90 28 23.00 53 53.81 78 83.68
4 1.34 29 24.07 54 55.08 79 84.74
5 1.87 30 25.31 55 56.34 80 85.77
6 2.45 31 26.48 56 57.60 81 86.77
7 3.07 32 27.66 57 58.86 82 87.76
8 3.74 33 28.84 58 60.11 83 88.73
9 4.45 34 30.03 59 61.36 84 89.68
10 5.20 35 31.19 60 62.61 85 90.60
11 5.98 36 32.44 61 63.86 86 91.50
12 6.80 37 33.66 62 65.10 87 92.36
13 7.64 38 34.90 63 66.34 88 93.20
14 8.50 39 36.14 64 67.56 89 94.02
15 9.40 40 37.36 65 68.81 90 94.80
16 10.32 41 38.64 66 69.97 91 95.50
17 11.27 42 39.89 67 71.16 92 96.26
18 12.24 43 41.14 68 72.34 93 96.93
19 13.23 44 42.40 69 73.52 94 97.55
20 14.23 45 43.66 70 74.69 95 98.13
21 15.26 46 44.92 71 75.93 96 98.66
22 16.32 47 46.19 72 77.00 97 99.10
23 17.40 48 47.45 73 78.14 98 99.50
24 18.50 49 48.73 74 79.27 99 99.80
25 19.61 50 50.00 75 80.39 100 100.00

Table 9-1 shows a horizontal vessel’s capacity as a function of the liquid


depth, both of which are given in percentages. Use of the table eliminates the
tedious calculations associated with Equation 9-2. The ends of the vessel are
assumed to be flat in Equation 9-2 and in the calculations for Table 9-1; how-
ever, most vessels have rounded ends. For accurate volume calculations, the
volume enclosed by the rounded ends should be included.
The liquid refrigerant mass is determined by

Mf = f Vf (9-3)
234 Chapter 9 Pressure Vessels and Refrigerant Management

where Mf is the liquid mass (kg) and f (kg/m3) is the density of the saturated
liquid in the vessel. Recall that the refrigerant mixture in the vessel is at equi-
librium, so both the vapor and liquid are at saturation conditions.

Example 9-1

Problem Consider a horizontally oriented vessel used as a high-pressure receiver in an


ammonia refrigeration system where the temperature inside the vessel is
32.22°C. The internal diameter is 1.22 m and the internal length is 2.44 m. The
liquid depth is known to be 40% of the diameter. Determine (1) the mass of liq-
uid ammonia in the receiver, (2) the ratio (%) of ammonia liquid volume in the
receiver to the receiver’s total volume, (3) the mass of ammonia vapor in
receiver, and (4) the ratio (%) of ammonia vapor mass to total ammonia mass in
the receiver.

Solution 1. Depth of liquid in high-pressure receiver = Hf = (0.40)(1.22) = 0.5 m


Use Equation 9-2 to obtain the liquid volume:

R – Hf
cos  = ---------------
R R = 0.61 m
– 0.5- = 0.200  = 78.46
= 0.61
-----------------------
0.61


V f = L R 2  ----------- – R  R – H f  sin 
 180
78.46
=  2.44 m      0.61 m  2  ------------- –  0.61 m   0.61 – 0.5 m  sin  78.46 
 180 
= 1.063 m 3

Using the saturation property table for ammonia, f (at 32.22°C) =


591.88 kg/m3. The liquid mass is

kg
M f =  f V f =  591.88 ------3-  1.063 m 3  = 629 kg (liquid)
 m 

2. To obtain the liquid volume to total volume ratio, first determine the total
receiver volume.

 
V total = --- D 2 L =  ---  1.22  2  2.44  =  2.85 m  3
4  4
Fundamentals of Refrigeration SI, Second Edition 235

Divide the calculated liquid volume above, 1.063 m3, into the total vol-
ume, 2.85 m3, multiplied by 100, for a liquid volume percentage of 37%.
Note that a measured liquid depth of 40% of the receiver height only
reflects 37% of the total volume due to the horizontal cylindrical shape.
3. To obtain the vapor mass, first determine the vapor volume inside the
receiver by subtracting the liquid volume from the total internal volume:

V v = V total – V f = 2.85 – 1.063 = 1.787 m 3

The saturation table gives the vapor specific volume at 32.22°C as vg =


0.104 m3/kg:

V
M v = -----v = 1.787
------------- = 17.2 kg (vapor)
vg 0.104

4. The ammonia vapor mass to total ammonia mass ratio (%) is given by

17.2
-------------------------  100%  = 2.66%  2.7%
629 + 17.2

The vapor mass computed for the high-pressure receiver vessel in


Example 9-1 is small compared to the liquid mass. For this receiver, the
percentage of the total mass that is vapor is only 2.7%. This is a typical
result, so when estimating the refrigerant mass in the vessel or the entire
system, the vapor mass is usually neglected.

When high-pressure receivers are used in small refrigeration systems oper-


ating on a seasonal schedule, the volume is such that the receiver can store the
entire refrigerant inventory. To move the refrigerant from the system to the
high-pressure receiver, the king valve (the main shut-off valve in the line sup-
plying liquid to the refrigeration system, as shown in Figure 9-2) is closed and
the compressor operates until no more refrigerant is pulled from the system’s
low-pressure side. Condensation of the vapor in the condenser and drainage of
the liquid into the high-pressure receiver complete the refrigerant’s movement.
At this point, the valve between the condenser and high-pressure receiver is
closed. Now, the refrigerant charge is in the high-pressure receiver. This pro-
cess is called pump down.
In these systems, the receiver must have a volume sufficient to store all the
refrigerant with a reasonable vapor space above the liquid surface. Addition-
ally, the receiver’s volume and the refrigerant charge in the system must always
provide a liquid volume in the vessel, even if the remaining system compo-
nents are fully charged. Typically, when using a horizontally oriented cylindri-
cal vessel, the vertical height of the vapor space at the top is limited to one-
236 Chapter 9 Pressure Vessels and Refrigerant Management

quarter to one-third of the internal diameter when containing the full refriger-
ant charge. The low liquid level limit for cylindrical vessels is usually about the
same: one-quarter to one-third of the internal diameter. To guarantee that the
high-pressure receiver will never contain only liquid, a vapor space is needed
above the liquid. If the vessel were inadvertently filled with liquid and the
valves were closed, a small temperature increase would generate huge internal
pressures and likely rupture the vessel. The normal practice is to size the
receiver so that when the complete refrigerant charge is in it, the receiver is
filled to approximately 80% to 88% of its total internal volume.
Large refrigeration systems (particularly industrial systems) operate con-
tinuously throughout the year, so the receiver vessels in these plants are not
sized to contain the complete charge in the system. Two approaches are used to
select high-pressure receivers for relatively large systems:

• Store a sufficient quantity of liquid refrigerant so that the refrigeration sys-


tem may maintain normal operation for a specified period of time, say 30
minutes. Consequently, the system liquid supply may be interrupted for a
period of time without shutting down the compressors.
• Provide sufficient storage volume to accommodate the largest unit or
refrigerated room in the system for pump down. This allows the largest unit
or room to be shut down for maintenance.

A diptube is a pipe entering the top of the high-pressure receiver that con-
tinues vertically down until it nearly reaches the bottom of the vessel. This tube
allows the removal of liquid refrigerant from the system when the receiver ves-
sel cannot contain all the liquid that must be stored.

Low-Pressure Receivers

The low-pressure receiver, used in recirculation systems, accomplishes both


the separation of vapor from liquid and the storage of refrigerant. Figure 9-4
shows a typical low-pressure vertically oriented receiver configured for the
separation of vapor and liquid. The liquid level shown is the controlled level
maintained by the system under design conditions. However, the total vessel
volume is selected in conjunction with individual concerns, each of which
results in a volume segment for the vessel. The reasons and details affiliated
with each volume segment are as follows:

• The vapor volume above the maximum operating liquid level must be suffi-
cient to separate the liquid from the vapor. The separation process depends
on several variables that are discussed later in this section.
• The volume between the maximum operating liquid level and the controlled
level accommodates the surge of liquid that may occur. Often this volume is
Fundamentals of Refrigeration SI, Second Edition 237

Figure 9-4 Typical low-pressure vertically oriented cylindrical receiver.


238 Chapter 9 Pressure Vessels and Refrigerant Management

referred to as the surge volume. About 30% of the vessel volume is reserved
for the surge volume. Liquid surge occurs from the defrost of an evaporator.
Also, liquid surge occurs in a recirculation system when the pumping is
stopped because the liquid-vapor return lines are pitched downward to the
low-pressure receiver. The interruption of electrical power is one instance
that would cause these lines to drain into the low-pressure receiver.
• The volume between the controlled level and the low level is called ballast.
This liquid quantity is necessary to start the system or to start an evaporator
after it has been out of service. When starting, the pump supplies refriger-
ant to the evaporators in the circuit (including any empty evaporators) at a
rate greater than that returned by the liquid-vapor return line, thus tempo-
rarily lowering the liquid level from the controlled level. Roughly 25% of
the vessel volume is designated for ballast.

Both vertical and horizontal orientations are used in the industry. When
pumped liquid recirculation is used, vertical orientation gives a higher net posi-
tive suction head (NPSH) for a given liquid volume, which is particularly import-
ant for ammonia because of its small liquid density. The vertical low-pressure
receiver also reduces the required floor space. However, the vertical receiver may
need a high ceiling in the mechanical room to accommodate its size. If ceiling
height is not adequate, the horizontal orientation must be used along with the
increased floor space. The NPSH requirement must be carefully checked when
using horizontal vessels in conjunction with pumped liquid recirculation.
Figure 9-4 shows a cylindrical vessel placed upright where the vapor/liquid
mixture inlet is on the right. Note that the inlet pipe inside the low-pressure
receiver directs the entering flow downward. A representative value for the
total low-pressure receiver volume is the sum of the internal volumes for the
evaporators and liquid/vapor lines.
The processes associated with the separation of liquid and vapor are compli-
cated. Presented here are the important principles that govern separation. Upon
entering the suction line, the liquid/vapor mixture appears as a stream of liquid
moving along the bottom of the pipe, while the remainder of the liquid in the
form of droplets sweeps through the pipe by the flow of vapor. After entering
the low-pressure receiver, most of the liquid stream falls toward the bottom of
the vessel due to gravity and joins the liquid already there. The remaining liquid
is contained in an array of droplets that vary in diameter. The key to separation
is to have the upward vapor velocity in the vessel small enough so that the large
droplets fall downward to the liquid below. Typically, the droplet size distribu-
tion is large, and small droplets will exit with the vapor. The separation cannot
be perfect, because increasing the vessel volume allocated to separation
decreases the droplet size escaping with the vapor flow. Consequently, small
droplets will escape, and vessel selection must be based on determining the larg-
est escaping droplet size. The designer usually resolves this issue using experi-
ence gained from the satisfactory operation of previous installations. However,
Fundamentals of Refrigeration SI, Second Edition 239

the consideration of the separation process yields reasonable guidelines that are
useful for sizing low-pressure receivers, as discussed in the following.

Vertical Vapor Flow


In the vertical vapor flow configuration shown in Figure 9-5, the liquid droplets
enter the space provided for separation. A droplet falling in a still vapor even-
tually reaches a maximum velocity called the terminal velocity. At the terminal
velocity, the gravitational pull on, or weight of, the droplet is balanced by the
resistance or drag force on the droplet pushing it up. An expression for the ter-
minal velocity of a droplet is obtained by equating these two forces.
Figure 9-6 shows such a droplet with the two opposing and equal forces
resulting in a zero net force. The gravitational force on the droplet is

d 3
F g = ---------   f –  v  (9-4)
6
where
Fg = force on droplet due to gravity, kg
d = droplet diameter, m
f = liquid density, kg/m3
v = vapor density, kg/m3
The drag force on the droplet results from its motion through the vapor and is

d 2 V 2
F d = C D  v  ---------  ------ (9-5)
 4   2g

where
FD = drag force on droplet, kg
CD = drag coefficient, dimensionless
V = relative velocity between droplet and surrounding vapor, m/s
g = gravitational acceleration, 9.81 m/s2
The drag coefficient depends on the droplet’s Reynolds number (v dV/µv),
where µv is the vapor viscosity. Table 9-2 shows values of the drag coefficient
for several values of the Reynolds number where the droplet shape is approxi-
mated as a sphere.
Setting the gravitational and drag force equal to each other yields an
expression for the terminal velocity:

4gd   f –  v 
Vt = ------------------------------- (9-6)
3 v C D

Figure 9-7 shows the terminal velocities for two refrigerants, ammonia and
R-22. The calculations use a 0.4 mm droplet size for ammonia and a droplet
size of 0.25 mm for R-22. These droplet sizes are selected because they match
240 Chapter 9 Pressure Vessels and Refrigerant Management

Figure 9-5 Operation of a vertically oriented low-pressure receiver.


Fundamentals of Refrigeration SI, Second Edition 241

Figure 9-6 Liquid droplet showing gravity and drag forces.

Table 9-2 Variation of Drag Coefficient with Reynolds Number for a Sphere
Reynolds Number Drag Coefficient
1 30.0
10 4.0
100 1.1
1000 0.4

the separating velocities reported in the 2006 ASHRAE Handbook—Refrigera-


tion (ASHRAE 2006). Also observe that the terminal velocity increases with
decreasing saturation temperature.
The terminal velocity is the highest velocity that a droplet will fall in a qui-
escent vapor. If the upward vapor velocity in a low-pressure vessel is equal to
the terminal velocity of a certain droplet size, then the droplet will neither fall
nor rise and is said to have the critical diameter. Larger droplets fall and
smaller droplets rise. Consequently, the terminal velocities given in Figure 9-7
are the separating velocities for vessels with upward vapor flow.
242 Chapter 9 Pressure Vessels and Refrigerant Management

Figure 9-7 Terminal velocities of ammonia and R-22 droplets.

Example 9-2

Problem Determine the diameter of a low-pressure receiver oriented vertically that con-
tains R-22 at –17.78°C. Assume that the critical droplet diameter is 0.25 mm.
The vapor flow entering the low-pressure receiver is 42.5 m3/min.
Fundamentals of Refrigeration SI, Second Edition 243

Solution Volumetric flow rate is given by the product of the cross-sectional area through
which it flows and the average flow velocity. In an upright cylindrical vessel,
the vapor exits at the top and so flows upward inside. If the cross-sectional area
is Av , then the volumetric flow rate is

Q = Av V

where V is the upward velocity. Taking the critical droplet diameter to be


0.25 mm, the vapor velocity must equal the terminal velocity Vt for this drop-
let. Droplets larger than the critical droplet diameter are separated from the
vapor. From Figure 9-7,

 t  36.6 m/min

now,


A v = --- D 2
4

where D is the vessel internal diameter. Substituting,


Q = --- D 2 V t
4
4Q =  4   42.5  = 1.22 m
D = -------- -----------------------
V t     36.6 

Note that the calculation in Example 9-2 assumes that the vapor velocity is
uniform at any cross section in the vessel’s vapor space. Therefore, the esti-
mated value of the vessel’s diameter is only valid if the uniform velocity
assumption is valid. Furthermore, the size of the vessel depends on the system’s
operation. If the system automatically starts and stops every day, the low-pres-
sure receiver’s temperature will be much higher when starting up. For example,
for a system operating at a –6.67°C evaporating temperature, the receiver tem-
perature may reach 7°C to 10°C. If the low-pressure receiver is sized based on
continuous operation, it will be much too small for automatic start up. Liquid
carryover may also occur if the vapor outlet is not at least 1 m from the vapor/
liquid mixture inlet.

Horizontal Vapor Flow


Separation of liquid and vapor in horizontal vessels employs a different con-
cept. Figure 9-8 shows a simplified horizontal vessel where the liquid/vapor
mixture enters at both ends and the vapor exits from the center. The entering
droplets are dragged along by the vapor in a horizontal direction toward the
244 Chapter 9 Pressure Vessels and Refrigerant Management

Figure 9-8 Critical droplet trajectories in horizontal receiver.

center of the vessel. Due to gravity, the droplets also begin falling toward the
liquid surface below. Separation is accomplished when the droplet impacts the
liquid surface but only occurs if the droplet has had sufficient time to fall all the
way through the space occupied by the vapor. In other words, the time a droplet
takes to move horizontally must be greater than the time needed to fall down-
ward and impact the liquid surface. Defining the residence time that a droplet
may be in the vessel as the horizontal distance traveled by the vapor inside the
vessel divided by the velocity of the vapor, we have an approach for the design
of the separating space. Calculation of droplet displacement, velocity, and
acceleration is possible using only the gravity and drag forces acting on a drop-
let previously investigated.
Figure 9-9 shows results that were obtained for ammonia and R-22 using the
previous droplet sizes for saturation temperatures of +4.44°C and –40°C. The
falling time is shown on the x-axis, and the distance fallen is shown on the y-
axis. For example, take saturated R-22 at –40°C where the droplet size is
0.25 mm. If the falling time is 0.7 s, then the distance fallen is about 0.6 m. The
dimensions of the vapor space establish the critical droplet diameter. With Fig-
ure 9-9, we may determine the vapor space dimensions using the indicated
droplet sizes as the critical sizes. Consider a low-side receiver vessel, cylindrical
in shape and horizontally oriented with the same internal features as those
shown in Figure 9-8. If the horizontal vapor velocity is V and the travel distance
Fundamentals of Refrigeration SI, Second Edition 245

©Business News Publishing Co. Reprinted with permission.

Figure 9-9 Results for ammonia and R-22 droplets.

is L/2, then the residence time is L/(2V). Entering Figure 9-9 with this residence
time, the distance dropped is determined. This is the required vertical dimension
for the vapor space to collect droplets equal to the critical size. By attempting
several combinations of vessel length (L), vessel internal diameter (D), and liq-
uid height (Hf), a suitable vessel may be sized.

Example 9-3

Problem A vessel, oriented horizontally, serves as a low-pressure receiver in a system


that uses ammonia as the refrigerant. The temperature of the ammonia is –40°C
The vessel’s internal configuration is similar to that shown in Figure 9-8 where
the vapor/liquid mixture enters from the top at each end and where the vapor
leaves moving upward from the center. The liquid in the vessel occupies 60% of
the vessel’s internal volume. The total vapor volume flow rate entering the ves-
sel is 79.3 m3/min. If the internal diameter of the vessel is fixed at 1.22 m, esti-
mate the length of this vessel for a critical droplet size equal to 0.4 mm Assume
that the ends of the vessel are flat.
246 Chapter 9 Pressure Vessels and Refrigerant Management

Solution Vessel internal diameter, D = 1.22 m


Vessel internal radius, R = 0.61 m
Liquid depth, Hf = (0.579)(1.22) = 0.71 m
Vapor depth, Hv = 1.22 – 0.71 = 0.51 m
Vapor volumetric flow rate, Q = 79.3 m3/min

Using Equation 9-2, calculate the vapor cross-sectional area:


R–H – 0.51- = 0.16,
cos  = ---------------v- = 0.61
--------------------------  = 80.79
R 0.61
A v = vapor cross-sectional area

= R 2  ----------- – R  R – H v  sin 
 180
80.79
=     0.61  2  ---------------- –  0.61   0.61 – 0.51   sin  80.79  
 180 
= 0.465 m 2

Use Figure 9-9 to estimate the residence time associated with a falling dis-
tance equal to Hv: t = residence time  0.4 s.
The horizontal travel distance due to the configuration inside is

L
--- = horizontal travel distance = Vt
2

where

Q
V = -----
Av

With the vapor flow divided, only half of the total flow enters the receiver
at either end. Looking at only the right (or left) half of the receiver, we have

L  0.5 Qt
--- = -------------------
2 Av
m 3 1 min
 0.5   79.3 ---------  --------------  0.4 s 
 min  60 s 
 ---------------------------------------------------------------------------
 0.465 m 2 
 0.568 or 0.57 m

Therefore, the total horizontal length required is (0.57)(0.57) = 0.325 m.


The 0.325 m estimated is based only on the residence time that a droplet of
Fundamentals of Refrigeration SI, Second Edition 247

critical diameter spends inside the receiver. Practical considerations, includ-


ing the turning of the vapor flow at the end of the vessel and locations where
the local vapor velocity exceeds the average velocity significantly, increase
the 0.325 m estimate.
Note that for Example 9-3, the average falling velocity is 0.51 m/0.4 s, or
1.28 m/s. The terminal velocity at the same conditions for a 0.4 mm critical
droplet is approximately 122 m/min, or 2 m/s. As expected, the droplet in hori-
zontal vapor flow has an average velocity less than its terminal velocity. This is
because in horizontal flow, the droplet velocity at the beginning of its fall is not
yet at the terminal velocity, Vt .

Surge Drums
Surge drums are vessels used with flooded evaporators to provide both storage
of the refrigerant and separation of vapor and liquid. The surge drum is
located a short distance above the coil and may be oriented horizontally or
vertically, depending on the space available. Flooded coils generate liquid
surge for two reasons. First, when the hot-gas defrost piping arrangement
directs high-pressure vapor to the top of the coil, the liquid in the coil is forced
downward toward the bottom of the coil and then back up the vertical leg and
into the surge drum. The size of the surge drum must accommodate this liquid
surge and prevent it from entering the suction line. A modification to this hot-
gas defrost method may include the addition of a suction line accumulator so
that liquid spilling out from the surge drum is trapped. The advantage is that a
smaller surge drum may be used.
The second reason flooded coils generate liquid surge originates from a
flooded coil operating at low load and experiencing a sudden increase in load.
The refrigerant in the coil begins to vigorously boil and drives a significant
fraction of the liquid into the surge drum.
The separation of liquid and vapor in a surge drum is governed by the same
principles discussed for low-pressure receivers. When surge drums feed one or
more air coils or water-chiller heat exchangers (shell-and-tube or plate-and-
frame), the ballast should be sized for the full amount of coil or heat exchanger
volume. During a “hot” start up (all system components at room temperature),
a quantity of liquid equal to the entire volume of the coils or heat exchangers is
instantly drained into the surge drum. Horizontal vessels used as surge drums
must be sized carefully to have the necessary residence time.

Flash Tanks and Intercoolers


The flash tank, applied to two-stage vapor compression systems, was investi-
gated in Chapter 3. The insertion of a flash tank as an intercooler increases the
refrigeration effect by interrupting the expansion process and cooling the
refrigerant vapor from the booster compressor. Recall that two separate com-
248 Chapter 9 Pressure Vessels and Refrigerant Management

pression stages are needed; no increase in the refrigeration effect occurs when
the condensed liquid is flashed all the way to the suction pressure.
Figure 9-10 shows a flash tank configured to serve as a cooler (or inter-
cooler) because it is placed between the booster and high-stage compressors.
The function of the flash tank is the separation of liquid and vapor, so it is usu-
ally sized for this purpose. An exception occurs when evaporators, operating at
intermediate temperatures, discharge refrigerant into the flash tank where liquid
surge volume is needed to handle the liquid pushed out from the evaporator and
into the suction line during a hot-gas defrost.
The flash tank in Figure 9-10 cools or desuperheats the discharge vapor
from the booster compressors by bubbling the refrigerant vapor through the
liquid. The vapor is in direct contact with the cooler liquid. The cooling of the
vapor is caused by a portion of the liquid in the flash tank evaporating. Inter-
cooling requires vigorous mixing between the vapor bubbles and liquid.
Because separation requires smooth flow patterns in opposition to the vigorous
pattern needed for intercooling, a compromise must be sought to accommodate
both processes. Another method to desuperheat the discharge vapor from the
booster compressor is to spray liquid refrigerant at a carefully regulated rate
into the line before it enters the flash tank.

Refrigerant Inventory
The mass of the refrigerant in a refrigeration system is first determined when
the system is designed. The quantity of refrigerant mass (or charge) is needed

Figure 9-10 Flash tank and intercooler.


Fundamentals of Refrigeration SI, Second Edition 249

so that the system is properly charged at the beginning of operation. Also,


safety requirements and reporting procedures to government agencies depend
on how much refrigerant is in the system.
By considering the liquid volume in a vessel resulting from the liquid
depth, we have completed an important task in the determination of a system’s
refrigerant charge. Exact calculations for the liquid volumes in the components
that contain both liquids and vapor are not available. However, estimates may
be made with the approximations given in the following.

Evaporator
Liquid Recirculation.
Bottom feed. Liquid fills approximately 80% of the internal volume.
Top feed. Liquid fills approximately 30% of the internal volume.
Direct Expansion. Liquid fills approximately 15% of the internal volume.

Condenser
Refrigerant Inside Tubes. Assume linear change of vapor to liquid, so liquid
volume is 30% to 50% of total volume. The total liquid and vapor mass in the
condenser is

Vf
M total cond = V cond  V +  -------------   f –  V 
 V cond

where Vcond is the total internal volume of the condenser, Vf is the condenser
internal volume occupied by liquid, v is the vapor density, and f is the liquid
density. Typically, the vapor density is much less than the liquid density in
refrigeration systems, so Mtotal cond may be approximated by

Vf
M total cond  V cond  -------------  f =  f V f
 V cond 

Liquid/ Vapor Return Lines


From evaporators in a recirculation system, liquid volume is

v  n – 1 
V pipe  ---------------------------------
  f +  v  n – 1 

where Vpipe is the internal volume of the pipe and n is the recirculation ratio.
The recirculation ratio is defined as the ratio of the refrigerant mass flow rate
supplying the evaporator-to-evaporating mass rate inside the evaporator.
250 Chapter 9 Pressure Vessels and Refrigerant Management

The Next Step


Chapter 10 discusses the refrigerants used as working fluids in vapor-compression
systems. The refrigerants now available to the industry have changed. This shift
occurred because several of the previously used refrigerants were found to con-
tribute to the depletion of the atmosphere’s ozone layer and to global warming.

Summary
Vessels in a refrigeration system serve three functions: store liquid refrigerant,
separate vapor from liquid, and protect the compressor. Suction line accumula-
tors are storage vessels located in the suction line upstream from the compres-
sor. They prevent liquid in the suction line from entering the compressor.
Receivers are vessels that are normally employed in larger refrigeration sys-
tems, particularly in industrial systems.
The high-pressure receiver, whose primary function is the storage of liquid
refrigerant, is placed immediately downstream from the condenser. The volume
of the high-pressure receiver should accommodate the liquid volume of the larg-
est subsystem so that repairs can be made while operating the remainder of the
plant. Another concern that influences the volume of the high-pressure receiver
is how long the liquid supply should be continued if the liquid flow from the
condensers is interrupted. This allows for a period of time that the compressors
continue to operate without shutdown. The receiver must also be sized to handle
time variations in the refrigerating load.
The low-pressure receiver provides both liquid storage and separation of liq-
uid and vapor. To accomplish the separation, the vapor volume must be suffi-
cient to allow gravity to pull the droplets downward for collection by the liquid.
Flash tanks and intercoolers are used in two-stage systems at an intermedi-
ate pressure for separation and desuperheating vapor. In a two-stage system,
the flash tank interrupts the expansion process, providing the cold liquid with
reduced compressor power input. Also, the same flash tank incorporates cool-
ing of the booster compressor discharge vapor, therefore limiting the final dis-
charge temperature from the high-stage compressor. Surge drums are vessels
that separate the liquid/vapor mixture exiting from flooded evaporators. After
studying Chapter 9, you should be able to

 explain the need and operation of a suction line accumulator;


 describe the two functions associated with high- and low-pressure receivers;
 calculate the liquid volume and liquid mass in vertically and horizontally
oriented receiver vessels;
 explain the meaning of critical droplet diameter;
 explain the liquid droplet separation process for vertically and horizontally
oriented vessels;
Fundamentals of Refrigeration SI, Second Edition 251

 determine the dimensions required for the vapor space in a low-pressure


receiver, oriented either vertically or horizontally, needed for the separation
of vapor and liquid;
 describe the function of the flash tank and intercooler applied to a two-stage
vapor-compression system;
 explain why surge drums are used in conjunction with flooded evaporators;
 calculate the liquid surge volume required in a low-pressure receiver; and
 calculate the approximate refrigerant mass in a system.

References and Bibliography


API. 2000. Recommended Practice 520, Sizing, Selection and Installation of
Pressure-Relieving Devices in Refineries. Washington, DC: American
Petroleum Institute.
ASHRAE. 2007. ANSI/ASHRAE Standard 15-2007, Safety Standard for
Refrigeration Systems. Atlanta: ASHRAE.
ASHRAE. 2006. ASHRAE Handbook—Refrigeration. Atlanta: ASHRAE.
ASME. 2007. Boiler and Pressure Vessel Code. New York: American Society
of Mechanical Engineers.
IIAR. 1999. ANSI/IIAR-1999, Equipment, Design and Installation of Ammonia
Mechanical Refrigeration Systems. Washington, DC: International Institute
of Ammonia Refrigeration.
Stoecker, W. 1988. Industrial Refrigeration. Troy, MI: Business News Publish-
ing Co.
252 Chapter 9 Pressure Vessels and Refrigerant Management

Skill Development Exercises for Chapter 9


Complete these questions by writing your answers on the worksheets at the back of this book.

9-1 Describe the considerations important to determine the size for a suction line
accumulator. Use the concepts presented in association with high- and low-
pressure receivers to assist in your explanation.
9-2 A cylindrical vessel, oriented horizontally, is used as a high-pressure receiver in
a refrigeration system using R-22. The internal diameter is 1.83 m and the
length is 3 m. The liquid depth in the receiver is 65% of the internal diameter.
The temperature of the R-22 is 29.4°C. (1) Find the mass of liquid using Equa-
tion 9-2, (2) verify your result using Table 9-1, and (3) calculate the percentage
of the total R-22 mass in the receiver that is vapor. Assume the ends of the ves-
sel are flat.
9-3 In an ammonia refrigeration system, a vertically oriented cylindrical vessel is
used as a low-pressure receiver. The temperature of the ammonia mixture
entering is –6.67°C, and the temperature inside the receiver is –6.67°C. The
volume flow rate of the vapor entering is 56.6 m3/min. For a critical droplet
diameter equal to 0.04 mm, estimate the required diameter of the receiver to
achieve separation. Assume the ends of the vessel are flat.
9-4 R-22 is used in a liquid recirculation refrigeration system where a low-pressure
receiver is 60% full of liquid, oriented horizontally, and at a temperature of –40°C.
The internal diameter of the vessel is 1.5 m, and its length is 3 m. If the vapor/liq-
uid mixture returned to the vessel enters at only one end and exits at the opposite
end, what is the largest vapor volume flow rate that the receiver can accept and
separate droplets at 0.25 mm in diameter? Assume the ends of the vessel are flat.
9-5 An ammonia refrigeration system has the following equipment and characteris-
tics for the low-pressure portion:
Refrigerant Ammonia
Low temperature –29°C
Recirculation ratio, n 4
Evaporator penthouse 4 evaporator units
Evaporator feed type bottom feed
Evaporator unit internal volume 0.24 m3
Liquid supply line internal diameter 5.25 cm
Liquid supply line length 152.4 m
Vapor/liquid return line internal diameter 154.05 mm
Vapor/liquid return line length 152.4 m
Condenser internal volume 0.93 m3
Determine (1) the liquid surge volume (m3) needed in the low-pressure
receiver in the event of a power failure, (2) the liquid surge volume (m3) asso-
ciated with the defrost of one evaporator coil, and (3) the total refrigerant mass
(kg) in the low-pressure portion of the system.
Refrigerant Selection

Study Objectives
Chapter 10 presents several concerns surrounding the selection and use of
refrigerants. The design of a refrigeration system involves selecting an appro-
priate refrigerant for the system. A number of considerations enter into this
decision: performance, safety, reliability, environmental impact, and eco-
nomic viability. The thermodynamic characteristics of the refrigerant, particu-
larly the saturation temperature and pressure, influence the performance of the
refrigeration system. Availability of the refrigerant is a concern. The refriger-
ant’s contribution to global warming may also be a factor in the selection pro-
cess. Recently, the production of several common refrigerants ceased due to
the refrigerants’ reduction of ozone concentration in the upper atmosphere of
the Earth.
Refrigerants are classified by ANSI/ASHRAE Standard 34-2007, Number
Designation and Safety Classification of Refrigerants (ASHRAE 2007a)
according to their flammability and toxicity, which relate directly to the safety
concerns associated with that refrigerant. ANSI/ASHRAE Standard 15-2007,
Safety Standard for Refrigeration Systems (ASHRAE 2007b) establishes
requirements and guidelines for the design of refrigeration systems. After
studying this chapter, you should be able to

 explain why the list of available refrigerants has changed;


 describe the characteristics of a refrigerant that are important to its classifi-
cation according to ASHRAE Standard 34;
 identify the refrigerant concentrations in air that characterize the toxicity
level for two refrigerants; and
 identify two standards that are important in the selection of refrigerants and
in the design of refrigeration systems.

Instructions
Read the material in Chapter 10. Verify the examples presented in the chapter
with your own calculations. At the end of the chapter, complete the skill devel-
opment exercises without referring to the text. Review those sections of the
chapter as needed to complete the exercises.
254 Chapter 10 Refrigerant Selection

Introduction

In the 1920s, the refrigerants used in domestic and small commercial systems
included sulfur dioxide, ethyl chloride, and methyl chloride, among others.
However, even low concentrations of these refrigerants are toxic, so inevitably
several of these systems developed leaks that caused serious injury and death.
These incidents drove the HVAC&R industry to develop safe refrigerants for
domestic use. The result was several new refrigerants. All were halocarbons,
which are compounds consisting of carbon and one or more halogen atoms:
chlorine, fluorine, and bromine. Specifically, these compounds are chlorofluo-
rocarbons (CFCs). The name originally given to this group of refrigerants was
freon. The first members included R-11 and R-12, which became available in
1931. Domestic refrigeration systems relied on R-12, while R-11 was used in
the water chillers that provided air conditioning for large buildings. These
refrigerants are nontoxic and nonflammable and enjoyed widespread applica-
tion for over 50 years. In the ensuing years, the application of CFCs expanded
to include aerosol propellants, cleaning agents for the microelectronics indus-
try, and blowing agents used in insulation. As a result, the quantities of CFCs
produced increased rapidly during the 1950s and 1960s. During this time, the
presence of R-11 was detected in the Earth’s atmosphere by very careful mea-
surements. The concentration measured was 50 parts per trillion by volume.
Investigations begun in the 1970s were directed at determining if CFCs had an
effect on the Earth’s atmosphere. A profile of these effects evolved over the fol-
lowing years.
Because CFCs are inert (chemically stable), their survival time in the atmo-
sphere allows them to rise into the upper atmosphere, passing through the ozone
(O3) in the stratosphere. At these high altitudes, the ultraviolet radiation from
the sun causes the chlorine atoms in the CFC molecules to break free. The chlo-
rine atoms then chemically react with the ozone molecules to ultimately pro-
duce diatomic oxygen (O2) and chlorine. Thus, the chlorine atoms are again free
to react with other ozone molecules. This ozone-destroying chemical reaction
continues until the chlorine atoms eventually make their way into the lower
atmosphere. On average, a single chlorine atom will remove about 100,000
ozone molecules during its lifetime in the atmosphere. This reduction of ozone
in the upper atmosphere allows more ultraviolet radiation to reach the Earth’s
surface, which in turn increases the risk of cancer in humans.
The first step to curtail the release of CFCs into the atmosphere was taken in
the late 1970s when CFCs were banned as aerosol propellants and blowing
agents for foam insulation by several countries: Canada, Norway, Sweden, and
the United States. In 1985, measurements were reported that showed a large
depletion of ozone was occurring above Antarctica during the spring seasons.
Further scientific investigations confirmed the ozone depletion and determined
that it was caused by the presence of chlorine in the atmosphere. With the added
evidence, reported at about the same time, of significant ozone depletion over
Fundamentals of Refrigeration SI, Second Edition 255

North America, Europe, and Japan, a number of countries made an agreement in


Montreal, Canada, through the United Nations. This agreement, The Montreal
Protocol on Substances That Deplete the Ozone Layer, was created in 1987 and
limited CFC production to one-half the amount produced in 1986 by the year
2000 (UNEP 1987). The United States, the European Community, and 24 other
countries supported this agreement. A 1990 meeting of the Montreal Protocol
completely eliminated production of CFCs by 2000. This new schedule com-
plied with the U.S. Clean Air Act of 1990. Subsequent meetings of the Montreal
Protocol have resulted in an end to CFC production in 1996 and placed attention
on other chemicals, including another group of refrigerants called hydrochloro-
fluorocarbons (HCFCs).
HCFCs are hydrogenated halocarbons, meaning that each carbon-containing
molecule has one hydrogen atom. The presence of the hydrogen atom causes the
molecule to be less chemically stable than the CFCs. Therefore, the HCFCs
have a lower tendency to deplete ozone. R-22 is a member of the HCFC family
whose worldwide scheduled curtailment date is 2030, as established by the
Montreal Protocol. The United States has adopted the following phase-out
schedule for R-22: January 1, 2010—freeze production and consumption (ban
on virgin R-22 unless used as feedstock or refrigerant in equipment manufac-
tured before January 1, 2010); January 1, 2020—ban on production and con-
sumption. Another halocarbon class are hydrofluorocarbons (HFCs), which
contain no chlorine and are not ozone depleting. HFCs are not regulated by the
Montreal Protocol.
The elimination of CFCs (particularly R-11 and R-12) is significant
because they can no longer be used in new refrigeration systems. As a result,
the HVAC&R industry has developed alternative refrigerants and is currently
conducting research that will lead to additional alternative refrigerants. Some
new refrigerants, as well as several CFCs, are discussed in this chapter. This
allows comparisons to be made among a variety of refrigerants. The selection
of refrigerants suitable for a particular application and the related concerns
associated with safety and the environment are also discussed.
The selection of a particular refrigerant for use in a refrigeration applica-
tion is complicated. Generally, design engineers consider five major points
when selecting a refrigerant for a specific application:

• Refrigeration system performance


• Safety of refrigerant—toxicity and flammability
• Reliability
• Environmental impact
• Economics

These points vary in relative importance to each other depending on the


particular application under consideration and government regulations, which
are unique to each country. In the sections that follow, refrigerant type, system
256 Chapter 10 Refrigerant Selection

capacity and efficiency, safety, and environmental concerns are investigated.


Reliability and economics, though important, receive less attention, as they are
not exclusively refrigeration topics.

Types of Refrigerants

The number designation of refrigerants originates with ASHRAE Standard 34


(ASHRAE 2007a). The prefix “R” is used by ASHRAE to designate a refrigerant.
Letter designations are also applied to the halocarbon refrigerants. The numbers
and letters are assigned as discussed in the following sections. Table 10-1 shows
information relating to several refrigerants.

Table 10-1 Physical Properties of Refrigerants


ASHRAE Name or Chemical Molecular Boiling Freezing Critical Critical Conc.
Designation Composition Formula Mass Point1 Point2 Temp.2 Press.3 Limit4
R-11 (CFC) Trichlorofluoromethane CCl3F 137.37 23.708 —110.47 197.96 4407.6 1100
2,2-dichloro-1,1,1-
R-123 (HCFC) CHCl2CF3 152.93 27.823 –107.15 183.68 3661.8 9100
trifluoroethane
R-12 (CFC) Dichlorodifluoromethane CCl2F2 120.93 –29.752 –157.05 111.97 4136.1 22,000
R-134a (HFC) Tetrafluoroethane CH2FCF3 102.3 –26.074 –103.3 113.26 4516.8 50,000
R-22 (HCFC) Chlorodifluoromethane CHClF2 86.48 –40.81 –157.42 96.145 4990.0 25,000
R-125 (HFC) Pentafluoroethane CHF2CF3 120.03 –48.09 –100.63 66.023 3617.7 69,000
R-717
Ammonia NH3 17.03 –33.327 –77.655 132.25 11333.0 300
(Inorganic)
R-13 (CFC) Chlorotrifluoromethane CClF3 104.47 –81.48 –181.15 28.85 3879 n/d
R-23 (HFC) Trifluoromethane CHF3 70.02 –82.018 –155.13 26.143 4832 41,000
R-744
Carbon dioxide CO2 44.01 –78.44 –56.5585 30.978 7377.3 40,000
(Inorganic)
R-170
Ethane C2H6 30.07 –88.581 –182.8 32.72 4872.2 7000
(Hydrocarbon)
R-290
Propane C3H8 44.10 –42.11 –187.62 96.74 4251.2 5000
(Hydrocarbon)
R-404A R-125/143a/134a
— 97.60 –46.222 — 72.046 3728.9 69,000
(zeotrope) (44%/52%/4%)
R-410A
R-32/125 (50%/50%) — 72.59 –51.446 — 71.358 4902.6 55,000
(zoetrope)
R-507A
R-125/143a (50%/50%) — 98.86 –46.741 — 70.617 3705 69,000
(azeotrope)
R-508A
R-23/116 (39%/61%) — 100.1 –87.60 — 10.192 3650.8 55,000
(azeotrope)
1. °C at 101.325 kPa
2. °C
3. kPa
4. ppm (volume/volume). Values reported subject to change.
5. at 527 kPa
Fundamentals of Refrigeration SI, Second Edition 257

Halocarbons

Numbering depends on the atoms that comprise the molecule. The first digit on
the right is the number of fluorine atoms, the second digit from the right is one
more than the number of hydrogen atoms, and the third digit from the right is
one less than the number of carbon atoms. When the third digit is “0”, it is not
written down. Acronyms are used by the industry to distinguish halocarbons
according to the degree to which the chemical compound harms the environ-
ment. These are summarized below:

Chlorofluorocarbons (CFCs): nonhydrogenated halocarbons. They contain


no hydrogen atoms and are harmful to the Earth’s ozone layer.
Hydrochlorofluorocarbons (HCFCs): hydrogenated halocarbons. They con-
tain one hydrogen atom and are slightly harmful to the Earth’s ozone layer.
Hydrofluorocarbons (HFCs): halocarbons that do not contain chlorine and
are not harmful to the Earth’s atmosphere.

Example chemical structures for representative refrigerants from each


group are shown in Figure 10-1.

Figure 10-1 Molecular structures of select halocarbon refrigerants.


258 Chapter 10 Refrigerant Selection

Inorganic Compounds
Inorganic compounds include the refrigerants ammonia (R-717) and carbon
dioxide (R-744). The two digits on the right equal the molecular weight of the
chemical. The third digit from the right is “7”.

Hydrocarbons
Several hydrocarbons may be used as refrigerants. This chapter discusses two:
ethane (R-170) and propane (R-290). Application of hydrocarbons as refriger-
ants is prevalent in the petrochemical and petroleum industries. In some Euro-
pean countries, the use of hydrocarbon refrigerants is encouraged in domestic
equipment.

Refrigerant Blends
A refrigerant blend is a mixture of two or more refrigerants where the constitu-
ents that makeup the mixture cannot be recovered by only distillation. Two
types of blends occur, zeotropes and azeotropes, where zeotropes are actually
nonazeotropes. In the effort by the refrigeration industry to develop alternative
refrigerants, refrigerant blends have received considerable interest in recent
years. Blends offer, through the selection of refrigerant constituents and their
relative compositions, the ability to obtain the desired characteristics. For
example, by adjusting the relative amounts of a high-pressure and a low-pres-
sure refrigerant, the vapor pressure of the blend can be made to correspond to
that of a particular CFC or HCFC that has been curtailed. In other situations,
blends are developed to improve refrigeration system operating characteristics,
such as compressor discharge temperature, or to improve lubricant circulation
by including a more miscible refrigerant into the blend.

Zeotropes
A zeotrope is a homogeneous mixture of at least two chemicals, where the
chemical composition of the liquid and vapor states are different when the
vapor and liquid are in equilibrium or in contact with each other. The zeo-
tropic refrigerant mixtures in use generally consist of two or more halocar-
bons. The properties of the zeotrope are different from those of the
constituents. An ideal zeotrope blend is shown in Figure 10-2, where the two
constituents are labeled as “i” and “j.” Observe that the concentrations of the
two constituents are different for the vapor and liquid at a given temperature
and pressure, as shown by the points “a” and “b” on the line. If a leak were to
occur causing a loss of refrigerant from the system, whether liquid or vapor,
the composition of the blend remaining in the system would change. This
would, in turn, change the properties of the resulting new refrigerant mixture.
As a consequence, two safety classifications are given for zeotropes. One is
Fundamentals of Refrigeration SI, Second Edition 259

Figure 10-2 Ideal zeotrope consisting of components i and j at a given pressure.

for the zeotrope as originally formulated, and the second is for that same zeo-
trope having undergone the greatest possible fractionation, a change in mix-
ture composition resulting from leakage of vapor or liquid, each of which has
a different composition.
These refrigerants are designated with a three-digit number, “4” being the
left-most digit. The two digits on the right indicate the sequence of that zeo-
trope’s commercial appearance.
Also observe, from Figure 10-2, that a zeotropic blend undergoing a liquid-
vapor phase change at constant pressure does not occur at a constant tempera-
ture, but rather over a range of temperatures. This temperature change that
occurs during a phase change is called temperature glide and depends on the
composition of the blend. Refrigerant blends with relatively large temperature
glides sometimes offer the potential for higher system operating efficiency. As
an illustration, by matching the temperature profiles of the refrigerant blend
and the fluid being cooled, it is possible to decrease the thermodynamic irre-
versibilities associated with heat transfer (i.e., to reduce the finite temperature
difference) in the evaporator. Furthermore, temperature glide results in frac-
tionation due to the separation of higher and lower boiling components. Here,
the vapor will contain an excess of the refrigerant with the lower boiling point
and the liquid an excess of the refrigerant with the higher boiling point.

Azeotropes
An azeotrope (sometimes called a constant boiling mixture) is a mixture that
has the peculiar property of generating a vapor that has the same composition
260 Chapter 10 Refrigerant Selection

as the liquid when the liquid and vapor are in equilibrium. Figure 10-3 is an
illustration of a typical azeotrope where this special behavior is observed to
occur where the saturated liquid and saturated vapor lines precisely coincide.
Over this concentration range, the azeotrope behaves as a single-component
fluid but still has properties that differ from either constituent alone.
Azeotropes are designated with a three-digit number, “5” being the left-
most digit. The two digits to the right indicate the sequence of that azeotrope’s
commercial appearance.
Azeotropes, like zeotropes, possess properties different from those of the
individual components. Additionally, compositions that produce an azeotrope
at one pressure may not be a perfect azeotrope at another pressure. Typically,
as pressure is increased, the azeotropic zone moves toward the high concentra-
tion of the low-temperature boiling refrigerant (component i in Figure 10-3).
Azeotrope R-502 (production now banned) showed this pressure effect and
was, therefore, prone to fractionation at higher operating pressures.
Most of the alternative halocarbons appearing recently are blends character-
ized as zeotropic, or nonazeotropic. Refrigerant R-410A has already been noted
as a zeotropic blend and has a relatively small temperature glide of only 0.11°C
at a pressure of about 1000 kPa. In contrast, zeotropic blend R-404A (44%,
52%, 4%; R-125, R-143a, R-134a) has a temperature glide of about 5.5°C at the
same pressure. An example of an azeotrope is R-507A (50%, 50%; R-143a, R-
125), where the azeotropic zone occurs over a relatively wide range of pressure.
It turns out that most of the new substitute halocarbon refrigerants are blends
and are near-azeotropic mixtures. Consequently, the saturated liquid and satu-
rated vapor lines for these mixtures do not coincide but are very near each other.

Figure 10-3 An azeotrope consisting of components i and j at a given pressure.


Fundamentals of Refrigeration SI, Second Edition 261

Saturation Pressure and Temperature of a Refrigerant


The pressure of the refrigerant in the evaporator and condenser influences the
selection of a refrigerant. In addition to keeping evaporator pressures above
atmospheric pressure, we desire reasonably low operating pressures to permit the
use of standard piping and vessels. Figure 10-4 shows the variation in saturation
pressure with temperature for most of the refrigerants we have considered.
Notice that the refrigerants considered fall into three groups. R-11 and R-123 are

Figure 10-4 Variation of saturation pressure with temperature for several refrigerants.
262 Chapter 10 Refrigerant Selection

in the high-temperature group suitable for use in centrifugal compressors and


have low saturation pressures when compared to the other refrigerants. The next
group is the intermediate-temperature refrigerants: R-12, R-134a, R-22, R-125,
R-717, R-290, R-600a, R-404A, R410A, and R-507A. The last group is the low-
temperature refrigerants. At a given temperature, their saturation pressure is
higher than other refrigerants. This group consists of R-13, R-23, R-170, R-744,
and R-508A.
Carbon dioxide (R-744) is interesting when used as a refrigerant. For exam-
ple, at atmospheric pressure, carbon dioxide cannot exist as a liquid, only as a
solid and vapor. However, when the pressure is greater than 517 kPa, carbon
dioxide cannot exist as a solid. So above 517 kPa (corresponding saturation
temperature is about –56°C), it behaves as a conventional refrigerant. Conse-
quently, in a carbon dioxide system, provision must be made to maintain pres-
sures sufficient for the pumping of carbon dioxide.

Refrigeration Capacity and Efficiency


Performance of a refrigeration system consists of refrigeration capacity and effi-
ciency. Refrigeration capacity is the cooling rate the system produces that
results from a certain volumetric flow rate of refrigerant. Refrigerant volumetric
flow rate, or volumetric capacity, depends on the refrigerant’s latent heat of
vaporization and the density of the refrigerant vapor as it enters the compressor.
A refrigerant’s normal boiling point is a predictor of its capacity. While
exceptions occur, the higher the boiling point, the lower the refrigerant’s volu-
metric cooling capacity. For example, R-134a and R-22, as seen in Table 10-1,
have –26.15°C and –40.76°C as the boiling point temperatures, respectively.
Table 10-2 shows that the volumetric vapor flow rate leaving the evaporator is
0.17 and 0.10 m3/min, respectively. Thus R-134a, with its higher boiling tem-
perature, has a lower volumetric cooling capacity compared to R-22. In contrast,
refrigerants with higher boiling points are likely to have greater operating effi-
ciency. The situation is compounded by the fact that higher boiling point refrig-
erants also are likely to have a higher pressure drops and possibly lower heat
transfer coefficients in the evaporator. Furthermore, additional refrigeration sys-
tem concerns include appropriate operating pressure, smaller compressor size,
and lower discharge temperature. Table 10-2 is extracted from the 2009
ASHRAE Handbook—Fundamentals (ASHRAE 2009) and shows the perfor-
mance characteristics of several refrigerants in a one-ton single-stage vapor-
compression refrigeration system operating with an evaporating temperature of
–15°C and a condensing temperature of 30°C.
Related to the refrigeration performance point are two important consider-
ations relevant to a system’s overall requirements. These are the temperature of
the cold space or product that must be maintained and the temperature of the
environment to which the heat is rejected. These involve evaporation and con-
densation processes. The temperature of the boiling refrigerant in the evaporator
is generally –12.22°C to –6.67°C below that of the cold space. As mentioned
Fundamentals of Refrigeration SI, Second Edition 263

Table 10-2 Comparative Refrigerant Performance per Ton of Refrigeration


Refrigerant Net Refrig- Specific Com- Com-
Evap- Con- Refrig- erant Liquid Volume pressor Power Coeffi- pressor
Chemical Name orator denser Com- erating Circu- Circu- of Suction Displace- Consump- cient of Discharge
or Composition Pressure, Pressure, pression Effect, lated, g/ lated, Gas, ment, tion, Perform- Temp.,
No. (% by mass) MPa MPa Ratio kJ/kg s L/s m3/kg L/s kW ance °C
170 Ethane 1.608 4.639 2.88 161.71 6.10 0.0219 0.0338 0.206 0.365 2.7 323
744 Carbon dioxide 2.254 7.18 3.19 133.23 3.88 0.0064 0.0168 0.065 0.192 2.69 343
1270 Propylene 0.358 1.304 3.64 286.17 3.46 0.0070 0.1299 0.449 0.220 4.5 315
290 Propane 0.286 1.075 3.76 277.90 3.53 0.0073 0.1562 0.551 0.218 4.5 309
502 R-22/115 (48.8/51.2) 0.343 1.312 3.83 105.95 9.43 0.0079 0.0508 0.479 0.228 4.38 311
507A R-125/143a (50/50) 0.379 1.459 3.85 110.14 9.07 0.0089 0.0508 0.461 0.239 4.18 308
404A R-125/143a/134a (44/52/4) 0.365 1.42 3.89 114.15 8.75 0.0086 0.0537 0.470 0.237 4.21 309
410A R-32/125 (50/50) 0.478 1.872 3.92 167.89 5.84 0.0056 0.0545 0.318 0.222 4.41 324
125 Pentafluoroethane 0.403 1.561 3.87 85.30 11.41 0.0098 0.0394 0.449 0.244 3.99 304
22 Chlorodifluoromethane 0.295 1.187 4.02 162.67 6.13 0.0052 0.0779 0.478 0.214 4.66 326
12 Dichlorodifluoromethane 0.181 0.741 4.09 117.02 8.49 0.0066 0.0923 0.784 0.212 4.7 311
500 R-12/152a (73.8/26.2) 0.214 0.876 4.09 139.68 7.08 0.0063 0.0939 0.665 0.212 4.66 314
407C R-32/125/134a (23/25/52) 0.288 1.26 4.38 163.27 6.11 0.0054 0.0805 0.492 0.222 4.5 321
600a Isobutane* 0.088 0.403 4.58 263.91 3.76 0.0069 0.4073 1.533 0.215 4.62 303
134a Tetrafluoroethane 0.163 0.767 4.71 148.03 6.71 0.0056 50.1214 0.814 0.216 4.6 310
124 Chlorotetrafluoroethane* 0.0088 0.443 5.03 117.83 8.41 0.0063 0.1711 11.439 0.214 4.62 303
717 Ammonia 0.235 1.162 4.94 1103.1 0.90 0.0015 0.5117 0.463 0.210 4.76 372
600 Butane* 0.056 0.283 5.05 292.24 3.53 0.0062 0.6446 2.274 0.218 4.74 303
11 Trichlorofluoromethane 0.02 0.125 6.25 155.95 6.36 0.0043 0.7689 4.891 0.197 5.02 316
123 Dichlorotrifluoroethane 0.016 0.109 6.81 142.28 7.02 0.0048 0.8914 6.259 0.204 4.9 306
113 Trichlorotrifluoroethane* 0.007 0.054 7.71 122.58 7.84 0.0051 1.6818 13.187 0.200 4.81 303
*Superheat required.

previously, this temperature difference is necessary to move heat from the cold
space to the refrigerant inside the evaporator. Similarly for the condenser, the
temperature of the refrigerant condensing inside is typically –12.22°C to
–6.67°C greater than that of the surrounding environment. The pressure in the
low side of the system should be greater than atmospheric pressure so that air
does not leak into the system. The pressure in the evaporator is the saturation
pressure that corresponds to the temperature of the refrigerant. However, the
high-temperature refrigerants R-11 and R-123 are exceptions in cases where
evaporator pressures are less than 69 kPa.
Table 10-1 gives physical properties of several refrigerants. One item
included is the boiling point at 101.325 kPa. If a cold space is to be main-
tained at –17.78°C, then R-134a is a potential refrigerant because its boiling
temperature is –26.1°C at atmospheric pressure, so it would provide both an
adequate temperature difference and positive pressure in the evaporator.
Other refrigerants listed in Table 10-1 are suitable as well.
The temperature in the condenser cannot fall below that of the surround-
ing environment. The saturation pressure of the refrigerant at this temperature
must be significantly below its critical pressure if the heat rejection process is
to occur at a constant or near-constant temperature. Recall Figure 2-5a in
which the critical point occurs where the saturation liquid and vapor curves
meet at the top. The critical pressure and temperature are given for several
refrigerants in Table 10-1. If a single refrigerant cannot be found to meet the
temperature requirements, then two or more refrigeration cycles using differ-
ent refrigerants coupled together in series may be used. This type of refrigera-
264 Chapter 10 Refrigerant Selection

tion system is called a cascade system and is normally applied to ultra-low-


temperature applications.
Several conclusions can be drawn from Table 10-2. For these refrigerants,
the COP is about the same; however, R-125 has the lowest at 3.67, while
ammonia (R-717) has the highest at 4.77 (except for R-11 and R-123). The
latent heat of ammonia is by far the highest of these refrigerants (1314 kJ/kg)
and is an advantage because the mass flow of ammonia needed to produce the
refrigeration is about one-fourth that of the halocarbons. However, one disad-
vantage of ammonia is its high vapor specific volume (0.51 m3/kg), which
results in a similar volume flow rate leaving the evaporator. Another disadvan-
tage of ammonia is the high temperature of the vapor discharge from the com-
pressor (98.8°C). Reciprocating compressors for ammonia systems employ
external cooling of the cylinder heads. The hydrocarbon refrigerants (ethane
and propane) provide reasonably efficient performance with low compressor
discharge temperatures. R-11 and R-123 are used with centrifugal compres-
sors, which have thermodynamic characteristics that result in reasonable wheel
tip speeds.
Comments for some other refrigerants are as follows:
R-12: a CFC and banned from further production. Use was intended for
reciprocating compressors applied to domestic systems and automotive air-
conditioning. Was widely used for large high- and low-temperature commer-
cial and industrial systems.
R-22: an HCFC, so production will cease in 2020 in developed countries.
Requires a smaller compressor than that of a system using R-12. Table 10-2
supports this conclusion.
R-134a: an HFC that is accepted as the best replacement for R-12. Operating
efficiencies are somewhat less with R-134a than with R-12. Table 10-2 sug-
gests this trend.
R-717: ammonia, common in large industrial refrigeration systems, where it
has been used for more than 100 years and will likely have continued use.
R-744: carbon dioxide, used in direct freezing of food products. Condensing
pressure is high, resulting in low-temperature applications such as the low-
temperature side of a cascade system.

Regarding compatibility of refrigerants with metals, a refrigerant is com-


patible when it does not chemically react with the system’s materials of con-
struction. Halocarbon refrigerants are compatible with most metals. However,
magnesium, zinc, and aluminum alloys that consist of more than 2% magne-
sium should not be used with halocarbon refrigerants. Ammonia should not be
used with copper or any other metal containing copper (such as brass). Alumi-
num alloys are compatible with ammonia when they do not contain copper.
The effect of refrigerants on elastomers and plastics should be carefully studied
so that appropriate materials are used in the system.
Fundamentals of Refrigeration SI, Second Edition 265

Consideration of reliability and expense are also important in the selection


of a refrigerant. While these two points are general engineering considerations,
several comments are pertinent to refrigeration systems. Chemical stability of
the refrigerant directly influences reliability. At the high and low temperatures
at which the refrigeration system operates, the refrigerant should not decom-
pose. Again, for reliability, the refrigerant must be compatible with all the
components that make up the system. Furthermore, the refrigerant must not
swell, embrittle, or dissolve the plastics and elastomers employed by the motor
parts and valve seals for good reliability.
Economic analyses of refrigeration systems are very important and influ-
ence many design decisions, including the selection of the refrigerant. Cer-
tainly, for large industrial systems, the cost of the refrigerant is significant, as
the system may easily contain more that 4500 kg of charge. The relatively low
cost of ammonia, compared to that of other refrigerants, is one reason for its
continued use in large systems. Economic analyses may involve a number of
different approaches, such as trade-off studies, where several alternative
designs are explored.

Safety of Refrigerants

ASHRAE classifies refrigerants according to safety. Two basic characteristics


—toxicity and flammability—are used as the basis for the classification. As
new refrigerants are developed, new classifications for these refrigerants are
determined. ASHRAE Standard 34 (ASHRAE 2007a) publishes the refrigerant
classifications and is updated periodically to include the latest developments.
The ASHRAE safety classification system is shown in Table 10-3. Two
levels of toxicity and three levels of flammability comprise the classifications.
The letters “A” and “B” correlate to nontoxic and toxic, respectively. The “B”
designation indicates there is evidence of toxicity when humans are exposed
to the refrigerant at a certain concentration in air. The concentration is the
quantity of refrigerant vapor (m3) per unit clean air (m3) and is usually
reported as parts per million (ppm, by volume). A refrigerant concentration
equal to 50 ppm (volume/volume) means that the volume of refrigerant pres-
ent in a particular volume of air occurs at the ratio:

50 ppm = 50 volume units refrigerant


-----------------------------------------------------------------
1,000,000 volume units air

Table 10-3 ASHRAE Safety Group (Classification of Refrigerants)


Flammability/ No Flame Moderately Highly
Toxicity Propagation Flammable Flammable
Toxicity Unidentified A1 A2 A3
Toxicity Identified B1 B2 B3
266 Chapter 10 Refrigerant Selection

where the volume unit may be m3, mm3, or any other unit. A refrigerant’s
flammability is divided into three categories: highly flammable, moderately
flammable, and nonflammable. The numbers used to designate these flamma-
bility categories are 3, 2, and 1, respectively. It should be noted that a refriger-
ant’s flammability refers to its ability to propagate a flame when the vapor is
mixed in air.
Table 10-4 shows the ASHRAE safety classifications of several refrigerants.
The halocarbons are nonflammable and nontoxic and so classified as “A1”
refrigerants. Ammonia, a “B2” refrigerant, is classified as toxic and moderately
flammable. The hydrocarbons listed in Table 10-4 are classified as “A3” refrig-
erants, nontoxic but highly flammable.
Other agencies have also classified refrigerants according to safety. For
example, Underwriters Laboratory (UL 2009) establishes its classification sys-
tem on the magnitude of the hazardous effects that a refrigerant has on life.

Toxicity

The word toxicity refers to the extent that exposure to a chemical adversely
affects human health. All refrigerants affect human health in different ways.
Additionally, the concentration of the refrigerant vapor in air influences the
severity of the human body’s response to the exposure.
Table 10-4 lists one indicator of a refrigerant’s vapor toxicity, the threshold
limit value. This quantity originates with the American Congress of Govern-
mental Industrial Hygienists (ACGIH) and is defined as the threshold limit
value time-weighted average (TLV-TWA) exposure that a worker can be
exposed to a substance for a normal 40-hour work week without any adverse
health effects.
Table 10-4 indicates that R-123, with a TLV-TWA equal to 10 ppm (v/v), is
considered toxic at low concentration levels. However, other organizations
indicate that a reasonable value of toxicity is more likely near 30 ppm (v/v).
Because R-123 is a new refrigerant, studies are being performed to determine a
reliable TLV-TWA. The other halocarbon refrigerants are noted to have signifi-
cantly higher TLV-TWAs. It should be noted that the halocarbon refrigerants
are odorless and colorless and can only be detected with appropriate special
instrumentation. Halocarbons are also heavier than air and tend to settle in low
areas, where at high concentrations, the vapors may cause asphyxiation (rather
than a toxic response) due to the vapors displacing the air.
Table 10-4 gives ammonia’s TLV-TWA as 25 ppm (v/v). The presence of
ammonia vapor is easily identified by its pungent odor, even at concentrations
greatly below levels that cause permanent health effects. On average, humans
detect the odor of ammonia vapor at 5 ppm (v/v). The National Institute of
Occupational Safety and Health (NIOSH) exposure reports another limiting
exposure called the immediately dangerous to life and health (IDLH) for
ammonia equal to 300 ppm. This is the level to which a healthy worker may be
exposed for a period of 30 minutes without any permanent health effects.
Fundamentals of Refrigeration SI, Second Edition 267

Table 10-4 Flammability and Environmental Characteristics of Several Refrigerants


Atmospheric
ASHRAE Name or Chemical TLV-TWA, LFL, Safety
ODP GWP Life,
Designation Composition Formula ppm, v/v % Group
years
R-11 Trichlorofluoro-
CCl3F 1.00 4000 50 100 None A1
(CFC) methane
R-123 2,2-dichloro-1,1,1-
(HCFC) trifluoroethane
CHCl2CF3 0.02 93 1.4 10a None A1

R-12 Dichlorodifluoro-
CCl2F2 1.0 8500 102 1000 None A1
(CFC) methane
R-134a
Tetrafluoroethane CH2FCF3 0 1300 14 1000 None A1
(HFC)
R-22 Chlorodifluoro-
CHClF2 0.055 1700 13.3 1000 None A1
(HCFC) methane
R-125
Pentafluoroethane CHF2CF3 0 2800 36 1000 None A1
(HFC)
R-717 NH3
Ammonia 0 0 <1 25 15 B2
(Inorganic)
R-13 Chlorotrifluoro- CClF3 1.00 11,700 640 1000 None A1
(CFC) methane
R-23 CHF3
Trifluoromethane 0 12,100 250 1000 None A1
(HFC)
R-744 CO2
Carbon dioxide 0 1 50–200 5000 None A1
(Inorganic)
R-170
(Hydro- Ethane C 2H 6 0 3 — n.a.b 3.3 A3
carbon)
R-290
(Hydro- Propane C3H8 0 3 — n.a.b 2.1 A3
carbon)
R-404A R-125/143a/134a
— 0 3260 blend 1000 None A1/A1
(Zeotrope) (44%/52%/4%)
R-410A R-32/125
— 0 1725 blend 1000 None A1/A1
(Zeotrope) (50%/50%)
R-507A R-125/143a
— 0 3300 blend 1000 None A1
(Azeotrope) (50%/50%)
R-508A R-23/116
— 0 11,860 blend 1000 None A1
(Azeotrope) (39%/61%)
a. TLV-TWA not established by ACGIH, estimated on basis of limited or incomplete toxicity testing.
b. Simple asphyxiant. Other significant physiological effects do not occur.
268 Chapter 10 Refrigerant Selection

The ASHRAE Standard 34 classification for toxicity is based on chronic


exposure limits defined by the TLV-TWA as follows:
Class A: toxicity unknown at concentration levels less than 400 ppm (v/v),
based on TLV-TWA.
Class B: toxicity known at concentrations less than 400 ppm (v/v), based on
TLV-TWA.

Flammability
ASHRAE Standard 34 (ASHRAE 2007a) classifies refrigerants into the three
flame categories as follows:

Class 1: nonflammable refrigerant, no flame propagation at any concentration


at room conditions.
Class 2: moderately flammable refrigerant, exhibits flame propagation at
concentrations greater than 10 kg/m3 with a heat of combustion less than
19 000 kJ/kg.
Class 3: highly flammable refrigerant, exhibits flame propagation at con-
centrations less than 1.0 kg/m3 with a heat of combustion greater than 19 000
kJ/kg.
Obviously, some refrigerants will burn when mixed with air. A refriger-
ant’s flammability is characterized by the concentration of the refrigerant
vapor in air (v/v), shown as a percentage. Thus, a flammable mixture is the
concentration (percentage) of vapor/air that supports the propagation of a
flame through the mixture. Generally, when a vapor produces a flammable
mixture, there is a concentration below which a flame will not propagate
across the mixture. This concentration is called the lower flammability limit
(LFL). Likewise, there is an upper concentration limit above which a flame
cannot propagate, called the upper flammability limit (UFL). For safety ques-
tions involving the leak of a refrigerant into the surrounding air, the LFL is
the important parameter.
The flammability limits are experimentally determined using a chamber,
vapor-handling equipment, and an ignition source (such as an electrical spark).
Different concentrations of the test vapor and air are introduced into the cham-
ber and mixed, and ignition is attempted. If the mixture ignites and propagates
a flame, the mixture is flammable. If the mixture does not propagate a flame, it
is nonflammable. Variation of the vapor mixture composition determines the
LFL and the UFL.
In Table 10-4, three of the listed refrigerants are flammable. They are
repeated in Table 10-5 (along with butane) with the addition of their UFLs.
Note that the hydrocarbon refrigerants have LFL values that are only sev-
eral percent of vapor in the air on a volume basis. Thus, a leak of hydrocarbon
Fundamentals of Refrigeration SI, Second Edition 269

Table 10-5 Flammability Limits for Several Refrigerants


Refrigerant LFL, %v/v UFL, %v/v
Butane 1.7 6.5
Propane 2.1 10.1
Ethane 3.0 12.5
Ammonia 15 28

vapor into a room may reach the LFL in a short time. In contrast, ammonia has
an LFL at a considerably higher concentration of 15%. This is the reason
ammonia is classified as a group 2 refrigerant and hydrocarbons are group 3
(highly flammable) refrigerants. Also note in Table 10-4 that the TLV-TWA
ammonia concentrations are much lower than the LFL for ammonia/air mix-
tures, which is 150,000 ppm (v/v).

Environmental Impact of Refrigerants

The environmental impact of refrigerants is summarized in Table 10-4 under


two parameters, ozone depletion potential (ODP) and global warming poten-
tial (GWP).

• ODP is an index that describes a refrigerant’s ability to destroy ozone in


the Earth’s atmosphere relative to that of R-11. The ODP for R-11 is
defined as 1.00.
• GWP is an index that describes a refrigerant’s ability to warm the Earth’s
atmosphere relative to that of carbon dioxide over a period of 100 years.
All chemicals have an average time of duration in the atmosphere. For
example, the atmospheric life of R-12 is 102 years while that of R-134a is
55 years.

Regarding ODP, Table 10-4 indicates that CFCs are high, HCFCs are a
small fraction of the CFC value, and HFCs have no ODP. In the United States,
halocarbon refrigerants cannot be vented to the atmosphere, and technicians
must be certified to operate refrigerant recovery equipment. Obviously, refrig-
erants selected should have a low or zero ODP.
GWP is another matter. Values of GWP for the halocarbons are consider-
ably higher (greater than 1000, with the exception of R-123) than an equivalent
quantity of carbon dioxide. Typically, GWP is an index giving the relative
quantity of infrared radiation that a gas can absorb compared to that of carbon
dioxide, which has an assigned value of 1.0 integrated over a time duration of
100 years. A refrigerant’s contribution to global warming may be significant if
its GWP index is high. Table 10-4 shows that the GWP for R-134a and R-23 at
270 Chapter 10 Refrigerant Selection

1300 ppm (v/v) and 12,100 ppm (v/v), respectively, coupled with their long
atmospheric life, make significant contributions to global warming.
Another measure of a refrigerant’s contribution to global warming is
called the total equivalent warming impact (TEWI). This measure of global
warming includes the carbon dioxide production connected with the genera-
tion of electrical energy to operate the compressor plus the influence of the
refrigerant itself if leaked into the atmosphere. Consequently, the TEWI
includes the energy efficiency of the refrigeration system. As an example,
consider a domestic refrigerator where refrigerants R-134a and isobutane are
being considered. The GWP of these refrigerants are 1300 ppm (v/v) and 11
ppm (v/v), respectively. Observe that the GWP index suggests that isobutane
has the far lesser impact on global warming. However, applying the TEWI
concept, less than 1% of the R-134a TEWI value originates from the direct
emission of R-134a into the atmosphere; the remainder is electrical power
consumption to operate the compressor. It turns out that the total TEWI val-
ues for R-134a and isobutane applied to a domestic refrigerator are only
about 1% different from each other. Consequently, it is energy efficiency that
is important in reducing the TEWI.

Codes and Standards


The importance of safety cannot be overstated. Both the refrigerant and the
system that contains it impact safety issues. Stated another way, the first
requirement of a successful refrigeration system is that it be safe. System reli-
ability and efficiency become important only after all safety concerns are met.
Safety issues are addressed in the three major activities associated with a
refrigeration system over its life span:

• Design
• Construction and installation
• Operation

During design of a refrigeration system, system components are selected that


conform to code requirements. The codes actually perform two functions for
the designer: to ensure that the refrigeration system complies with standard
industry practice for safety and that professional liability is managed.
Construction and installation may involve a new system or a change
applied to an existing system. The installer faces situations not anticipated by
the designer and so makes judgments and adjustments to complete the job. This
is especially true for the installation of industrial systems due to their large size
and complexity.
Operation of the system is also important to system safety. Correct mainte-
nance procedures are essential for technicians to avoid personal injuries and
damage to the equipment. The dominant code in the U.S. HVAC&R industry is
ASHRAE Standard 15 (ASHRAE 2007b), which is continuously maintained by
Fundamentals of Refrigeration SI, Second Edition 271

ASHRAE and consists of 13 sections and 10 appendices. Sections 4 through 7


use the refrigerant classifications from ASHRAE Standard 34 (ASHRAE
2007a) and suggest applications for each of the refrigerant classifications.
Installation, vessels, and pressure relief are handled in sections 8 and 9. Section
10 is concerned with system operating and testing and is particularly important
for industrial systems. Section 11 covers general requirements, such as signs,
identification and maintenance. Appendix A concerns the new blends of
nonazeotropic mixtures, while Appendix B identifies the procedures to use with
the emergency release of refrigerants. The remaining Appendices in ASHRAE
Standard 15 contain useful information for applying the requirements specified
by the standard.
Other important codes are referred to in ASHRAE Standard 15 that auto-
matically become part of the code. These include, but are not limited to, the
following:

• Boiler and Pressure Vessel Code (ASME 2007)


• ANSI/ASME B31.5, Refrigeration Piping and Heat Transfer Components
(ASME 2006)
• ANSI/ASHRAE Standard 34, Number Designation and Safety Classifica-
tion of Refrigerants (ASHRAE 2007a)
• NFPA 70: National Electrical Code (NFPA 2010)
• IIAR-1999, Equipment, Design and Installation of Ammonia Mechanical
Refrigeration Systems (IIAR 1999)
• ANSI/UL 207, Refrigerant-Containing Components and Accessories, Non-
electrical (UL 2009)
• Basic National Mechanical Code (BOCA 1990)

Pressure vessels greater than 6 in. inside diameter and internal pressure greater
than 15 psig are subject to the specifications in the Boiler and Pressure Vessel
Code (ASME 2007). Pressure gauges and valve bodies are not treated as pressure
vessels.
Section 7 of ASHRAE Standard 15 presents the restrictions on refrigerant
use due to safety concerns. This section gives the calculation procedure for
determining the maximum allowed refrigerant quantity in a refrigeration sys-
tem that has direct contact with the occupied space in a building. In this situa-
tion, the evaporator coil is located inside the occupied space. This calculation
assumes that all the refrigerant disperses into the occupied space, whose vol-
ume is computed according to the rules given in ASHRAE Standard 15. The
idea behind the standard’s calculation of occupied building space volume is to
determine the smallest possible volume that the refrigerant could disperse into,
yielding the highest possible refrigerant concentration in the building—the
worst case scenario. Consult ASHRAE Standard 15 for details concerning all
restrictions on refrigerant use.
272 Chapter 10 Refrigerant Selection

Example 10-1

Problem A commercial fabrication shop has floor dimensions of 25 by 30 m and is built


on a concrete slab. The roof is flat and is 5 m above the floor. The facility’s
occupied space, restricted to authorized personnel, is served by one rooftop
refrigeration system where the evaporator is placed in contact with the occu-
pied space. The refrigeration system uses R-404A with a charge of 20.4 kg.
Walls of the shop continue to the roof, no plenum volumes exist anywhere in
the building, and the shop area is nonventilated. All doors are tight fitting and
closed. Assume Table 10-1 limit concentration values apply for this applica-
tion. A machinery room (separate well-ventilated room) is required if the
refrigerant charge, upon completely dispersing into the occupied space,
exceeds the concentration value given in Table 10-1. Determine if a machinery
room is required.

Solution Occupied volume = (25)(30)(5) m = 3750 m3


From Table 10-1, the concentration limit for R-404A is 69,000 ppm (v/v).
Convert units to kg/1000 m3.

m 3 R-404A-
69,000 ppm = 69,000
----------------------------------------------
1,000,000 m 3 air
m 3 R-404A-
= 69
-----------------------------------
1000 m 3 air
Next, determine the mass of 69 m3 of R-404A using the ideal gas equation
of state and assuming room conditions of 21°C and 101.325 kPa.

pV = mRT
kg
Molecular mass  R-404A  = 97.60 -------------
kg mol
J
R = 8314 ----------------------
kg mol  K
J
8314- = 85.18 ---------------------
R R-404A = ------------ -
97.60 kg mol  K

pV -
m = ---------------------
R R-404A T
 101.325   1000   69 
= ----------------------------------------------------
 85.18   21 + 273.15 
kg R-404A-
= 280 --------------------------
1000 m 3
Fundamentals of Refrigeration SI, Second Edition 273

So, the maximum allowed refrigerant charge in this fabrication shop is

280 kg R-404A-
 3750 m 3   ------------------------------------ = 1050 kg R-404A
 1000 m 3 

See that the maximum allowed quantity of refrigerant R-404A, according


to Table 10-1, is 1050 kg, which in this situation is far greater than the 20.4 kg
charge. Consequently, a machinery room is not required in this instance, and
the evaporator coil can be located in the occupied space.

Two items mentioned here suggest the wide range of concern in the com-
plex piping arrangements of industrial refrigeration systems. The first item is
the sudden deceleration of liquid in a pipe, where the resultant impact forces
can be sufficient to rupture a pipe elbow or similar fitting. Visualize liquid
moving in a pipe as a cylinder filling the cross section in the pipe. Then, when
the liquid suddenly stops (as in striking an elbow or end cap), large forces
result. The pressure rise is due to the momentum change experienced by the
liquid cylinder. Thus,

 cylinder mass   velocity 


pA = -----------------------------------------------------------------------
cylinder deceleration time
(10-1)
 LA   V 
= ------------------------
t

where p is the pressure (kg/m2), A is the inside cross-sectional area of the


pipe (m2), L is the length of the liquid cylinder (m),  is the liquid density
(kg/m3), V is the initial velocity of the liquid in the pipe (m/s), and t is the
time over which the liquid decelerates to zero velocity. The product, pA, is
the force that results when the liquid cylinder is brought to zero velocity in a
pipe. Dividing both sides by A yields an expression for the pressure during
the liquid’s deceleration.

Example 10-2

Problem A pipe 30.5 m in length contains liquid R-22 at –6.67°C and 413.7 kPa. At the
end of the pipe is a solenoid operated shutoff valve. The velocity of the flowing
liquid is 1.83 m/s. If the solenoid valve requires 0.02 s to close, estimate the
momentary overpressure exerted by the refrigerant in the pipe.

Solution From Equation 10-1, we know that

LV
p = -----------
t
274 Chapter 10 Refrigerant Selection

and for a very slightly subcooled liquid, the R-22 saturation table gives: f =
1300 kg/m3. With L = 30.5 m, V = 1.83 m/s, and t = 0.02 s, we have

1300   30.5   1.83 -


p = -----------------------------------------------
 0.02   9.81 
= 370 000 kg/m 2

where
9.81 = gravitational constant, m/s2
Thus, observe that the predicted 370 000 kg/m2 is considerable and could
cause damage.

The second item of safety concern is pipe failure due to trapped liquid. Liq-
uid is incompressible; regardless of how high the pressure becomes, the vol-
ume of the liquid remains constant. However, when a liquid warms, its volume
increases due to a small amount of expansion. The consequence of these two
thermodynamic features of liquids is that when the liquid is trapped in a vol-
ume with no vapor pockets and begins to warm, the volume expansion by the
liquid exerts great pressures on the pipe’s walls.
An example of a trapped liquid is a single pipe with a valve on either end. If
cold liquid is passing through the pipe, and both valves are closed simultane-
ously, the liquid is trapped. This situation is shown in Figure 10-5a. If the liq-
uid warms, the pressure inside the pipe greatly increases. Figure 10-5b shows
another situation where liquid may be trapped in a pipe between a solenoid-
operated valve and a check valve.
Figure 10-6 shows the pressure increase that occurs when liquid ammonia
is trapped with an initial temperature of –17.78°C, assuming that the pipe wall
remains rigid. As you can see, with modest increases in temperature, the pres-
sures become very large indeed.

The Next Step


Chapter 11 discusses several types of refrigeration system. The loads considered
are food product cooling and freezing and the cooling of secondary fluids.

Summary
This chapter considered several refrigerants important to the HVAC&R
industry. Refrigerants numerically designated as R-11, R-12, R-13, and the
azeotrope R-502 have been banned by the Montreal Protocol because they
contribute to the depletion of ozone in the upper atmosphere.
The selection of a refrigerant for use in a system depends on the evapo-
rating pressure, the condensing pressure, flammability, toxicity, and other
Fundamentals of Refrigeration SI, Second Edition 275

Figure 10-5 Possible pipe configurations that could trap liquid: (a) pipe between two shut-off
valves and (b) pipe between a check valve and a solenoid operated valve.

Figure 10-6 Pressure increase from warming trapped ammonia.


276 Chapter 10 Refrigerant Selection

considerations. To aid in the selection process, ASHRAE has devised a


refrigerant classification system based on two criteria, flammability and tox-
icity. A refrigerant that is nonflammable and not toxic is identified as a
group A1 refrigerant. Codes and standards mandate technical aspects of the
refrigeration system with regard to safety. ASHRAE Standard 15 is the pre-
mier code document for mechanical refrigeration systems. You should now
be able to

 describe the importance of the element chlorine in the destruction of ozone;


 describe the differences between CFCs, HCFCs, and HFCs;
 list six types of refrigerants and give an example of each;
 describe similarities and differences of zeotropic and azeotropic mixtures;
 list and describe the five points that are considered when selecting a refrig-
erant;
 identify the two characteristics that ASHRAE Standard 34 uses to classify
refrigerants, giving examples of refrigerants from the list in this chapter;
 identify and give examples for the three temperature groups of refrigerants;
 identify the TLV-TWA toxicity level for a refrigerant listed in this chapter
and explain what this value means;
 name and describe the three indices associated with the environmental
impact of refrigerants;
 name two codes and/or standards related to refrigerants and indicate their
importance to the HVAC&R field;
 define the term limit concentration value as it applies to a refrigerant; and
 identify two safety issues associated with piping in an industrial refrigera-
tion system.

References and Bibliography


ASHRAE. 2007a. ANSI/ASHRAE Standard 34-2007, Number Designation and
Safety Classification of Refrigerants. Atlanta: ASHRAE.
ASHRAE. 2007b. ANSI/ASHRAE Standard 15-2007, Safety Standard for
Refrigeration Systems. Atlanta: ASHRAE.
ASHRAE. 2009. ASHRAE Handbook–Fundamentals. Atlanta: ASHRAE.
ASME. 2007. Boiler and Pressure Vessel Code. New York: American Society
of Mechanical Engineers.
ASME. 2006. ANSI/ASME B31.5-2006, Refrigeration Piping and Heat Trans-
fer Components. New York: American Society of Mechanical Engineers.
BOCA. 1990. Basic National Mechanical Code. Country Club Hills, IL: Build-
ing Officials and Code Administration International, Inc.
IIAR. 1999. ANSI/IIAR-1999, Equipment, Design and Installation of Ammonia
Mechanical Refrigeration Systems. Washington, DC: International Institute
of Ammonia Refrigeration.
Fundamentals of Refrigeration SI, Second Edition 277

NFPA. 2010. NFPA 70: National Electrical Code. Quincy, MA: National Fire
Protection Association Inc.
Nuckolls, A. 1933. The Comparative Life, Fire, and Explosion Hazards of
Common Refrigerants. Miscellaneous Hazard No. 2375. Northbrook, IL:
Underwriters Laboratory.
UL. 2009. ANSI/UL 207-2009, Refrigerant-Containing Components and
Accessories, Nonelectrical. Northbrook, IL: Underwriters Laboratories.
UNEP. 1987. The Montreal Protocol on Substances That Deplete the Ozone
Layer. New York: United Nations Environment Programme.
278 Chapter 10 Refrigerant Selection

Skill Development Exercises for Chapter 10


Complete these questions by writing your answers on the worksheets at the back of this book.

10-1 What is the significance of the Montreal Protocol to the HVAC&R industry?
10-2 A refrigeration system is required to maintain a cold space at –28.89°C, when
the refrigerant condensing temperature could be as high as 35°C. If the refrig-
erating load is 700 kW, suggest a refrigerant for this system. Support your
selection with technical reasons.
10-3 A refrigerant not listed in the tables included in Chapter 10 is R-113. Using
ASHRAE Standard 34 classification criteria, identify the features of this refrig-
erant discernible from the designation.
10-4 A chiller unit using R-123 as the refrigerant is located in a mechanical room of
a building. The room’s dimensions are 6.1 m wide, 9.1 m long, and 3.66 m
high. The pressure and temperature in the room are 101.325 kPa and 21.1°C,
respectively. Determine the mass of R-123 that must leak into the room to pro-
duce the maximum exposure that a worker is permitted to experience over a
40-hour work week.
10-5 Figure 10-5 shows two configurations that could result in a trapped liquid.
Sketch and label one other configuration that may also trap liquid.
10-6 Consider the same commercial fabrication as given in Example 10-1, where
the refrigerant is changed to propane (R-290). With a refrigerant charge of
25.4 kg R-290, does ASHRAE Standard 15 indicate that a machinery room
is required?
10-7 A 101.6 mm, schedule 40 pipe 30.5 m long contains liquid ammonia at –6.67°C
and 345 kPa. A solenoid-operated shutoff valve is located at the end of the pipe
where the fluid is heading. The velocity of the flowing liquid is 1.83 m/s. If the
solenoid valve takes 0.02 s to close, estimate the momentary overpressure exerted
by the refrigerant in the pipe.
Product Cooling,
Freezing Loads, and
Cooling Secondary Fluids
Study Objectives
This chapter presents the refrigerating loads associated with the cooling and
freezing of food products. The rates at which these processes occur are particu-
larly important because of their influence on food quality. Some of the equip-
ment used to cool and freeze food products (including the air-blast freezer,
plate freezer, spiral freezer, and fluidized bed freezer) are examined to show
how the principles are applied in practice. Fluids other than the refrigerant,
generally referred to as secondary fluids, may be used to transport heat from a
particular point in a process or location in a plant via heat exchanger to the
refrigeration system. Characteristics of the heat exchanger and several second-
ary fluids are presented in conjunction with the operation and performance of
the system. After studying Chapter 11, you should be able to

 explain the cooling process for unfrozen food products;


 explain the freezing process associated with food products;
 identify an important source of cooling and freezing information for food
products;
 estimate the quantity of heat that must be removed from a food product
when cooled from an initial temperature to a final temperature;
 estimate the freezing time associated with food products that are simple in
shape and small in size;
 describe how an air-blast freezer, a plate freezer, and an immersion freezer
operate;
 explain why a scraped-surface heat exchanger is used for liquid food
products; and
 explain why heat exchangers are needed with refrigeration systems for
cooling a secondary fluid.

Instructions
Read the material in Chapter 11. At the end of the chapter, complete the skill
development exercises without referring to the text. Review again those sec-
tions of the chapter as needed to complete the exercises.
280 Chapter 11 Product Cooling, Freezing Loads, and Cooling Secondary Fluids

Introduction

This chapter concerns the determination of the refrigerating load. The refriger-
ating load is significant, as it directly impacts the capacity of the refrigeration
system. Up to this point, we have assumed that the load is known when consid-
ering the refrigeration system. This chapter focuses on the refrigerating load
that originates with the cooling and freezing of food products and with the
cooling of liquids.
In the context of food refrigeration, the word product generally refers to
food, and we will follow that convention here. Similarly, cooling implies a
temperature reduction of the food product. The final temperature may be above
or below the freezing temperature of the product. Furthermore, freezing
involves the conversion of the product’s internal fluid to a solid by the removal
of the freezing latent heat (or latent heat of fusion).
Refrigeration of food products is the largest application within the cate-
gory of industrial refrigeration. Food products stored at temperatures below
normal room temperature have extended storage time, or shelf life. This is
why food products are refrigerated. Figure 11-1 shows the increase in shelf

Adapted from a figure ©Academic Press, Inc.

Figure 11-1 Variation of strawberry shelf life with storage temperature.


Fundamentals of Refrigeration SI, Second Edition 281

life for strawberries with decreasing temperature. The trend of increasing


shelf life with decreasing storage temperature holds for all food products. This
chapter investigates the quantity of heat that must be removed from the prod-
uct to obtain a suitable storage temperature.
Storage temperatures below 0°C drastically reduce the growth rates of
microorganisms, which in turn reduces the rate of deterioration of the food
product due to microbial activity. Additionally, low storage temperatures
decrease the enzymatic and oxidation chemical reactions in the product, which,
for example, relate to the ripening of fruit. Upon freezing, the ice crystals
within the product decrease the quantity of water that may participate in these
reactions that cause deterioration.
The quality of the food product is affected by the rate at which the freezing
process is accomplished. However, the particular food product influences the
freezing time that yields the best quality. For some products, quick freezing
times are needed so that only small ice crystals form, thereby minimizing the
structural damage to the product and preserving the product’s texture. Other
products are not sensitive to structural changes caused by the formation of ice
crystals and do not require quick freezing times. Still other products (such as
lettuce) cannot be stored at temperatures below freezing because the structural
damage is so severe that it destroys the texture.
Other factors may contribute to the refrigerating load. Heat gain through
the walls of the enclosure and by air leakage are major sources of heat. Other
sources include the heat given off by people, lights, and electrical equipment
(such as the motors that power forklift trucks). Accurate determination of the
refrigerating load requires the estimation of all heat contributions that must be
accommodated by the refrigeration system. The heat from the product—as
well as heat gains from conduction through the walls, air leakage into the
space, and other sources—must be determined. Consequently, depending on
the application, information from this chapter and the references listed at the
end may be needed to determine the refrigerating load.
It is important that the refrigeration system be neither oversized nor under-
sized in any of these applications. Generally, if the refrigeration system is over-
sized, it may not operate for a large fraction of the day. In such instances, if the
application is comfort air conditioning, the relative humidity inside the cooled
space may not be sufficiently reduced to achieve acceptable occupant comfort.
On the other hand, if the space is a refrigerated food warehouse that consists of
several large cooler and freezer rooms, then an oversized system would result
in the refrigeration system never operating at full capacity because the load
would always be less than the design load. Besides greater-than-needed capital
equipment, a refrigerating system operating at part load has lower system effi-
ciency. As a result, the energy needed to operate the refrigeration system would
be greater, resulting in higher operating costs. Experience also shows that part-
load operation may result in higher maintenance costs.
Objectives of the refrigeration system are also not met when the system is
undersized in relation to the load. With comfort air conditioning, the system
282 Chapter 11 Product Cooling, Freezing Loads, and Cooling Secondary Fluids

may operate over a too-high fraction of the day to meet the desired indoor tem-
perature. Additionally, the relative humidity may be too low. A more severe sit-
uation exists when the system is undersized so that available refrigerating
capacity is not sufficient to meet the desired indoor temperature. With a refrig-
erated food warehouse, the required indoor temperature may not be achieved
during the summer if the system is undersized. Consequently, both undersizing
and oversizing the refrigerating load lead to undesirable system performance.
Unfortunately, it is the refrigeration plant itself that is usually blamed for bad
performance.
This chapter extends the consideration of cooling load to include the cool-
ing of liquids other than the refrigerant that are circulated to another location
where they absorb heat. These liquids are generally referred to as secondary
fluids. With this configuration, a heat exchanger is employed, and the refriger-
ant is confined to only the refrigeration system. The flow rate and temperature
change of the secondary fluid determines the needed refrigeration capacity.
Another important cooling application explored involves the maintenance
of the interior space of a structure at a specific temperature. The structure may
be a building containing people, a warehouse storing chilled and frozen food
products, a cabinet used to display refrigerated food in a store, or a box for
storing perishable foods in the home. The refrigerating load in all these situa-
tions is called a space cooling load, because the interior space is maintained at
a set temperature.
The accurate determination of space cooling loads is difficult and is outside
the scope of our treatment of refrigeration. The variation of sunlight and out-
door temperature results in a significant change to the structure’s outside sur-
face temperature throughout the day. When the outside surface is exposed to
direct sunlight, its temperature increases to a value that can be, depending on
how the structure is constructed, much higher than the outdoor air temperature.
This higher surface temperature increases the heat conduction rate through the
wall. However, the heat that enters through the outside surface arrives at the
inside surface of the wall at a later time. This time delay influences the space
cooling load, particularly in buildings air conditioned for comfort. When the
structure is outside, the outdoor temperature variation is an important variable
in assessing the space cooling load.

Cooling Food Products


Every fruit and vegetable that cannot be frozen has an optimum temperature
for storage. The recommended source of this important information is Table 1
of Chapter 11 from the 2006 ASHRAE Handbook—Refrigeration (ASHRAE
2006), in which the temperature, relative humidity, and resulting shelf life are
given. Table 1 is reproduced in Appendix C, Table C-1, of this text.
Several thermal properties of food products are given in Chapter 9 of Refrig-
eration (ASHRAE 2006), such as the specific heat (cp) of the product material
in the unfrozen state, which is given in Table 3; this table is reproduced in
Fundamentals of Refrigeration SI, Second Edition 283

Appendix C of this book as Table C-2. The amount of heat that must be
removed from the product when cooling to the storage temperature is called the
product load. The resulting cooling load on the refrigeration system may be
continuous or intermittent depending on the cooling process. When the product
reaches the storage temperature, it no longer contributes to the refrigerating
load. A notable exception concerns the storage of fruits and vegetables, where
the process of respiration continues at storage temperatures above freezing. Res-
piration occurs because the fruit or vegetable is still alive, and the biological
activity taking place generates heat. Chapter 9, Table 9, of Refrigeration
(ASHRAE 2006) provides values for the heat of respiration for fresh fruits and
vegetables and is reproduced in Appendix C of this book as Table C-3.
The product cooling load involving temperatures above freezing is given by

Q = Mc p  T i – T f  (11-1)

where Q is the quantity of heat removed (kJ), M is the product mass (kg), cp is
the specific heat (kJ/kg·K), Ti is the initial temperature of the product (°C), and
Tf is the final, or storage, temperature of the product (°C).

Example 11-1

Problem If 6895 kg of Imperator carrots enter a chilling cooler at 33.3°C and are chilled
to 0°C each day, determine the product load.

Solution Appendix C, Table C-2, gives the specific heat above freezing for topped-
mature carrots as 3.94 kJ/kg·K. Using Equation 11-1 gives

Q =  6895   3.94   33.3 – 0  = 900,000 kJ

which is the heat that must be removed over a 24-hour period. The rate of cool-
ing is 900 000/24 or 37 500 kJ. This is 10.4 kW of refrigerating capacity.

If Example 11-1 required that the carrots be chilled in only 10 h, then the
cooling rate is 900 000/10 or 90 000 kJ/h, which is 25 kW of refrigeration.
Consequently, the cooling time must also be considered when calculating the
refrigerating load.
From Example 11-1, we see that product chilling is a process that involves
the variable or transient flow of heat from the product. The product, when placed
in a space of constant lower temperature, cools with its outside temperature fall-
ing more rapidly than its interior temperature. Additionally, the heat transfer is
driven by the temperature difference between the space or air temperature and
the surface temperature of the product. Therefore, as the product cools, the heat
transfer rate from the product reduces with time, and a temperature distribution
exists inside the product as the cooling takes place. The determination of the heat
284 Chapter 11 Product Cooling, Freezing Loads, and Cooling Secondary Fluids

transfer rate and the temperature distribution is possible, but difficult, to deter-
mine by calculation alone.

Example 11-2
Problem For the 6895 kg of Imperator carrots undergoing the cooling described in
Example 11-1, what would be the refrigeration requirement for these carrots
upon reaching 0°C due to only their heat of respiration?

Solution At a storage temperature of 0°C, Appendix C, Table C-3, shows that the heat of
respiration for Imperator carrots is 3936 kJ/day per tonne of produce. Thus,

6895 kg carrots- = 6.9 tonnes (mass)


-------------------------------------
1000 kg/tonne

The refrigeration load due to the heat of respiration is

 6.9 tonnes   3936 kJ/day per tonne of produce  = 27,158 kJ/day

where dividing by 24 h gives 1132 kJ/h, or about 0.3 kW of refrigeration.


While this particular refrigeration load is small, a carrot product quantity of
690 tonnes and a higher storage temperature of 10°C would result in a refriger-
ation load equal to 64 kW of refrigeration due to respiration.

Cooling time is important for two reasons. The first reason has already
been identified as the establishment of the product cooling load on the refriger-
ation system. The other reason is that the cooling time influences product qual-
ity. If the cooling time is too long, product quality is adversely affected. In
practice, determination of the cooling time for a particular product is based on
acceptable food quality and safety. For example, the U.S. Department of Agri-
culture has established that the chilling of beef must occur within a 72-hour
period. This is the maximum cooling time for the interior temperature of the
beef carcass to reach 0°C to 1.1°C, as discussed in Chapter 17 of ASHRAE
Handbook—Refrigeration (ASHRAE 2006).
Figure 11-2 shows the configuration of the beef carcass when undergoing
chilling and the measurement of the “deep-round” temperature (i.e., the center
temperature of the largest portion of the carcass, which is always the warmest
part during chilling). The variation of three beef carcass temperatures with
cooling time is shown in Figure 11-3. The surface temperature is observed to
drop the most quickly, while the deep round temperature takes the longest. The
intermediate temperature curve is the average temperature of all parts of the
beef carcass. Examination of Figure 11-3 reveals that the determination of the
cooling time is uncertain for two reasons. First, because temperature curves
flatten over time, it is difficult to identify when the temperature reaches the
Fundamentals of Refrigeration SI, Second Edition 285

Figure 11-2 Temperature measurement of the deep round portion in a beef carcass.

Figure 11-3 Variation of three beef carcass temperatures.


286 Chapter 11 Product Cooling, Freezing Loads, and Cooling Secondary Fluids

needed storage temperature. Second, the variation of temperature at several


positions in the product must be measured over the entire cooling time.
However, applying knowledge of heat transfer yields a new plot, which is
sometimes called the Gurney-Lurie chart after the researchers who first used it.
This chart plots the temperature as a dimensionless temperature ratio on a log-
arithmic scale against time. The cooling data plotted in Figure 11-3 is replotted
using the Gurney-Lurie format to give Figure 11-4.
The dimensionless temperature ratio is equal to

T–T
 = ---------------r- (11-2)
Ti – Tr

where T (°C) is the temperature measurement of the product at time t, Ti (°C) is


the initial temperature of the product before the cooling begins, Tr is the space
or air temperature in which the product is placed for cooling (°C), and  is the
dimensionless temperature ratio. The cooling curves now appear as almost
straight lines and provide reasonably accurate estimates of the cooling times.
Semi-log graph paper, as used in Figure 11-4, is convenient for making the
cooling charts. Fortunately, experience and data gained from the testing of var-
ious food products is available to assist in the determination of time constants.

Figure 11-4 Gurney-Lurie chart of Figure 11-3 data.


Fundamentals of Refrigeration SI, Second Edition 287

Example 11-3

Problem The initial temperature of a beef carcass after slaughter is 40.55°C. The room
air temperature where the carcass is placed after slaughter is maintained at 0°C.
What is the cooling time needed if the deep-round temperature must reach
4.44°C?

Solution Calculate .

 4.44 – 0 
 = --------------------------- = 0.110
 40.55 – 0 

From Figure 11-4, read the needed cooling time to be approximately 37 h.


The 2006 ASHRAE Handbook—Refrigeration, Chapter 10 (ASHRAE 2006),
describes product cooling time estimates based on the “equivalent heat transfer
dimensionality” method, which, while having improved accuracy, is more
complicated to use.

Freezing Food Products

Appendix C, Table C-1, contains a column for the highest freezing tempera-
ture. This is the temperature at which freezing begins inside the product. Note
that all products begin freezing at a temperature below 0°C. This occurs
because the liquid in the product is not pure water but rather a mixture of sev-
eral chemicals (such as sugar and water). As the freezing process occurs, the
water concentration in the liquid decreases, resulting in a slightly lower freez-
ing temperature for the remaining liquid. Therefore, the freezing process
occurs over a small temperature range rather than at one specific temperature.
Figure 11-5 shows temperature measurements made inside several fish fil-
let configurations while inside a blast freezer. Also plotted in Figure 11-5 is the
air temperature supplied by the blast freezer. Observe that for each fish fillet
tested, the freezing process is characterized by an almost-constant temperature.
Careful examination of Figure 11-5 reveals that the freezing temperature is
somewhat below 0°C and that it decreases slightly as freezing occurs. Interest-
ingly, at the product’s recommended storage temperature, typically 20% of the
water remains in the liquid state.
The cooling of a product to a temperature below the freezing temperature
involves three steps:

• Product cooling down to the freezing temperature


• Product freezing at a nearly constant temperature
• Product cooling down to a subfreezing temperature
288 Chapter 11 Product Cooling, Freezing Loads, and Cooling Secondary Fluids

Figure 11-5 Temperature measurements inside fish fillets in air-blast freezer.

In the form of an equation, the total heat removed is given by

Q total = Q unfrozen + Q freeze + Q frozen (11-3)

where Qunfrozen is the heat removed during cooling down to the product freez-
ing temperature (kJ), Qfreeze is the latent heat removed during the freezing pro-
cess (kJ), and Qfrozen is the heat removed after freezing is complete to achieve
the storage temperature (kJ). We may also write

Q total = Mc p unfrozen  T initial – T freeze 


(11-4)
+ Mh L + Mc p frozen  T freeze – T final 

where
M = product mass, kg
cp, unfrozen = specific heat of the unfrozen product, kJ/kg·°C
Fundamentals of Refrigeration SI, Second Edition 289

cp, frozen = specific heat of the frozen product, kJ/kg·°C


hL = product freezing latent heat, Btu/lbkJ/kg
Tinitial = initial product temperature, °C
Tfreeze = product freezing temperature, °C
Tfinal = final or storage product temperature, °C

An interesting feature of the unfrozen and frozen product specific heats


should be noted: the frozen value is roughly one-half of the unfrozen value. This
is because food products contain a large proportion of water, and the specific
heat of ice (2.05 kJ/kg·°C) is about one-half the value for water (4.19 kJ/kg·°C).

Example 11-4
Problem Blueberries at a rate of 90.1 kg/min enter a continuous fluidized bed freezer
generating a constant air temperature of –20.56°C. The blueberries enter the
freezer at 26.67°C and exit from the freezer at a temperature of –15°C. Calcu-
late the refrigerating capacity to cool, freeze, and further cool only the blueber-
ries passing through the freezer.

Solution In this example, a fixed mass of product cannot be identified. However, we real-
ize that the product flows into the fluidized bed freezer at the rate of 90.7 kg/min
and that this rate may be expressed as a mass divided by an increment of time.
This is equivalent to writing M/t, where the mass, M, is the product mass mov-
ing into the freezer over time t. The same concept applies to Qtotal, the heat
transfer associated with mass M.
Dividing both sides of Equation 11-4 by t gives
Q total Mc p unfrozen Mh Mc p frozen
------------- = ------------------------------  T initial – T freeze  + ----------L- + -------------------------  T freeze – T final 
t t t t

Furthermore, we can write

M- = m· and Q total
-------------
·
---- = Q
t t

where the dot above the quantity denotes a time rate of change or flow. Substi-
tuting m· and Q· total into the above equation gives
·
Q = m· c p unfrozen  T initial – T freeze  + m· h L + m· c p frozen  T freeze – T final 

For this example, m· = 90.7 kg/min, and from Appendix C, Table C-2, for
blueberries, we have
cp,unfrozen = 3.81 kJ/kg·°C
cp,frozen = 2.05 kJ/kg·°C
290 Chapter 11 Product Cooling, Freezing Loads, and Cooling Secondary Fluids

hL = 283 kJ/kg
Tfreeze = –1.61°C

So,

·  90.7   3.81   26.67 –  – 1.61   +  90.7   284 


Q = 
 +  90.7   2.05    – 1.61   – 20.56   
= 9773 + 25,759 + 3523
= 39,055 kJ/min

or that
·
Q = 39,055
---------------- = 650 kW, refrigerating capacity
60

Equation 11-4 is useful for estimating the refrigerating capacity needed, but it
does not provide any information regarding the freezing time. The determination
of the freezing time, as already mentioned, is important for the preservation of
food product quality. Unfortunately, the freezing time is also difficult to predict.
Freezing time depends on numerous variables: supply-air temperature, velocity
of supply air impacting the product, size of product, shape of product, product
thermal conductivity, product heat capacity, product material density, product
packaging, and the initial temperature of the product, among others.
To put the situation into focus, Example 11-4 identified the refrigerating
load in conjunction with the blueberry flow rate through the freezer. What was
not specified is the period of time (residence time) that the blueberries must be
inside the freezer to be frozen upon exiting for the given operating conditions.
In an actual installation, the fan power, air infiltration rate, enclosure heat
losses, and conveyor drive heat gains (if an inside freezer) all contribute to the
refrigerating load in addition to that of the product. For example, a typical indi-
vidual quick-freezing (IQF) freezer requires approximately 844 kW of refriger-
ating capacity to freeze 5.44 tonnes per hour. A rough estimate is 140 kW of
refrigeration per tonne of product.
Complicated calculation procedures have been applied to the prediction of
freezing times but have only provided approximate values to date. This effort is
an area of active research, and improved procedures will eventually result. In
simple terms, the refrigeration system, by means of a cooling medium, such as
air in a blast freezer, only influences the surface temperature of the product. The
thermal characteristics of the product regulate the flow of heat from the interior
to the surface as the freezing interface moves inward from the surface toward the
center. Figure 11-5 shows this by noting the relatively long freezing time for the
6.35 cm thick fish fillets as compared to the short freezing time for the 2.54 cm
thick fillets. Obviously, product thickness affects freezing time; a greater time is
needed for the freezing interface to move through the thicker product.
Fundamentals of Refrigeration SI, Second Edition 291

Equations have been developed for the prediction of freezing times. Their
value is not so much in the accuracy of the time predicted but in identifying the
changes in freezing time resulting from process changes (such as air tempera-
ture and air velocity). Furthermore, freezing times may be estimated using new
products and freezing equipment before installation. The first equation devel-
oped for predicting the freezing time is the Planck equation:

h L D D 1
t freezing = -----------------------------------  ----  -----
- + ----- (11-5)
 T freeze – T air  N 4k h c

where
tfreezing = freezing time, h
 = product material density, kg/m3
hL = freezing latent heat, kJ/kg
Tfreeze = product freezing temperature, °C
Tair = supply air temperature, °C
D = distance to product center parallel to direction of airflow, m
N = shape factor: 2 for slab, 4 for cylinder, or 6 for sphere
k = product material thermal conductivity, kW/m·°C
hc = surface convection heat transfer coefficient, kW/m2·°C

The variables in Planck’s equation require knowledge of the food product


and the convection heat transfer coefficient. Rough estimates for these vari-
ables may be obtained using the following equations:

Density

 = 1.1p water

where the density of water is 1000 kg/m3.


Thermal Conductivity

k (above freezing temperature)


k = 0.148 + 0.493fw (vegetables and fruits)
k = 0.080 + 0.52fw (meats)

where fw is the fraction of water in the product.


Convection Heat Transfer Coefficient

h c = 25.67V 0.6  W/m 2 ·°C 

where V is the air velocity at the product (m/s).


292 Chapter 11 Product Cooling, Freezing Loads, and Cooling Secondary Fluids

Planck’s equation, in addition to applying to the freezing process, may also


be used in cooling situations when the product’s initial temperature is greater
than the freezing temperature. Here, the total enthalpy change replaces the
latent heat of freezing.

Example 11-5

Problem Consider again the blueberries in Example 11-4 that pass through the fluidized-
bed IQF freezer. The upward air passes through the bed of blueberries at a tem-
perature of –26.1°C with a velocity of 3.75 m/s. Approximate the blueberries as
spheres with a diameter of 12.7 mm Estimate the freezing time for the blueber-
ries at their freezing temperature.

Solution Using Planck’s equation, we first obtain the necessary variables:

  (1.1)(1000) = 1100 kg/m3


hL = 283 kJ/kg
Tfreeze = –1.61°C
Tair = –26.1°C
Blueberry diameter = 12.7 mm; Distance D = 12.7/2 = 6.35 mm
N = 6 (spherical shape)
k  0.148 + 0.493 fw = 0.148 + 0.493(0.846) = 0.565 W/m·K
Appendix C, Table C-2, gives fw = 0.8461
hc  25.67V 0.6 = (25.67)(3.75)0.6 = 56.7 W/m2·K

Substituting,

h L D D 1
t freezing = -----------------------------------  ----  -----
- + -----
 T freeze – T air   N  4k h c

 1100   284   6.35   1/1000  6.35   1/1000 - + ---------


------------------------------------ 1-
= -------------------------------------------- -------------------------------------
 – 1.61 –  – 26.1   6  4   0.565  56.7
= 273 s

Exercise caution when using Planck’s equation, particularly when assigning


numerical values to the variables. Measured values for product density are not
generally available and vary among product samples. The product’s freezing
latent heat usually depends on the quantity of water in the product and the value
of the latent heat of fusion (freezing) for water. The product’s thermal conduc-
tivity is lower when unfrozen than when frozen by a factor of about two. Also,
Fundamentals of Refrigeration SI, Second Edition 293

the initial freezing temperature and the final temperature when the product is
fully frozen are different and are not accounted for in Planck’s equation.
However, even with all these cautions, Planck’s equation remains a popular
formula for predicting freezing times. Again, experience with specific food
products is sometimes available and should be reviewed when estimating
freezing times. In the example with blueberries, the actual freezing times are
from 2 to 2.5 times the value predicted in Example 11-5. For example, in an
actual IQF freezer, the freezing time for blueberries is from about 10 to 11
minutes. To obtain greater accuracy in estimating freezing times for food prod-
ucts, methods involving advanced heat transfer calculations are described in
Chapter 10 of the 2006 ASHRAE Handbook—Refrigeration (ASHRAE 2006).

Food Freezing Equipment


As we have seen, a food product to be frozen must be exposed to a low-
temperature medium for an adequate period of time to remove the sensible
and freezing latent heat. The freezing process may be categorized depend-
ing on the manner in which the heat is removed from the product. Three
categories of freezing are as follows:
Air-blast freezing. High-velocity air directed toward the product absorbs heat
from the product by convection. Evaporators or liquid-to-air heat exchangers
maintain the air at a suitable temperature.
Contact freezing. The refrigerant located on one side of a metal barrier
removes heat from the product by heat conduction through the metal.
Immersion freezing. The food product is submerged in a low-temperature
brine cooled by evaporators in a refrigeration system.
The selection of a freezing method is dependent on matching the preceding
processing steps with the subsequent packaging steps associated with complete
product production.

Air-Blast Systems
Low-temperature, high-velocity air passing in direct contact over a small-size
product achieves rapid freezing times (i.e., IQF). Examples of products that fall
into this category include peas, corn kernels, strawberries, and blueberries.
Figure 11-6 shows one application of IQF where a fluidized bed is gener-
ated using an upward flow of air passing through a mesh conveyor belt carrying
the product from the inlet to the outlet of the unit. The airflow rate is sufficient
to partially suspend the food product in the airstream to obtain high convection
heat transfer coefficients. Sometimes this partial suspension of the material in
an airflow is called dense phase fluidization. This is shown in Figure 11-6a.
A fluidized bed is depicted in Figure 11-6b. Notice that the flow path above
the product expands, reducing the air velocity and preventing the loss of product
294 Chapter 11 Product Cooling, Freezing Loads, and Cooling Secondary Fluids

(a)

(b)

Figure 11-6 (a) Upward airflow to achieve fluidization and (b) fluidized bed freezer.

from the conveyor. Use of the fluidized bed concept is limited to product shapes
and sizes capable of being suspended in an airstream.
Figure 11-7 shows several configurations of the air-blast freezer. An advan-
tage of the air-blast freezer is its accommodation of products with awkward
shapes and variable sizes. When the product is packaged, the packaging material
serves as a barrier between the product and the moving cold air. Figure 11-7a is a
stationary freezing tunnel where the product is placed on trays, which, in turn,
are placed on racks. The racks are arranged to provide gaps that allow the mov-
ing air to pass closer to the product. Baffles must be installed to ensure that air
flows equally over the product. The four rows of carts shown in Figure 11-7a
must be loaded sequentially, one at a time beginning with the right-most cart.
Fundamentals of Refrigeration SI, Second Edition 295

A push-through tunnel, shown in Figure 11-7b, is a variation on the station-


ary freezing tunnel. The movement of product into and out of the push-through
tunnel is more convenient than the stationary configuration, but both configura-
tions handle the product in a batch fashion. The tunnel must also be loaded one
row at a time. Freezing times are generally longer when compared to IQF
because of the lower air velocity next to the product and because of the pres-
ence of the packaging material.
Automation of the air-blast freezer has resulted in a number of mechanical
configurations, all of which provide continuous movement of the product
through the air-blast unit. Two such configurations are shown in Figures 11-7c
and 11-7d where a multitier belt freezer and a spiral belt freezer are shown,
respectively. Successful operation of either type freezer in providing uniform
freezing of the product requires that the product be uniformly distributed over
the belt. In this way, the moving air is evenly directed past all the product. The
air may either flow perpendicularly through the belt or it may flow parallel to
the belt.
The multitier belt configuration reduces the floor space that would otherwise
be needed if only a single belt was used. Also, the multitier arrangement allows
belts to travel at different rates. For example, the product may be “crust frozen”
on the first belt and then “bulk frozen” on the slower moving second belt.
The spiral belt freezer configuration in Figure 11-7d provides the highest belt
area per unit floor space. A large number of tiers (40 or more) may be stacked
upon one another using a rotating drum. Special mechanical features of the col-
lapsible conveyor belt allow the spiral motion to occur, which also eliminates any
product transfer points inside the freezer. Figure 11-7d shows the airflow pattern
as horizontal with no baffling. Other spiral freezer designs use baffles to split and
direct the airflow in a manner to decrease freezing times. Additionally, the air-
flow pattern may be arranged such that the coldest air first contacts the coldest
product. This is generally referred to as a parallel arrangement.

Contact Freezers

The most common contact freezer is the plate freezer, shown in Figure 11-8.
The product is frozen when it is held between two refrigerated plates. Evapora-
tion of refrigerant inside the plates causes the refrigeration. The barrier
between the product and refrigerant is the plate material and the packaging
material, if present. Direct contact between the product and the metal plate pro-
motes rapid freezing times. Compared to air-blast systems, plate freezers are
more efficient, cost more to purchase, and cost less to operate. The refrigerant
is transported to the plates through flexible lines so that, after the product is
frozen, the plates may be moved apart. In this way, the product may be
removed from the freezer. Manual or automatic systems may be used to push
the product from the freezer onto a conveyor belt. However, the mechanical
complexity of the plate freezer results in the need for careful maintenance.
296 Chapter 11 Product Cooling, Freezing Loads, and Cooling Secondary Fluids

(a)

(b)

(c)

(d)

Figure 11-7 (a) Air-blast stationary freezing tunnel, (b) air-blast push-through freezing tunnel,
(c) multitier belt air-blast freezer, and (d) horizontal airflow spiral belt air-blast
freezer.
Fundamentals of Refrigeration SI, Second Edition 297

Figure 11-8 Contact plate freezer.

Immersion Freezing
A refrigerated brine held in a tank may serve as a freezer. Typical brine solu-
tions are glycol or sodium chloride mixed with water. The product may either
be immersed in the brine solution or it may be sprayed when passing through
the tank. Turkeys and chickens inside plastic bags are a common application of
immersion freezing using a propylene glycol solution. The advantages of
immersion freezers include the acceptability of irregularly shaped products and
the high convection heat transfer coefficient at the product surface. A disadvan-
tage is that the handling of the product is greater than that required in the air-
blast systems. Also, the brine solution must be washed from the surface of the
product after freezing is complete. However, with fish products, such as crab
sections or whole dressed fish, a sodium chloride solution brine may contact
the product without the danger of contamination.

Cooling Liquid-Food Products


We now restrict our attention to liquid foods, such as ice cream and other dairy
products. The removal of heat from the liquid product is generally accomplished
with greater efficiency before it is packaged. A common method used to freeze
a liquid product is the scraped-surface heat exchanger, typically tubular in
298 Chapter 11 Product Cooling, Freezing Loads, and Cooling Secondary Fluids

shape. The outer shell functions as the evaporator for the refrigeration system,
absorbing heat from the liquid product passing through the interior. Figure 11-9
depicts the scraped-surface heat exchanger, where the scraper blades continu-
ally remove frozen product from the refrigerated surface.
To accomplish the freezing of liquid foods, the residence time of the prod-
uct in the scraped-surface heat exchanger must be sufficient to reduce the tem-
perature of the product to several degrees Fahrenheit below the freezing
temperature. This causes the formation of ice crystals that are removed from
the refrigerated surface by the action of the rotating blades. As a result, the
convection heat transfer coefficient on the refrigerated surface is high, which
improves the heat transfer rate from what it would otherwise be. When the
product temperature is several degrees below the freezing temperature, about
60% to 80% of the freezing latent heat is removed and the product is a frozen
slurry. The slurry then flows in properly sized transport piping to be placed in

©Academic Press Inc. Reprinted with permission.

Figure 11-9 Scraped-surface heat exchanger.


Fundamentals of Refrigeration SI, Second Edition 299

appropriate packaging. At this point, further freezing of the product may occur
by using an air-blast freezer or plate freezer.
Either batch or continuous feed of liquid foods may be applied with the
scraped-surface heat exchanger. With the batch system, a specific quantity of
liquid product is placed inside the heat exchanger and the freezing continued
until the product reaches the desired condition; then it is taken out of the unit.
A continuous-feed liquid-food system uses a scraped-surface heat exchanger in
which the residence time of the product inside must be adequate to achieve the
needed product condition upon exit.

Cooling Secondary Fluids

Whenever the refrigerant cannot be delivered to the location where the heat
must be removed, an alternative is to introduce another fluid cooled by the
refrigeration system, which is then pumped to the place where the heat is
removed. A typical arrangement is to pump the other fluid through a heat
exchanger that simultaneously serves as the evaporator for the refrigeration
system. In this situation, the refrigeration system has the objective of cooling
another fluid. Alternative names for the other fluid are secondary coolant, heat
transfer fluid, brine, secondary refrigerant, or secondary fluid. The secondary
fluid remains a liquid throughout the entire system. Applications of secondary
fluids occur in the food processing industry, ice rinks, environmental test facil-
ities, and many other industrial processes.
A secondary fluid may be used in conjunction with a refrigeration system
for several reasons. For example, refrigeration systems designed to minimize
the refrigerant charge typically confine the refrigerant to only the refrigeration
equipment. Transport of the refrigerant through the piping to reach the point
where the heat is removed is therefore eliminated. To accomplish this, a sec-
ondary fluid must be cooled and transported to that same point. The design of
such a system requires consideration of two competing concerns:

• heat transfer rate into the secondary fluid during transport versus the quan-
tity of pipe insulation needed to reduce the heat transfer rate
• pipe diameter and consequent pressure drop versus the pumping power
needed to circulate the fluid

These two considerations must be balanced to arrive at the best overall sec-
ondary fluid arrangement while satisfying the overall refrigerating load. An
application where this would be considered is the refrigeration of the display
cases in a food supermarket, where ammonia is a candidate refrigerant. In this
instance, the ammonia charge is relatively small and confined to the machinery
room, which significantly reduces the potential exposure of the public to an
ammonia leak.
300 Chapter 11 Product Cooling, Freezing Loads, and Cooling Secondary Fluids

Secondary Fluid Properties

Information about the physical properties of secondary fluids is necessary for


the successful application of these fluids to refrigeration systems. Before inves-
tigating in specialized fluids, water is sometimes used as a secondary fluid,
particularly in situations where freezing temperatures are highly unlikely. An
example occurs in the commercial building industry with water chillers: a
refrigeration system using an available refrigerant (such as R-134a, R-123,
etc.) cools water to an above-freezing temperature and circulates the water to
coils throughout the building to provide cooling.
Water is an excellent secondary fluid because its viscosity is relatively low
and its specific heat is relatively high. Physical properties of saturated water for
several temperatures are given in Table 11-1. While these properties are given
for saturated water, they are also valid for subcooled water when evaluated at
the saturated temperature, because the pressure has negligible influence on
these properties.
A solution of calcium chloride and water is an example of a common
refrigeration brine. Figure 11-10 shows the variation of specific heat, specific
gravity, viscosity, and thermal conductivity of calcium chloride brines. The
specific gravity is defined as the ratio of the density of the fluid in question
divided by the density of water. Thus, water has a specific gravity of one. The
dynamic viscosity is given in units of centipoise. Figure 11-10 shows that cal-
cium chloride brines may be used at temperatures below 0°C without danger of
freezing.
Observe in Figure 11-10 that the calcium chloride brine specific gravity
and viscosity are higher than those of water, which results in greater pumping
energy to achieve the needed circulation rate.
Higher pumping costs are associated with brines, a significant disadvan-
tage when applying brines to refrigeration systems. When estimating the per-
formance of secondary fluids in a system, normal practice is to base the
physical property values on the average temperature of the fluid as it passes
through a particular component.

Table 11-1 Physical Properties of Saturated Water

Density, Thermal
Temperature, Specific Heat, Viscosity,
Conductivity,
°C kg/m3 kJ/kg·K mPa/s
W/m·K
0 998.72 4.223 1.790 0.5615
5 998.40 4.212 1.567 0.5712
10 998.56 4.199 1.306 0.5803
15 997.76 4.191 1.121 0.5907
Fundamentals of Refrigeration SI, Second Edition 301

Figure 11-10 Properties of calcium chloride brines: (a) specific heat, (b) specific gravity, (c)
viscosity, and (d) thermal conductivity.
302 Chapter 11 Product Cooling, Freezing Loads, and Cooling Secondary Fluids

Another brine used in refrigeration applications is sodium chloride brine,


which has physical properties similar to those of calcium chloride brine. Both
brines must be treated to inhibit corrosion and control the production of depos-
its. The treatment for calcium chloride typically involves the routine testing for
pH to maintain the appropriate alkalinity and the addition of corrosion inhibi-
tors. Application of brines to refrigeration occurs in the industrial refrigeration
field and in ice skating rinks.
Another example of a secondary fluid is the ethylene glycol and water solu-
tion, whose physical properties are given in Figure 11-11. Generally, ethylene
glycol is inhibited by the manufacturer for corrosion control. The inhibitor’s
basic function is to form a protective surface barrier between the metal surface
and the solution. Propylene glycol is another inhibited glycol that has an
unusual feature: it is nontoxic. For this reason, propylene glycol finds wide
application in the food industry when cooling and freezing food products.
Figure 11-11 shows that the freezing temperature of the ethylene glycol
solution is below the freezing temperature of pure water, depending on the con-
centration. This is seen by following a particular curve of constant concentra-
tion (say 60%) toward the left and noting where it strikes the water line (or
freezing line when below 0°C). Note that the freezing temperature of some gly-
col/water solution concentrations are lower than the freezing temperature of
pure ethylene glycol.
Furthermore, the specific gravity and viscosity of ethylene glycol aqueous
(water) solutions are greater than those for only water. Therefore, these solu-
tions also require high pumping costs to circulate the fluid. Because the viscos-
ity of propylene glycol aqueous solutions is somewhat higher than that of
ethylene glycol solutions, the pumping costs are even higher. Also, the freeze
protection provided by any of these secondary fluids depends on the concentra-
tion of the solution.

Refrigerating Secondary Fluids

Figure 11-12 shows a vapor-compression system cooling a secondary fluid in a


closed system. Observe that the evaporator absorbs heat from the circulating
fluid, thereby decreasing its temperature. The refrigerating load is some dis-
tance from the refrigeration system, which is met by the pumped secondary
fluid. The expansion tank is needed to accommodate the changes in volume of
the secondary fluid resulting from temperature changes. For example, if the
system shuts down, the secondary fluid warms and expands into the volume
provided by the expansion tank.
It should be mentioned that starting a refrigeration system when the sec-
ondary fluid is warm results in a refrigerating load that is greater than the
design load. This higher load is usually referred to as a pulldown load. It
depends on the quantity of secondary fluid in the system and on the fluid’s
properties, namely specific heat (cp). Additionally, higher-than-design loads
Fundamentals of Refrigeration SI, Second Edition 303

Figure 11-11 Properties of aqueous solutions of ethylene glycol: (a) specific heat, (b) specific
gravity, (c) viscosity, and (d) thermal conductivity.

result from sudden decreases in the required temperature. For this reason, sys-
tems are sometimes intentionally oversized.
Figure 11-12 also shows the importance of the heat exchanger in that it
serves as the link between the refrigeration system and the secondary fluid
flow loop. Figure 11-13a depicts a counterflow heat exchanger with labeled
inlet and outlet fluid streams. The counterflow arrangement is the simplest
configuration. Other more elaborate configurations are used in practice (most
notably the shell-and-tube geometry), but all exchangers operate under the
same fundamental principles. We shall confine our attention to the counterflow
heat exchanger.
Disregarding the internal flow arrangement and construction of the heat
exchanger, the application of the conservation of energy reveals that the heat
transfer from fluid 1, the heat transfer to fluid 2, and the heat transfer rate
through the internal surface are all equal. Strictly speaking, this is only true if
the heat exchanger is operating at steady-state and steady flow conditions,
304 Chapter 11 Product Cooling, Freezing Loads, and Cooling Secondary Fluids

Figure 11-12 Refrigeration system with closed secondary fluid system.

which means that the operating characteristics (all temperatures and flows) of
the heat exchanger do not change with time. We also note that if fluid 1 is
cooler than fluid 2, energy conservation gives the same result, except the direc-
tion of heat transfer is reversed. The equations for the three heat transfer rates
are as follows:

Heat Transfer Rate from Fluid 1

·
Q = m· 1 c p1  T 1 i – T 1 o  (11-6)

Heat Transfer Rate to Fluid 2

·
Q = m· 2 c p2  T 2 o – T 2 i  (11-7)

Heat Transfer Rate Across Internal Area

·  T 1 i – T 2 o  –  T 1 o – T 2 i 
Q = UA ------------------------------------------------------------------ (11-8)
T 1 i – T 2 o
ln  -------------------------
T 1 o – T 2 i
Fundamentals of Refrigeration SI, Second Edition 305

(a)

(b)

Figure 11-13 Schematics of two heat exchangers: (a) a counterflow heat exchanger and (b) an
evaporator cooling a secondary fluid.
306 Chapter 11 Product Cooling, Freezing Loads, and Cooling Secondary Fluids

where
·
Q = heat transfer rate, kW
m·  m·
1 2 = mass flow rates of fluids 1 and 2, kg/h
cp1, cp2 = specific heat of fluids 1 and 2, kJ/kg·°C
T = fluid temperature, °C
U = overall heat transfer coefficient, kW/m2·K
A = internal heat transfer area, m2

Equation 11-8 results from the heat transfer rate through the heat
exchanger’s internal surface area available for heat transfer, which is given by

·
Q = UAT m (11-9)

where Tm is the log-mean temperature difference between the two fluids.
Equation 11-9 was first used in Chapter 4 on evaporators when liquid coolers
were studied. Generally, when considering heat exchangers in a system, the
entering temperatures, the fluid flow rates, the specific heat of the fluids, and
the product UA are known. Equations 11-6, 11-7, and 11-8 have three
·
unknowns— Q , T1,o, and T2,o—which are the heat transfer rate and the two
·
outlet temperatures, respectively. With Equations 11-6, 11-7, and 11-8, Q may
be eliminated to give

m· 1 c p1  T 1 i – T 1 o  = m· 2 c p2  T 2 o – T 2 i  (11-10)

 T 1 i – T 2 o  –  T 1 o – T 2 i 
m· 1 c p1  T 1 i – T 1 o  = UA ------------------------------------------------------------------ (11-11)
T 1 i – T 2 o
ln  -------------------------
 T 1 o – T2 i 

Solving for the outlet temperature, T1,o is found to be

 
 1 – eD 
T 1 o = T 1 i –  T 1 i – T 2 i   --------------------------
· c - (11-12)
m 1 p1
– e D
 --------------
·
m 2 c p2

where

1   --------------
1 
D = UA  -------------- –
 m· 1 c p1  m· 2 c p2
Fundamentals of Refrigeration SI, Second Edition 307

The outlet temperature T1,o is dependent only on inlet fluid conditions


and the physical characteristics of the heat exchanger, namely U and A.
Equation 11-10, with T1,o determined, may be used to compute T2,o.
Equations 11-10, 11-11, and 11-12 are valid if the fluids do not change
phase when passing through the heat exchanger. However, in our situation
where the evaporator of the refrigeration system serves as one side of the heat
exchanger, a change in phase does indeed occur. The refrigerant enters as a
mixture of liquid and vapor and leaves as a vapor. If the pressure drop of the
refrigerant as it flows through the heat exchanger is negligible, and if the
refrigerant leaves as a saturated vapor, then the temperature of the refrigerant
inside the heat exchanger remains constant. It turns out that this is a special
case of the counterflow heat exchanger that we have just considered.
Figure 11-13b shows the heat exchanger operating as an evaporator, where
the temperature of the evaporating refrigerant is identified as Te. Applying
energy conservation, where the energy absorbed by the evaporating refrigerant
equals the energy lost by the secondary fluid, gives:

·  Te – Ti  –  Te – To 
Q = UA ------------------------------------------------- = m· c p  T o – T i  (11-13)
Te – Ti
ln  -----------------
 T e – T o

Rearranging yields

Te – Ti 
UA = ln  ----------------
---------
· -
m cp  T e – T o

and further rearrangement gives


·
T o = T i +  T e – T i   1 – e – UA   m cp  (11-14)

From Equation 11-14, the secondary fluid outlet temperature is computed


from the inlet and evaporating temperatures, fluid flow rate, specific heat of the
selected fluid, and physical characteristics of the heat exchanger through UA.

Example 11-6
Problem A refrigeration system using ammonia as the refrigerant is used to cool a 60%
(by volume) solution of ethylene glycol and water from –12.22°C to –20.56°C.
The pressure drop of the ammonia as it passes through the evaporator is negli-
gible. The ammonia enters the evaporator as a saturated liquid/vapor mixture
and exits as a saturated vapor at a temperature of –28.89°C. A flow rate of
63.5 kg/h or 3810 kg/min of the ethylene glycol solution is needed to meet the
refrigerating load. Determine the ammonia flow rate needed to meet the refrig-
erating load and the UA product for this evaporator.
308 Chapter 11 Product Cooling, Freezing Loads, and Cooling Secondary Fluids

Solution The ammonia heat transfer rate is


·
Q am = m· am c p am  T am out – T am in  = m· am  h am out – h am in 

The brine heat transfer rate is


·
Q br = m· br c p br  T br in – T br out 

Equating the heat transfer rates gives


· ·
Q am = Q br
m· am  h am out – h am in  = m· c br p br  T br in – T br out 

and so
m· br c p br  T br in – T br out 
m· am = -------------------------------------------------------------
-
 h am out – h am in 

By using Figure 11-11, evaluate cp,br .


c p br  @–12.22°C  + c p br  @–20.56°C 
c p br = ------------------------------------------------------------------------------------------------
-
2
 2.93 + 2.89  kJ/kg·°C
 ------------------------------------------------------- = 2.91 kJ/kg·°C
2

Using the thermodynamic saturation table, evaluate the enthalpy of ammonia.


h am in  sat. liq. –28.89°C  = 49.25 kJ/kg
h am out  sat. vap. –28.89°C  = 1406.56 kJ/kg
1.06 kg/s   2.91 kJ/kg·°C   – 12.22°C –  – 20.56°C  - = 0.0189 kg/s
m· am = ---------------------------------------------------------------------------------------------------------------------------------
 1406.56 – 49.25 kJ/kg

The UA product is now determined using

m· cp  T o – T i 
UA = -------------------------------------------------
 Te – Ti  –  Te – To 
-------------------------------------------------
 Te – Ti 
ln  ----------------------
  T e – T o 

kJ
 1.06 kg/s   2.91 -------------  – 20.56 –  – 12.22  
 kg·°C
= ---------------------------------------------------------------------------------------------------------------- = 2140 W/°C
– 28.89 –  – 12.22  –  – 28.89 –  – 20.56   
----------------------------------------------------------------------------------------------------------
– 28.89 –  – 12.22 
ln  ---------------------------------------------
– 28.89 –  – 20.56 
Fundamentals of Refrigeration SI, Second Edition 309

The Next Step


Chapter 12 presents four applications of refrigeration systems, including super-
markets that use refrigeration racks and display cases; refrigerated warehouses
that use large, two-stage compression systems; meat processing involving
chilling and storage; and lyophilization that produces vaccines and antibiotics.

Summary
The largest application in the industrial refrigeration field is the cooling of food
products. Refrigerated food has significantly longer storage times and, when
frozen, storage may extend to several months. Food product thermal character-
istics, such as specific heat above the freezing temperature, specific heat below
the freezing temperature, the highest freezing temperature, recommended stor-
age temperature, latent heat of freezing, water content, heat generation rate due
to respiration of fruits and vegetables, and an approximate storage life, have
been collected and tabulated by ASHRAE. This information, along with the
amount of food product associated with the particular process in question, per-
mits quantitative estimation of the heat that must be removed to attain a lower
temperature.
The cooling to a temperature above freezing is approximated by a straight
line when the logarithm of a dimensionless temperature ratio is plotted against
time. This feature allows for the convenient determination of the cooling time,
because only a few experimental points are needed to define the straight line. If
the cooling continues to a temperature lower than the freezing temperature, the
product temperature tends to remain constant during the freezing process and
only drops again when the product is completely frozen.
Planck’s equation (shown in this chapter as applied to slabs, cylinders, and
spheres) may be used to estimate the freezing time of simple geometrically
shaped food products. For complicated objects, the approximations made in the
use of Planck’s equations yield greater uncertainty in the estimated freezing time.
The combination of the total heat removed and the time period over which
the cooling occurs establishes the refrigerating load. Methods used to cool and
freeze food products include: air-blast systems, contact freezing, and immer-
sion freezing. Air-blast freezing directs a cold airstream at the product, where
the air temperature is maintained by the evaporator of a refrigeration system.
Contact freezing uses a plate freezer, where the refrigerant and product are on
opposite sides of a metal plate. Immersion freezing involves the dunking or
spraying of the product with a propylene glycol/water solution or a sodium
chloride/water solution, where the temperature of the solution is regulated by
an evaporator. For liquid foods, a scraped-surface heat exchanger may be used
to partially freeze the product into an ice crystal slurry.
The refrigerating loads associated with the cooling of secondary fluids and
the cooling of enclosed spaces have different characteristics. Consideration of
these loads extends the product cooling and freezing loads considered in the
310 Chapter 11 Product Cooling, Freezing Loads, and Cooling Secondary Fluids

previous chapter. A secondary fluid is any fluid (different from the refrigerant)
that is cooled and pumped to the location where the cooling is needed.
Water is generally used as the secondary fluid in commercial buildings
when multiple cooling coils are distributed throughout the building. Other flu-
ids are needed when the operating temperatures are below the freezing tem-
perature of water. Example secondary fluids for low-temperature applications
include aqueous brine solutions of calcium chloride and sodium chloride and
aqueous solutions of ethylene glycol and propylene glycol.
A heat exchanger, also serving as an evaporator, couples the refrigeration
system to the secondary fluid flow loop. Performing an energy balance on the
heat exchanger yields the equations that describe their performance. After
studying Chapter 11, you should now be able to

 explain how to construct a Gurney-Lurie chart for the cooling of a food


product;
 use food product thermal properties and characteristics to estimate the quan-
tity of heat that must be removed to obtain a specific temperature;
 explain the origin of the respiration heat load for some fruits and vegetables;
 explain why the highest freezing temperature of food products is lower than
0°C;
 describe the freezing process in a food product;
 define all the variables in Planck’s equation for freezing and explain how
estimates may be obtained for all the variables;
 estimate both the quantity of heat that must be removed from a food product
when undergoing freezing and the time needed to accomplish the freezing;
 explain the operations of the air-blast, plate, and immersion freezers;
 explain the operation of a scraped-surface heat exchanger in conjunction
with liquid food products; and
 draw a diagram of a vapor-compression refrigeration system cooling a sec-
ondary fluid and explain how the cooling is accomplished with the heat
exchanger.

References and Bibliography


ASHRAE. 1997. ASHRAE Handbook—Fundamentals, Chapter 19. Atlanta:
ASHRAE.
ASHRAE. 2006. ASHRAE Handbook—Refrigeration. Atlanta: ASHRAE.
ASHRAE. 2009. ASHRAE Handbook—Fundamentals. Atlanta: ASHRAE.
Singh, R., and D. Heldman. 1993. Introduction to Food Engineering. San
Diego, CA: Academic Press, Inc.
Fundamentals of Refrigeration SI, Second Edition 311

Skill Development Exercises for Chapter 11


Complete these questions by writing your answers on the worksheets at the back of this book.

11-1 Whole lean pork shoulders weighing 454 kg are brought into a stationary
freezing tunnel that has an initial temperature of 4.44°C. Over a 12-hour
period, the pork is cooled and frozen to a temperature of –23.33°C. Determine
the refrigerating capacity that must be available from the refrigeration system
supplying the stationary tunnel.

11-2 The rate of temperature drop for any frozen product is always much faster than
the temperature drop rate when it was unfrozen. Explain the reason why, ignor-
ing the zero rate during the transition from unfrozen to frozen.

11-3 Golden Delicious apples are harvested over a 20-day harvest time. The apples
are placed in boxes, with each box containing 26.8 kg of apples. Each day,
2000 boxes of apples that have an initial temperature of 23.89°C are brought
into the storage cooler. The temperature of the storage cooler is 1.67°C. The
average weight of each box when empty is 2 kg, and the box material has a
specific heat of 2.5 kJ/kg·°C. Additionally, assume that each day’s load is fully
cooled before the next day’s load is brought into the storage cooler. Estimate
the highest product load resulting from the apple harvest for the storage cooler.

11-4 Explain why Planck’s equation cannot be applied to cooling processes below
the freezing temperature of the product.

11-5 A refrigerated warehouse receives a shipment of unfrozen turkeys at a tempera-


ture of 6.67°C. They are to be stored in a cooler room maintained at 0°C. The air
velocity in the cooler is 0.25 m/s. The turkeys are stored in boxes with 4 to 5 tur-
keys in each box. Assume that each box may be treated as a sphere 0.91 m in
diameter and that the box material may be neglected. Using Planck’s equation,
estimate the time interval for the turkeys to obtain a 0°C temperature. If the
shipment consisted of 27215 kg of turkeys, what would be the increase in refrig-
erating capacity needed during cool down of the turkeys?

11-6 The heat exchanger shown in Figure 11-14 serves as an evaporator for a vapor-
compression system. The refrigerant temperature in the evaporator is constant
at –9.4°C. A secondary fluid, a 50% by volume solution of ethylene glycol and
water, flows at 27.21 kg/min, where the heat transfer rate is 8 kW. For this heat
exchanger, UA is known to be 1.58 kW/K. Calculate the inlet and outlet tem-
peratures for the ethylene glycol solution.
312 Chapter 11 Product Cooling, Freezing Loads, and Cooling Secondary Fluids

Figure 11-14 Schematic diagram of the heat exchanger in Skill Development Exercise 11-6.
Practical Guide to
Refrigeration Systems

Study Objectives
Chapter 12 presents four refrigeration applications, excluding air conditioning, for
human comfort. The first three applications relate to the food industry, which
highlights the importance of refrigeration in this field. The last system described
concerns the pharmaceutical industry, where refrigeration is applied to achieve
relatively low temperatures. While the refrigeration systems vary in size and oper-
ating conditions, they all utilize the vapor-compression system and thus have the
four basic components that make up this cycle: compressor, condenser, expansion
device, and evaporator. Because all operate with evaporator temperatures below
the freezing temperature of water, all are capable of defrosting the evaporator coil.
Additionally, features concerning the design and operation of the refrigeration
systems are given in order to show that many technical issues must be addressed
in order for a system to operate as intended. Obviously, the information presented
here is relevant only to the technology applied to these systems, and other sources
of technical information should be consulted for additional information. After
studying Chapter 12, you should be able to

 explain what a compressor rack is and why it is used;


 explain the operation of a display case in a food supermarket;
 describe how the evaporator coil in a display case containing frozen food in
a supermarket undergoes defrost;
 describe how the refrigeration system in a refrigerated warehouse accom-
plishes compression of the refrigerant;
 explain why the refrigerant is typically piped to several locations in a refrig-
erated warehouse;
 describe how evaporator coils are defrosted in a refrigerated warehouse;
 give two example applications that a refrigeration system may have in a
food processing plant;
 describe one method of defrost for the evaporator coils in a food processing
plant; and
 describe a refrigeration system that is applied to a pharmaceutical applica-
tion for which a low temperature is required.
314 Chapter 12 Practical Guide to Refrigeration Systems

Instructions
Read the material in Chapter 12. At the end of the chapter, complete the skill
development exercises without referring to the text. Review again those sec-
tions of the chapter as needed to complete the exercises.

Introduction

Refrigeration systems are applied when heat removal or cold conditions are
required. This chapter presents four applications for vapor-compression refrig-
eration systems: supermarkets, refrigerated warehouses, meat-processing
plants, and pharmaceutical plants. The previous chapters in this text contain the
information needed to understand the design and operation of these refrigera-
tion systems. However, each particular application presents unique circum-
stances that the designer must recognize when designing and installing the
refrigeration systems.
The first application involves commercial refrigeration in supermarkets,
where compressor racks and display cases are characteristic features. Com-
pressor racks consist of an open steel framework inside of which are located a
number of compressors operating in parallel to meet the refrigerating capacity.
Distributed throughout the supermarket are refrigerated display cases contain-
ing fresh and frozen food configured in several ways. Depending on the tem-
perature requirements, both single-stage and two-stage vapor-compression
refrigeration systems are used in modern supermarkets.
The second and third applications presented here are refrigerated ware-
houses and a food processing plant represented by a beef processing plant.
These are similar in that both are industrial-sized systems serving the food
industry. Ammonia is the predominant refrigerant in these systems, and when
low temperature requirements must be met, two-stage vapor compression is
used. Also, the evaporators are generally flooded, or fed with recirculated liq-
uid. However, because ammonia is generally the refrigerant, direct-expansion-
fed coils are only suitable for temperatures above freezing. Vessels are needed
in these refrigerated systems to provide a surge volume and refrigerant inven-
tory storage when the system shuts down.
The last application presented is the freeze-drying process, also known as
lyophilization, as applied in the production of medical vaccines and antibiotics.
This application is rather specialized, where the cold temperatures supplied by
refrigeration in conjunction with the low pressures developed by the vacuum
pumps sublime the frozen water in the medical solutions leaving the vaccine or
antibiotic in a dry state. In this condition, the medical product can be stored at
room temperatures for considerable lengths of time.
Fundamentals of Refrigeration SI, Second Edition 315

Refrigeration Racks and Display Cases


The primary application of refrigeration compressor racks and display cases is
in retail food stores—supermarkets and convenience stores. The focus of our
concern is the refrigeration system that maintains the food at an appropriate
temperature for storage. Typically, the refrigeration equipment in a supermar-
ket consists of display cases, storage refrigerators, and processing refrigerators.
In this equipment, the food may be displayed for immediate sale; undergo final
processing, as with beef and pork meat; or be stored in specialized refrigerated
rooms for later sale. There are two temperature ranges that characterize refrig-
eration in supermarkets: medium temperature and low temperature. The evapo-
rator temperature range for medium-temperature equipment is from –18°C to
5°C, where the food product temperature may be above or below the freezing
temperature of water. The low-temperature range varies from –40°C to –18°C,
where the product temperatures are all below freezing. Display cases are not
designed to cool the food product but rather to maintain its storage tempera-
ture.
While supermarkets and convenience stores utilize refrigeration for air con-
ditioning as well, air conditioning for human comfort is not considered here.
However, the ambient conditions inside the supermarket do influence the per-
formance of display cases. Important variables include temperature, humidity,
and air movement adjacent to the display case. For open display cases, for
example, an increase in the ambient relative humidity from 35% to 50% dou-
bles the water condensation rate on the evaporator coil (ASHRAE 2006).

Display Cases
Display cases are cabinets in which refrigerated food is stored in a manner
accessible by the customer. Several basic configurations of display cases may
be used in a supermarket. It is important that the display case has the appropri-
ate features so that it accommodates the special characteristics of the food that
is stored. A display case may be open or closed. Food may be arranged in the
open type in a vertical or horizontal display where the customer simply reaches
into the cabinet and obtains the desired food item. The closed type employs
glass doors to shield the food from the space outside the cabinet. Consequently,
the customer must open the door to obtain the desired food item. Both display
case configurations are popular in supermarkets. Of all open display cases in
supermarkets, about two-thirds are medium-temperature units and one-third
are low-temperature units.
Figure 12-1 shows a vertical open display case with convenient access to
the food products inside the cabinet. To reduce the cooling load, air is
directed down over the front and captured along the lower edge. This refrig-
erated-air jet tends to separate the supermarket ambient air from the air flow-
ing over the food product inside the display case. Consequently, the heat
transfer between the outside air and inside air of the display case is less than
316 Chapter 12 Practical Guide to Refrigeration Systems

Figure 12-1 Example of a vertical open display case.

it would be otherwise. Figure 12-2 shows a schematic diagram of a vertical


open display case with four horizontal shelves, where arrows indicate the
downward movement of air at the front of the unit.
Figure 12-3 shows a horizontal open (i.e., chest-type) display case where
product is accessible by reaching down from the top. Due to the higher density
of cold air, an advantage of the horizontal configuration is that the air inside
tends to remain in the display case even though it is open.
A closed display case is shown in Figure 12-4. The glass doors provide a
barrier to ambient air infiltration when closed. Thus, infiltration only occurs
when the doors are open, which is a small fraction of the time. These cases
may be placed next to each other with no space between them. They provide
good food product variety in a relatively small floor space. Figure 12-5 is a dia-
gram of a multiple-shelf closed vertical display case.

Cooling Load

The cooling load for a display case includes interactions with the product,
internal materials of the display case, and heat transfer from the surroundings.
Recalling that heat transfer is driven by a temperature difference, a display case
gains heat in a number of different ways:

• Conduction through the display case walls to the interior. For closed dis-
play cases, heat is also transmitted through the glass doors.
Fundamentals of Refrigeration SI, Second Edition 317

Figure 12-2 Diagram of a multiple-shelf open vertical display case showing downward air
movement at front.

Figure 12-3 Horizontal open (chest-type) display case.


318 Chapter 12 Practical Guide to Refrigeration Systems

Figure 12-4 Closed food display case.

Figure 12-5 Diagram of a multiple-shelf closed vertical display case.


Fundamentals of Refrigeration SI, Second Edition 319

• Infiltration (heat transfer by convection) by ambient air entering the interior


through the air curtain. With closed display cases, the infiltration load is
significantly reduced by the glass doors.
• Convection through the addition of evaporator fan-motor heat, light-fixture
heat, defrost heat, an anti-sweat heater, and product pull down, all of which
occur inside the refrigerated space.
• Radiation as a result of the temperature difference between the inside sur-
face temperatures of the display case and the surface temperatures of the
surrounding walls and objects.

The heat gains for display cases may be divided into sensible and latent
loads. The sensible portion of the total load consists of conduction, radiation,
and the convection gains indicated. The infiltration is also made up of two parts:
sensible and latent. The sensible part is that which is due to temperature differ-
ence between the refrigerated air in the display case and the ambient air making
its way into the display case. The latent portion consists of a fraction of the total
infiltration load and the food product’s latent heat of respiration. Consequently,
the latent load is dependent on the heat content of the moisture added to the
refrigerated air inside the display case. Anti-sweat heaters are electrical resis-
tance heaters generally applied to low-temperature open and closed vertical
cases. ASHRAE (2006) reports that infiltration is the dominant load component
of open vertical medium temperature display cases and contributes about 80%
of the total load. In contrast, the horizontal open (chest-type) low-temperature
display case for frozen food has radiation as its largest contribution to the total
load, about 40% of the total.

Supermarket Refrigeration Systems

Refrigeration systems in a supermarket may be one of several basic types:

1. A single compressor piped to an evaporator using liquid and suction lines


across the supermarket (sometimes referred to as a conventional system
piped to a single condenser). Variations include several single compressors
connected to a larger condenser, or a single compressor connected to sev-
eral evaporators, each with independent liquid and suction lines and con-
trol. These systems are called a multiple evaporator systems and are
common in supermarkets.
2. The refrigeration rack, also referred to as a multiplex or parallel compres-
sor rack, common in supermarkets. This system consists of two or more
compressors piped in parallel with common suction and discharge lines.
The compressors are connected to one or more large condensers and gener-
ally use the same lubricating oil management system. The compressor rack
serves several evaporators, where each evaporator is piped with indepen-
dent liquid and suction lines spanning the distance between the display
320 Chapter 12 Practical Guide to Refrigeration Systems

case and the machinery room. This arrangement permits individual control
of each evaporator and display case. The condensers are usually located
outdoors (i.e., on the supermarket’s roof) to provide lower condensing tem-
peratures.

3. Loop refrigeration systems, similar to the multiplex rack in that the com-
pressors operate in parallel using shared suction and discharge headers.
However, the evaporators are not piped with long liquid and suction lines
transporting refrigerant; rather, a common liquid supply line and common
suction line are routed throughout the supermarket. The individual display
case liquid and suction lines are connected to the two loops as appropriate—
normally near the location of the display case.

4. Secondary loop systems similar in configuration to the loop system, but in


which a secondary coolant, rather than the refrigerant, is routed through
the supermarket. The secondary coolant is chilled in the machinery room
by the primary refrigeration system. A centrifugal pump is typically used to
circulate the secondary coolant throughout the supermarket and is selected
to deliver sufficient flow for the system’s pressure drop to meet the display
case cooling loads. Paramount to the success of a secondary loop system is
the selection of the secondary coolant, which generally must have low vis-
cosity to minimize pumping costs and have, at the same time, good heat
transfer characteristics. A heat exchanger in the machinery room, usually a
plate-and-frame type, cools the secondary coolant upon return from the dis-
play cases. In supermarket applications, the secondary coolant remains as a
liquid at all temperatures, but two secondary coolants that would involve a
phase change (liquid to and from vapor) are being considered—carbon
dioxide and water-based ice slurries. With secondary coolants involving a
phase change at the heat exchanger (evaporator) in the display case, the
evaporator itself must be designed to accommodate the volume of vapor
generated, and the suction line must be carefully sized as well to handle the
vapor. Secondary loop systems are low-charge refrigerant systems, because
the primary refrigeration system is confined to the machinery room. Also,
leakage of refrigerant from the system is greatly reduced because the
refrigeration rack is fabricated and assembled at the factory, where the pip-
ing joints are higher quality and thus more reliable. A conventional or mul-
tiplex supermarket refrigeration system typically leaks up to 35% of its
refrigerant charge per year. The potential reduction of refrigerant leakage
using a secondary loop system is considerable—up to 80% or 90% less
than that of a multiplex system. On the other hand, the secondary loop sys-
tem has several disadvantages. These include the inherent thermodynamic
loss associated with the addition of the chiller’s heat exchanger, the energy
needed to circulate the secondary coolant throughout the supermarket, and
additional thermal insulation required for both supply and return secondary
coolant lines. Despite these disadvantages, secondary loop supermarket
refrigeration systems have been built in recent years that show significant
energy savings over multiplex compressor rack systems.
Fundamentals of Refrigeration SI, Second Edition 321

5. Distributed refrigeration systems, are also low-refrigerant-charge systems


resulting from the compressors being located in the individual display
cases. Thus, the liquid and suction lines are short. With distributed sys-
tems, supermarkets generally use a closed-loop, water-cooled system to
supply the condensers in the display cases with cool water. This concept
also captures the display case condenser heat for transport to the outdoors
and prevents it from entering the supermarket’s interior and adding to its
air-conditioning load. Distributed systems offer an important advantage
regarding energy conservation: the shorter suction lines result in lower
pressure drop and less heat transfer between the evaporator and compres-
sor, thus giving a lower saturated suction temperature.

When considering the design of a supermarket refrigeration system, the


cooling load requirements of the display cases must match the capacity of the
selected refrigeration system. To achieve this, the manufacturer’s ratings of the
display cases and of the refrigeration system equipment must be consulted. To
aid in this effort, ANSI/ASHRAE Standard 72. Method of Testing Open Refrig-
erators (ASHRAE 2005) provides standard test methods by which open and
closed supermarket display cases are evaluated for refrigeration load require-
ments at specified ambient air conditions (e.g., 75°F and 55% RH).

Parallel Compressor Rack Refrigeration System

The parallel compressor rack system is prevalent in supermarkets and therefore


the focus for our deeper consideration of supermarket refrigeration. The term
rack refers to the frame structure in which the compressors and associated
equipment are located. Figure 12-6 shows a compressor rack representative of
those installed in supermarkets. In a supermarket, the display case cooling load
varies over a large range, making multiple parallel compressors a good choice.
The compressor rack system may use either an air-cooled condenser or a
water-cooled condenser and may be located in the compressor rack or out-
doors.
A simplified schematic diagram of a parallel compressor rack system is
shown in Figure 12-7, in which three compressors are utilized in conjunction
with only one evaporator. While this setup is not typical in supermarkets, the
use of one condenser and one receiver for high-pressure liquid is. Note that
the basic four components of the vapor-compression system—compressor,
condenser, expansion device, and evaporator—are present in this system.
The additional component in Figure 12-7, the receiver, stores liquid that is
not being circulated by the system. Receivers provide the ability to accommo-
date system operating condition adjustments; provide good condenser draining;
and permit the storage of liquid refrigerant, thereby improving system control
and operation. When the system cooling load is high, the refrigerant quantity
circulating in the system is low, which causes the receiver to store an excess of
refrigerant. When the cooling load is low, the circulating quantity of refrigerant
322 Chapter 12 Practical Guide to Refrigeration Systems

Figure 12-6 Compressor rack used in a supermarket.

Figure 12-7 Supermarket refrigeration system with three compressors in parallel.

is high, and the excess refrigerant stored in the receiver is low. Consequently,
the designer of the refrigeration system must carefully consider the volume
needed for the receiver so that it functions as intended in the system. Another
function of the receiver is to store the complete refrigerant charge in the event of
a system pump down. If the receiver does not have sufficient volume to accept
Fundamentals of Refrigeration SI, Second Edition 323

the refrigerant charge, an auxiliary vessel must be used to obtain the needed
capacity.
Not shown in Figure 12-7 is a suction pressure control switch to turn ON
and OFF the compressors as needed to match the display case cooling load.
These controls can also rotate the lead compressor so that each of the compres-
sors has approximately equal running time.
The piping in a supermarket refrigeration system is very important and
involves many details and design rules that must be followed. Several but not
all of these details are presented here to provide familiarity. The suction piping
should be brought to a position above the compressors using a common suction
header and should be arranged in a manner such that the quantity of oil return
to the compressors is equal to each. To achieve good oil return, symmetry in
the piping arrangement is a key feature. Also, appropriate sizing of the suction
line is needed to achieve a vapor velocity adequate for oil return to the com-
pressors. Because oil separates from the refrigerant vapor, it moves along the
pipe’s internal surface and accumulates in low regions. If this occurs, the oil
flow is disrupted, and the possible accumulation of liquid refrigerant could
generate slugs of liquid traveling toward the compressor that cause slugging.
Thus the horizontal suction line must not have any low regions and must be
pitched down toward the compressors to drain the oil. Another issue with suc-
tion line sizing is that the pressure drop in both the suction risers and horizontal
lengths should be as low as is practical for efficient system operation. Some-
times, designers generously size horizontal suction lines to reduce the pressure
drop and overcome a fraction of the pressure drop penalty of the suction riser.
When several display cases are operating at different medium temperatures,
they may still be connected to a parallel compressor rack. The configuration
needed to accommodate the multiple evaporators is shown in Figure 12-8,
where three evaporators are shown, each operating at a different temperature.
Evaporator 1 operates at 10°C, Evaporator 2 at 1.69°C, and Evaporator 3 at –
6.67°C. Evaporator pressure regulators (EPRs) are located downstream in the
suction lines of the two higher-temperature evaporators and are adjusted to
maintain the desired temperatures there. An EPR is an automatic pressure-regu-
lating valve that maintains a preset upstream pressure and, thus, temperature in
the evaporator located upstream. An EPR regulates the upstream pressure by the
force of a spring in compression holding a diaphragm against a port. When the
upstream pressure increases and reaches the setpoint, the diaphragm rises and
allows vapor to flow through the valve, thus maintaining the setpoint pressure.
To pass a greater vapor flow, the diaphragm must rise further, but this also
increases the pressure somewhat above the setpoint. Pilot-operated valves are
also used as EPRs for larger sizes, where the pressure upstream controls the
position of the power piston, which is connected to the throttling plug. A check
valve is located in the suction line downstream from the coldest evaporator,
which prevents flow of refrigerant from the higher-temperature evaporators to
the low-temperature evaporator when the compressor rack is supplying refriger-
ant to the higher-temperature evaporators. However, when all three evaporators
324 Chapter 12 Practical Guide to Refrigeration Systems

Figure 12-8 Supermarket refrigeration system with three evaporators using evaporator
pressure regulators to maintain design temperature and three compressors in
parallel.

require refrigerant, such as during start-up, the compressor operates at the suc-
tion pressure needed to cool the highest-temperature evaporator first, then at the
suction pressure needed for the middle-temperature evaporator, and last at the
suction pressure needed for the lowest-temperature evaporator. To accomplish
this, the compressor must have sufficient capacity to handle the total cooling
load at the suction pressure of the lowest-temperature evaporator. Also, the low-
est-temperature evaporator only receives refrigerant when the higher-tempera-
ture coils are satisfied. To avoid starving the lowest temperature evaporator, the
lowest-temperature evaporator generally has 60% or more of the total cooling
load.
To achieve low temperature conditions (–18°C to –40°C) in supermarket
display cases, one of two approaches is usually taken: two-stage compression or
cascade compression systems. In commercial refrigeration, two-stage compres-
sion refrigeration is sometimes referred to as a compound system. As previously
discussed in Chapter 3, these systems connect compressors in series in order to
obtain a lower suction pressure. In supermarket applications, compound sys-
tems are only applied when evaporator temperatures are less than –18°C. Cas-
Fundamentals of Refrigeration SI, Second Edition 325

cade systems can reach lower temperatures than compound systems by utilizing
the features of different refrigerants.
To maintain good performance of the display cases that have evaporators
operating at temperatures below freezing, defrost is required. The methods
used in supermarkets to defrost evaporators include hot-gas defrost, electric
defrost, compressor cycling, and air defrost.

Hot-Gas Defrost
Compressor discharge gas is circulated through the evaporator’s tubes, melting
the ice that has accumulated on the fins and tubes of the coil. During hot-gas
defrost, valves shut down the evaporator’s normal refrigeration and direct com-
pressor discharge gas through the coil. Because discharge gas flow is limited
by operation of the system, typically only one in four evaporators can undergo
defrost at any one time. The control of hot-gas defrost for an evaporator
requires thoughtful selection of valves and operation during a defrost. Some-
times, high-pressure saturated gas (vapor) taken from the top of the receiver is
used to carry out a hot-gas defrost.

Electric Defrost
Electric resistance heaters in the evaporator coil melt ice by conduction and
convection. The electric heaters may also be located in the air path upstream
from the coil, where the heated air passing through the coil melts the ice. Gen-
erally, electric defrost requires a longer time to complete the defrost but is less
complicated than a hot-gas defrost system.

Compressor Cycling
Compressor cycling involves turning off the compressor and allowing suffi-
cient time for the ice on the coil to melt as a result of air circulating through the
display case. Defrost by this method is slow and is only practical for display
cases operating at temperatures of 1°C and above.

Air Defrost
Ambient air from the supermarket is diverted to the coil, thereby melting the
ice. The temperature of the supermarket must be greater than a minimum value
in order to achieve an adequate defrost. Winter temperatures inside a supermar-
ket may prevent good defrost and, therefore, could be supplemented with elec-
tric resistance heaters.

Refrigerated Warehouses
Many food products, pharmaceuticals, and other items require refrigeration to
extend shelf life. Large quantities of these products are stored in specially
designed structures called refrigerated warehouses. These structures have
insulated walls and ceilings that incorporate water-vapor-retarding materials
326 Chapter 12 Practical Guide to Refrigeration Systems

to reduce the heat and moisture gains to the inside. The more effective the
insulation and water-vapor retarders are, the smaller the refrigeration system
needs to be in order to maintain the desired temperatures inside the ware-
house. Chapter 11 presents information showing the wide diversity of food
products that are stored in refrigerated warehouses. A large dock facilitates
the movement of product in and out and is an integral part of a refrigerated
warehouse. Refrigerated warehouses larger than about 1900 m2 in floor area
generally use a central refrigerating system as opposed to smaller rooftop
units distributed throughout the building. Figure 12-9 shows a photograph of
the dock area, freezer and cooler rooms, machinery room, and the administra-
tive office of a refrigerated warehouse.
Typically, a refrigerated warehouse consists of several rooms, some of
which may be coolers and others which are freezers. Coolers operate at inter-
mediate temperatures several degrees above freezing and extend the shelf life
of perishable food products. The temperature in a particular cooler depends on
the product being stored. Appendix C, Table C-1, gives a list of the storage
temperature requirements for a number of food products, where, for example,
tangerines are noted to have a recommended storage temperature of 4°C to
7°C. A storage temperature below 0°C is acceptable for some fruits and mel-
ons, as their liquid contains a solution of water and dissolved substances that
somewhat depress the freezing temperature. Coolers also store fresh meat and
fish at temperatures just above freezing.

Figure 12-9 Aerial photograph of a refrigerated warehouse showing the dock area, cooler
and freezer rooms, machinery room, and administration office.
Fundamentals of Refrigeration SI, Second Edition 327

Relative humidity is another important variable in the storage of perishable


food products. The recommended relative humidity for tangerines is between
85% and 90%. In fact, most food products require relative humidity in this
range for good product shelf life.
Freezers are those rooms in a refrigerated warehouse operating at tempera-
tures below freezing for the storage of frozen products. Many meat products,
when frozen, require storage temperatures of nearly –24°C. Additionally, fro-
zen fruits are stored at this temperature as well. Ice cream, on the other hand,
requires a –29°C storage temperature to minimize the growth of ice crystals.
Relative humidity requirements range from about 60% to 95% for most prod-
ucts stored in freezers. Recall from the psychrometric chart in Figure 4-3 that
while the relative humidity may be relatively high, the actual moisture content
in the freezer’s air is quite low.
For a refrigerated warehouse to meet its requirements for food product stor-
age, it must achieve the storage temperatures for all the products stored. This is
accomplished in a refrigerated warehouse by incorporating a number of cooler
and freezer rooms all operating at the needed temperatures. If the product
remains the same throughout the year, then those rooms serving as coolers and
freezers maintain their assignments over the year. Figure 12-10 shows a sche-

Figure 12-10 Schematic diagram showing the floor plan and product rack location of a typi-
cal refrigerated warehouse consisting of one cooler room and two freezer
rooms (doors and other details not shown).
328 Chapter 12 Practical Guide to Refrigeration Systems

matic diagram of a refrigerated warehouse floor plan consisting of one cooler


room operating at –2°C and two freezer rooms operating at –32°C. However,
some rooms in a refrigerated warehouse may serve as coolers or freezers during
the year, depending on the product storage requirements. These special rooms
are called convertible rooms, where the evaporators, refrigerant piping, and con-
trols must be able to accommodate both cooler and freezer operation.
Chapter 11 identifies the refrigerating load associated with the cooling and
freezing of food products. This portion of the total load may be significant if
large quantities of the product brought into the refrigerated warehouse are at a
temperature above that needed for storage. In a refrigerated warehouse, a number
of heat-gain sources, as follows, contribute to the total refrigeration load:

• Product load, due to temperature change and internal heat generation for
vegetables and fruits.
• Heat transmission through walls, floor, and roof, driven by the tempera-
ture difference outdoors and the cooler or freezer storage temperature.
Walls and roofs exposed to sunlight have higher heat gains than those that
are shaded.
• Infiltration through cooler, freezer, and dock doors—convection heat
transfer, due to the exchange of air, occurs by motion of the air whenever a
door is open.
• Internal load—all energy used in a cooler, freezer, or dock including
lights, evaporator fan motors, forklift trucks, and people is ultimately con-
verted to thermal energy in the space and appears as a heat gain and load on
the refrigeration system.
• Latent load—the refrigeration load due the moisture content of the air;
influenced by the amount of air infiltration that occurs.
• Defrost load—when evaporator coils defrost, heat is released to the cooler
or freezer space and must subsequently be removed by the evaporators.

If the refrigerated warehouse also provides freezing of the product using


special equipment, such as blast freezers, then the refrigeration system must be
sized in order to handle this additional load, which may be considerable.
The designer of refrigerated warehouses must carefully account for all these
loads in order to correctly assess the total load of the facility. This total load cal-
culation must also account for the timing of activities in the warehouse. For
example, if the blast freezers are operated only during the day and new product
is brought to the warehouse only in the morning, then these loads must be added
to the other loads that occur at that time. This is the design load, and it is this
load that establishes the capacity of the refrigeration system and its components.
Inherent in determining the design refrigeration load of a system is thorough
knowledge of how the refrigerated warehouse will be operated. If the facility’s
refrigeration load significantly increases by virtue of increasing entering prod-
uct quantity, rate, and/or temperature, the refrigeration system may not be capa-
Fundamentals of Refrigeration SI, Second Edition 329

ble of maintaining the needed storage temperature, resulting in a temperature


rise to an unacceptable level. The solution is to either control the incoming
product or enlarge the refrigeration system.

Compression System

A refrigerated warehouse consisting of coolers and freezers operating at freezer


temperatures at or below –29°C generally uses a two-stage vapor-compression
system. If the warehouse contained only coolers, then only one-stage compres-
sion would be needed. Figure 12-11 shows a diagram of a two-stage refrigera-
tion system serving one intermediate-temperature evaporator (cooler) and two
low-temperature evaporators (freezers). The low-temperature evaporators in the
freezers are fed with recirculated refrigerant, while the cooler evaporator is fed
by direct expansion. Not shown in the diagram are the multiple evaporator coils
associated with each of the cooler and freezer rooms, control valves, isolation
valves, and the other components and controls needed for safe operation and

Figure 12-11 Schematic diagram of a two-stage vapor-compression refrigeration system


applied to a refrigerated warehouse consisting of cooler and freezer rooms.
330 Chapter 12 Practical Guide to Refrigeration Systems

maintenance of the system. This diagram is very similar to the two-stage system
shown in Chapter 3 with the exception of multiple evaporators.
Multiple compressors are typically used for the booster and high-compression
stages in refrigerated warehouses. Reciprocating compressors have been popular
for many years, but today rotary screw compressors are preferred, especially in
large installations. Figure 12-12 shows the interior of a machinery room from a
refrigerated warehouse consisting of several rotary screw compressors and hori-
zontally oriented vessels behind the compressors. Recall that rotary screw com-
pressors are less efficient when operated at less than rated capacity. So, to achieve
flexible and efficient operation with screws, if three compressors are applied to the
given stage, two would be the same capacity and together able to meet the full sys-
tem capacity. The third compressor would be approximately one half of the larger
compressors and serve as a trim compressor operating, when needed, to provide
the most efficient combination of the three. Incidentally, reciprocating compres-
sors with cylinder unloading are nearly as efficient at reduced capacity as they are
at full capacity. A variation of the two-stage compression approach to refrigeration
is to utilize the economizer port on a rotary screw compressor. This is simply a
hole located somewhere between the suction and discharge ports on the side of the
screw compressor’s case inside of which the screw rotates. Refrigerant vapor at an
intermediate pressure may be introduced here and compressed to the discharge
pressure. Thus, liquid subcooling is possible using a shell-and-tube heat
exchanger, where a fraction of the liquid from the receiver is used to refrigerate

Photograph courtesy Ron Vallort and Associates, 2007.

Figure 12-12 A refrigerated warehouse machinery room showing low-stage and high-stage
rotary screw compressors and horizontally oriented vessels.
Fundamentals of Refrigeration SI, Second Edition 331

the remaining high-pressure liquid to the evaporators. The power savings result
from not having to compress the refrigerant vapor from suction to this intermedi-
ate pressure. Using the economizer port affords some of the benefit of two-stage
compression (compression of the flash gas) but with only the one compressor
stage. However, the full benefit of two-stage operation cannot be attained with an
economizer port alone because desuperheating does not occur. Another limitation
of the economizer port is that its position is fixed and, depending on the suction
and discharge pressures, it may not be possible for the compressor to maintain the
desired intermediate pressure.
Ammonia is the refrigerant of choice for new refrigerated warehouses. In
the past, refrigerant R-22 was sometimes used in refrigerated warehouses, but
the ozone depletion potential of R-22 was sufficient for its near-future curtail-
ment by the Montreal Protocol (UNEP 1987). As a consequence, R-22 is no
longer considered for newly constructed refrigerated warehouses. Ammonia
has good operating efficiency and relatively high refrigerating capacity due to
its high latent heat. For example, ammonia requires about a third of the flow
rate of R-22 to deliver the same refrigeration.

Evaporators and Piping Systems


In a refrigerated warehouse, evaporators are distributed throughout in the cool-
ers, freezers, and docks. Typically, the evaporators are located above the floor
either by a suitable structure near the ceiling or in an enclosure above the roof
called a penthouse. In a penthouse arrangement, the evaporators are placed in
the penthouse where the airflow through the evaporators is directed down and
turned in the appropriate horizontal direction for good distribution and circula-
tion in the cooler or freezer space. Penthouses allow for improved access to the
evaporator coils, controls, and associated piping, which improves ease of main-
tenance. Evaporators suspended from the ceiling must also be oriented to pro-
vide good air distribution in the cooler or freezer. However, suspended
evaporators can only be serviced by people using floor lifts to reach the coils.
Three basic types of refrigerant feed are used with evaporators in refriger-
ated warehouses—direct expansion, flooded, and recirculated. These methods
were discussed in Chapter 3. For refrigerated warehouses that use a central
refrigeration system, where the compressors and vessels are located in a
machinery room and the condensers placed nearby outdoors, flooded and recir-
culation refrigerant feed methods are common. Regulating the flow of ammo-
nia by direct expansion is not practical for low-temperature coils, because the
degree of superheat needed in the coil provides a significant penalty in refriger-
ating capacity of the coil. However, for refrigerated docks and coolers, direct-
expansion coils are sometimes used, as the superheat and consequent tempera-
ture difference penalty are not as large.
Since evaporators are located throughout the refrigerated warehouse in
coolers, freezers, and docks, insulated steel piping of an appropriate diameter
to obtain an acceptable pressure drop transports the refrigerant between the
332 Chapter 12 Practical Guide to Refrigeration Systems

machinery room and the evaporators. Valves and controls regulate the flow of
the liquid refrigerant to the coil at a rate accommodating the actual refrigerat-
ing load. The refrigerant’s pressure drop in the suction line, given in terms of
the corresponding saturation temperature drop, is usually from 0.5°C to 1°C
per 30 m of equivalent pipe length. These suction lines returning from the
evaporators are vapor lines for flooded and direct-expansion evaporators. How-
ever, for recirculated coils, the suction lines return both vapor and liquid and so
must have their diameters selected for both the vapor and liquid flows. Again,
the 0.5°C to 1°C per 30 m of equivalent length of pipe is typical for suction
lines. Additionally, it is important to pitch down to the machinery room wet
suction lines in order for the liquid to drain properly and to empty in the event
of a power failure. Additionally, the vessels in the machinery room must have
sufficient volume to accommodate the liquid refrigerant returning from the wet
suction lines. Suction lines are insulated so that the heat gain and temperature
rise of the refrigerant from the evaporator to the machinery room is minimized.
The methods available for defrosting coils in a refrigerated warehouse are
air, electric, water, and hot gas. Most common in refrigerated warehouses are
the hot-gas and water methods. The coil must be relatively free of frost and/or
ice for good performance. A coil plugged with frost has reduced airflow, which
results in decreased refrigerating capacity.

Air Defrost
For air defrost to be effective, the air temperature in the room or dock must be
about 2°C so that enough heat transfer occurs from the air to the coil to melt
the ice. When there is no forced airflow through the coil, the defrost time is
unacceptably long. Defrost times are decreased when the evaporator’s fan is
operated, but even then the defrost time may be very long. Air defrost may be
applied in the dock area and in coolers, which have suitable operating tempera-
tures.

Electric Defrost
Electric resistance heaters placed inside an evaporator coil can accomplish
defrost in a reasonable length of time. The resistance heaters must be carefully
distributed and be in close contact with the coil in order for the heat to quickly
melt the frost. While the first cost of electric defrost is low compared to that of
water and hot-gas defrosts, its operating cost is the highest.

Water Defrost
The use of water sprayed onto a coil is an effective method to remove frost.
The temperature of the water is about 15°C, which is a compromise between
the speed at which the defrost occurs and the fogging near the coil that invari-
ably takes place with water defrost. While the advantages of water defrost
include a convenient source to accomplish the defrost, it is not generally used
Fundamentals of Refrigeration SI, Second Edition 333

in refrigerated warehouses because the temperature inside the freezer or cooler


increases too much.

Hot-Gas Defrost
Hot-gas defrost is by far the most popular method for defrost in a refrigerated
warehouse. The source of the hot gas is the discharge gas from the compres-
sors, so it is a by-product of the refrigeration system. Thus, a hot-gas line must
be supplied to each coil in the refrigerated warehouse. The basic steps to
accomplish a defrost using hot gas are to shut off the liquid supply to the coil,
allow the liquid to evaporate, close the outlet of the coil, and supply the coil
with high-pressure hot gas. The pressure inside the coil must be high enough so
that the saturation temperature is great enough to melt the frost on the outer
surface of the coil. The suction line conveys the hot gas discharged from the
coil during the defrost. The concept of hot-gas defrost is the same for flooded,
recirculated, and direct-expansion coils, but the control for each is somewhat
different. An important point to note for hot-gas defrost is that the sequence
and speed at which the valves control the hot gas flow should be sufficiently
slow so that a slug of liquid is not introduced into the suction line and propelled
toward the machinery room, causing a loud “bang” and possible failure of the
piping. The industry term to describe this necessary slow control sequence is
soft defrost.

System Operation
The refrigeration load depends on the season and movement of product in and/or
out of the refrigerated warehouse. Compressors operate in parallel in each com-
pression stage to meet the load. For those times when the load is quite small, a
swing compressor may operate as a single-stage compressor. The operation of
the system in this manner can reduce the energy consumption rate compared to
that of operating in a two-stage compression configuration with rotary screw
compressors unloaded to very low capacity levels.
Energy conservation involves reducing the consumption of energy. Gener-
ally, to achieve efficient system operation by reducing compressor power con-
sumption, the condenser temperature should be as low and the evaporator
temperature as high as practical. However, there are restrictions on these two
operating variables: the high temperature must be sufficient for the operation of
control valves and provide for an adequate hot-gas defrost if hot-gas defrost is
used. Obviously, the low temperature must be adequate for all evaporators to
operate and meet their refrigerating loads. Energy consumption may be reduced
by heat recovery from the compressor discharge gas when its heat is transferred
to a circulating fluid under the freezer floor to prevent frost heave. Operating
strategies important for energy conservation include many items. Some of these
are pertinent to a specific facility but may include: freezer door management,
cooler door management, dock door management, freezer and cooler room
lighting, timely evaporator defrost, and many more. In combination, these and
334 Chapter 12 Practical Guide to Refrigeration Systems

similar strategies can significantly decrease the energy consumption of a facil-


ity. Furthermore, good maintenance practices also reduce energy consumption.
An example of this is the appropriate chemical treatment of evaporative con-
denser water to eliminate scale buildup on the tubes, thereby maintaining nearly
original condenser performance.
Energy storage, unlike energy conservation or recovery, does not save
energy. Rather, it decreases the cost of energy. In a refrigerated warehouse, it is
possible to operate the refrigeration system at night when the cost of electrical
power is less. During the day, shut off the refrigeration system to allow the
product to warm 0.5°C or 1°C. This is acceptable for some food products, but it
is not acceptable for other products, such as ice cream and fish. If this approach
is feasible, then the refrigeration energy expense can be reduced by application
of this approach to energy storage.

Meat Processing
The processing of red meat—including beef, pork, and lamb—for consumers
involves the application of refrigeration from the time the animal is slaughtered
until the final product is purchased at the supermarket. In this section, we focus
on the beef processing industry while acknowledging that hogs, lambs, poultry,
fish, dairy, vegetables, and bakery products are processed in much the same
way using refrigeration. However, each food product has special characteristics
and, therefore, requires special methods to achieve the best quality for con-
sumption. The 2006 ASHRAE Handbook—Refrigeration (ASHRAE 2006)
provides a good overview of how refrigeration is applied to a number of prod-
uct types.
When an animal is slaughtered, the meat undergoes changes that involve
physical, chemical, and organoleptic characteristics. In other words, enzymatic
changes, bacteria growth, and meat aging are all initiated simultaneously. If the
meat were left alone, it would quickly spoil in a matter of hours. Refrigeration
of the meat after slaughter dramatically reduces these injurious changes in the
meat. The intent is to provide an adequate storage time for processing and stor-
ing the meat before selection and purchase by the consumer.
In order to apply refrigeration to beef processing in a useful way, the
designer of the refrigeration system must understand the temperature require-
ments for preserving meat. The animal body temperature is approximately
39°C before slaughter. Directly after slaughter, the meat remains supple,
shrinkable, and rich in glucids. For one to three hours after slaughter, the meat
remains in this state. After about three hours, the degradation of the glucids
increases acidity, which causes the muscles to become rigid. This condition is
called rigor mortis and exists from about 12 to 48 hours after slaughter. During
this time, the meat is not ready for human consumption. As more time passes,
certain enzymes cause additional biochemical changes in the muscles. The
time frame for these changes is approximately two days to two weeks from the
time of slaughter, after which the meat is said to be “aged” or “mature,” due to
Fundamentals of Refrigeration SI, Second Edition 335

muscle relaxation in the absence of spoilage or deterioration, and ready for


human consumption. This result is achieved by the appropriate application of
refrigeration.
To ensure good quality meat for sale in supermarkets, the refrigeration sys-
tem must lower the temperature of a carcass as rapidly as is practical after it is
dressed. Obviously, refrigeration can only influence the surface temperature of
the carcass. Heat transfer from inside the carcass is governed by the thermal
properties of the meat. The internal heat (energy) of the carcass must move
through the interior meat and out from the surface. For good results, a beef car-
cass should be cooled to a uniform temperature, usually from 1°C to 2°C,
within 72 hours—the maximum time allowed for the complete beef carcass to
be cooled. Beef refrigerated in this manner is considered to be fresh and is
advertised this way in supermarkets. Figure 12-13 is a photograph of several
sides of beef in a chill room.

Photograph courtesy Ron Vallort and Associates, 2007.

Figure 12-13 Sides of beef in a chill room.


336 Chapter 12 Practical Guide to Refrigeration Systems

Beef may also undergo freezing, obtaining temperatures from –18°C to


–50°C using blast freezers, spiral freezers, and plate freezers, among others.
Freezing halts the meat’s aging process and enzyme action. The shelf life of
frozen meat products is significantly increased from 6 to 18 months.

Refrigerant Compression

Beef processing and temperature requirements dictate refrigeration system


design decisions. Typical industrial meat-processing plants use ammonia as a
refrigerant because of the large cooling loads. When dressed, the beef carcass
is split into two sides of beef, which must be chilled to a uniform temperature
of approximately 1°C within 72 hours. Each side weighs from 135 to 450 kg,
and the average weight is approximately 225 kg. While the specific heat of the
meat ranges from 2.1 to 3.3 kJ/(kg·°C), the average is 3.1 kJ/(kg·°C)
(ASHRAE 2006). If a chill room has a capacity of 500 beef carcasses per day
and a meat chill rate of 28°C in 20 hours, then the total (sensible and latent)
product load is about 350 kW. A meat-processing plant may slaughter many
more cattle per day and, in such a situation, generally uses multiple chill
rooms. This permits the chilling process to commence after two or three hours
from the entrance of the first side of beef. The chill room remains in operation
after slaughter and through the night. After this period, the beef is moved to
holding rooms for two to three days, after which the chill room is washed and
sanitized in preparation for the next day of operation. The cooling load for an
equivalent amount of beef in a holding room is about one-third that of the chill
room. Sometimes the beef is moved to refrigerated transport vehicles, where
the final heat removal takes place as if in a holding room. Additionally, if the
beef is frozen at the plant, much lower refrigerant temperatures are needed
from the refrigeration system than those for the chill and holding rooms.
Beef processing plants that require only intermediate-temperature chill and
holding rooms are best served by a one-stage vapor-compression system. See
Chapter 3 for information concerning one-stage systems. For large systems,
multiple compressors operated in parallel provide the ability to follow varia-
tions in refrigerating load throughout the day. For beef processing plants oper-
ating at both low temperatures for freezing beef and at intermediate
temperatures for beef chill and holding rooms, a two-stage vapor compression
system is well suited. Figure 12-14 is a diagram of a two-stage compression
system similar to that given in Figure 3-22 in Chapter 3. Observe how the two-
stage system accommodates the multiple temperatures of the processing plant,
namely the chill and holding rooms and the freezing units. The intermediate
and low temperatures at which the refrigeration system operates are dictated by
the temperatures needed in processing the beef. The lowest temperature of the
intermediate-temperature evaporators sets that temperature, and the lowest
temperature of the low-temperature evaporator sets that temperature. As a
result, the ideal optimal intermediate temperature may not be possible for the
Fundamentals of Refrigeration SI, Second Edition 337

Figure 12-14 Diagram of a two-stage compression refrigeration system for a beef processing
plant consisting of chill room, holding room, and freezer-room evaporators.

system. Recall that the two-stage system’s performance is influenced by the


intermediate temperature (see Chapter 3).
Ammonia is by far the most common refrigerant used in beef processing
plants, as it has many desirable features. One important feature is the low liq-
uid flow rate needed to supply evaporators relative to the next most popular
refrigerant for industrial systems, which is R-22. A concern with selection of
refrigerants at the present time is the curtailment in the near future of HCFC
refrigerants, of which R-22 is a member. For this reason, new construction of
industrial-size refrigeration systems now exclusively utilizes ammonia as the
refrigerant or possibly in conjunction with carbon dioxide as a secondary cool-
ant or in a cascade system.

Evaporators and Piping


The evaporators in chill and holding rooms generally consist of finned-coil
(evaporator) units. These units are equipped with defrost equipment and with
integral mounted fans to circulate air through the room. The usual fin spacing
338 Chapter 12 Practical Guide to Refrigeration Systems

for coils in this application is 1 to 1.4 mm Occasionally, variable fin spacing is


used to reduce the harmful effects of ice formation on the coil. The finned-coil
units may be placed on the floor of the room, but more often they are placed
above the floor just below the ceiling. Figure 12-15 shows a finned-coil evapo-
rator mounted on the ceiling of a chill room.
Virtually all chill and holding rooms in beef processing plants use ammonia
as the refrigerant in the finned-coil evaporator units. There is an important rea-
son for this—ammonia has a very high heat transfer coefficient (about triple that
of R-22) that causes a given finned-coil unit to have a very high refrigerating
capacity. In order to fully utilize the complete coil heat transfer surface, systems
use flooded or recirculated (see Chapter 4) finned coils. Direct-expansion units
do not utilize the complete coil heat transfer surface because a fraction of the
refrigerant in the coil must be superheated. Furthermore, direct-expansion-fed
coils must pass the flash gas generated in the valve through the coil and cannot
provide the good coil liquid flooding that flooded and recirculated coils do. As a
result, a coil fed by direct expansion is normally 15% to 20% dry and, therefore,
derated in refrigerating capacity by this amount by manufacturers.
Multiple coils can operate at different temperatures above that of the suc-
tion line back to the compressor. To accomplish this, coil temperature can be

Photograph courtesy of Kansas State University, 2007.

Figure 12-15 Finned coil in a chill room.


Fundamentals of Refrigeration SI, Second Edition 339

regulated by evaporator pressure regulators (EPRs), located downstream from


the coil in the suction line, which adjust the pressure in the coil. These controls
consist of a throttling valve that drops the pressure to that of the suction line.
Obviously, an EPR valve causes inefficiency in the refrigeration system, the
amount of which is directly proportional to the pressure drop. Typically, the
EPR valve is actuated by coil pressure or temperature, whose function is to
maintain a constant coil temperature.
Operation of the finned-coil evaporator units in the chill room begins when
the carcasses are brought into the room. This is usually done with the carcasses
suspended above the floor on hooks attached to an overhead rail for easy trans-
port. The rails and hooks are spaced so that none of the beef sides touch one
another and so adequate air movement occurs around each beef side. It is gen-
eral practice to maintain the air temperature in the chill room between 0°C and
5°C throughout the process. During the initial time of cooldown, the evapora-
tive loss of moisture from the carcass is high due to the large temperature dif-
ference between the carcass and the air temperature. In the United States, it is
common for processors to spray chilled water (containing a small amount of
chlorine or acetic acid) onto the carcasses to accelerate the chilling rate. Air
chilling is still preferred in Europe, where improvements have also reduced the
time to fully chill. The moisture evaporation rate from warm carcasses is so
great that most chill rooms contain fog when first starting the cooldown.
Evaporator selection for a chill room or holding room can only be approxi-
mated by a designer. There are two reasons for this:

• The refrigeration varies throughout the chilling process, so the load can
only be approximated.
• Long-term field performance of the finned-coil evaporator units is difficult
to determine.

The beef-chilling rooms must have evaporator coils that are sufficient in
capacity to remove the moisture and control the relative humidity. Moisture
condensing in the chill room must be prevented to eliminate the dripping of
water onto the beef carcasses. To accomplish this, coils of relatively small sur-
face area operating with a relatively large temperature difference are needed.
Sometimes, cooling alone is not sufficient to remove the moisture generated,
and reheat may be required to achieve the desired chill-room conditions. For
finned-coil units, a typical temperature difference is 5°C for most applications
but is much higher for beef-chilling rooms. The influence that the evaporator
has on shrinkage is also important but equally difficult to determine, so a
designer’s experience is the best guide.
The evaporators in chilling and holding rooms operate with refrigerant
temperatures below freezing, so frost builds up on the coil surface. Methods of
defrost are discussed in the following sections.
340 Chapter 12 Practical Guide to Refrigeration Systems

Hot-Gas Defrost
Hot discharge gas from the compressors routed through the evaporator coils is
an effective means of defrost. By activation of appropriate valves, hot gas is
gently introduced into the evaporator coil, where the gas partially condenses.
The heat released by the hot gas in the coil melts the frost on the outside of the
coil, taking from as little as 5 minutes, up to 20 minutes. Usually, the evapora-
tor air fans are OFF, permitting as much of the melted ice as possible to fall to
the pan below. The water is then piped out of the room using heat-traced lines.
After the defrost is complete, the valves are sequenced again to first boil out to
suction all the liquid refrigerant that may be in the coil and then to continue
refrigeration by admitting liquid refrigerant. Hot-gas defrost is also commonly
applied to evaporators in refrigerated warehouses.

Coil-Spray Defrost
Water is sprayed directly onto the coil surface, which supplies sufficient heat to
melt the ice. The air fans are turned OFF, and both the refrigerant supply line
and the suction line are closed. Provision is made for pressure relief of the coil
into the suction line during defrost. The water temperature should be between
10°C and 25°C to prevent water from refreezing on the coil and ensure that the
drain pan and drain lines remain open to flow. The time to defrost is from about
5 to 15 min, depending on the thickness of the frost and water temperature,
among other variables. It is typical for fog to form around the coil undergoing
water defrost, which again is captured by the coil when its operation continues.

Electric Defrost
An array of electrical resistance heating elements placed throughout the coil
can defrost a coil. When defrost is needed, refrigerant flow is stopped by
appropriate valves, air fans are stopped, and electrical power is directed
through the heating elements. When the defrost is complete, operation of the
coil resumes. Provision is made for pressure relief of the coil into the suction
line during defrost.

Air Defrost
With air defrost, the refrigerant supply and suction lines are shut OFF, but the
air fans continue to operate. Defrost is accomplished by heat transfer from the
room air passing through the coil into the coil’s fins and tubes. The pressure in
the coil is allowed to increase during defrost, but pressure relief is provided
from the coil to suction. Some refrigeration continues by the coil under an air
defrost, but it is substantially less than that under normal operation. Air defrost
by room air requires double or triple the time of a hot-gas or coil-spray defrost.
Fundamentals of Refrigeration SI, Second Edition 341

Pharmaceutical Refrigeration
An application of refrigeration in the pharmaceutical industry involves the lyo-
philization of a vaccine or antibiotic for long-term storage. The prefix “lyo-”
comes from the Greek word lyein, which means “to loosen.” The lyophilization
process is achieved by freeze drying, or refrigerating the product to a low tem-
perature under high vacuum conditions. The essence of this process as applied
to vaccines or antibiotics is to place the medical solution into a glass vial and
freeze it to obtain a frozen material, at which time the water is drawn off by sub-
limation. The dry material that remains in the vial is a medical product that has
a longer storage life. Typically, the vials are of a size that corresponds to the
physician’s dosage rate for the patient. To reconstitute the dry product, sterile
water is injected. When a vaccine product is lyophilized and, thus, dormant, it
may be stored for considerable periods of time. But when sterile water is
injected into it, the vaccine becomes “live” and viable.

Lyophilization
Lyophilization consists of the lyophilizer circuit and a vacuum chamber. The lyo-
philizer circuit is a cabinet inside of which the medical solutions are loaded onto
movable shelves. In the pharmaceutical industry, the term lyphilizer is shortened
to lyo, which we use here as well. Figure 12-16 is a schematic diagram of the lyo
circuit, vacuum chamber, and circuiting secondary coolant chilled by the refrig-
eration system. The lyo cabinet is connected by means of a large pipe to the vac-
uum chamber containing refrigeration coils. A typical lyo cabinet shape is that of
a cube with dimensions 8 ft on each side. Figure 12-17 shows a production lyo
cabinet. The vacuum chamber is somewhat larger in volume than the lyo cabinet.
The medical water solution is placed into vials, which vary in size from 5 to 50
mL, depending on the intended use and dosage. The vials are loaded at room
temperature and arranged in an array supported by shallow trays inside the lyo
cabinet. The lyo shelves may be moved vertically up or down as needed. Fluted
rubber stoppers partially seal the vials, which permits the escape of water vapor
during the freeze-drying process that occurs later.
The production facility must be an ultraclean area wherever the vaccine or
antibiotic is exposed to air. In some production facilities, all facets of product
handling are fully automated. However, many production facilities are manu-
ally operated.

Operating Sequence
The operating sequence for lyophilization involves the following steps.

1. After the lyo cabinet’s trays (Figure 12-18) are fully loaded with vials and
the rubber fluted stoppers are pushed into the tops of the vials, the cabinet
door is shut, which produces a vacuum-tight seal. The refrigeration system
342 Chapter 12 Practical Guide to Refrigeration Systems

Figure 12-16 Schematic diagram of a lyophilizer circuit and vacuum chamber.

is then started. Figure 12-19 is a schematic of a typical refrigeration system


applied to lyophilization. This system utilizes two refrigeration systems
that operate in parallel to pull the lyo cabinet and its product down to a tem-
perature that satisfies the specific protocol for the vaccine or antibiotic
being produced. Three parallel refrigeration systems may also be used.
2. Freezing occurs due to the circulation of a chilled secondary fluid moving
through a network of passages inside the solid metal shelves. Each tray is
connected to the chilled secondary fluid headers by means of flexible hoses.
The selection of the secondary fluid is important because fluid viscosity
increases with decreasing temperature. Temperatures below –50°C may be
necessary to meet the product’s production protocol. One secondary fluid
used with lyo systems is a silicone polymer, which has relatively low viscos-
ity compared to other secondary fluids.
3. The secondary fluid is chilled in the direct-expansion evaporators and is cir-
culated by a pump through the cabinet’s trays, which gradually reduces the
temperature of the trays. Usually a spare circulation pump is also installed,
but it is not shown in Figure 12-16. Both (or three) refrigeration systems are
used simultaneously to reduce the temperature of the cabinet, as required by
the product’s protocol. The length of time for this freezing cycle to occur
depends on the product’s mass and the temperature to which it must be
Fundamentals of Refrigeration SI, Second Edition 343

Photograph courtesy of Millrock Technology, 2008.

Figure 12-17 Lyophilization cabinet (product chamber) on left, refrigeration equipment on


right, and vacuum chamber at the back.

Photograph courtesy of Millrock Technology, 2008.

Figure 12-18 Lyophilization cabinet movable trays that secure and seal the product.
344 Chapter 12 Practical Guide to Refrigeration Systems

Figure 12-19 Schematic diagram of the refrigeration system associated with the lyophilizer
circuit and vacuum chamber.

brought. Also, as the product temperature drops, the capacity of the refriger-
ation systems decreases. Consequently, careful attention is placed on the
product’s freezing protocol, and emphasis is placed on ensuring that the
refrigeration systems can meet that protocol in both time and temperature.

4. When the desired temperature of the product is achieved, refrigeration to


the direct-expansion evaporators is stopped, and refrigeration on the direct-
expansion coils inside the vacuum chamber is started. At this point, the
vacuum pumps are also started. Generally, the vacuum chamber is pulled
down to a pressure of approximately 200 μm Hg but rarely less than 50 m
Hg. At about the same time, the electric heater begins to warm the second-
ary fluid circulating through the trays inside the lyo cabinet in order to sub-
lime the ice. The water vapor produced moves toward the refrigerated coils
in the vacuum chamber where it forms frost covering the coil. After a time,
the coil is defrosted, and the water exits by means of a drain at the bottom
of the vacuum chamber.

5. When all the ice in the vials has sublimed, the refrigeration systems are
shut down and the lyo cabinet slowly warms back to the ambient tempera-
ture. The vials are then sealed very carefully by moving the trays vertically
closer together. This action forces the stoppers fully into the necks of the
vials. After the vials are sealed, the lyo cabinet door is opened, the trays are
withdrawn, and the vials are stored in an appropriate manner. Depending
Fundamentals of Refrigeration SI, Second Edition 345

on the particular product being manufactured, a compete production cycle


may require about eight hours from start to finish.

Refrigeration
From Figure 12-19, observe that the refrigeration system is rather straightforward.
The system shown utilizes two compounded compressors to achieve the pressure
ratio needed for the low evaporating temperature. For example, these compressors
could be the semihermetic type internally compounded, meaning that the two
compressors are manufactured as one unit. Thus the low-stage and the high-stage
compressors are coupled together, where liquid injection would be used between
the low and high stages to obtain good performance. Successfully applying liquid
injection of the refrigerant between the stages is tricky, because good mixing of
the liquid refrigerant with the vapor must occur in a small volume.
As shown in Figure 12-19, a subcooler is not included in the refrigeration
system. If a subcooler were used, the efficiency of the refrigeration cycle
would be improved. The provisions made in the refrigeration system to accom-
plish the return of oil to the compressor are complex and cannot easily be
shown in Figure 12-19. However, oil return is important to maintain reliable
operation of the refrigeration system. The suction trap protects the compressor
and is shown in Figure 12-19. Not shown is how the trapped liquid is removed
from the trap to maintain operation.
Generally, chilled water is used for condensing. Compared to cooling tower
water, chilled water is clean and colder, allowing the lyo refrigeration system to
operate at lower condensing temperatures, possibly in the vicinity of 21°C. The
low condensing temperature keeps the compressor discharge pressure low, as
well as the pressure ratio across the compressor. Obviously, this improves the
efficiency of the refrigeration cycle over what it would have been using cooling
tower water.
Halocarbon refrigerants are typically used in this application and, conse-
quently, copper tubing is commonly used for the refrigerant piping. The copper
tube fittings and the flexible hose connections are likely to develop small leaks
over time due mostly to vibration. Because these leaks are small, they are diffi-
cult to discover and repair. Unfortunately, the leaked refrigerant enters the
atmosphere and contributes to the degradation of the Earth’s ozone layer and to
global warming. Government regulations require that quantities of refrigerant
that escape to the atmosphere greater than a minimum threshold be reported to
the appropriate authorities and that the leaks be repaired. One solution to leak-
age from copper refrigerant lines is to replace all lines with stainless steel tub-
ing with welded fittings and valves.
Older refrigeration systems that use CFC refrigerants, such as R-13 or R-502,
will likely convert to an alternative refrigerant, such as R-507a. If a conversion in
refrigerants were made from R-13 to R-507a, then a reduction in performance
would result and possibly cause the lyo refrigeration system to not meet a medi-
cal product’s protocol. In this situation, the refrigeration system designer could
346 Chapter 12 Practical Guide to Refrigeration Systems

consider the addition of a subcooler and possibly regain the performance and
efficiency of the original system.

Summary
This chapter discussed how refrigeration is used in four applications: super-
markets, refrigerated warehouses, beef-processing plants, and pharmaceutical
lyophilization. Each of these applications had singular characteristics that
resulted in unique features associated with its refrigeration system. For exam-
ple, supermarkets typically use factory-assembled parallel low-stage and paral-
lel high-stage compressors mounted in open steel racks. On the other hand,
refrigerated warehouses and meat-processing plants use large custom-designed
industrial refrigeration systems that consist of multiple evaporators, long pip-
ing runs, storage vessels, and multiple compressors to meet the facilities’
requirements. The lyophilization of vaccines and antibiotics in the pharmaceu-
tical industry is a freeze-drying process using low-temperature refrigeration
and vacuum pumps to remove water from the medical solutions. After studying
Chapter 12, you should now be able to

 explain how a supermarket food display case operates to maintain food at


its appropriate storage temperature;
 describe two refrigeration system configurations used in supermarkets;
 explain why a refrigerated warehouse requires rooms held at different tem-
peratures;
 explain the difference between cooler rooms, freezer rooms, and convertible
rooms in a refrigerated warehouse;
 describe the chilling process of a beef carcass;
 describe what characteristics the evaporator coils should have in a chill
room to maintain relative humidity at an acceptable level;
 describe the lyophilization process as applied in the pharmaceutical industry;
and
 describe how the refrigeration system operates during the lyophilization
process in producing vaccines and antibiotics.

References
ASHRAE. 2005. ANSI/ASHRAE Standard 72-2005, Method of Testing Open
Refrigerators. Atlanta: ASHRAE.
ASHRAE. 2006. ASHRAE Handbook—Refrigeration. Atlanta: ASHRAE.
UNEP. 1987. The Montreal Protocol on Substances that Deplete the Ozone
Layer. New York: United Nations Environment Programme.
Fundamentals of Refrigeration SI, Second Edition 347

Skill Development Exercises for Chapter 12


Complete these questions by writing your answers on the worksheets at the back of this book.

12-1 Identify the refrigeration system components that are common to each of the
four applications described in this chapter. Identify at least one unique differ-
ence between each of the four applications.
12-2 Explain why an evaporator pressure regulator is needed in a supermarket utiliz-
ing multiple evaporators operating at different intermediate temperatures. Does
the use of EPRs improve or degrade the operating efficiency of the refrigera-
tion system? Explain why.
12-3 Why is ammonia generally selected as the refrigerant for use in refrigeration
systems applied to warehouses and meat-processing plants?
12-4 Consider the lyo refrigeration system shown schematically in Figure 12-19.
Redraw that part of the schematic diagram for this system to show the addition
of a liquid subcooler, its location, and its connections to operate. What would
be the advantage of adding a liquid subcooler to this cycle?
12-5 A lyo refrigeration system operates with refrigerant R-123. From the log sheets
recorded by a technician over the past three months, refrigerant R-123 has been
added to the system on a monthly basis as follows:

Month No. Quantity R-123 Added (kg)


1 10.9
2 15.4
3 22.2

Assuming that each month contains 30 days, what is the average daily leak rate
of R-123 for each month? Is there reason for concern? Use the Internet to
access the U.S. EPA regulations concerning refrigerant leak rates and deter-
mine if there is cause for concern. Is there another reason for concern?
Appendix A
Sample Refrigerants
The sample refrigerants found in Appendix A are taken from the 1997
ASHRAE Handbook—Fundamentals. See Chapter 11, “References and Bibli-
ography,” for publication information.
350 Appendix A Sample Refrigerants

Table A-1
Fundamentals of Refrigeration SI, Second Edition 351

Table A-2
352 Appendix A Sample Refrigerants

Table A-3
Fundamentals of Refrigeration SI, Second Edition 353

Table A-4
354 Appendix A Sample Refrigerants

Table A-5
Fundamentals of Refrigeration SI, Second Edition 355

Table A-5
356 Appendix A Sample Refrigerants

Table A-6
Fundamentals of Refrigeration SI, Second Edition 357

Table A-7
358 Appendix A Sample Refrigerants

Table A-8
Appendix B
Pressure-Enthalpy Diagrams
The pressure-enthalpy diagrams for refrigerants found in Appendix B are taken
from the 1997 ASHRAE Handbook—Fundamentals. See Chapter 11, “Refer-
ences and Bibliography,” for publication information.
360
Appendix B Pressure-Enthalpy Diagrams

Figure B-1 Pressure-enthalpy diagram for R-22.


Fundamentals of Refrigeration SI, Second Edition 361

Pressure-enthalpy diagram for R-23.


Figure B-2
362
Appendix B Pressure-Enthalpy Diagrams

Figure B-3 Pressure-enthalpy diagram for R-123.


Fundamentals of Refrigeration SI, Second Edition 363

Pressure-enthalpy diagram for R-134a.


Figure B-4
364
Appendix B Pressure-Enthalpy Diagrams

Figure B-5 Pressure-enthalpy diagram for R-404A.


Fundamentals of Refrigeration SI, Second Edition 365

Pressure-enthalpy diagram for R-717.


Figure B-6
366
Appendix B Pressure-Enthalpy Diagrams

Figure B-7 Pressure-enthalpy diagram for R-744.


Appendix C
Supplemental Tables
The supplemental tables found in Appendix C are taken from the 2006
ASHRAE Handbook—Refrigeration. See Chapter 12, “References and Bibliog-
raphy,” for publication information.
368 Appendix C Supplemental Tables

Table C-1 Storage Requirements of Vegetables, Fresh Fruits, and Melons


Highest
Storage Relative Ethylene Approximate Observations
Common Name Scientific Freezing Ethylene Respiration
Temp., Humidity, Production Postharvest and Beneficial
(Other Common Name) Name Temp., a Sensitivityb Ratec
°C % Rate Life CAd Conditions
°C
Acerola (Barbados cherry) Malpighia glabra 0 85 to 90 –1.4 6 to 8 weeks
African horned melon Cucumis africanus 13 to 15 90 Low Moderate 3 to 6 months
(kiwano)
Amaranth (pigweed) Amaranthus spp. 0 to 2 95 to 100 Very low Moderate 10 to 14 days
Anise (fennel) Foeniculum vulgare 0 to 2 90 to 95 –1.1 2 to 3 weeks
Apple
Not chilling sensitive Malus pumila –1 90 to 95 –1.5 Very high High Low 3 to 6 months 2 to 3% O2
1 to 2% CO2
Chilling sensitive Malus pumila cv. 4 90 to 95 –1.5 Very high High Low 1 to 2 months 2 to 3% O2
Yellow Newton, 1 to 2% CO2
Grimes golden,
McIntosh
Apricot Prunus armeniaca –0.5 to 0 90 to 95 –1.1 Moderate Moderate Low 1 to 3 weeks 2 to 3% O2
2 to 3% CO2
Artichokes
Chinese Stachys affinia 0 90 to 95 Very low Very Low 1 to 2 weeks
Globe Cynara acolymus 0 95 to –1.2 Very low Low High 2 to 3 weeks 2 to 3% O2
100 3 to 5% CO2
Jerusalem Helianthus –0.5 to 0 90 to 95 –2.5 Very low Low Low 4 months
tuberosus
Arugula Eruca vesicaria var. 0 95 to 100 Very low High Moderate 7 to 10 days
sativa
Asian pear (nashi) Pyrus serotina 1 90 to 95 –1.6 High High Low 4 to 6 months
P. pyrifolia
Asparagus, green or white Asparagus 2.5 95 to 100 –0.6 Very low Moderate Very high 2 to 3 weeks 5 to 12% CO2
officinalis
Atemoya Annona squamosa x 13 85 to 90 High High 2 to 4 weeks 3 to 5% O2
A. cherimola 5 to 10% CO2
Avocado
Fuchs, Pollock Persea americana 13 85 to 90 –0.9 High High Moderate 2 weeks
cv. Fuchs, Pollock
Fuerte, Hass Persea americana 3 to 7 85 to 90 –1.6 High High Moderate 2 to 4 weeks 2 to 5% O2
cv. Fuerte, Hass 3 to 10% CO2
Lula, Booth Persea americana 4 90 to 95 –0.9 High High Moderate 4 to 8 weeks
cv. Lula, Booth
Babaco (mountain papaya) Carica 7 85 to 90 1 to 3 weeks
candamarcensis
Banana Musa paradisiaca 13 to 15 90 to 95 –0.8 Moderate High Low 1 to 4 weeks 2 to 5% O2
var. sapientum 2 to 5% CO2
Barbados cherry see Acerola
Beans
Fava (broad) Vicia faba 0 90 to 95 1 to 2 weeks
Lima Phaseolous lunatus 5 to 6 95 –0.6 Low Moderate Moderate 5 to 7 days
Long (yard-long) Vigna sesquipedalis 4 to 7 90 to 95 Low Moderate 7 to 10 days
Snap (wax, green) Phaseolus vulgaris 4 to 7 95 –0.7 Low Moderate Moderate 7 to 10 days 2 to 3% O2
4 to 7% CO2
Winged Psophocarpus 10 90 4 weeks
tetragonolobus
Beet
Bunched Beta vulgaris 0 98 to 100 –0.4 Very low Low Low 10 to 14 days
Topped Beta vulgaris 0 98 to 100 –0.9 Very low Low Low 4 months
Berries
Blackberry Rubus spp. –0.5 to 0 90 to 95 –0.8 Low Low Moderate 3 to 6 days 5 to 10% O2
15 to 20% CO2
Blueberry Vaccinium –0.5 to 0 90 to 95 –1.3 Low Low Low 10 to 18 days 2 to 5% O2
corymbosum 12 to 20% CO2
Cranberry Vaccinium 2 to 5 90 to 95 –0.9 Low Low Low 8 to 16 weeks 1 to 2% O2
macrocarpon 0 to 5% CO2
Fundamentals of Refrigeration SI, Second Edition 369

Table C-1 Storage Requirements of Vegetables, Fresh Fruits, and Melons (Continued)
Highest
Storage Relative Ethylene Approximate Observations
Common Name Scientific Freezing Ethylene Respiration
Temp., Humidity, Production Postharvest and Beneficial
(Other Common Name) Name Temp., a Sensitivityb Ratec
°C % Rate Life CAd Conditions
°C
Dewberry Rubus spp. –0.5 to 0 90 to 95 –1.3 Low Low 2 to 3 days
Elderberry Rubus spp. –0.5 to 0 90 to 95 –0.9 Low Low 5 to 14 days
Loganberry Rubus spp. –0.5 to 0 90 to 95 –1.3 Low Low 2 to 3 days
Raspberry Rubus idaeus –0.5 to 0 90 to 95 –0.9 Low Low Moderate 3 to 6 days 5 to 10% O2
15 to 20% CO2
Strawberry Fragaria spp. 0 90 to 95 –0.8 Low Low Low 7 to 10 days 5 to 10% O2
15 to 20% CO2
Bittermelon (bitter gourd) Momordica 10 to 12 85 to 90 Low Moderate Moderate 2 to 3 weeks 2 to 3% O2
5% CO2
Black salsify (scorzonera) Scorzonera 0 to 1 95 to 98 Very low Low 6 months
hispanica
Bok choy Brassica chinensis 0 95 to 100 Very low High 3 weeks
Breadfruit Artocarpus altilis 13 to 15 85 to 90 2 to 4 weeks
Broccoli Brassica oleracea 0 95 to 100 –0.6 Very low High Moderate 10 to 14 days 1 to 2% O2
var. Italica 5 to 10% CO2
Brussels sprouts Brassica oleracea 0 95 to 100 –0.8 Very low High Moderate 3 to 5 weeks 1 to 2% O2
var. Gemnifera 5 to 7% CO2
Cabbage
Chinese (Napa) Brassica 0 95 to 100 –0.9 Very low High Low 2 to 3 months 1 to 2% O2
campestris 0 to 6% CO2
var. Pekinensis
Common, early crop Brassica oleracea 0 98 to 100 –0.9 Very low High Low 3 to 6 weeks
var. Capitata
Common, late crop Brassica oleracea 0 95 to 100 –0.9 Very low High Low 5 to 6 months 3 to 5% O2
var. Capitata 3 to 7% CO2
Cactus leaves (nopalitos) Opuntia spp. 5 to 10 90 to 95 Very low Moderate 2 to 3 weeks
Cactus fruit (prickly pear Opuntia spp. 5 85 to 90 –1.8 Very low Moderate 2 to 6 weeks
fruit)
Caimito see Sapotes
Calamondin see Citrus
Canistel see Sapotes
Carambola (starfruit) Averrhoa 9 to 10 85 to 90 –1.2 Low 3 to 4 weeks
carambola
Carrot
Topped Daucus carota 0 98 to 100 –1.4 Very low High Low 3 to 6 months No CA benefit
Bunched, immature Daucus carota 0 98 to 100 –1.4 Very low High Moderate 10 to 14 days Ethylene causes
bitterness
Cashew, apple Anacardium 0 to 2 85 to 90 5 weeks
occidentale
Cassava (yucca, manioc) Manihot esculenta 0 to 5 85 to 90 Very low Low Low 1 to 2 months No CA benefit
Cauliflower Brassica oleracea 0 95 to 98 –0.8 Very low High Moderate 3 to 4 weeks 2 to 5% O2
var. Botrytis 2 to 5% CO2
Celeriac Apium graveolens 0 98 to 100 –0.9 Very low Low Low 6 to 8 months 2 to 4% O2
var. Rapaceum 2 to 3% CO2
Celery Apium graveolens 0 98 to 100 –0.5 Very low Moderate Low 1 to 2 months 1 to 4% O2
var. Dulce 3 to 5% CO2
Chard Beta vulgaris 0 95 to 100 Very low High 10 to 14 days
var. Cida
Chayote Sechium edule 7 85 to 90 Low 4 to 6 weeks
Cherimoya (custard apple) Annona cherimola 13 90 to 95 –2.2 High High Very high 2 to 4 weeks 3 to 5% O2
5 to 10% CO2
Cherries
Sour Prunus cerasus 0 90 to 95 –1.7 Low 3 to 7 days 3 to 10% O2
10 to 12% CO2
Sweet Prunus avium –1 to 0 90 to 95 –2.1 Low 2 to 3 weeks 10 to 20% O2
20 to 25% CO2
Chicory see Endive
Chiles see Peppers
Chinese broccoli (gailan) Brassica 0 95 to 100 Very low High 10 to 14 days
alboglabra
Chives Allium 0 95 to 100 Very low High 2 to 3 weeks
schoenoprasum
370 Appendix C Supplemental Tables

Table C-1 Storage Requirements of Vegetables, Fresh Fruits, and Melons (Continued)
Highest
Storage Relative Ethylene Approximate Observations
Common Name Scientific Freezing Ethylene Respiration
Temp., Humidity, Production Postharvest and Beneficial
(Other Common Name) Name Temp., a Sensitivityb Ratec
°C % Rate Life CAd Conditions
°C
Cilantro (Chinese parsley) Coriandrum 0 to 2 95 to 100 Very low High High 2 weeks
sativum
Citrus
Calamondin orange Citrus reticulta 9 to 10 90 –2.0 Low 2 weeks
x. Fortunella spp.
Grapefruit
CA, AZ, dry areas Citrus paradisi 14 to 15 85 to 90 –1.1 Very low Moderate Low 6 to 8 weeks 3 to 10% O2
5 to 10% CO2
FL, humid areas Citrus paradisi 10 to 15 85 to 90 –1.1 Very low Moderate Low 6 to 8 weeks 3 to 10% O2
5 to 10% CO2
Kumquat Fortunella 4 90 to 95 Low 2 to 4 weeks
japponica
Lemon Citrus limon 10 to 13 85 to 90 –1.4 Low 1 to 6 months 5 to 10% O2
0 to 10% CO2
Store at 32 to
40°F0 to 4°C
for <1 mo.
Lime (Mexican, Tahitian Citrus aurantifolia; 9 to 10 85 to 90 –1.6 Low 6 to 8 weeks 5 to 10% O2
or Persian) C. latifolia 0 to 10% CO2
Orange
CA, AZ, dry areas Citrus sinensis 3 to 9 85 to 90 –0.8 Very low Moderate Low 3 to 8 weeks 5 to 10% O2
0 to 5% CO2
FL, humid areas Citrus sinensis 0 to 2 85 to 90 –0.8 Very low Moderate Low 8 to 12 weeks 5 to 10% O2
0 to 5% CO2
Blood orange Citrus sinensis 4 to 7 90 to 95 –0.8 Low 3 to 8 weeks 5 to 10% O2
0 to 5% CO2
Seville (sour) Citrus aurantium 10 85 to 90 –0.8 Low Low 12 weeks
Pomelo Citrus grandis 7 to 9 85 to 90 –1.6 Low 12 weeks
Tangelo (minneola) Citrus reticulata 7 to 10 85 to 95 –0.9 Low
x paradisi
Tangerine (mandarin) Citrus reticulata 4 to 7 90 to 95 –1.1 Very low Moderate Low 2 to 4 weeks
Coconut Cocos nucifera 0 to 2 89 to 85 –0.9 1 to 2 months
Collards and kale Brassica oleracea 0 95 to 100 –0.5 Very low High High 10 to 14 days
var. Acephala
Corn, sweet and baby Zea mays 0 95 to 98 –0.6 Very low Low High 5 to 8 days 2 to 4% O2
5 to 10% CO2
Cucumber Cucumis sativus 10 to 12 85 to 90 –0.5 Low High Low 10 to 14 days 3 to 5% O2
0 to 5% CO2
Cucumber, pickling Cucumis sativus 4 95 to 100 Low High 7 days 3 to 5% O2
3 to 5% CO2
Currants Ribes sativum; R. –0.5 to 0 90 to 95 –1.0 Low Low 1 to 4 weeks
nigrum; R. rubrum
Custard apple see Cherimoya
Daikon (Oriental radish) Raphanus sativus 0 to 1 95 to 100 Very low Low 4 months
Dasheen see Taro
Date Phoenix dactylifera –18 to 0 75 –15.7 Very low Low Low 6 to 12
months
Dill see Herbs
Durian Durio zibethinus 4 to 6 85 to 90 6 to 8 weeks 3 to 5% O2
5 to 15% CO2
Eggplant Solanum melongena 10 to 12 90 to 95 –0.8 Low Moderate Low 1 to 2 weeks 3 to 5% O2
0% CO2
Endive (escarole) Cichorium endivia 0 95 to 100 –0.1 Very low Moderate High 2 to 4 weeks
Belgian endive Cichorium intybus 2 to 3 95 to 98 Very low Moderate 2 to 4 weeks Light causes
(Witloof chicory) greening
3 to 4% O2
4 to 5% C2
Feijoa (pineapple guava) Feijoa selloiana 5 to 10 90 Moderate Low 2 to 3 weeks
Fennel see Anise
Fig, fresh Ficus carica –0.5 to 0 85 to 90 –2.4 Moderate Low Low 7 to 10 days 5 to 10% O2
15 to 20% CO2
Fundamentals of Refrigeration SI, Second Edition 371

Table C-1 Storage Requirements of Vegetables, Fresh Fruits, and Melons (Continued)
Highest
Storage Relative Ethylene Approximate Observations
Common Name Scientific Freezing Ethylene Respiration
Temp., Humidity, Production Postharvest and Beneficial
(Other Common Name) Name Temp., a Sensitivityb Ratec
°C % Rate Life CAd Conditions
°C
Garlic Allium sativum 0 65 to 70 –0.8 Very low Low Low 6 to 7 months 0.5% O2
5 to 10% CO2
Ginger Zingiber officinale 13 65 Very low Low 6 months No CA benefit
Gooseberry Ribes grossularia –0.5 to 0 90 to 95 –1.1 Low Low Low 3 to 4 weeks
Granadilla see Passionfruit
Grapee
Table grape Vitis vinifera –0.5 to 0 90 to 95 –2.7 Very low Low Low 1 to 6 months 2 to 5% O2
1 to 3% CO2
to 4 weeks: 5 to
10% O2
10 to 15% CO2
American grape Vitis labrusca –1 to –0.5 90 to 95 –1.4 Very low Low Low 2 to 8 weeks
Grapefruit see Citrus Low
Guava Psidium guajava 5 to 10 90 Low Moderate Moderate 2 to 3 weeks
Herbs, fresh culinary 5 to 10% O2
5 to 10% CO2
Basil Ocimum basilicum 10 90 Very low High 7 days
Chives Allium 0 95 to 100 –0.9 Low Moderate
schoenorasum
Dill Anethum 0 95 to 100 –0.7 Very low High 1 to 2 weeks
graveolens
Epazote Chenopodium 0 to 5 90 to 95 Very low Moderate 1 to 2 weeks
ambrosioides
Mint Mentha spp. 0 95 to 100 Very low High 2 to 3 weeks
Oregano Origanum vulgare 0 to 5 90 to 95 Very low Moderate 1 to 2 weeks
Parsley Petroselinum 0 95 to 100 –1.1 Very low High Very high 1 to 2 months
crispum
Perilla (shiso) Perilla frutescens 10 95 Very low Moderate 7 days
Sage Salvia officinalis 0 90 to 95 2 to 3 weeks
Thyme Thymus vulgaris 0 90 to 95 2 to 3 weeks
Horseradish Amoracia rusticana –1 to 0 98 to 100 –1.8 Very low Low 10 to 12
months
Husk tomato see Tomatillo
Jaboticaba Myrciaria 13 to 15 90 to 95 2 to 3 days
cauliflora =
Eugenia cauliflora
Jackfruit Artocarpus 13 85 to 90 Moderate Moderate 2 to 4 weeks
heterophyllus
Jerusalem artichoke see Artichoke
Jicama (yambean) Pachyrrhizus erosus 13 to 18 85 to 90 Very low Low Low 1 to 2 months
Jujube (Chinese date) Ziziphus jujuba 2.5 to 10 85 to 90 –1.6 Low Moderate 1 month
Kaki see Persimmon
Kale see Collards and
kale
Kiwano see African
horned melon
Kiwifruit (Chinese Actinidia chinensis 0 90 to 95 –0.9 Low High Low 3 to 5 months 1 to 2% O2
gooseberry) 3 to 5% CO2
Kohlrabi Brassica oleracea 0 98 to 100 –1.0 Very low Low Low 2 to 3 months
var. Gongylodes
Lo Bok see Daikon
Langsat (lanzone) Aglaia sp.; Lansium 11 to 14 85 to 90 2 weeks
sp.
Leafy greens
Cool-season various 0 95 to 100 –0.6 Very low High 10 to 14 days
Warm-season various 7 to 10 95 to 100 –0.6 Very low High 5 to 7 days
Leek Allium porrum 0 95 to 100 –0.7 Very low Moderate Moderate 2 months 1 to 2% O2
2 to 5% CO2
Lemon see Citrus
Lettuce Lactuca sativa 0 98 to 100 –0.2 Very low High Low 2 to 3 weeks 2 to 5% O2
0% CO2
Lime see Citrus
372 Appendix C Supplemental Tables

Table C-1 Storage Requirements of Vegetables, Fresh Fruits, and Melons (Continued)
Highest
Storage Relative Ethylene Approximate Observations
Common Name Scientific Freezing Ethylene Respiration
Temp., Humidity, Production Postharvest and Beneficial
(Other Common Name) Name Temp., a Sensitivityb Ratec
°C % Rate Life CAd Conditions
°C
Longan Dimocarpus longan 1 to 2 90 to 95 –2.4 2 to 4 weeks
= Euphoria longan
Loquat Eriobotrya 0 90 –1.9 3 weeks
japonica
Luffa (Chinese okra) Luffa spp. 10 to 12 90 to 95 Low Moderate 1 to 2 weeks
Lychee (litchi) Litchi chinensis 1 to 2 90 to 95 Moderate Moderate Low 3 to 5 weeks 3 to 5% O2
3 to 5% CO2
Malanga (tania, new Xanthosoma 7 70 to 80 Very low Low 3 months
cocoyam) sagittifolium
Mamey see Sapotes
Mandarin see Citrus
Mango Mangifera indica 13 85 to 90 –1.4 Moderate Moderate Moderate 2 to 3 weeks 3 to 5% O2
5 to 10% CO2
Mangosteen Garcinia 13 85 to 90 Moderate High 2 to 4 weeks
mangostana
Melons
Cantaloupes and other Cucurbita melo 2 to 5 95 –1.2 High Moderate Low 2 to 3 weeks 3 to 5% O2
netted melons var. reticulatus 10 to 15% CO2
Casaba Cucurbita melo 7 to 10 85 to 90 –1.0 Low Low 3 to 4 weeks 3 to 5% O2
5 to 10% CO2
Crenshaw Cucurbita melo 7 to 10 85 to 90 –1.1 Moderate High 2 to 3 weeks 3 to 5% O2
5 to 10% CO2
Honeydew, orange-flesh Cucurbita melo 5 to 10 85 to 90 –1.1 Moderate High Low 3 to 4 weeks 3 to 5% O2
5 to 10% CO2
Persian Cucurbita melo 7 to 10 85 to 90 –0.8 Moderate High 2 to 3 weeks 3 to 5% O2
5 to 10% CO2
Mint see Herbs
Mombin see Spondias
Mushrooms Agaricus, other 0 90 –0.9 Very low Moderate High 7 to 14 days 3 to 21% O2
genera l5 to 15% CO2
Mustard greens Brassica juncea 0 90 to 95 Very low High 7 to 14 days
Nashi see Asian pear
Nectarine Prunus persica –0.5 to 0 90 to 95 –0.9 Moderate Moderate Low 2 to 4 weeks 1 to 2% O2
3 to 5% CO2
Internal
breakdown
at 37 to 50°F3 to
10°C
Okra Abelmoschus 7 to 10 90 to 95 –1.8 Low Moderate High 7 to 10 days Air
esculentus 4 to 10% CO2
Olives, fresh Olea europea 5 to 10 85 to 90 –1.4 Low Moderate Low 4 to 6 weeks 2 to 3% O2
0 to 1% CO2
Onion
Mature bulbs, dry Allium cepa 0 65 to 70 –0.8 Very low Low Low 1 to 8 months 1 to 3% O2
5 to 10% CO2
Green Allium cepa 0 95 to 100 –0.9 Low High Moderate 3 weeks 2 to 4% O2
10 to 20% CO2
Orange see Citrus
Papaya Carica papaya 7 to 13 85 to 90 –0.9 Low 1 to 3 weeks 2 to 5% O2
5 to 8% CO2
Parsley see Herbs
Parsnips Pastinaca sativa 0 95 to 100 –0.9 Very low High Low 4 to 6 months Ethylene causes
bitterness
Passionfruit Passiflora spp. 10 85 to 90 Very high Moderate Very high 3 to 4 weeks
Peach Prunus persica –0.5 to 0 90 to 95 –0.9 High Moderate Low 2 to 4 weeks 1 to 2% O2
3 to 5% CO2
Internal
breakdown
at 37 to 50°F3 to
10°C
Pear, Americane Pyrus communis –1.5 to –0.5 90 to 95 –1.7 High High Low 2 to 7 months Cultivar variations
1 to 3% O2
0 to 5% CO2
Peas
Fundamentals of Refrigeration SI, Second Edition 373

Table C-1 Storage Requirements of Vegetables, Fresh Fruits, and Melons (Continued)
Highest
Storage Relative Ethylene Approximate Observations
Common Name Scientific Freezing Ethylene Respiration
Temp., Humidity, Production Postharvest and Beneficial
(Other Common Name) Name Temp., a Sensitivityb Ratec
°C % Rate Life CAd Conditions
°C
In pods (snow, snap, and Pisum sativum 0 to 1 90 to 98 –0.6 Very low Moderate Very high 1 to 2 weeks 2 to 3% O2
sugar peas) 2 to 3% CO2
Southern peas (cowpeas) Vigna sinensis = 4 to 5 95 6 to 8 days
V. unguiculata
Pepino (melon pear) Solanum muricatum 5 to 10 95 Low Moderate 4 weeks
Peppers
Bell pepper or paprika Capsicum annuum 7 to 10 95 to 98 –0.7 Low Low Low 2 to 3 weeks 2 to 5% O2
2 to 5% CO2
Hot peppers (chiles) Capsicum annuum 5 to 10 85 to 95 –0.7 Low Moderate 2 to 3 weeks 3 to 5% O2
and C. frutescens 5 to 10% CO2
Persimmon (kaki) Dispyros kaki 3 to 5% O2
5 to 8% CO2
Fuyu Dispyros kaki 0 90 to 95 –2.2 Low High Low 1 to 3 months
var. Fuyu
Hachiya Dispyros kaki 0 90 to 95 –2.2 Low High Low 2 to 3 months
var. Hachiya
Pineapple Ananas comosus 7 to 13 85 to 90 –1.1 Low Low Low 2 to 4 weeks 2 to 5% O2
5 to 10% CO2
Plantain Musa paradisiaca 13 to 15 90 to 95 –0.8 Low High 1 to 5 weeks
var. paradisiaca
Plums and prunes Prunus domestica –0.5 to 0 90 to 95 –0.8 Moderate Moderate Low 2 to 5 weeks 1 to 2% O2
0 to 5% CO2
Pomegranate Punica granatum 5 90 to 95 –3.0 Low 2 to 3 months 3 to 5% O2
5 to 10% CO2
Potato
Early crop Solanum tuberosum 10 to 15 90 to 95 –0.8 Very low Moderate Low 10 to 14 days No CA benefit
Late crop Solanum tuberosum 4 to 12 95 to 98 –0.8 Very low Moderate Low 5 to 10 No CA benefit
months
Pumpkin Cucurbita maxima 12 to 15 50 to 70 –0.8 Very low Moderate Low 2 to 3 months
Quince Cydonia oblonga –0.5 to 0 90 –2.0 Low High 2 to 3 months
Raddichio Cichorium intybus 0 to 1 95 to 100 4 to 8 weeks
Radish Raphanus sativus 0 95 to 100 –0.7 Very low Low Low 1 to 2 months 1 to 2% O2
2 to 3% CO2
Rambutan Nephelium 12 90 to 95 High High 1 to 3 weeks 3 to 5% O2
lappaceum 7 to 12% CO2
Rhubarb Rheum 0 95 to 100 –0.9 Very low Low Low 2 to 4 weeks
rhaponticum
Rutabaga Brassica napus 0 98 to 100 –1.1 Very low Low Low 4 to 6 months
var. Napobrassica
Sage see Herbs
Salsify (vegetable oyster) Trapopogon 0 95 to 98 –1.1 Very low Low Low 2 to 4 months
porrifolius
Sapotes
Black sapote Diospyros 13 to 15 85 to 90 –2.3 2 to 3 weeks
ebenaster
Caimito (star apple) Chrysophyllum 3 90 –1.2 3 weeks
cainito
Canistel (eggfruit) Pouteria 13 to 15 85 to 90 –1.8 3 weeks
campechiana
Mamey sapote Calocarpum 13 to 15 90 to 95 High High 2 to 3 weeks
mammosum
Sapodilla (chicosapote) Achras sapota 15 to 20 85 to 90 High High 2 weeks
White sapote Casimiroa edulis 20 85 to 90 –2.0 2 to 3 weeks
Scorzonera see Black salsify
Shallot Allium cepa 0 to 2.5 65 to70 –0.7 Low Low
var. ascalonicum
Soursop Annona muricata 13 85 to 90 1 to 2 weeks
Spinach Spinacia oleracea 0 95 to 100 –0.3 Very low High Low 10 to 14 days 5 to 10% O2
5 to 10% CO2
Spondias Spondias spp. 13 85 to 90 1 to 2 weeks
(mombin, wi apple, jobo,
hogplum)
Sprouts from seeds 0 95 to 100 5 to 9 days
374 Appendix C Supplemental Tables

Table C-1 Storage Requirements of Vegetables, Fresh Fruits, and Melons (Continued)
Highest
Storage Relative Ethylene Approximate Observations
Common Name Scientific Freezing Ethylene Respiration
Temp., Humidity, Production Postharvest and Beneficial
(Other Common Name) Name Temp., a Sensitivityb Ratec
°C % Rate Life CAd Conditions
°C
Alfalfa sprouts Medicago sativa 0 95 to 100 7 days
Bean sprouts Phaseolus sp. 0 95 to 100 7 to 9 days
Radish sprouts Raphanus sp. 0 95 to 100 5 to 7 days
Squash
Summer, soft rind Cucurbita pepo 7 to 10 95 –0.5 Low Moderate Low 1 to 2 weeks 3 to 5% O2
(courgette) 5 to 10% CO2
Winter, hard rind Cucurbita moschata; 12 to 15 50 to 70 –0.8 Low Moderate Low 2 to 3 months Large differences
(calabash) C. maxima among varieties
Star apple see Sapotes
Starfruit see Carambola
Sweet potato or yam Ipomea batatas 13 to 15 85 to 95 –1.3 Very low Low Low 4 to 7 months
Sweetsop Annona squamosa; 7 85 to 90 High High 4 weeks 3 to 5% O2
(sugar apple, custard Annona spp. 5 to 10% CO2
apple)
Tamarillo (tree tomato) Cyphomandra 3to 4 85 to 95 Low Moderate 10 weeks
betacea
Tamarind Tamarindus indica 2 to 7 90 to 95 –3.7 Very low Very Low 3 to 4 weeks
Taro (cocoyam, Colocasia esculenta 7 to 10 85 to 90 –0.9 Low 4 months No CA benefit
eddoe, dasheen)
Thyme see Herbs
Tomatillo (husk tomato) Physalis ixocarpa 7 to 13 85 to 90 Very low Moderate Low 3 weeks
Tomato
Mature, green Lycopersicon 10 to 13 90 to 95 –0.5 Very low High Low 2 to 5 weeks 3 to 5% O2
esculentum 2 to 3% CO2
Firm, ripe Lycopersicon 8 to 10 85 to 90 –0.5 High Low Low 1 to 3 weeks 3 to 5% O2
esculentum 3 to 5% CO2
Turnip root Brassica campetris 0 95 –1.0 Very low Low Low 4 to 5 months
var. Rapifera
Water chestnut Eleocharis dulcis 1 to 2 85 to 90 2 to 4 months
Watercress (garden cress) Lepidium sativum; 0 95 to 100 –0.3 Very low High High 2 to 3 weeks
Nasturtium
officinales
Watermelon Citrullus vulgaris 10 to 15 90 –0.4 Very low High Low 2 to 3 weeks No CA benefit
Yam Dioscorea spp. 15 70 to 80 –1.1 Very low Low 2 to 7 months
Yucca see Cassava
b
Note: Recommendations in this table are general guidelines. Recommended storage conditions Detrimental effects include yellowing, softening, increased decay, abscission,
and expected postharvest life for a specific produce item may be different from those listed and browning.
c
here because of variations in growing conditions and postharvest care. Also, new cultivars At recommended storage temperature.
(varieties) of a particular item may require different conditions and have a very different Low = <20 mg CO2/(kg·h)
expected postharvest life from that listed in the table. Empty cells indicate that no data are Moderate = <40 mg CO2/(kg·h)
available. For updates on guidelines, refer to the University of California Web site at http:// High = <60 mg CO2/(kg·h)
Very high = >60 mg CO2/(kg·h)
postharvest.ucdavis.edu. d
CA = controlled atmosphere.
aVery low = <0.1 L/(kg·h) at 20°C e
For a more complete listing of grapes and pears, see International Institute of
Low = 0.1 to 1.0 L/(kg·h) Refrigeration.
Moderate = 1.0 to 10.0 L/(kg·h)
High = 10 to 100 L/(kg·h)
Very high = >100 L/(kg·h)
Fundamentals of Refrigeration SI, Second Edition 375

Table C-2 Unfrozen Composition Data, Initial Freezing Point, and Specific Heats of Foods*
Moisture Initial Specific Heat Specific Heat Latent
Carbohydrate
Content, Protein, Freezing Above Below Heat of
% % Fat, % Total, % Fiber, % Ash, % Point, Freezing, Freezing Fusion,
Food Item xwo xp xf xc xfb xa °C kJ/(kg·K) kJ/(kg·K) kJ/kg
Vegetables
Artichokes, globe 84.94 3.27 0.15 10.51 5.40 1.13 –1.2 3.90 2.02 284
Jerusalem 78.01 2.00 0.01 17.44 1.60 2.54 –2.5 3.63 2.25 261
Asparagus 92.40 2.28 0.20 4.54 2.10 0.57 –0.6 4.03 1.79 309
Beans, snap 90.27 1.82 0.12 7.14 3.40 0.66 –0.7 3.99 1.85 302
lima 70.24 6.84 0.86 20.16 4.90 1.89 –0.6 3.52 2.07 235
Beets 87.58 1.61 0.17 9.56 2.80 1.08 –1.1 3.91 1.94 293
Broccoli 90.69 2.98 0.35 5.24 3.00 0.92 –0.6 4.01 1.82 303
Brussels sprouts 86.00 3.38 0.30 8.96 3.80 1.37 –0.8 3.90 1.91 287
Cabbage 92.15 1.44 0.27 5.43 2.30 0.71 –0.9 4.02 1.85 308
Carrots 87.79 1.03 0.19 10.14 3.00 0.87 –1.4 3.92 2.00 293
Cauliflower 91.91 1.98 0.21 5.20 2.50 0.71 –0.8 4.02 1.84 307
Celeriac 88.00 1.50 0.30 9.20 1.80 1.00 –0.9 3.90 1.89 294
Celery 94.64 0.75 0.14 3.65 1.70 0.82 –0.5 4.07 1.74 316
Collards 90.55 1.57 0.22 7.11 3.60 0.55 –0.8 4.01 1.86 302
Corn, sweet, yellow 75.96 3.22 1.18 19.02 2.70 0.62 –0.6 3.62 1.98 254
Cucumbers 96.01 0.69 0.13 2.76 0.80 0.41 –0.5 4.09 1.71 321
Eggplant 92.03 1.02 0.18 6.07 2.50 0.71 –0.8 4.02 1.83 307
Endive 93.79 1.25 0.20 3.35 3.10 1.41 –0.1 4.07 1.69 313
Garlic 58.58 6.36 0.50 33.07 2.10 1.50 –0.8 3.17 2.19 196
Ginger, root 81.67 1.74 0.73 15.09 2.00 0.77 — 3.75 1.94 273
Horseradish 78.66 9.40 1.40 8.28 2.00 2.26 –1.8 3.70 2.12 263
Kale 84.46 3.30 0.70 10.01 2.00 1.53 –0.5 3.82 1.86 282
Kohlrabi 91.00 1.70 0.10 6.20 3.60 1.00 –1.0 4.02 1.90 304
Leeks 83.00 1.50 0.30 14.15 1.80 1.05 –0.7 3.77 1.91 277
Lettuce, iceberg 95.89 1.01 0.19 2.09 1.40 0.48 –0.2 4.09 1.65 320
Mushrooms 91.81 2.09 0.42 4.65 1.20 0.89 –0.9 3.99 1.84 307
Okra 89.58 2.00 0.10 7.63 3.20 0.70 –1.8 3.97 2.05 299
Onions 89.68 1.16 0.16 8.63 1.80 0.37 –0.9 3.95 1.87 300
dehydrated flakes 3.93 8.95 0.46 83.28 9.20 3.38 — — — 13
Parsley 87.71 2.97 0.79 6.33 3.30 2.20 –1.1 3.93 1.94 293
Parsnips 79.53 1.20 0.30 17.99 4.90 0.98 –0.9 3.74 2.02 266
Peas, green 78.86 5.42 0.40 14.46 5.10 0.87 –0.6 3.75 1.98 263
Peppers, freeze-dried 2.00 17.90 3.00 68.70 21.30 8.40 — — — 7
sweet, green 92.19 0.89 0.19 6.43 1.80 0.30 –0.7 4.01 1.80 308
Potatoes, main crop 78.96 2.07 0.10 17.98 1.60 0.89 –0.6 3.67 1.93 264
sweet 72.84 1.65 0.30 24.28 3.00 0.95 –1.3 3.48 2.09 243
Pumpkins 91.60 1.00 0.10 6.50 0.50 0.80 –0.8 3.97 1.81 306
Radishes 94.84 0.60 0.54 3.59 1.60 0.54 –0.7 4.08 1.77 317
Rhubarb 93.61 0.90 0.20 4.54 1.80 0.76 –0.9 4.05 1.83 313
Rutabaga 89.66 1.20 0.20 8.13 2.50 0.81 –1.1 3.96 1.92 299
Salsify (vegetable oyster) 77.00 3.30 0.20 18.60 3.30 0.90 –1.1 3.65 2.05 257
Spinach 91.58 2.86 0.35 3.50 2.70 1.72 –0.3 4.02 1.75 306
Squash, summer 94.20 0.94 0.24 4.04 1.90 0.58 –0.5 4.07 1.74 315
winter 87.78 0.80 0.10 10.42 1.50 0.90 –0.8 3.89 1.87 293
Tomatoes, mature green 93.00 1.20 0.20 5.10 1.10 0.50 –0.6 4.02 1.77 311
ripe 93.76 0.85 0.33 4.64 1.10 0.42 –0.5 4.08 1.79 313
Turnip 91.87 0.90 0.10 6.23 1.80 0.70 –1.1 4.00 1.88 307
greens 91.07 1.50 0.30 5.73 3.20 1.40 –0.2 4.01 1.74 304
Watercress 95.11 2.30 0.10 1.29 1.50 1.20 –0.3 4.08 1.69 318
Yams 69.60 1.53 0.17 27.89 4.10 0.82 — 3.47 2.06 232
Fruits
Apples, fresh 83.93 0.19 0.36 15.25 2.70 0.26 –1.1 3.81 1.98 280
dried 31.76 0.93 0.32 65.89 8.70 1.10 — 2.57 2.84 106
Apricots 86.35 1.40 0.39 11.12 2.40 0.75 –1.1 3.87 1.95 288
Avocados 74.27 1.98 15.32 7.39 5.00 1.04 –0.3 3.67 1.98 248
Bananas 74.26 1.03 0.48 23.43 2.40 0.80 –0.8 3.56 2.03 248
Blackberries 85.64 0.72 0.39 12.76 5.30 0.48 –0.8 3.91 1.94 286
Blueberries 84.61 0.67 0.38 14.13 2.70 0.21 –1.6 3.83 2.06 283
Cantaloupes 89.78 0.88 0.28 8.36 0.80 0.71 –1.2 3.93 1.91 300
Cherries, sour 86.13 1.00 0.30 12.18 1.60 0.40 –1.7 3.85 2.05 288
sweet 80.76 1.20 0.96 16.55 2.30 0.53 –1.8 3.73 2.12 270
Cranberries 86.54 0.39 0.20 12.68 4.20 0.19 –0.9 3.91 1.93 289
Currants, European black 81.96 1.40 0.41 15.38 0.00 0.86 –1.0 3.71 1.95 274
red and white 83.95 1.40 0.20 13.80 4.30 0.66 –1.0 3.85 1.98 280
376 Appendix C Supplemental Tables

Table C-2 Unfrozen Composition Data, Initial Freezing Point, and Specific Heats of Foods* (Continued)
Moisture Initial Specific Heat Specific Heat Latent
Carbohydrate
Content, Protein, Freezing Above Below Heat of
% % Fat, % Total, % Fiber, % Ash, % Point, Freezing, Freezing Fusion,
Food Item xwo xp xf xc xfb xa °C kJ/(kg·K) kJ/(kg·K) kJ/kg
Dates, cured 22.50 1.97 0.45 73.51 7.50 1.58 –15.7 2.31 2.30 75
Figs, fresh 79.11 0.75 0.30 19.18 3.30 0.66 –2.4 3.70 2.25 264
dried 28.43 3.05 1.17 65.35 9.30 2.01 — 2.51 4.13 95
Gooseberries 87.87 0.88 0.58 10.18 4.30 0.49 –1.1 3.95 1.96 293
Grapefruit 90.89 0.63 0.10 8.08 1.10 0.31 –1.1 3.96 1.89 304
Grapes, American 81.30 0.63 0.35 17.15 1.00 0.57 –1.6 3.71 2.07 272
European type 80.56 0.66 0.58 17.77 1.00 0.44 –2.1 3.70 2.16 269
Lemons 87.40 1.20 0.30 10.70 4.70 0.40 –1.4 3.94 2.02 292
Limes 88.26 0.70 0.20 10.54 2.80 0.30 –1.6 3.93 2.03 295
Mangos 81.71 0.51 0.27 17.00 1.80 0.50 –0.9 3.74 1.95 273
Melons, casaba 92.00 0.90 0.10 6.20 0.80 0.80 –1.1 3.99 1.87 307
honeydew 89.66 0.46 0.10 9.18 0.60 0.60 –0.9 3.92 1.86 299
watermelon 91.51 0.62 0.43 7.18 0.50 0.26 –0.4 3.97 1.74 306
Nectarines 86.28 0.94 0.46 11.78 1.60 0.54 –0.9 3.86 1.90 288
Olives 79.99 0.84 10.68 6.26 3.20 2.23 –1.4 3.76 2.07 267
Oranges 82.30 1.30 0.30 15.50 4.50 0.60 –0.8 3.81 1.96 275
Peaches, fresh 87.66 0.70 0.90 11.10 2.00 0.46 –0.9 3.91 1.90 293
dried 31.80 3.61 0.76 61.33 8.20 2.50 — 2.57 3.49 106
Pears 83.81 0.39 0.40 15.11 2.40 0.28 –1.6 3.80 2.06 280
Persimmons 64.40 0.80 0.40 33.50 0.00 0.90 –2.2 3.26 2.29 215
Pineapples 86.50 0.39 0.43 12.39 1.20 0.29 –1.0 3.85 1.91 289
Plums 85.20 0.79 0.62 13.01 1.50 0.39 –0.8 3.83 1.90 285
Pomegranates 80.97 0.95 0.30 17.17 0.60 0.61 –3.0 3.70 2.30 270
Prunes, dried 32.39 2.61 0.52 62.73 7.10 1.76 — 2.56 3.50 108
Quinces 83.80 0.40 0.10 15.30 1.90 0.40 –2.0 3.79 2.13 280
Raisins, seedless 15.42 3.22 0.46 79.13 4.00 1.77 — 2.07 2.04 52
Raspberries 86.57 0.91 0.55 11.57 6.80 0.40 –0.6 3.96 1.91 289
Strawberries 91.57 0.61 0.37 7.02 2.30 0.43 –0.8 4.00 1.84 306
Tangerines 87.60 0.63 0.19 11.19 2.30 0.39 –1.1 3.90 1.93 293
Whole Fish
Cod 81.22 17.81 0.67 0.0 0.0 1.16 –2.2 3.78 2.14 271
Haddock 79.92 18.91 0.72 0.0 0.0 1.21 –2.2 3.75 2.14 267
Halibut 77.92 20.81 2.29 0.0 0.0 1.36 –2.2 3.74 2.18 260
Herring, kippered 59.70 24.58 12.37 0.0 0.0 1.94 –2.2 3.26 2.27 199
Mackerel, Atlantic 63.55 18.60 13.89 0.0 0.0 1.35 –2.2 3.33 2.23 212
Perch 78.70 18.62 1.63 0.0 0.0 1.20 –2.2 3.71 2.15 263
Pollock, Atlantic 78.18 19.44 0.98 0.0 0.0 1.41 –2.2 3.70 2.15 261
Salmon, pink 76.35 19.94 3.45 0.0 0.0 1.22 –2.2 3.68 2.17 255
Tuna, bluefin 68.09 23.33 4.90 0.0 0.0 1.18 –2.2 3.43 2.19 227
Whiting 80.27 18.31 1.31 0.0 0.0 1.30 –2.2 3.77 2.15 268
Shellfish
Clams 81.82 12.77 0.97 2.57 0.0 1.87 –2.2 3.76 2.13 273
Lobster, American 76.76 18.80 0.90 0.50 0.0 2.20 –2.2 3.64 2.15 256
Oysters 85.16 7.05 2.46 3.91 0.0 1.42 –2.2 3.83 2.12 284
Scallop, meat 78.57 16.78 0.76 2.36 0.0 1.53 –2.2 3.71 2.15 262
Shrimp 75.86 20.31 1.73 0.91 0.0 1.20 –2.2 3.65 2.16 253
Beef
Brisket 55.18 16.94 26.54 0.0 0.0 0.80 — 3.19 2.33 184
Carcass, choice 57.26 17.32 24.05 0.0 0.0 0.81 –2.2 3.24 2.31 191
select 58.21 17.48 22.55 0.0 0.0 0.82 –1.7 3.25 2.24 194
Liver 68.99 20.00 3.85 5.82 0.0 1.34 –1.7 3.47 2.16 230
Ribs, whole (ribs 6-12) 54.54 16.37 26.98 0.0 0.0 0.77 — 3.16 2.32 182
Round, full cut, lean and fat 64.75 20.37 12.81 0.0 0.0 0.97 — 3.39 2.18 216
full cut, lean 70.83 22.03 4.89 0.0 0.0 1.07 — 3.52 2.12 237
Sirloin, lean 71.70 21.24 4.40 0.0 0.0 1.08 –1.7 3.53 2.11 239
Short loin, porterhouse steak, lean 69.59 20.27 8.17 0.0 0.0 1.01 — 3.49 2.14 232
T-bone steak, lean 69.71 20.78 7.27 0.0 0.0 1.27 — 3.49 2.14 233
Tenderloin, lean 68.40 20.78 7.90 0.0 0.0 1.04 — 3.45 2.14 228
Veal, lean 75.91 20.20 2.87 0.0 0.0 1.08 — 3.65 2.09 254
Pork
Backfat 7.69 2.92 88.69 0.0 0.0 0.70 — 2.17 2.98 26
Bacon 31.58 8.66 57.54 0.09 0.0 2.13 — 2.70 2.70 105
Fundamentals of Refrigeration SI, Second Edition 377

Table C-2 Unfrozen Composition Data, Initial Freezing Point, and Specific Heats of Foods* (Continued)
Moisture Initial Specific Heat Specific Heat Latent
Carbohydrate
Content, Protein, Freezing Above Below Heat of
% % Fat, % Total, % Fiber, % Ash, % Point, Freezing, Freezing Fusion,
Food Item xwo xp xf xc xfb xa °C kJ/(kg·K) kJ/(kg·K) kJ/kg
Belly 36.74 9.34 53.01 0.0 0.0 0.49 — 2.80 3.37 123
Carcass 49.83 13.91 35.07 0.0 0.0 0.72 — 3.08 3.10 166
Ham, cured, whole, lean 68.26 22.32 5.71 0.05 0.0 3.66 — 3.47 2.22 228
country cured, lean 55.93 27.80 8.32 0.30 0.0 7.65 — 3.16 2.31 187
Shoulder, whole, lean 72.63 19.55 7.14 0.0 0.0 1.02 –2.2 3.59 2.20 243
Sausage
Braunschweiger 48.01 13.50 32.09 3.13 0.0 3.27 — 3.01 2.40 160
Frankfurter 53.87 11.28 29.15 2.55 0.0 3.15 –1.7 3.15 2.31 180
Italian 51.08 14.25 31.33 0.65 0.0 2.70 — 3.10 2.37 171
Polish 53.15 14.10 28.72 1.63 0.0 2.40 — 3.14 2.36 178
Pork 44.52 11.69 40.29 1.02 0.0 2.49 — 2.95 2.43 149
Smoked links 39.30 22.20 31.70 2.10 0.0 4.70 — 2.82 2.45 131
Poultry Products
Chicken 65.99 18.60 15.06 0.0 0.0 0.79 –2.8 4.34 3.32 220
Duck 48.50 11.49 39.34 0.0 0.0 0.68 — 3.06 2.45 162
Turkey 70.40 20.42 8.02 0.0 0.0 0.88 — 3.53 2.28 235
Egg
White 87.81 10.52 0.0 1.03 0.0 0.64 –0.6 3.91 1.81 293
dried 14.62 76.92 0.04 4.17 0.0 4.25 — 2.29 2.10 49
Whole 75.33 12.49 10.02 1.22 0.0 0.94 –0.6 3.63 1.95 252
dried 3.10 47.35 40.95 4.95 0.0 3.65 — 2.04 2.00 10
Yolk 48.81 16.76 30.87 1.78 0.0 1.77 –0.6 3.05 2.25 163
salted 50.80 14.00 23.00 1.60 0.0 10.60 –17.2 3.01 3.79 170
sugared 51.25 13.80 22.75 10.80 0.0 1.40 –3.9 3.07 2.54 171
Lamb
Composite of cuts, lean 73.42 20.29 5.25 0.0 0.0 1.06 –1.9 3.60 2.14 245
Leg, whole, lean 74.11 20.56 4.51 0.0 0.0 1.07 — 3.62 2.14 248
Dairy Products
Butter 17.94 0.85 81.11 0.06 0.0 0.04 — 2.40 2.65 60
Cheese
Camembert 51.80 19.80 24.26 0.46 0.0 3.68 — 3.10 3.34 173
Cheddar 36.75 24.90 33.14 1.28 0.0 3.93 –12.9 2.77 3.07 123
Cottage, uncreamed 79.77 17.27 0.42 1.85 0.0 0.69 –1.2 3.73 1.99 266
Cream 53.75 7.55 34.87 2.66 0.0 1.17 — 3.16 2.91 180
Gouda 41.46 24.94 27.44 2.22 0.0 3.94 — 2.87 2.77 138
Limburger 48.42 20.05 27.25 0.49 0.0 3.79 –7.4 3.03 2.82 162
Mozzarella 54.14 19.42 21.60 2.22 0.0 2.62 — 3.15 2.46 181
Parmesan, hard 29.16 35.75 25.83 3.22 0.0 6.04 — 2.58 2.94 97
Processed American 39.16 22.15 31.25 1.30 0.0 5.84 –6.9 2.80 2.75 131
Roquefort 39.38 21.54 30.64 2.00 0.0 6.44 –16.3 2.80 3.36 132
Swiss 37.21 28.43 27.45 3.38 0.0 3.53 –10.0 2.78 2.88 124
Cream
Half and half 80.57 2.96 11.50 4.30 0.0 0.67 — 3.73 2.16 269
Table 73.75 2.70 19.31 3.66 0.0 0.58 –2.2 3.59 2.21 246
Heavy whipping 57.71 2.05 37.00 2.79 0.0 0.45 — 3.25 2.32 193
Ice Cream
Chocolate 55.70 3.80 11.0 28.20 1.20 1.00 –5.6 3.11 2.75 186
Strawberry 60.00 3.20 8.40 27.60 0.30 0.70 –5.6 3.19 2.74 200
Vanilla 61.00 3.50 11.00 23.60 0.0 0.90 –5.6 3.22 2.74 204
Milk
Canned, condensed, sweetened 27.16 7.91 8.70 54.40 0.0 1.83 –15.0 2.35 — 91
Evaporated 74.04 6.81 7.56 10.04 0.0 1.55 –1.4 3.56 2.08 247
Skim 90.80 3.41 0.18 4.85 0.0 0.76 — 3.95 1.78 303
Skim, dried 3.16 36.16 0.77 51.98 0.0 7.93 — 1.80 — 11
Whole 87.69 3.28 3.66 4.65 0.0 0.72 –0.6 3.89 1.81 293
dried 2.47 26.32 26.71 38.42 0.0 6.08 — 1.85 — 8
Whey, acid, dried 3.51 11.73 0.54 73.45 0.0 10.77 — 1.68 — 12
sweet, dried 3.19 12.93 1.07 74.46 0.0 8.35 — 1.69 — 11
Nuts, Shelled
Almonds 4.42 19.95 52.21 20.40 10.90 3.03 — 2.20 — 15
Filberts 5.42 13.04 62.64 15.30 6.10 3.61 — 2.09 — 18
Peanuts, raw 6.5 25.80 49.24 16.14 8.50 2.33 — 2.23 — 22
dry roasted with salt 1.55 23.68 49.66 21.51 8.00 3.60 — 2.08 — 5
378 Appendix C Supplemental Tables

Table C-2 Unfrozen Composition Data, Initial Freezing Point, and Specific Heats of Foods* (Continued)
Moisture Initial Specific Heat Specific Heat Latent
Carbohydrate
Content, Protein, Freezing Above Below Heat of
% % Fat, % Total, % Fiber, % Ash, % Point, Freezing, Freezing Fusion,
Food Item xwo xp xf xc xfb xa °C kJ/(kg·K) kJ/(kg·K) kJ/kg
Pecans 4.82 7.75 67.64 18.24 7.60 1.56 — 2.17 — 16
Walnuts, English 3.65 14.29 61.87 18.34 4.80 1.86 — 2.09 — 12
Candy
Fudge, vanilla 10.90 1.10 5.40 82.30 0.0 0.40 — 1.90 — 36
Marshmallows 16.40 1.80 0.20 81.30 0.10 0.30 — 2.02 — 55
Milk chocolate 1.30 6.90 30.70 59.20 3.40 1.50 — 1.83 — 4
Peanut brittle 1.80 7.50 19.10 69.30 2.00 1.50 — 1.77 — 6
Juice and Beverages
Apple juice, unsweetened 87.93 0.06 0.11 11.68 0.10 0.22 — 3.87 1.78 294
Grapefruit juice, sweetened 87.38 0.58 0.09 11.13 0.10 0.82 — 3.85 1.78 292
Grape juice, unsweetened 84.12 0.56 0.08 14.96 0.10 0.29 — 3.77 1.82 281
Lemon juice 92.46 0.40 0.29 6.48 0.40 0.36 — 3.99 1.73 309
Lime juice, unsweetened 92.52 0.25 0.23 6.69 0.40 0.31 — 3.99 1.73 309
Orange juice 89.01 0.59 0.14 9.85 0.20 0.41 –0.4 3.90 1.76 297
Pineapple juice, unsweetened 85.53 0.32 0.08 13.78 0.20 0.30 — 3.81 1.81 286
Prune juice 81.24 0.61 0.03 17.45 1.00 0.68 — 3.71 1.87 271
Tomato juice 93.90 0.76 0.06 4.23 0.40 1.05 — 4.03 1.71 314
Cranberry-apple juice drink 82.80 0.10 0.0 17.10 0.10 0.0 — 3.73 1.84 277
Cranberry-grape juice drink 85.60 0.20 0.10 14.00 0.10 0.10 — 3.81 1.80 286
Fruit punch drink 88.00 0.0 0.0 11.90 0.10 0.10 — 3.87 1.78 294
Club soda 99.90 0.0 0.0 0.0 0.0 0.10 — 4.17 1.63 334
Cola 89.40 0.0 0.0 10.40 0.0 0.10 — 3.90 1.76 299
Cream soda 86.70 0.0 0.0 13.30 0.0 0.10 — 3.83 1.79 290
Ginger ale 91.20 0.0 0.0 8.70 0.0 0.0 — 3.95 1.73 305
Grape soda 88.80 0.0 0.0 11.20 0.0 0.10 — 3.89 1.77 297
Lemon-lime soda 89.50 0.0 0.0 10.40 0.0 0.10 — 3.90 1.76 299
Orange soda 87.60 0.0 0.0 12.30 0.0 0.10 — 3.86 1.78 293
Root beer 89.30 0.0 0.0 10.60 0.0 0.10 — 3.90 1.76 298
Chocolate milk, 2% fat 83.58 3.21 2.00 10.40 0.50 0.81 — 3.78 1.83 279
Miscellaneous
Honey 17.10 0.30 0.0 82.40 0.20 0.20 — 2.03 — 57
Maple syrup 32.00 0.00 0.20 67.20 0.0 0.60 — 2.41 — 107
Popcorn, air-popped 4.10 12.00 4.20 77.90 15.10 1.80 — 2.04 — 14
oil-popped 2.80 9.00 28.10 57.20 10.00 2.90 — 1.99 — 9
Yeast, baker’s, compressed 69.00 8.40 1.90 18.10 8.10 1.80 — 3.55 2.17 230
*Composition data from USDA (1996). Initial freezing point data from Table 1 in Chapter 30 of the 1993 ASHRAE Handbook—Fundamentals. Specific heats calculated from equa-
tions in this chapter. Latent heat of fusion obtained by multiplying water content expressed in decimal form by 334 kJ/kg, the heat of fusion of water (Table 1 in Chapter 30 of the
1993 ASHRAE Handbook—Fundamentals).
Fundamentals of Refrigeration SI, Second Edition 379

Table C-3 Heat of Respiration of Fresh Fruits and Vegetables Held at Various Temperaturesa
Heat of Respiration (mW/kg)
Commodity 0°C 5°C 10°C 15°C 20°C 25°C Reference
Apples
Yellow, transparent 20.4 35.9 — 106.2 166.8 — Wright et al. (1954)
Delicious 10.2 15.0 — — — — Lutz and Hardenburg (1968)
Golden Delicious 10.7 16.0 — — — — Lutz and Hardenburg (1968)
Jonathan 11.6 17.5 — — — — Lutz and Hardenburg (1968)
McIntosh 10.7 16.0 — — — — Lutz and Hardenburg (1968)
Early cultivars 9.7-18.4 15.5-31.5 41.2-60.6 53.6-92.1 58.2-121.2 — IIR (1967)
Late cultivars 5.3-10.7 13.6-20.9 20.4-31.0 27.6-58.2 43.6-72.7 — IIR (1967)
Average of many 6.8-12.1 15.0-21.3 — 40.3-91.7 50.0-103.8 — Lutz and Hardenburg (1968)
cultivars
Apricots 15.5-17.0 18.9-26.7 33.0-55.8 63.0-101.8 87.3-155.2 — Lutz and Hardenburg (1968)
Artichokes, globe 67.4-133.4 94.6-178.0 16.2-291.5 22.9-430.2 40.4-692.0 — Rappaport and Watada (1958),
Sastry et al. (1978)
Asparagus 81.0-237.6 162.0-404.5 318.1-904.0 472.3-971.4 809.4-1484.0 — Lipton (1957), Sastry et al. (1978)
Avocados *b *b — 183.3-465.6 218.7-1029.1 — Biale (1960), Lutz and Hardenburg
(1968)
Bananas
Green *b *b †b 59.7-130.9 87.3-155.2 — IIR (1967)
Ripening *b *b †b 37.3-164.9 97.0-242.5 — IIR (1967)
Beans
Lima, unshelled 31.0-89.2 58.2-106.7 — 296.8-369.5 393.8-531.5 — Lutz and Hardenburg (1968),
Tewfik and Scott (1954)
shelled 52.4-103.8 86.3-180.9 — — 627.0-801.1 — Lutz and Hardenburg (1968),
Tewfik and Scott (1954)
Snap *b 101.4-103.8 162.0-172.6 252.2-276.4 350.6-386.0 — Ryall and Lipton (1972),
Watada and Morris (1966)
Beets, red, roots 16.0-21.3 27.2-28.1 34.9-40.3 50.0-68.9 — — Ryall and Lipton (1972),
Smith (1957)
Berries
Blackberries 46.6-67.9 84.9-135.8 155.2-281.3 208.5-431.6 388.0-581.9 — IIR (1967)
Blueberries 6.8-31.0 27.2-36.4 — 101.4-183.3 153.7-259.0 — Lutz and Hardenburg (1968)
Cranberries *b 12.1-13.6 — — 32.5-53.8 — Anderson et al. (1963), Lutz and
Hardenburg (1968)
Gooseberries 20.4-25.7 36.4-40.3 — 64.5-95.5 — — Lutz and Hardenburg (1968),
Smith (1966)
Raspberries 52.4-74.2 91.7-114.4 82.4-164.9 243.9-300.7 339.5-727.4 — Haller et al. (1941), IIR (1967),
Lutz and Hardenburg (1968)
Strawberries 36.4-52.4 48.5-98.4 145.5-281.3 210.5-273.5 303.1-581.0 501.4-625.6 IIR (1967), Lutz and Hardenburg
(1968), Maxie et al. (1959)
Broccoli, 55.3-63.5 102.3-474.8 — 515.0-1008.2 824.9-1011.1 1155.2-1661.0 Morris (1947), Lutz and Hardenburg
sprouting (1968), Scholz et al. (1963)
Brussels sprouts 45.6-71.3 95.5-144.0 187.2-250.7 283.2-316.7 267.2-564.0 — Sastry et al. (1978), Smith (1957)
Cabbage
Penn Statec 11.6 28.1-30.1 — 66.4-94.1 — — Van den Berg and Lentz (1972)
White, winter 14.5-24.2 21.8-41.2 36.4-53.3 58.2-80.0 106.7-121.2 — IIR (1967)
spring 28.1-40.3 52.4-63.5 86.3-98.4 159.1-167.7 — — Sastry et al. (1978), Smith (1957)
Red, early 22.8-29.1 46.1-50.9 70.3-824.2 109.1-126.1 164.9-169.7 — IIR (1967)
Savoy 46.1-63.0 75.2-87.3 155.2-181.9 259.5-293.4 388.0-436.5 — IIR (1967)
Carrots, roots
Imperator, Texas 45.6 58.2 93.1 117.4 209.0 — Scholz et al. (1963)
Main crop, 10.2-20.4 17.5-35.9 29.1-46.1 86.8-196.4 — — Smith(1957)
United Kingdom at 18°C
Nantes, Canadad 9.2 19.9 — 64.0-83.9 — — Van den Berg and Lentz (1972)
Cauliflower
Texas 52.9 60.6 100.4 136.8 238.1 — Scholz et al. (1963)
United Kingdom 22.8-71.3 58.2-81.0 121.2-144.5 199.8-243.0 — — Smith (1957)
Celery
New York, white 21.3 32.5 — 110.6 191.6 — Lutz and Hardenburg (1968)
United Kingdom 15.0-21.3 27.2-37.8 58.2-81.0 115.9-124.1 — — Smith(1957)
at 18°C
Utah, Canadae 15.0 26.7 — 88.3 — — Van den Berg and Lentz (1972)
380 Appendix C Supplemental Tables

Table C-3 Heat of Respiration of Fresh Fruits and Vegetables Held at Various Temperaturesa (Continued)
Heat of Respiration (mW/kg)
Commodity 0°C 5°C 10°C 15°C 20°C 25°C Reference
Cherries
Sour 17.5-39.3 37.8-39.3 — 81.0-148.4 115.9-148.4 157.6-210.5 Hawkins (1929), Lutz and
Hardenburg (1968)
Sweet 12.1-16.0 28.1-41.7 — 74.2-133.4 83.4-94.6 — Gerhardt et al. (1942), Lutz and
Hardenburg (1968), Micke et al.
(1965)
Corn, sweet with 126.1 230.4 332.2 483.0 855.5 1207.5 Scholz et al. (1963)
husk, Texas
Cucumbers, *b *b 68.4-85.8 71.3-98.4 92.1-142.6 — Eaks and Morris (1956)
California at 13°C
Figs, Mission — 23.5-39.3 65.5-68.4 145.5-187.7 168.8-281.8 252.2-281.8 Claypool and Ozbek (1952), Lutz
and Hardenburg (1968)
Garlic 8.7-32.5 17.5-28.6 27.2-28.6 32.5-81.0 29.6-53.8 — Mann and Lewis (1956), Sastry et al.
(1978)
Grapes
Labrusca, Concord 8.2 16.0 — 47.0 97.0 114.4 Lutz (1938), Lutz and Hardenburg
(1968)
Vinifera, Emperor 3.9-6.8 9.2-17.5 2.42 29.6-34.9 — 74.2-89.2 Lutz and Hardenburg (1968),
Pentzer et al. (1933)
Thompson 5.8 14.1 22.8 — — — Wright et al. (1954)
seedless
Ohanez 3.9 9.7 21.3 — — — Wright et al. (1954)
Grapefruit
California Marsh *b *b *b 34.9 52.4 64.5 Haller et al. (1945)
Florida *b *b *b 37.8 47.0 56.7 Haller et al. (1945)
Horseradish 24.2 32.0 78.1 97.0 132.4 — Sastry et al. (1978)
Kiwifruit 8.3 19.6 38.9 — 51.9-57.3 — Saravacos and Pilsworth (1965)
Kohlrabi 29.6 48.5 93.1 145.5 — — Sastry et al. (1978)
Leeks 28.1-48.5 58.2-86.3 159.1-202.2 245.4-346.7 — — Sastry et al. (1978), Smith (1957)
Lemons, California, *b *b *b 47.0 67.4 77.1 Haller et al. (1945)
Eureka
Lettuce
Head, California 27.2-50.0 39.8-59.2 81.0-118.8 114.4-121.2 178.0 — Sastry et al. (1978)
Texas 31.0 39.3 64.5 106.7 168.8 2.4 at 27°C Lutz and Hardenburg, (1968), Watt
and Merrill (1963)
Leaf, Texas 68.4 86.8 116.9 186.7 297.8 434.5 Scholz et al. (1963)
Romaine, Texas — 61.6 105.2 131.4 203.2 321.5 Scholz et al. (1963)
Limes, Persian *b *b 7.8-17.0 17.5-31.0 20.4-55.3 44.6-134.8 Lutz and Hardenburg (1968)
Mangoes *b *b — 133.4 222.6-449.1 356.0 Gore (1911), Karmarkar and Joshe
(1941b), Lutz and Hardenburg
(1968)
Melons
Cantaloupes *b 25.7-29.6 46.1 99.9-114.4 132.4-191.6 184.8-211.9 Lutz and Hardenburg (1968), Sastry
et al. (1978), Scholz et al. (1963)
Honeydew — *b 23.8 34.9-47.0 59.2-70.8 78.1-102.3 Lutz and Hardenburg (1968), Pratt
and Morris (1958), Scholz (1963)
Watermelon *b *b 22.3 — 51.4-74.2 — Lutz and Hardenburg (1968),
Scholz et al. (1963)
Mintl 23.8-44.5 89.0 225.6-270.1 311.6-403.6 492.7-673.7 762.7-940.8 Hruschka and Want (1979)
Mushrooms 83.4-129.5 210.5 — — 782.2-938.9 — Lutz and Hardenburg (1968),
Smith (1964)
Nuts (kind not 2.4 4.8 9.7 9.7 14.5 — IIR (1967)
specified)
Okra, Clemson *b — 259.0 432.6 774.5 1024 Scholz et al. (1963)
at 29°C
Olives, Manzanillo *b *b — 64.5-115.9 114.4-145.5 121.2-180.9 Maxie et al. (1959)
Onions
Dry, Autumn 6.8-9.2 10.7-19.9 — 14.7-28.1 — — Van den Berg and Lentz (1972)
Spicef
White Bermuda 8.7 10.2 21.3 33.0 50.0 83.4 Scholz et al. (1963)
at 27°C
Green, New Jersey 31.0-65.9 51.4-202.2 107.2-174.6 195.9-288.6 231.6-460.8 290.0-622.2 Lutz and Hardenburg (1968)
Fundamentals of Refrigeration SI, Second Edition 381

Table C-3 Heat of Respiration of Fresh Fruits and Vegetables Held at Various Temperaturesa (Continued)
Heat of Respiration (mW/kg)
Commodity 0°C 5°C 10°C 15°C 20°C 25°C Reference
Oranges
Florida 9.2 18.9 36.4 62.1 89.2 105.2 at 27°C Haller et al. (1945)
California, w. navel *b 18.9 40.3 67.4 81.0 107.7 Haller et al. (1945)
California, *b 13.6 34.9 37.8 52.4 62.1 Haller et al. (1945)
Valencia
Papayas *b *b 33.5 44.6-64.5 — 115.9-291.0 Jones (1942), Pantastico (1974)
Parsleyl 98.0-136.5 195.9-252.3 388.8-486.7 427.4-661.9 581.7-756.8 914.1-1012.0 Hruschka and Want (1979)
Parsnips
United Kingdom 34.4-46.1 26.2-51.9 60.6-78.1 95.5-127.1 — — Smith (1957)
Canada, Hollow 10.7-24.2 18.4-45.6 — 64.0-137.2 — — Van den Berg and Lentz (1972)
Crowng
Peaches
Elberta 11.2 19.4 46.6 101.8 181.9 266.7 Haller et al. (1932)
at 27°C
Several cultivars 12.1-18.9 18.9-27.2 — 98.4-125.6 175.6-303.6 241.5-361.3 Lutz and Hardenburg (1968)
Peanuts
Curedh 0.05 at 1.7°C 0.5 at 30°C Thompson et al. (1951)
Not cured, 42.0 at 30°C Schenk (1959, 1961)
Virginia Bunchi
Dixie Spanish 24.5 at 30°C Schenk (1959, 1961)
Pears
Bartlett 9.2-20.4 15.0-29.6 — 44.6-178.0 89.2-207.6 — Lutz and Hardenburg (1968)
Late ripening 7.8-10.7 17.5-41.2 23.3-55.8 82.4-126.1 97.0-218.2 — IIR (1967)
Early ripening 7.8-14.5 21.8-46.1 21.9-63.0 101.8-160.0 116.4-266.7 — IIR (1967)
Peas
Green-in-pod 90.2-138.7 163.4-226.5 — 530.1-600.4 728.4-1072.2 1018.4-1118.3 Lutz and Hardenburg (1968),
Tewfik and Scott (1954)
shelled 140.2-224.1 234.7-288.7 — — 1035-1630 — Lutz and Hardenburg (1968),
Tewfik and Scott (1954)
Peppers, sweet *b *b 42.7 67.9 130.0 — Scholz et al. (1963)
Persimmons 17.5 34.9-41.7 59.2-71.3 86.3-118.8 Gore (1911), Lutz and Hardenburg
(1968)
Pineapple
Mature green *b *b 165 38.3 71.8 105.2 at 27°C Scholz et al. (1963)
Ripening *b *b 22.3 53.8 118.3 185.7 Scholz et al. (1963)
Plums, Wickson 5.8-8.7 11.6-26.7 26.7-33.9 35.4-36.9 53.3-77.1 82.9-210.5 Claypool and Allen (1951)
Potatoes
California white, rose
immature *b 34.9 41.7-62.1 41.7-91.7 53.8-133.7 Sastry et al. (1978)
mature *b 17.5-20.4 19.7-29.6 19.7-34.9 19.7-47.0 Sastry et al. (1978)
very mature *b 15.0-20.4 20.4 20.4-29.6 27.2-35.4 Sastry et al. (1978)
Katahdin, Canada j *b 11.6-12.6 23.3-30.1 Van den Berg and Lentz (1972)
Kennebec *b 10.7-12.6 12.6-26.7 Van den Berg and Lentz (1972)
Radishes
With tops 43.2-51.4 56.7-62.1 91.7-109.1 207.6-230.8 368.1-404.5 469.4-571.8 Lutz and Hardenburg (1968)
Topped 16.0-17.5 22.8-24.2 44.6-97.0 82.4-97.0 141.6-145.5 199.8-225.5 Lutz and Hardenburg (1968)
Rhubarb, topped 24.2-39.3 32.5-53.8 91.7-134.8 118.8-168.8 Hruschka (1966)
Rutabaga, 5.8-8.2 14.1-15.1 31.5-46.6 Van den Berg and Lentz (1972)
Laurentian, Canadak
Spinach
Texas 136.3 328.3 530.5 682.3 Scholz et al. (1963)
United Kingdom, 34.4-63.5 81.0-95.5 173.6-222.6 549.0-641.6 Smith (1957)
summer at 18°C
winter 51.9-75.2 86.8-186.7 202.2-306.5 578.1-722.6 Smith (1957)
at 18°C
Squash
Summer, yellow, †b †b 103.8-109.1 222.6-269.6 252.2-288.6 Lutz and Hardenburg (1968)
straight-neck
b *b
Winter butternut * — — — 219.7-362.3 Lutz and Hardenburg (1968)
382 Appendix C Supplemental Tables

Table C-3 Heat of Respiration of Fresh Fruits and Vegetables Held at Various Temperaturesa (Continued)
Heat of Respiration (mW/kg)
Commodity 0°C 5°C 10°C 15°C 20°C 25°C Reference
Sweet Potatoes
Cured, Puerto Rico *b *b †b 47.5-65.5 Lewis and Morris (1956)
Yellow Jersey *b *b †b 65.5-68.4 Lewis and Morris (1956)
Noncured *b *b *b 84.9 160.5-217.3 Lutz and Hardenburg (1968)
Tomatoes
Texas, mature *b *b *b 60.6 102.8 126.6 Scholz et al. (1963)
green at 27°C
ripening *b *b *b 79.1 120.3 143.1 Scholz et al. (1963)
at 27°C
California, mature *b *b *b 71.3-103.8 88.7-142.6 Workman and Pratt (1957)
green
Turnip, roots 25.7 28.1-29.6 63.5-71.3 71.3-74.2 — Lutz and Hardenburg (1968)
Watercressl 44.5 133.6 270.1-359.1 403.6-581.7 896.3-1032.8 1032.9-1300.0 Hruschka and Want (1979)
a h
Column headings indicate temperatures at which respiration rates were determined, within 1 Shelled peanuts with about 7% moisture. Respiration after 60 hours curing
K, except where the actual temperatures are given. was almost negligible, even at 30°C.
b
The symbol * denotes a chilling temperature. The symbol † denotes the temperature is bor- i
Respiration for freshly dug peanuts, not cured, with about 35-40% moisture.
derline, not damaging to some cultivars if exposure is short. During curing, peanuts in the shell were dried-about 5-6% moisture, and in
c
Rates are for 30 to 60 days and 60 to 120 days storage, the longer storage having the higher roasting are dried further-about 2% moisture.
rate, except at 0°C, where they were the same. j
d
Rates are for 30 to 60 days and 120 to 180 days storage, respiration increasing with time only Rates are for 30-60 days and 120-180 days with rate declining with time at 5°C
at 15°C. but increasing at 15°C as sprouting started.
e k
Rates are for 30 to 60 days storage. Rates are for 30-60 days and 120-180 days; rates increased with time, espe-
f cially at 15°C where sprouting occurred.
Rates are for 30 to 60 days and 120 to 180 days storage; rates increased with time at all tem-
l
peratures as dormancy was lost. Rates are for 1 day after harvest.
g
Rates are for 30 to 60 days and 120 to 180 days; rates increased with time at all temperatures.
Fundamentals of Refrigeration SI, Second Edition 383

References and Bibliography


ANLA. 2004. American standard for nursery stock. ANSI/ANLA Standard
Z60.1-2004. American Nursery & Landscape Association, Washington,
D.C.
Hunt, A.B. 1987. Protecting perishable foods during transport by truck. USDA
Handbook 669. U.S. Department of Agriculture, Washington, D.C.
IIR. 1986. General principles for the freezing, storage and thawing of food-
stuffs: Recommendations for the processing and handling of frozen foods,
3rd ed. International Institute of Refrigeration, Paris.
IIR. 1990. Principles of refrigerated preservation of perishable foodstuffs:
Manual of refrigerated storage in the warmer developing countries. Inter-
national Institute of Refrigeration, Paris.
IIR. 1993. Cold stores guide. International Institute of Refrigeration, Paris.
IIR. 2000. Recommendations for chilled storage of perishable produce. Inter-
national Institute of Refrigeration, Paris.
ISO. 1974. Avocados—Guide for storage and transport. Standard 2295. Inter-
national Organization for Standardization, Geneva.
Kader, A.A. 1986. Biochemical basis for effect of controlled and modified
atmospheres on fruits and vegetables. Food Technology 40(5):99–100 and
102–104.
Scott, V.N. 1989. Interaction of factors to control microbial spoilage of refrig-
erated foods. Journal of Food Protection 52(6):431–435.
Schlimme, D.V., M.A. Smith, and L.M. Ali. 1991. Influence of freezing rate,
storage temperature, and storage duration on the quality of cooked turkey
breast roll. ASHRAE Transactions 97(1):214–220.
Schlimme, D.V., M.A. Smith, and L.M. Ali. 1991. Influence of freezing rate,
storage temperature, and storage duration on the quality of turkey frank-
furters. ASHRAE Transactions 97(1):221–227.
Tressler, D.K. and C.F. Evers. 1957. The freezing preservation of foods,
3rd ed. AVI Publishing, Westport, CT.
Valenzuela, S.G., D.D. Delgado, and D.R. Ramirez. 1972. Handling, storage
and transport systems for exported refrigerated perishable foods. Revista
del Instituto de Investigaciones Technologicas, Bogotá, Columbia.
Webb, B.H., A.H. Johnson, and J.A. Alford. 1973. Fundamentals of Dairy
Chemistry. AVI Publishing, Westport, CT.
Skill Development
Exercises
Complete the student information form and answer the following questions to
the Skill Development Exercises for each chapter. To receive credit, all ques-
tions must be answered, and student information must be provided. Where
computation is required, show your work in the space provided. Email (pre-
ferred) or fax your answer sheets to the address or number provided below. The
correct answers with explanations will be returned to you.

ASHRAE Learning Institute


Kelly Arnold, Administrative Assistant for Professional Development
Email: edu@ashrae.org
Fax: 770-539-2161
Fundamentals of Refrigeration SI, Second Edition

Student Information

Chapter 1 Skill Development Exercises


Name ________________________________________________________

Company/Department _________________________________________

Address ______________________________________________________

City ______________________________ State ______ Zip ____________

Telephone _________________________ Fax ________________________

E-mail ________________________________________________________

Student Number ______________________________________________

Skill Development Exercises for Chapter 1


Total number of questions: 6

1-1 After arriving home from a long day at work, you open your refrigerator door
and remove a bottle containing a cold beverage. What refrigeration-cycle pro-
cesses occurred to produce your cold drink?
Fundamentals of Refrigeration SI, Second Edition

1-2 Describe in your own words the meaning of the word refrigeration.
Chapter 1 Skill Development Exercises

1-3 For the vapor-compression refrigeration system, name the four major compo-
nents and their functions.
Fundamentals of Refrigeration SI, Second Edition

1-4 Describe in your own words the purpose of the heat pump-system.

Chapter 1 Skill Development Exercises


1-5 Explain the similarities and differences between the fields of air conditioning,
commercial refrigeration, and industrial refrigeration.
Fundamentals of Refrigeration SI, Second Edition

1-6 Name and describe three applications of refrigeration systems in both the air-
conditioning and industrial-refrigeration fields.
Chapter 1 Skill Development Exercises
Fundamentals of Refrigeration SI, Second Edition

Student Information

Chapter 2 Skill Development Exercises


Name ________________________________________________________

Company/Department _________________________________________

Address ______________________________________________________

City ______________________________ State ______ Zip ____________

Telephone _________________________ Fax ________________________

E-mail ________________________________________________________

Student Number ______________________________________________

Skill Development Exercises for Chapter 2


Total number of questions: 8

2-1 A water-cooled condenser is used to remove heat from a vapor-compression


refrigeration system. The temperature of the water entering the condenser is
10°C and the exiting temperature is 13.89°C. If the mass flow rate of water
through the condenser is 1.59 kg/s, what is the rate of heat rejected by the con-
denser (kJ/min)?
Fundamentals of Refrigeration SI, Second Edition

2-2 A vessel, cylindrical in shape and installed vertically, has a height and diameter
of 5.0 m and 1.5 m, respectively. When partially filled with liquid ammonia, a
Chapter 2 Skill Development Exercises

pressure gauge indicates 641.9 kPa (gage) for the internal pressure. If the
height of the liquid level is 3.0 m above the vessel’s bottom, what is the quality
of the ammonia in the vessel? The atmospheric pressure is 100 kPa.
Fundamentals of Refrigeration SI, Second Edition

2-3 Consider the generic component shown in Figure 2-7 where the following
information is known:

Chapter 2 Skill Development Exercises


m· 1 = 2.0 kg, m· 2 = 4.0 kg/s, m· 3 = 1.0 kg/s, m· 4 = 2.0 kg/s,
h 1 = 80 kJ/kg, h 2 = 30 kJ/kg,
h 3 = 120 kJ/kg, h 4 = 20 kJ/kg, h 5 = 40 kJ/kg,
·
Q out = 100 kJ

Using mass and energy balances on the component, determine m· 5 (kg/s)


and the power output (kJ/s).
Fundamentals of Refrigeration SI, Second Edition

2-4 An industrial refrigeration system is employed to maintain a freezer room at


–23.33°C. For this system, the evaporating temperature is –28.89°C and the
Chapter 2 Skill Development Exercises

condensing temperature is 35°C. What is the maximum COP for this refriger-
ation system?
Fundamentals of Refrigeration SI, Second Edition

2-5 A display freezer cabinet in a supermarket is claimed to operate at a cold tem-


perature of –17.78°C and reject heat to the surrounding air at 43.33°C. While

Chapter 2 Skill Development Exercises


maintaining these temperatures, the claim further states that 12,660 kJ/h are
withdrawn from the cold space, requiring 0.447 kW power input to the com-
pressor. Evaluate this claim and, using thermodynamics, determine whether it is
true.
Fundamentals of Refrigeration SI, Second Edition

2-6 A heat-pump cycle is applied to a residential building whose interior is kept at


20°C when the outdoor temperature is –1.11°C. The rate of heat transfer
Chapter 2 Skill Development Exercises

through the building’s structure (walls, roof, etc.) is 1530 kJ/h per degree Cel-
sius temperature difference between the indoor and outdoor temperatures. Find
the minimum power needed to drive this heat-pump cycle (kW).
Fundamentals of Refrigeration SI, Second Edition

2-7 The suction line to an ideal compressor in a low-temperature application sup-


plies R-23 saturated vapor at 62.22°C. If the refrigerant flow rate is 10 kg/min

Chapter 2 Skill Development Exercises


and the discharge pressure is 1034.21 kPa, what is the power (kJ/min) input to
the compressor?
Fundamentals of Refrigeration SI, Second Edition

2-8 An expansion device is used to control the flow of R-22 to an ideal evaporator.
Saturated liquid refrigerant at 26.67°C with a flow of 20 kg/min enters the
Chapter 2 Skill Development Exercises

expansion device. The temperature of the evaporator is maintained at –20.55°C,


and the refrigerant leaving the evaporator is saturated vapor. Determine (1) the
pressure in the evaporator (kPa); (2) the quality of refrigerant downstream
from the expansion device; and (3) the rate of heat transfer to the evaporator
(kJ/min).
Fundamentals of Refrigeration SI, Second Edition

Student Information

Chapter 3 Skill Development Exercises


Name ________________________________________________________

Company/Department _________________________________________

Address ______________________________________________________

City ______________________________ State ______ Zip ____________

Telephone _________________________ Fax ________________________

E-mail ________________________________________________________

Student Number ______________________________________________

Skill Development Exercises for Chapter 3


Total number of questions: 9

3-1 Consider an ideal single-stage system, as shown in Figure 3-1, using R-22 as
the refrigerant. If the evaporating temperature is 7.22°C, the condensing tem-
perature is 35°C, and the refrigerant mass flow rate is 0.453 kg/min, calculate
(1) enthalpies for all state points in the cycle, (2) refrigerating capacity rate, (3)
power needed to operate the compressor, (4) the heat rejection rate from the
condenser, and (5) the COP. Additionally, describe the effect of the evaporating
temperature increase on Q· in , Q· out , and W· c .
Fundamentals of Refrigeration SI, Second Edition

3-1 (contd.)
Chapter 3 Skill Development Exercises
Fundamentals of Refrigeration SI, Second Edition

3-2 Reconsider Example 3-2, where an ideal two-stage refrigeration system oper-
ates with R-22 as the refrigerant and incorporates a flash tank. The evaporat-

Chapter 3 Skill Development Exercises


ing temperature drops to –40°C. Remaining the same are the condensing
temperature at 35°C and the intermediate pressure at 482.63 kPa. On the basis
of 3.52 kW of refrigerating capacity, determine (1) the enthalpies for all
refrigerant state points in the cycle, (2) refrigerant mass flow rates through the
booster and high-stage compressors, (3) the heat transfer rate rejected by the
condenser, (4) the ideal compressor power input (isentropic efficiency is 1) for
both the booster and high-stage compressors, and (5) the COP and kW/ton.
Fundamentals of Refrigeration SI, Second Edition

3-2 (contd.)
Chapter 3 Skill Development Exercises
Fundamentals of Refrigeration SI, Second Edition

3-3 A subcooler is used in a refrigeration system where ammonia serves as the


refrigerant. The subcooler is configured as shown in Figure 3-10. The entering

Chapter 3 Skill Development Exercises


liquid mass flow is 45.36 kg/min. The liquid leaving the condenser is saturated
at 26.67°C. The liquid leaving the subcooler is 15.55°C and is at the condens-
ing pressure. The liquid and vapor inside the vessel are saturated at –1.11°C.
Calculate the mass flow rates of the subcooled liquid and the mass flow rate of
the vapor to the compressor.
Fundamentals of Refrigeration SI, Second Edition

3-4 A compressor operates in a system using R-134a as the refrigerant where the
mass flow rate is 3.63 kg/min. The conditions at the suction port of the com-
Chapter 3 Skill Development Exercises

pressor are 172.37 kPa and –6.67°C. The pressure at the discharge port is
1206.6 kPa. This compressor is known to have an isentropic efficiency of 80%.
Calculate (1) actual refrigerant temperature at the discharge port of the com-
pressor and (2) actual power input requirement to the compressor.
Fundamentals of Refrigeration SI, Second Edition

3-5 Rework Exercise 3-2 where all the conditions remain the same but both the
booster and high-stage compressors have an isentropic efficiency of 75%. R-22

Chapter 3 Skill Development Exercises


is the refrigerant. Also, determine the booster and high-stage compressor dis-
charge temperatures.
Fundamentals of Refrigeration SI, Second Edition

3-5 (contd.)
Chapter 3 Skill Development Exercises
Fundamentals of Refrigeration SI, Second Edition

3-6 Sketch and label a diagram showing the major components and interconnecting
piping for a two-stage mechanically pumped liquid recirculation system. The

Chapter 3 Skill Development Exercises


refrigerating load is divided between a low-temperature and an intermediate-
temperature load.
Fundamentals of Refrigeration SI, Second Edition

3-7 Explain why the thermal performance of the direct-expansion coil is less than
that of an equally sized flooded or liquid-recirculated coil operating under the
Chapter 3 Skill Development Exercises

same conditions. Why is this important for coils used in low-temperature appli-
cations?
Fundamentals of Refrigeration SI, Second Edition

3-8 Why does a pressure drop in the refrigerant flowing through an evaporator coil
increase the evaporating temperature of the coil?

Chapter 3 Skill Development Exercises


Fundamentals of Refrigeration SI, Second Edition

3-9 List the similarities and differences between unitary systems and field-erected
refrigeration systems.
Chapter 3 Skill Development Exercises
Fundamentals of Refrigeration SI, Second Edition

Student Information

Chapter 4 Skill Development Exercises


Name ________________________________________________________

Company/Department _________________________________________

Address ______________________________________________________

City ______________________________ State ______ Zip ____________

Telephone _________________________ Fax ________________________

E-mail ________________________________________________________

Student Number ______________________________________________

Skill Development Exercises for Chapter 4


Total number of questions: 5

4-1 A water pipe whose surface temperature is 7.22°C passes through a basement
where the air temperature is 20°C. What is the maximum relative humidity that
the air may have before condensation occurs on the surface of the pipe?
Fundamentals of Refrigeration SI, Second Edition

4-2 An air coil has an inlet face area of 2.6 m2 operating at the following air inlet
conditions: average air velocity = 2.03 m/s; relative humidity = 70%; dry-bulb
Chapter 4 Skill Development Exercises

temperature = 32.22°C. The outlet dry-bulb temperature is 23.89°C, and the


temperature of the wetted surface inside the coil is 18.33°C. Using the psy-
chrometric chart, find the relative humidity of the air leaving the coil and the
refrigerating rate for this coil at these conditions.
Fundamentals of Refrigeration SI, Second Edition

4-3 The fins on an air coil are fabricated from copper sheets 0.07874 cm thick. The
coil’s tubing is also copper with a nominal outside diameter equal to 1.27 cm

Chapter 4 Skill Development Exercises


(actual outside diameter is 1.5875 cm). The spacing between tubes follows the
pattern shown in Figure 4-11, where the horizontal distance between tube cen-
terlines is 3.81 cm and the vertical distance between centers is 4.7 cm. The air-
side convection heat transfer coefficient is 45.43 W/m2·°C and the thermal con-
ductivity of copper is 1271.93 W/m2·°C. Use Figures 4-10 and 4-11 to estimate
the fin efficiency.
Fundamentals of Refrigeration SI, Second Edition

4-4 Explain the physical events that occur with a refrigerant in a flooded air coil
with (1) a sudden decrease in refrigeration load and (2) a sudden increase in
Chapter 4 Skill Development Exercises

load.

4-5 A drinking-water cooler is designed to supply 6.667°C water at a continuous


flow rate of 3.674 kg/h. A shell-and-coil cooler is used to chill the water. If the
temperature of the supply water is 26.67°C, estimate the product of UA for the
coil. Assume that the refrigerant enters the coil as a saturated liquid and leaves
as a saturated vapor at 1.67°C with no pressure drop.
Fundamentals of Refrigeration SI, Second Edition

Student Information

Chapter 5 Skill Development Exercises


Name ________________________________________________________

Company/Department _________________________________________

Address ______________________________________________________

City ______________________________ State ______ Zip ____________

Telephone _________________________ Fax ________________________

E-mail ________________________________________________________

Student Number ______________________________________________

Skill Development Exercises for Chapter 5


Total number of questions: 7

5-1 Describe the suction, compression, and discharge processes for a reciprocating
compressor in conjunction with the pressure-volume diagram. Also, discuss
the two limiting cases where the suction and discharge pressures are equal and
the suction pressure is sufficiently low that the compressor no longer pumps
refrigerant.
Fundamentals of Refrigeration SI, Second Edition

5-1 (contd.)
Chapter 5 Skill Development Exercises

5-2 A reciprocating compressor has the following specifications: number of cylin-


ders = 6, bore = 11.43 cm, stroke = 11.43 cm, volume ratio rvc = 0.04, and oper-
ating speed = 1200 rpm. This compressor is planned for use in an ammonia
refrigeration system. When the evaporating saturation temperature is –12.22°C
and the condensing temperature is 35°C, the power required by the compressor
is –73.7 kW, which produces 257.4 kW of refrigeration. For this compressor
operating at these conditions, estimate the following quantities: pressure ratio
across the compressor, clearance volumetric efficiency, actual volumetric effi-
ciency, and isentropic compressor efficiency.
Fundamentals of Refrigeration SI, Second Edition

5-2 (contd.)

Chapter 5 Skill Development Exercises


Fundamentals of Refrigeration SI, Second Edition

5-3 Consider a reciprocating compressor that consists of eight cylinders with cylin-
der dimensions of 10.16 × 12.065 cm for the bore and stroke, respectively. The
Chapter 5 Skill Development Exercises

refrigerant is CO2 (R-744), which enters the suction port as a saturated vapor at
–40°C and is discharged at a pressure corresponding to a –17.78°C condensing
pressure. If the operating speed of the compressor is 1500 rpm, determine (1)
compressor displacement rate (cm3/s), (2) refrigerant mass flow rate (kg/min),
(3) the pressure ratio across the compressor, and (4) the ideal (minimum)
power (kW) to drive the compressor.
Fundamentals of Refrigeration SI, Second Edition

5-4 Describe the features that result in the reciprocating compressor operating as a
variable-volume-ratio compressor. Compare the reciprocating compressor to

Chapter 5 Skill Development Exercises


the fixed-volume-ratio rotary twin-screw compressor.

5-5 A rotary twin-screw compressor is used in a single-stage industrial refrigera-


tion system with R-22 as the refrigerant. The evaporating temperature is –15°C
and the condensing temperature is 40.56°C. The exponent in Equation 5-18 is
adjusted to the value of 1.2 to account for heat transfer from the refrigerant
vapor to the lubricating oil. Under the condition of negligible pressure drop in
the suction and discharge piping, estimate the volume ratio, Vi, that is required
for the screw compressor to achieve a matching discharge pressure.
Fundamentals of Refrigeration SI, Second Edition

5-5 (contd.)
Chapter 5 Skill Development Exercises

5-6 For a centrifugal compressor, estimate the speed of the impeller tip that is
required to compress saturated R-123 vapor at 4.44°C to a pressure corre-
sponding to a condensing temperature of 37.78°C.
Fundamentals of Refrigeration SI, Second Edition

5-6 (contd.)

Chapter 5 Skill Development Exercises


5-7 An R-134a refrigeration system utilizes a compressor where the refrigerating
capacity and power requirement specified by the manufacturer are based on
saturated liquid leaving the condenser and saturated vapor entering the com-
pressor. At conditions of –6.67°C evaporating temperature and 37.78°C con-
densing temperature, predict the percentage of change in refrigerating capacity
and power required if the vapor leaving the evaporator and entering the com-
pressor has –6.67°C of superheat.
Fundamentals of Refrigeration SI, Second Edition

5-7 (contd.)
Chapter 5 Skill Development Exercises
Fundamentals of Refrigeration SI, Second Edition

Student Information

Chapter 6 Skill Development Exercises


Name ________________________________________________________

Company/Department _________________________________________

Address ______________________________________________________

City ______________________________ State ______ Zip ____________

Telephone _________________________ Fax ________________________

E-mail ________________________________________________________

Student Number ______________________________________________

Skill Development Exercises for Chapter 6


Total number of questions: 6

6-1 Using equations, explain why extended surfaces, or fins, are used with air-
cooled condensers.
Fundamentals of Refrigeration SI, Second Edition

6-2 A vapor-compression refrigeration system using R-134a provides a refrigerat-


ing capacity of 10.55 kW. The conditions at the exit of the evaporator are
Chapter 6 Skill Development Exercises

7.22°C and 241 kPa. The condenser operates at 60°C and 862 kPa. Estimate
the heat rejection ratio for this condenser.
Fundamentals of Refrigeration SI, Second Edition

6-3 In a manufacturer’s catalog, a particular air-cooled condenser is noted to have a


refrigerating capacity of 42.2 kW (at the evaporator) when the condensing tem-

Chapter 6 Skill Development Exercises


perature is 48.89°C and the evaporating temperature is 4.44°C. The air enters
the condenser at 32.22°C and exits at a temperature of 43.33°C. The refrigerant
is R-22. Determine the product UA for this condenser at the given conditions,
and estimate the condensing temperature if the refrigeration load drops to half
its original value, or 21.1 kW. (Hint: Begin with Figure 6-1 and approximate
the heat rejection ratio.)
Fundamentals of Refrigeration SI, Second Edition

6-4 Consider again the air-cooled condenser in problem 6-3 above operating at the
same conditions and capacity of 42.2 kW. However, it is now known that the
Chapter 6 Skill Development Exercises

relative humidity of the air entering the condenser is 60%. Find (1) the change
in enthalpy of the air as it passes through the condenser, (2) the mass flow rate
(kg/min) of the flowing air, and (3) the volumetric flow rate (m3/kg) at the inlet
to the condenser.
Fundamentals of Refrigeration SI, Second Edition

6-5 Consider an evaporative condenser starting operation with an ambient wet-bulb


temperature greater than the temperature of the water in the sump. Explain

Chapter 6 Skill Development Exercises


what happens to the air and water using a psychrometric chart and the operat-
ing temperature of the condenser.

6-6 A water chiller using R-123 is checked to determine whether purging of the
noncondensable gases is needed. Pressure and temperature measurements are
carefully made at a location in the system where the liquid and vapor are in
equilibrium downstream from the condenser. The results are 71.7 kPa and
43.33°C. The barometric pressure is measured using a barometer that indicates
754.9 mm. Should purging be recommended in this situation?
Fundamentals of Refrigeration SI, Second Edition

Student Information

Chapter 7 Skill Development Exercises


Name ________________________________________________________

Company/Department _________________________________________

Address ______________________________________________________

City ______________________________ State ______ Zip ____________

Telephone _________________________ Fax ________________________

E-mail ________________________________________________________

Student Number ______________________________________________

Skill Development Exercises for Chapter 7


Total number of questions: 6

7-1 Calculate the pressure drop for a 30.5 m length of a 15.2 cm steel Schedule 40
ammonia suction line. The mass rate flow is 45.4 kg/min from a –28.9°C evap-
orating temperature.
Fundamentals of Refrigeration SI, Second Edition

7-1 (contd.)
Chapter 7 Skill Development Exercises

7-2 Using the equations presented in the section “Fluid Flow in Pipes,” show that
the pressure drop over a length of pipe is inversely proportional to D raised to
the fifth power

1
 p  ------
 D 5
Fundamentals of Refrigeration SI, Second Edition

7-3 A steel piping layout for an ammonia liquid line is shown the figure below,
where all the pipe runs are horizontal. The ammonia liquid enters at a pressure of

Chapter 7 Skill Development Exercises


448 kPa (gage) and 4.44°C and has a mass flow rate of 136 kg/min. Estimate the
pressure drop for this piping layout and determine if the ammonia flashes before
it reaches the exit.
Fundamentals of Refrigeration SI, Second Edition

7-4 Calculate the influence that a pressure drop in a suction line has on the com-
pressor power to operate a refrigeration system using R-134a that consists of
Chapter 7 Skill Development Exercises

four components: compressor, condenser, expansion valve, and evaporator. The


refrigerating capacity is 14 kW. The condensing pressure is 1207 kPa, and the
refrigerant exits the condenser as a saturated liquid. The evaporating pressure
is 345 kPa, and superheated vapor leaves the evaporator at 15.55°C. The suc-
tion line between the evaporator and compressor is 4 m long, 1.27 cm nominal
diameter type-K copper tube (actual inside diameter is 1.34 cm). The compres-
sor’s isentropic efficiency is 70%. The pressure drop in the other lines that con-
vey refrigerant, in the condenser, and in the evaporator may be neglected. It is
suggested to proceed through the following intermediate steps:

1. Calculate the pressure drop in the suction line using R-134a thermody-
namic data. (Hint: Viscosity of the superheated vapor may be approximated
by the saturated vapor viscosity at the same temperature.)
2. Estimate the actual inlet conditions to the compressor. (Hint: Enthalpy dif-
ference from the inlet to exit of the suction line is approximately p   .)
3. Calculate the actual thermodynamic conditions at the discharge of the com-
pressor.
4. Calculate the actual compressor power input.
5. Recalculate the compressor power input if the suction line has no pressure
drop.
6. Compare the two estimated compressor powers by calculating a percentage
change relative to the no-pressure-drop ideal situation.
Fundamentals of Refrigeration SI, Second Edition

7-4 (contd.)

Chapter 7 Skill Development Exercises


Fundamentals of Refrigeration SI, Second Edition

7-5 The figure below shows a schematic of an evaporative condenser. The centrifu-
gal pump planned for use has the performance characteristics shown in
Chapter 7 Skill Development Exercises

Figure 7-9. The required flow is 1.06 m3/min with pressure difference of
110 kPa. Determine the following: impeller diameter, power needed to drive the
pump, the pump’s efficiency at this condition, the pump’s required NPSH, and
whether the height provided by the sump and inlet pipe is sufficient for this
application.
Fundamentals of Refrigeration SI, Second Edition

7-6 A recirculation system that uses ammonia as the refrigerant has an insulated
steel line delivering liquid to a cluster of evaporators. The refrigerating load

Chapter 7 Skill Development Exercises


at the evaporators is 700 kW, and the evaporating temperature is –40°C. The
circulation ratio is 4. The liquid line is a 50 mm Schedule 40 (internal diame-
ter of 5.25 cm) with roughness equal to 0.000381 cm. The equivalent length
of the pipe is 305 m with an elevation from the supply to the evaporator of
9.1 m, and the pressure drop across the balancing valve at the evaporator is
69 kPa. Determine the pressure that must be provided by the pumping system
to deliver the required ammonia to this evaporator cluster, and estimate the
temperature of the ammonia in the low-pressure receiver so that the liquid
entering the evaporator is –40°C.
Fundamentals of Refrigeration SI, Second Edition

Student Information

Chapter 8 Skill Development Exercises


Name ________________________________________________________

Company/Department _________________________________________

Address ______________________________________________________

City ______________________________ State ______ Zip ____________

Telephone _________________________ Fax ________________________

E-mail ________________________________________________________

Student Number ______________________________________________

Skill Development Exercises for Chapter 8


Total number of questions: 7

8-1 Refrigerant R-134a is used in conjunction with a capillary tube that has a bore
of 0.762 mm and length of 4060 mm. The operating conditions are 1448 kPa
condensing pressure and 0°C subcooling. Assume that heat transfer to the capil-
lary tube is negligible. Determine the mass flow of R-134a through the capillary
tube (kg/h). Also, if the same capillary tube operating at the same condensing
pressure had a quality of 20% at the inlet to the tube, what would be the new R-
134a mass flow rate?
Fundamentals of Refrigeration SI, Second Edition

8-1 (contd.)
Chapter 8 Skill Development Exercises

8-2 Explain why the thermostatic expansion valve should be called a superheat-
controlled valve.
Fundamentals of Refrigeration SI, Second Edition

8-3 R-22 passes through a thermostatic expansion valve without external equalizer
lines. As supplied by the manufacturer, this valve is set to open at an evaporator

Chapter 8 Skill Development Exercises


temperature of 1.67°C when the superheat is 5.56°C. What is the pressure dif-
ference between the top and bottom sides of the diaphragm needed to open the
valve? If the entering pressure to the evaporator is 690 kPa and the pressure
drop from the entrance to the exit of the evaporator is 13.8 kPa, what is the
degree of superheat needed to open the valve?
Fundamentals of Refrigeration SI, Second Edition

8-4 An expansion valve has a refrigerating capacity equal to 42.2 kW when the
pressure difference across the valve is 483 kPa. The conditions at which this
Chapter 8 Skill Development Exercises

valve is rated include only R-22 liquid (no vapor bubbles) at 37.8°C entering
and an evaporating temperature of 4.44°C. Estimate the new rating of this
valve if the pressure difference increases to 1172 kPa.
Fundamentals of Refrigeration SI, Second Edition

8-5 Explain the control action of a low-side float valve when the refrigeration load
increases. Use a plot of the refrigerant mass flow rate versus the suction pres-

Chapter 8 Skill Development Exercises


sure to explain your answer.

8-6 Reconsider the ideal Brayton refrigeration cycle from Example 8-3 where the
high-side pressure is now decreased to 1379 kPa (gage). All other values for the
cycle remain the same, including the air mass flow rate, which is 125.6 kg/min
or 2.1 kg/s. Determine the following: the net work rate required by the com-
pressor, the refrigeration rate, and the COP.
Fundamentals of Refrigeration SI, Second Edition

8-6 (contd.)
Chapter 8 Skill Development Exercises

8-7 Reconsider Example 8-3 again, where all the cycle information is unchanged
with the exception that the isentropic efficiencies of the compressor and tur-
bine (turbo expander) are now 80%. Calculate the net work rate required by the
compressor, the refrigeration rate, and the COP.
Fundamentals of Refrigeration SI, Second Edition

8-7 (contd.)

Chapter 8 Skill Development Exercises


Fundamentals of Refrigeration SI, Second Edition

Student Information

Chapter 9 Skill Development Exercises


Name ________________________________________________________

Company/Department _________________________________________

Address ______________________________________________________

City ______________________________ State ______ Zip ____________

Telephone _________________________ Fax ________________________

E-mail ________________________________________________________

Student Number ______________________________________________

Skill Development Exercises for Chapter 9


Total number of questions: 5

9-1 Describe the considerations important to determine the size for a suction line
accumulator. Use the concepts presented in association with high- and low-
pressure receivers to assist in your explanation.
Fundamentals of Refrigeration SI, Second Edition

9-2 A cylindrical vessel, oriented horizontally, is used as a high-pressure receiver in


a refrigeration system using R-22. The internal diameter is 1.83 m and the
Chapter 9 Skill Development Exercises

length is 3 m. The liquid depth in the receiver is 65% of the internal diameter.
The temperature of the R-22 is 29.4°C. (1) Find the mass of liquid using Equa-
tion 9-2, (2) verify your result using Table 9-1, and (3) calculate the percentage
of the total R-22 mass in the receiver that is vapor. Assume the ends of the ves-
sel are flat.
Fundamentals of Refrigeration SI, Second Edition

9-3 In an ammonia refrigeration system, a vertically oriented cylindrical vessel is


used as a low-pressure receiver. The temperature of the ammonia mixture

Chapter 9 Skill Development Exercises


entering is –6.67°C, and the temperature inside the receiver is –6.67°C. The
volume flow rate of the vapor entering is 56.6 m3/min. For a critical droplet
diameter equal to 0.04 mm, estimate the required diameter of the receiver to
achieve separation. Assume the ends of the vessel are flat.
Fundamentals of Refrigeration SI, Second Edition

9-4 R-22 is used in a liquid recirculation refrigeration system where a low-pressure


receiver is 60% full of liquid, oriented horizontally, and at a temperature of –40°C.
Chapter 9 Skill Development Exercises

The internal diameter of the vessel is 1.5 m, and its length is 3 m. If the vapor/liq-
uid mixture returned to the vessel enters at only one end and exits at the opposite
end, what is the largest vapor volume flow rate that the receiver can accept and
separate droplets at 0.25 mm in diameter? Assume the ends of the vessel are flat.
Fundamentals of Refrigeration SI, Second Edition

9-5 An ammonia refrigeration system has the following equipment and characteris-
tics for the low-pressure portion:

Chapter 9 Skill Development Exercises


Refrigerant Ammonia
Low temperature –29°C
Recirculation ratio, n 4
Evaporator penthouse 4 evaporator units
Evaporator feed type bottom feed
Evaporator unit internal volume 0.24 m3
Liquid supply line internal diameter 5.25 cm
Liquid supply line length 152.4 m
Vapor/liquid return line internal diameter 154.05 mm
Vapor/liquid return line length 152.4 m
Condenser internal volume 0.93 m3
Determine (1) the liquid surge volume (m3) needed in the low-pressure
receiver in the event of a power failure, (2) the liquid surge volume (m3) asso-
ciated with the defrost of one evaporator coil, and (3) the total refrigerant mass
(kg) in the low-pressure portion of the system.
Fundamentals of Refrigeration SI, Second Edition

Student Information

Chapter 10 Skill Development Exercises


Name ________________________________________________________

Company/Department _________________________________________

Address ______________________________________________________

City ______________________________ State ______ Zip ____________

Telephone _________________________ Fax ________________________

E-mail ________________________________________________________

Student Number ______________________________________________

Skill Development Exercises for Chapter 10


Total number of questions: 7

10-1 What is the significance of the Montreal Protocol to the HVAC&R industry?
Fundamentals of Refrigeration SI, Second Edition

10-2 A refrigeration system is required to maintain a cold space at –28.89°C, when


the refrigerant condensing temperature could be as high as 35°C. If the refrig-
Chapter 10 Skill Development Exercises

erating load is 700 kW, suggest a refrigerant for this system. Support your
selection with technical reasons.

10-3 A refrigerant not listed in the tables included in Chapter 10 is R-113. Using
ASHRAE Standard 34 classification criteria, identify the features of this refrig-
erant discernible from the designation.
Fundamentals of Refrigeration SI, Second Edition

10-4 A chiller unit using R-123 as the refrigerant is located in a mechanical room of
a building. The room’s dimensions are 6.1 m wide, 9.1 m long, and 3.66 m

Chapter 10 Skill Development Exercises


high. The pressure and temperature in the room are 101.325 kPa and 21.1°C,
respectively. Determine the mass of R-123 that must leak into the room to pro-
duce the maximum exposure that a worker is permitted to experience over a
40-hour work week.
Fundamentals of Refrigeration SI, Second Edition

10-5 Figure 10-5 shows two configurations that could result in a trapped liquid.
Sketch and label one other configuration that may also trap liquid.
Chapter 10 Skill Development Exercises
Fundamentals of Refrigeration SI, Second Edition

10-6 Consider the same commercial fabrication as given in Example 10-1, where
the refrigerant is changed to propane (R-290). With a refrigerant charge of

Chapter 10 Skill Development Exercises


25.4 kg R-290, does ASHRAE Standard 15 indicate that a machinery room
is required?
Fundamentals of Refrigeration SI, Second Edition

10-7 A 101.6 mm, schedule 40 pipe 30.5 m long contains liquid ammonia at –6.67°C
and 345 kPa. A solenoid-operated shutoff valve is located at the end of the pipe
Chapter 10 Skill Development Exercises

where the fluid is heading. The velocity of the flowing liquid is 1.83 m/s. If the
solenoid valve takes 0.02 s to close, estimate the momentary overpressure exerted
by the refrigerant in the pipe.
Fundamentals of Refrigeration SI, Second Edition

Student Information

Chapter 11 Skill Development Exercises


Name ________________________________________________________

Company/Department _________________________________________

Address ______________________________________________________

City ______________________________ State ______ Zip ____________

Telephone _________________________ Fax ________________________

E-mail ________________________________________________________

Student Number ______________________________________________

Skill Development Exercises for Chapter 11


Total number of questions: 6

11-1 Whole lean pork shoulders weighing 454 kg are brought into a stationary
freezing tunnel that has an initial temperature of 4.44°C. Over a 12-hour
period, the pork is cooled and frozen to a temperature of –23.33°C. Determine
the refrigerating capacity that must be available from the refrigeration system
supplying the stationary tunnel.
Fundamentals of Refrigeration SI, Second Edition

11-1 (contd.)
Chapter 11 Skill Development Exercises

11-2 The rate of temperature drop for any frozen product is always much faster than
the temperature drop rate when it was unfrozen. Explain the reason why, ignor-
ing the zero rate during the transition from unfrozen to frozen.
Fundamentals of Refrigeration SI, Second Edition

11-3 Golden Delicious apples are harvested over a 20-day harvest time. The apples
are placed in boxes, with each box containing 26.8 kg of apples. Each day,

Chapter 11 Skill Development Exercises


2000 boxes of apples that have an initial temperature of 23.89°C are brought
into the storage cooler. The temperature of the storage cooler is 1.67°C. The
average weight of each box when empty is 2 kg, and the box material has a
specific heat of 2.5 kJ/kg·°C. Additionally, assume that each day’s load is fully
cooled before the next day’s load is brought into the storage cooler. Estimate
the highest product load resulting from the apple harvest for the storage cooler.
Fundamentals of Refrigeration SI, Second Edition

11-4 Explain why Planck’s equation cannot be applied to cooling processes below
the freezing temperature of the product.
Chapter 11 Skill Development Exercises

11-5 A refrigerated warehouse receives a shipment of unfrozen turkeys at a tempera-


ture of 6.67°C. They are to be stored in a cooler room maintained at 0°C. The air
velocity in the cooler is 0.25 m/s. The turkeys are stored in boxes with 4 to 5 tur-
keys in each box. Assume that each box may be treated as a sphere 0.91 m in
diameter and that the box material may be neglected. Using Planck’s equation,
estimate the time interval for the turkeys to obtain a 0°C temperature. If the
shipment consisted of 27215 kg of turkeys, what would be the increase in refrig-
erating capacity needed during cool down of the turkeys?
Fundamentals of Refrigeration SI, Second Edition

11-5 (contd.)

Chapter 11 Skill Development Exercises


11-6 The heat exchanger shown in Figure 11-14 serves as an evaporator for a vapor-
compression system. The refrigerant temperature in the evaporator is constant
at –9.4°C. A secondary fluid, a 50% by volume solution of ethylene glycol and
water, flows at 27.21 kg/min, where the heat transfer rate is 8 kW. For this heat
exchanger, UA is known to be 1.58 kW/K. Calculate the inlet and outlet tem-
peratures for the ethylene glycol solution.
Fundamentals of Refrigeration SI, Second Edition
Chapter 11 Skill Development Exercises

Figure 11-14 Schematic diagram of the heat exchanger in Skill Development Exercise 11-6.

11-6 (contd.)
Fundamentals of Refrigeration SI, Second Edition

Student Information

Chapter 12 Skill Development Exercises


Name ________________________________________________________

Company/Department _________________________________________

Address ______________________________________________________

City ______________________________ State ______ Zip ____________

Telephone _________________________ Fax ________________________

E-mail ________________________________________________________

Student Number ______________________________________________

Skill Development Exercises for Chapter 12


Total number of questions: 5

12-1 Identify the refrigeration system components that are common to each of the
four applications described in this chapter. Identify at least one unique differ-
ence between each of the four applications.
Fundamentals of Refrigeration SI, Second Edition

12-2 Explain why an evaporator pressure regulator is needed in a supermarket utiliz-


ing multiple evaporators operating at different intermediate temperatures. Does
Chapter 12 Skill Development Exercises

the use of EPRs improve or degrade the operating efficiency of the refrigera-
tion system? Explain why.

12-3 Why is ammonia generally selected as the refrigerant for use in refrigeration
systems applied to warehouses and meat-processing plants?
Fundamentals of Refrigeration SI, Second Edition

12-4 Consider the lyo refrigeration system shown schematically in Figure 12-19.
Redraw that part of the schematic diagram for this system to show the addition

Chapter 12 Skill Development Exercises


of a liquid subcooler, its location, and its connections to operate. What would
be the advantage of adding a liquid subcooler to this cycle?
Fundamentals of Refrigeration SI, Second Edition

12-5 A lyo refrigeration system operates with refrigerant R-123. From the log sheets
recorded by a technician over the past three months, refrigerant R-123 has been
Chapter 12 Skill Development Exercises

added to the system on a monthly basis as follows:

Month No. Quantity R-123 Added (kg)


1 10.9
2 15.4
3 22.2

Assuming that each month contains 30 days, what is the average daily leak rate
of R-123 for each month? Is there reason for concern?
ASHRAE LEARNING INSTITUTE
Self-Directed Learning Course Evaluation Form

Course Title: Fundamentals of Refrigeration (SI), Second Edition (2016)

On a scale of 1 to 5, circle the number that corresponds to your feeling about the statements below.
(1 = strongly agree, 5 = strongly disagree, 3 = undecided)

Strongly Strongly
Course Content Agree Undecided Disagree

1. The objectives of the course were clearly stated. 1 2 3 4 5


2. The course content supported the stated objectives. 1 2 3 4 5
3. The content quality and format of the course material make it valuable as a future reference. 1 2 3 4 5
4. The quality and clarity of the charts and diagrams enhanced your ability 1 2 3 4 5
to understand the course concepts.
5. The organization of course material supported effective mastery of the topic. 1 2 3 4 5
6. The material presented will be of practical use to you in your work. 1 2 3 4 5
7. The degree of difficulty (level) of this course was correct to meet your 1 2 3 4 5
needs and expectations.

General

1. Which description best characterizes your primary job function?


_____Architect* _____Developer _____Manufacturer _____Sales
_____Code Agency _____Educator/Research _____Marketing _____Specifier
_____Consultant _____Energy Conservation _____Owner _____Student
_____Contractor/Installer _____Facilities Engineer _____Plant Engineer _____Utilities
_____Consumer/User _____Government _____Policy Maker/Regulator

_____Other (please specify) _______________________________________________________________________________


*Are you a registered architect? ___No ___Yes, AIA Membership Number (required)_____________________

2. Which describes your educational background?


_____High School _____Master's Degree—Engineering
_____Associates Degree/Certificate Program _____Master's Degree—Other Than Engineering
_____Bachelor's Degree—Engineering Technology _____Doctoral Degree—Engineering
_____Bachelor's Degree—Engineering _____Doctoral Degree—Other Than Engineering
_____Bachelor's Degree—Other Than Engineering

_____Other (please specify)_______________________________________________________________________________

3. Approximately how many hours did it take you to complete this course?
_____10 hours _____20 hours _____30 hours _____40 hours _____Other (please specify)___________

4. What topics would you suggest for future courses? ______________________________________________________________


______________________________________________________________________________________________________
______________________________________________________________________________________________________
______________________________________________________________________________________________________
______________________________________________________________________________________________________
General comments regarding any aspect of the course, including suggestions for improvement:

_________________________________________________________________________________________________________

_________________________________________________________________________________________________________

_________________________________________________________________________________________________________

_________________________________________________________________________________________________________

_________________________________________________________________________________________________________

_________________________________________________________________________________________________________

_________________________________________________________________________________________________________

_________________________________________________________________________________________________________

_________________________________________________________________________________________________________

_________________________________________________________________________________________________________

_________________________________________________________________________________________________________

_________________________________________________________________________________________________________

_________________________________________________________________________________________________________

_________________________________________________________________________________________________________

_________________________________________________________________________________________________________

_________________________________________________________________________________________________________

_________________________________________________________________________________________________________

_________________________________________________________________________________________________________

_________________________________________________________________________________________________________

_________________________________________________________________________________________________________

_________________________________________________________________________________________________________

Name (optional) __________________________________________________________________________________________

Phone (optional) __________________________________________________________________________________________

E-mail (optional) __________________________________________________________________________________________

Return to: ASHRAE, Education Department, 1791 Tullie Circle NE, Atlanta, GA 30329
Email: edu@ashrae.org Fax: 404-321-5478
Flexible and Effective Continuing Education
for HVAC&R Professionals

This essential up-to-date revision of ASHRAE’s Fundamentals of Refrig-


eration addresses the three major areas of refrigeration: comfort cooling
(air conditioning), commercial refrigeration, and industrial refrigeration.
Each of twelve chapters builds stepwise in a logical progression from an
understanding of basic principles to a consideration of practical details to
their implications in commercial applications. Skill Development Exercises
at the end of each chapter help readers assess their understanding of the
material and apply what they learn to real-world situations. Answers to to
these exercises can be submitted to earn PDH, CEU, or LU credits.

Fundamentals of Refrigeration is an indispensable introduction to refrigera-


tion for engineering graduates who wish to broaden their expertise, as
well as veteran technicians seeking to brush up on current theory and
practice.

1791 Tullie Circle


Atlanta, GA 30329-2305
Telephone: 404/636-8400
Fax: 404/321-5478
E-mail: edu@ashrae.org
www.ashrae.org/ali

ISBN 978-1-939200-14-3 (paperback)


ISBN 978-1-939200-15-0 (PDF)

9 781939 200143
Product Code: 98046 11/16

SDL_cover_SI.indd 2 10/26/2016 9:20:50 AM

You might also like